You are on page 1of 243

SSC CGL TOPIC

WISE QUESTIONS
1. Percentage
2. Profit & Loss
3. Discount
4. Simple interest
5. Ratio
6. Proportion
7. Age & Partnership
8. Mixture
9. Average
10. Time & Work
11. Time & Distance
12. Train
13. Race
14. Boat & Stream
15. Number System
16. LCM & HCF
17. Simplification
18. Algebra
19. Trigonometry
20. Geometry (Triangle)
21. Geometry (Circle)
22. Mensuration 2D
23. Mensuration 3D
24. Data Interpretation
25. Statistics
26. Probability
INDEX

1. Percentage 01 – 12
2. Profit & Loss 13 – 23
3. Discount 24 – 34
4. Simple interest 35 – 45
5. Ratio 46 – 50
6. Proportion 51 – 54
7. Age & Partnership 55 – 57
8. Mixture 58 – 61
9. Average 62 – 67
10. Time & Work 68 – 81
11. Time & Distance 82 – 90
12. Train 91 – 94
13. Race 95 – 97
14. Boat & Stream 98 – 100
15. Number System 101 – 111
16. LCM & HCF 112 – 117
17. Simplification 118 – 121
18. Algebra 122 – 144
19. Trigonometry 145 – 172
20. Geometry (Triangle) 173 – 197
21. Geometry (Circle) 198 – 207
22. Mensuration 2D 208 – 216
23. Mensuration 3D 217 – 230
24. Data Interpretation 231 – 237
25. Statistics 238 – 239
26. Probability 240 – 241

https://sscstudy.com/
Join Telegram- Maths by Aditya Ranjan PERCENTAGE

PERCENTAGE
01

1. The length and breadth of a rectangle are 4. In the election, 8% of the voters did not cast
increased by 8% and 5%, respectively. By how their votes. Two candidates contested the
much percentage will the area of the rectangle election in which the winning cadidate got 48%
increase? of the total votes and won election by 1100
votes. Find the total number of voters in the
election, given that all the votes cast were valid?


r
SSC CGL 01/12/2022 (Shift- 01)

si
(a) 13.4% (b) 15.4%
SSC CGL 02/12/2022 (Shift- 01)
(c) 12.4% (d) 16.4%
2.
an by
The price of a scooter increases successively
(a) 25500
(c) 27500
(b) 26500
(d) 28500

n
by10%, 5% and 15%. What is the total 5. In an election between two candidates, the
percentage increase in price of scooter? defeated candidate secured 42% of the valid
ja votes polled and lost the election by 7,68,400

R s
votes. If 82,560 votes were declared invalid
and 20% people did NOT cast their vote, then
a th

the invalid votes were what percentage


(rounded off to 1 decimal place) of the votes
SSC CGL 01/12/2022 (Shift- 02)
which people did NOT cast?
ty a

33 21
(a) 32 % (b) 34 %
40 40
di M


11 31
(c) 30 % (d) 36 %
40 40

3. Raju, Ravi and Ashok contested an election.
5% votes polled were invalid. Raju got 30%
of the total votes. Ravi got 32% of the total SSC CGL 02/12/2022 (Shift- 02)
votes. The winner got 5136 more votes than
(a) 10.6 Percent (b) 9.8 Percent
the person who received the least number of
(c) 12.9 Percent (d) 6.8 Percent
votes. Find the total number of votes polled.
6. A district has 10,24,000 inhabitants. If the
population increases at the rate 2.5% per
A

annum, find the number of inhabitants at the


end of three years.

10,24,000


SSC CGL 01/12/2022 (Shift- 04)
SSC CGL 02/12/2022 (Shift- 02)
(a) 171200 (b) 64200 (a) 11,20,736 (b) 11,02,736
(c) 171220 (d) 172100 (c) 10,75,840 (d) 10,64,850

Aditya Ranjan (Excise Inspector) Selected Selection 1


https://sscstudy.com/
Join Telegram- Maths by Aditya Ranjan PERCENTAGE

7. The reduction of 20% in the price of rice


enables a person to obtain 50 kg more for
Rs.450. Find the original price of rice per kg.


SSC CGL 03/12/2022 (Shift- 03)
kg
(a) 26% (b) 27%
kg
(c) 25% (d) 24%
SSC CGL 02/12/2022 (Shift- 02)
12. The cost of apples is increased by 20% and
(a) Rs.1 (b) Rs.2 then decreased by 20%. What is the net
(c) Rs.1.25 (d) Rs.2.25 pecentage decrease?
8. In an election between two candidates, 12%
of voters did not cast their votes. The winner

by obtaining 68% of the total votes defeated
his contestant by 2880 votes. What was the SSC CGL 03/12/2022 (Shift- 04)
total number of voters who cast their votes (a) 4% (b) 3%
in the election? (c) 5% (d) 6%

r
13. 0.5 is what percentage of 20?

si
0.5, 20
an by

(a) 25%
SSC CGL 05/12/2022 (Shift- 01)
(b) 0.25%

n
SSC CGL 02/12/2022 (Shift- 03) (c) 2.5% (d) 0.025%
(a) 5280 (b) 8000 14. Mr. X has a monthly income of $26,500 and
ja
R s
(c) 4000 (d) 6000 his monthly expenditure is $20,500. The next
year, his salary is increased by 12% and
a th

1 expenditure is increased by 6%. His savings


9. Ram loses 12 % of his money and after
2 increase by how much percent?
spending 75% of the remainder, is left with X $26,500
Rs.630. How much money did Ram have
ty a

$20,500 12%
initially?
6%
di M

1
12 %
2 SSC CGL 05/12/2022 (Shift- 03)
(a) 32.5% (b) 27.3%
(c) 32.0% (d) 34.7%
15. During the first year, the population of a town
SSC CGL 03/12/2022 (Shift- 02) increased by 12%.The next year, due to some
(a) Rs.2,080 (b) Rs.2,880 contagious disease, it decreased by 8%. At the
(c) Rs.2,205 (d) Rs.2,808 end of the second year, the population was
64,400. Find the population of the town at
10. Number p is 15% more than 150. If k is 15%
beginning of the first year.
less than p, then k is equal to:
A


p, 150 15% k, p 15%

k

SSC CGL 03/12/2022 (Shift- 03)

(a) 136. 324 (b) 166. 625

(c) 116. 328 (d) 146. 625
SSC CGL 05/12/2022 (Shift- 04)
11. If the price of salt decreases by 20%, then
by what percentage should consumption be (a) 50,00 (b) 54,750
increased to keep the expenditure same? (c) 62,500 (d) 50,500

Aditya Ranjan (Excise Inspector) Selected Selection 2


https://sscstudy.com/
Join Telegram- Maths by Aditya Ranjan PERCENTAGE

16. Two candidates P and Q contested in an 19. A man loses 20% of his money and after
election. 70% of the registered voters are P spending 85% of the remainder, he is left with
supporters. If 60% of the P supporters and Rs.120. How much did he have at first?
30% of the Q supporters are expected to vote
for candidate P, then what percentage of the
registered voters are expected to vote for
candidate P?
P Q
P P
SSC CGL 06/12/2022 (Shift- 04)
Q
(a) Rs.1,200 (b) Rs.1,000
P
P (c) Rs.800 (d) Rs.500
20. A mask manufacturing company manufactured
SSC CGL 06/12/2022 (Shift- 01) ‘X’ number of masks in 2018. It increased its
(a) 30% (b) 51% manufacturing capacity by 30% in 2019 and

r
(c) 26% (d) 47% further increased its manufacturing by 15%

si
17. The population of country A decreased by p% in 2020. In 2021, due to the machinery
and the population of country B decreased by breakdown, its manufacturing declined by
an by
q% from the year 2020 to 2021. Here 'p' is
greater than ‘q’. Let 'x' be the ratio of the
40%. What is the value of ‘X’ if it
manufactured 179400 masks in 2021?

n
population of country A to the population of
'X'
country B in the given year. What is the
ja
percentage decrease in 'x' from 2020 to 2021?
R s
A p%
a th

B q%
'p', 'q' 'x'
A B
ty a

'x' 'X'
SSC CGL 07/12/2022 (Shift- 01)
di M

SSC CGL 06/12/2022 (Shift- 02)


(a) 180000 (b) 230000
100  p – q  100  p – q  (c) 200000 (d) 210000
(a) (b)
100 – q  100 + p
21. 20% of the inhabitants of a village having died
100  p – q  100  p – q  of malaria, a panic set in, during which 30%
(c) (d) of the remaining inhabitants left the village.
100 – p 100 + q
The population was then reduced to12,000.
18. If the price of petrol is increased by 28%, by What was the number of inhabitants initially
what percentage should the consumption be (Consider integral part only)
decreased by the consumer, if the expenditure
on petrol remains unchanged? (Correct to 2
A

decimal places)



SSC CGL 07/12/2022 (Shift- 02)
SSC CGL 06/12/2022 (Shift- 03)
(a) 21000 (b) 21428
(a) 12.35% (b) 21.88%
(c) 20.25% (d) 25.75% (c) 21500 (d) 30428

Aditya Ranjan (Excise Inspector) Selected Selection 3


https://sscstudy.com/
Join Telegram- Maths by Aditya Ranjan PERCENTAGE

22. The length and breadth of a cuboid is


increased by 20% and 25%, respectively, while

its height is reduced by 30%. What must be
the total percentage increase/decrease in the
volume of the cuboid?



SSC CGL 08/12/2022 (Shift- 03)

SSC CGL 07/12/2022 (Shift- 03) (a) 7,00,000 (b) 10,00,000
(a) Increase by 5% (b) Increase by 3%
(c) 8,97,993 (d) 9,92,519
(c) Decrease by 2% (d) Decrease by 4%
23. Three candidates were participating in an 26. Three candidates P, Q and R participated in
election. The person at third place got 20% an election. P got 35% more votes than Q,
of the total votes while the difference between and R got 15% more votes than Q. P overtook

r
the votes of the winner and the first runner R by 2,412 votes. If 90% voters voted and
up was 20% of the total votes. If the difference

si
no invalid or illegal votes were cast, then what
between the votes of the first runner up and
was the number of voters in the voting list?
the second runner up was 37,000, how many
an by
votes did the winner receive? P, Q R

n
P Q R Q
P R
ja

R s



a th




SSC CGL 07/12/2022 (Shift- 04) SSC CGL 09/12/2022 (Shift- 01)
ty a

(a) 1,80,000 (b) 1,85,000


(a) 46,900 (b) 42,800
(c) 1,75,000 (d) 1,95,000
di M

24. In an election between two candidates, a (c) 42,210 (d) 48,500


candidate who gets 76 % of the total votes
polled is elected by a majority of 1480 votes. 27. In an election contested between two
What is the total number of votes polled? candidates, 15% of the total voters did not
(Consider integral part only) cast their votes and 100 votes got disqualified.
The candidate who won the election won it
by securing 45% of the total votes and won

by a margin of 400 votes. Find the total

number of voters?

SSC CGL 08/12/2022 (Shift- 01)

A

(a) 2800 (b) 1800


(c) 2846 (d) 1846
25. In the first year the population of a town
decreased by 5% due to the Corona virus first

wave. In the next year it decreased again by
5% due to the second wave and in the third
SSC CGL 09/12/2022 (Shift- 02)
year it increased by 5%. At the end of the
third year the population was 9,47,625. What (a) 6,000 (b) 3,600
was the population at the beginning of the
first year? (c) 10,000 (d) 3,500

Aditya Ranjan (Excise Inspector) Selected Selection 4


https://sscstudy.com/
Join Telegram- Maths by Aditya Ranjan PERCENTAGE

28. During the first year, the strength of a school 31. Out of an earning of Rs.720, Pankaj spends
increased by 12%, in the second year it 65%. How much does he save?
decreased by 12% and in the third year it
Rs.720
increased by 10%. At the end of the third year
its strength was nearly 10842. What was the
strength at the beginning of the first year?
SSC CGL 12/12/2022 (Shift- 03)
(a) Rs.250 (b) Rs.252

(c) Rs.253 (d) Rs.251

32. The expenditure on the cost of living of a man
and his family is 60% of his salary. He gets

two increment of 30% and 60% in his salary.
SSC CGL 09/12/2022 (Shift- 03) If his saving increases by 250% of his initial
saving, then the new cost of living expenditure
(a) 8000 (b) 10000 is what percentage (rounded off to the nearest

r
(c) 6500 (d) 12000 integer) of the increased salary?

si
29. In an election between two candidates, a
candidate who gets 64% of the votes polled
an by
is elected by a majority of 252 votes. What
is the total number of votes polled?

n


ja

R s
SSC CGL 12/12/2022 (Shift- 04)
a th


(a) 32.7 Percent (b) 43.6 Percent
SSC CGL 09/12/2022 (Shift- 04)
(c) 29.5 Percent (d) 36.6 Percent
ty a

(a) 950 (b) 900


33. The number of units manufactured by a
(c) 850 (d) 800 company A was 12500 units in 2019 and
di M

30. A number is decreased by 20% to get another 10625 units in 2020. While in company B,
number. The number so obtained is increased the production fell from 34000 units in 2019
by 200% to get a third number. The difference to 30600 units in 2020. If X and Y are the
of the third number and the original number percentage decrease in the number of units
is what percentage more or less than the manufactured by company A and B
difference of second and the third number? respectively from 2019 to 2020, then what
will be the ratio of X and Y?

A



B
A


X Y

A B
SSC CGL 12/12/2022 (Shift- 02) X Y
(a) More, 15.7 Percent
(b) Less, 15.7 Percent SSC CGL 13/12/2022 (Shift- 01)

(c) More, 12.5 Percent (a) 5 : 3 (b) 3 : 4

(d) Less, 12.5 Percent (c) 3 : 2 (d) 8 : 5

Aditya Ranjan (Excise Inspector) Selected Selection 5


https://sscstudy.com/
Join Telegram- Maths by Aditya Ranjan PERCENTAGE

34. The sum of two numbers is 680. If the bigger (a) 408 (b) 310
number is decreased by 15% and the smaller
(c) 308 (d) 410
number is increased by 15%, then the
resultant numbers are equal. Find the smaller 38. If radius of a sphere is decreased by 48%, then
number. by what percent does its surface area
decrease?
%

SSC CGL TIER - II 02/03/2023
SSC CGL 13/12/2022 (Shift- 02)
(a) 82.91% (b) 72.96%
(a) 307 (b) 285
(c) 289 (d) 304 (c) 78.98% (d) 86.26%
35. Shiva’s monthly salary is Rs.75,000. He 39. Two candidates contested an election.One of
spends Rs.12,000 on household items, them got 64% of the votes and won by 434
Rs.14,000 on LIC and mutual funds, Rs.15,000 votes.What was the total number of votes polled?

r
on his children’s education, and keeps
Rs.10,000 for miscellaneous expenses. His

si
savings are Rs.21,000 per month and he
donates the rest of the amount to an or
an by
phanage. The percentage of his income that
SSC CGL TIER- II 03/03/2023
he donates is:

n
(a) 1550

ja (b) 1345
R s
(c) 1680

a th

(d) 1684

40. Mrs. Deepa Devi saves 30% of her salary. If he
gets Rs. 42,000 per month as salary, what is

ty a

his monthly expenditure?


SSC CGL 13/12/2022 (Shift- 03)
(a) 3% (b) 6%

di M

(c) 8% (d) 4%

36. Two numbers are, respectively, 10% and 25%
more than the third number. The ratio of the two SSC CGL TIER- II 06/03/2023
numbers is: (a) Rs. 29,300
% % (b) Rs. 29,200

(c) Rs. 29,400
SSC CGL TIER - II 02/03/2023
(d) Rs. 29,100
(a) 22 : 25
41. If 20% of (A + B) = 30% of (A – B), then what
(b) 19 : 25
percentage of B is equal to A?
A

(c) 23 : 25
(A + B) 20% = (A – B) B
(d) 18 : 25
A
37. A basket contains 350 eggs. If 12% of the eggs
are rotten, how many eggs are good enough SSC CGL TIER- II 07/03/2023
to be sold? (a) 400%
% (b) 300%
(c) 500%
SSC CGL TIER - II 02/03/2023 (d) 100%

Aditya Ranjan (Excise Inspector) Selected Selection 6


https://sscstudy.com/
Join Telegram- Maths by Aditya Ranjan PERCENTAGE

ANSWER KEY
1.(a) 2.(a) 3.(a) 4.(c) 5.(d) 6.(b) 7.(d) 8.(a) 9.(b) 10.(d)

11.(c) 12.(a) 13.(c) 14.(a) 15.(c) 16.(b) 17.(a) 18.(b) 19.(b) 20.(c)

21.(b) 22.(a) 23.(b) 24.(c) 25.(b) 26.(a) 27.(a) 28.(b) 29.(b) 30.(d)

31.(b) 32.(a) 33.(c) 34.(c) 35.(d) 36.(a) 37.(c) 38.(b) 39.(a) 40.(c)

41.(c)

r
si
an by
n
ja
R s
a th
ty a
di M
A

Aditya Ranjan (Excise Inspector) Selected Selection 7


https://sscstudy.com/
Join Telegram- Maths by Aditya Ranjan PERCENTAGE

SOL U TION S
1. (a) 5. (d)
Let total valid votes be 100
Old New
Length 100 108 ATQ,
Breadth 100 105 (58 – 42)% = 768400
Area 10000 11340 16% = 768400
11340 – 10000 100% = 4802500
% Change =  100
10000
 Valid votes = 4802500
= 13.4%
Ivalid votes = 82560

r
2. (a)
Total votes = 4885060
Old New

si
No. of people who did not cast their votes
10% 10 11
5%
15%
20
20
an by 21
23
=
20
100
 4885060

n
4000 5313 = 1221265

ja 5313 – 4000  Required% =


82560
 100
R s
% Increase =  100
4000 1221265
a th

33 = 6.8%
= 32 % 6 (b)
40
3. (a) 1
2.5% =
ty a

Let total votes be 100 40


Initial Final
Raju Ravi Ashok
di M

I. 40 41
30 32 33
II. 40 41
3  5136
III. 40 41
100  171200
64000 68921
4. (c)
64000  1024000
Let total votes be
1024000
100 68921   68921
64000
= 16 × 68921
–8%
= 1102736
7. (d)
A

92 Let total amount be 100 new price


 For Rs. 20 = 50 kgs.
For Rs. 100 = 250 kgs.
450
48 44  New price =
250

450 100
 Old price = 
4  1100 250 80
100  27500 = 2.25

Aditya Ranjan (Excise Inspector) Selected Selection 8


https://sscstudy.com/
Join Telegram- Maths by Aditya Ranjan PERCENTAGE

8. (a) 12. (a)


Let total votes be Initial Final
I. 5 6
100 II. 5 4
25 24
–12% 25 – 24 1
Net% Decrease = =
25 25
= 4%
Votes Polled 88
13. (c)
ATQ,
0.5
 100
68 20 20
Winner Looser 100 5
= = = 2.5%

r
40 2
ATQ,
14. (a)

si
48  2880
2880 Income 26500 53
88 
48
an by
= 5280.
 88 = =
Expediture 20500 41

n
Old New
9. (b)
Income 53 53 + 6.36 = 59.36
1
12 % =
25
%
ja Exp. 41 41 + 2.46 = 43.46
R s
2 2
Savings 12 15.90
Let total money be Rs. 200
a th

ATQ, 15.90 – 12
 % change in Savings = 100
175 25 12
   630
1 100
ty a

195
= = 32.5%
630 6
 200   4  200
di M

175 15. (c)


= 2880 Old New
10. (d)
I. 100 112
Overall change in 150
II. 100 92
15  15 10000 10304
= 15  15  =  2.25
100
10304  64400
2.25 6.75
Now,  150 = 64400
100 2 10000   10000
10304
6.75 = 62500
 k = 150 – = 146.625
2 16. (b)
A

11. (c) Let total votes be 100


Initial Final P Q
Price 5 4 Supporters 70 30
Exp. 1 1 60%   30%
Consumption 4 5 42  9
51
1  Expected votes of P = 42 + 9
% Increase = = 25%
4 = 51%

Aditya Ranjan (Excise Inspector) Selected Selection 9


https://sscstudy.com/
Join Telegram- Maths by Aditya Ranjan PERCENTAGE

17. (a) 21. (b)

A x Let total initial population be 100x


= ..........(1)
B 1 ATQ,
A B
80 70
2020 x 1 100x    12000
100 100
(100 – p) x (100 – q)
2021  100x  21428
100 100
22. (a)
A1
(100 – p) Old New
 = ..........(2)
B1 (100 – q) Length 5 6
From (1) and (2) Breadth 4 5
Height 10 7
(100 – p) x
% Change = x – Volume 20 21
(100 – q)
 100

r
x 1
 % Change =  100  5% Increase

si
20
(100 – q– 100  p) x
=  100 23. (b)
an by (100 – q) x
Let total votes be 100, then
100 (p– q)

n
= A B C
100 – q 50 30 20
18. (b) ja ATQ,
R s
Old New 10  37000
a th

Price 100 128 50  185000


Exp. 1 1 24. (c)
Consp. 128 100 Let, total votes polled = 100, then
ty a

A B
28
 Required% =  100 76 24
di M

128
ATQ,
= 21.88%
19. (b) 76 – 24  52  1480
Let initial money = 100 1480
100   100
ATQ, 52
= 2846
15
 80  120 25. (b)
100
120 Old New
100   100  100
15  80 I. 20 19
= 1000 II. 20 19
A

20. (c) III. 20 21


Old New 8000 7581
2019 100 130 ATQ,
2020 100 115
7581  947625
2021 100 60
947625
1000 897 8000   8000
7581
897  179400
1000  200000 = 10,00,000

Aditya Ranjan (Excise Inspector) Selected Selection 10


https://sscstudy.com/
Join Telegram- Maths by Aditya Ranjan PERCENTAGE

26. (a) 31. (b)


Let, votes Q get = 100, then Let total earning = 100
P Q R ATQ,
135 100 120 100%  720
ATQ, 720
35%   35 = 252
20  2412 100
2412 32. (a)
1
20 Let his inintial salary be 100.
2412 100 +30%  130 
Income 100  +60%  208
Total votes polled =  350 
20 90
Exp. 60
= 46900
27. (a) +250% 140
Savings 40 
ATQ,
Total votes = invalid votes + votes not casted  New Exp. = 208 – 140 = 68

r
+ winner votes + looser votes 100% = 15% + 68
100 + 45% + 45% – 400 Required% =  100 = 32.7%

si
208
 5% = 300
 100% = 6000
an by 33. (c)
28. (b) 2019  2020  Net decrease %Decrease

n
Old New 1875
I. 100 112 A 12500  10625  1875   100 = 15%
12500
II. ja
100 88 B 34000  30600  3400  10%
R s
III. 10 11
15 3
a th

100000 108416  Required ratio = =


10 2
108416  10842
34. (c)
10842
100000   100000 A + B = 680
ty a

108416
 10000 17 23
A= B
di M

29. (b) 20 20
Let total votes polled = 100
A 23
A B  =
B 17
64 36  40  680
ATQ, 17  289
28  252 35. (d)
252 Salary = 75000
100   100 = 900
28 Exp.  12000
30. (d) 14000
Let original no. be 100, then. 15000
A

A B C 10000
100  80  240 51000
ATQ, Savings = 21000
240 – 100 = 140 Donation = 75000 – 72000
240 – 80 = 160 = 3000
20 3000
 Required% =  100  Required% =  100
160 75000
= 12.5% Loss = 4%

Aditya Ranjan (Excise Inspector) Selected Selection 11


https://sscstudy.com/
Join Telegram- Maths by Aditya Ranjan PERCENTAGE

36. (a) 39. (a)


Let, three numbers be Total votes = 100%
A B C Wining votes = 64%
110 125 100 Loosing votes = 36%
 A : B = 110 : 125 A.T.Q,
= 22 : 25 64 – 36  434
37. (c) 28  434
Total eggs = 350 100  1550
40. (c)
12
Rotten eggs =  350
100 Salary  100%  42000
Expenditure = 100 – 30 = 70%
(100 – 12)
Good enough =  350 42000
100 70   70
100

r
88 = 29400
=  350
100

si
41. (c)
= 308
an by 20 30
38. (b) Given, (A B) = (A – B)
100 100
Initial Final

n
2A + 2B = 3A – 3B
Radius 100 52
5B = A
S. Ar (Radius)²
ja 2500 676
R s
B 1
2500 – 676 =
Required% =  100 A 5
a th

2500
 A = 500% of B
= 72.96%
ty a

......-------......
di M
A

Aditya Ranjan (Excise Inspector) Selected Selection 12


https://sscstudy.com/
Join Telegram- Maths by Aditya Ranjan PROFIT & LOSS

PROFIT & LOSS


02

1. A dishonest merchant sells goods at a 12.5% 5. A pen was sold for Rs.166.44 with a profit
loss on the cost price, but uses 28 g weight of 14%. If it were sold for Rs.154.76, then
instead of 36 g. What is his percentage profit what would have been the percentage of profit
or loss? or loss?

g g

SSC CGL 01/12/2022 (Shift- 01) SSC CGL 02/12/2022 (Shift- 01)

r
(a) 6.25% loss (b) 12.5% gain (a) 5% loss (b) 6% profit

si
(c) 18.75 gain (d) 10.5% loss (c) 5% profit (d) 6% loss
2. The difference between the cost price and 6. The price of some wooden furniture increases
an by
selling price of a pair of shoes is Rs.1,200.
If the profit is 15%, the selling price is:
by 65% when it passes through three hands.
If the first and second sellers made a profit

n
of 20% and 25%, respectively, then find the
profit percentage of the third seller.
ja


R s


a th

SSC CGL 01/12/2022 (Shift- 02)



(a) Rs.8,200 (b) Rs.9,200
(c) Rs.8,000 (d) Rs.9,000
SSC CGL 02/12/2022 (Shift- 03)
ty a

3. Kapil sells a mobile to Sachin at a gain of 15%


(a) 10% (b) 12.5%
and Sachin again sells it to Rohit at a profit
(c) 16% (d) 8%
di M

of 12%. If Rohit pays Rs.322, what is the cost


price of the mobile for Kapil? 7. By selling a car for Rs.2,78,000, a dealer gains
25%. If the profit is reduced to 18%, then the
selling price will be:



SSC CGL 01/12/2022 (Shift- 03) SSC CGL 02/12/2022 (Shift- 04)
(a) Rs.350 (b) Rs.450 (a) Rs.2,62,432 (b) Rs.2,65,432
(c) Rs.250 (d) Rs.325 (c) Rs.2,72,432 (d) Rs.2,60,432
4. A horse was sold for Rs.60,000 at a profit of 8. A trader has a weighing balance that shows
A

20%. For what price should be have sold to 1300 g for a kg. He further marks up his cost
gain a 30% profit? price by 15%. The net profit percentage is:
kg
g

SSC CGL 01/12/2022 (Shift- 04) SSC CGL 03/12/2022 (Shift- 01)
(a) Rs.64,000 (b) Rs.65,000 (a) 48.5% (b) 49.5%
(c) Rs.50,000 (d) Rs.55,000 (c) 50% (d) 45%

Aditya Ranjan (Excise Inspector) Selected Selection 13


https://sscstudy.com/
Join Telegram- Maths by Aditya Ranjan PROFIT & LOSS

9. If the cost price is 72% of the selling price, 14. A tradesman marks his goods 20% above his
then what is the percentage of profit? (Correct cost price. If he allows his customer 20%
to 2 decimal places) discount on the marked price, how much profit
or loss does he make, if any:

SSC CGL 03/12/2022 (Shift- 03)

(a) 38.89% (b) 35.75%

(c) 32.25% (d) 28.75%
10. A dishonest trader says to customers that he
sells his goods at a cost price, but he uses a SSC CGL 05/12/2022 (Shift- 02)
false weight and gains 12.5% as profit. How
(a) 2% loss (b) 4% loss
many grams does he use to weigh 1 kg?
(c) 1% profit (d) 3% profit

15. A businessman cheats by using faulty weights,


to the tune of 12% each time, when buying

r
and selling material. What is the rise in his
kg profit percentage using faulty weights only?

si


SSC CGL 03/12/2022 (Shift- 04)
an by
(a) 900 g (b) 880.5 g

n
(c) 850 g (d) 888.8 g
11. Ram sold a plot for Rs.4,00,000 at a 20% loss.
For what price should he sell the plot to gain
ja SSC CGL 05/12/2022 (Shift- 03)
R s
a 5% profit?
(a) 22% (b) 25.44%
a th

(c) 24% (d) 26.32%


16. Apples were purchased at 100 for Rs.350 and
SSC CGL 05/12/2022 (Shift- 01) sold at Rs.48 a dozen. What is the profit or
ty a

(a) Rs.5,25,000 (b) Rs.5,20,000 loss percentage?


(c) Rs.5,20,000 (d) Rs.5,05,000
di M


12. A dishonest dealer sells a product at 11% loss
on cost price, but uses 22% less weight. What
is his percentage profit or loss?
SSC CGL 05/12/2022 (Shift- 04)

2 3
(a) Profit 14 % (b) Loss 18 %
SSC CGL 05/12/2022 (Shift- 02) 7 5
(a) 14.1% loss (b) 14.1% gain (c) Loss 10% (d) Profit 12%
(c) 11.4% gain (d) 11.4% loss
17. A man lost 15% by selling a mobile for
13. A shopkeeper earns 10% on an investment
Rs.4,675. What will be his gain percentage by
A

but loses 20% on another investment. If the


ratio of the two investments is 2 : 3, then selling it for Rs.6,050?
the combined loss percentage is:



SSC CGL 06/12/2022 (Shift- 01)
SSC CGL 05/12/2022 (Shift- 02)
(a) 5.5% (b) 7% (a) 10.5% (b) 9.5%
(c) 6% (d) 8% (c) 3.5% (d) 10%

Aditya Ranjan (Excise Inspector) Selected Selection 14


https://sscstudy.com/
Join Telegram- Maths by Aditya Ranjan PROFIT & LOSS

18. A shopkeeper has 2220 kg of rice. A part of 22. The initial profit percentage on the sale of
which he sells at a 20% profit and the rest an item was 55%.If the cost price of the item
at a 12% profit. He gains 18% on the whole. went up by 24% but the selling price remained
The quantity (in kg) sold at a 12% profit is: the same, what would be the new profit
kg percentage?



kg
SSC CGL 06/12/2022 (Shift- 02)
(a) 555 (b) 1210 SSC CGL 07/12/2022 (Shift- 01)
(c) 1665 (d) 425 (a) 36% (b) 25%
19. Piyush sold a guitar to Anuj at 16% gain and (c) 28% (d) 33%
Anuj sold it to Mayank at 32% gain. If Mayank
1
paid Rs.3,828 for the guitar, what amount did 23. A shopkeeper sells an article at 13 % loss.

r
Piyush pay for the same? 2
If he sells it for Rs.1,274 more, then he gain

si

11%. What is the cost price of the article?

an by
1
13 %
2

n


ja SSC CGL 06/12/2022 (Shift- 02)
R s
(a) Rs.2,500 (b) Rs.3,200 SSC CGL 07/12/2022 (Shift- 02)
a th

(c) Rs.1,600 (d) Rs.2,800 (a) Rs.4,874 (b) Rs.4,800


20. The selling price of 28 items is equal to the (c) Rs.5,200 (d) Rs.5,274
cost price of 17 items. What is the percentage 24. A shopkeeper bought a cycle for Rs.1,200 and
ty a

of profit or loss? sold it for Rs.1,500. Find his profit/loss


percentage.
di M


SSC CGL 06/12/2022 (Shift- 03)

2 2
(a) 39 % loss (b) 39 % profit SSC CGL 07/12/2022 (Shift- 03)
7 7
(a) 3% profit (b) 15% loss
2 2 (c) 30% loss (d) 25% profit
(c) 33 % profit (d) 33 % loss
7 7 25. Ramesh claims that he is selling onions at
21. Three-fourth of a consignment was sold at a Rs.36 per kg, which costs him Rs.40 per kg,
profit of 8% and the rest at a loss of 4%. If but he gives 800 grams instead of 1 kg. Find
A

there was an overall profit of Rs.600, then Ramesh’s percentage gain or loss.
find the value of the consignment. kg
kg
kg

SSC CGL 06/12/2022 (Shift- 04) SSC CGL 07/12/2022 (Shift- 04)
(a) Rs.15,000 (b) Rs.6,000 (a) Gain 13.5% (b) Loss 13.5%
(c) Rs.18,000 (d) Rs.12,000 (c) Loss 12.5% (d) Gain 12.5%

Aditya Ranjan (Excise Inspector) Selected Selection 15


https://sscstudy.com/
Join Telegram- Maths by Aditya Ranjan PROFIT & LOSS

26. A shopkeeper advertises for selling cloth at 30. A dealer sells his goods at a profit of 20%,
7% loss. However, by using a false scale of but uses a weight of 800 g in place of a kg
length 1 metre he actually gains 24%. What weight. Find his real gain percentage
will be the actual length he uses instead of
1 metre ?
kg 800 g

SSC CGL 09/12/2022 (Shift- 01)
(a) 40% (b) 44%
(c) 50% (d) 42%
31. Sofia sold an iPhone at the cost of Rs.46,068
SSC CGL 08/12/2022 (Shift- 01) at a loss of 12%. At what cost will she have
to sell it to get a profit of 18%?
(a) 75 cm (b) 31 cm

(c) 76 cm (d) 93 cm

27. A person having bought goods for Rs.400 sells

r
half of it at a gain of 5%. At what gain

si
percentage must he sell the remainder, so as SSC CGL 09/12/2022 (Shift- 02)
to gain 25% on the whole? (a) Rs.61,773 (b) Rs.65,773
an by

32.
(c) Rs.58,350 (d) Rs.52,350
In a certain shop, the profit is 130% of the

n
cost. If the cost increases by 28% and the
selling price remains constant, then what is
ja the profit percentage to the nearest whole
R s
SSC CGL 08/12/2022 (Shift- 02) number?
a th

(a) 30% (b) 25%


(c) 20% (d) 45%
28. Mohit purchased a table for Rs.1,260 and due
ty a

to some scratches on its top, he sold it for


Rs.1,197. What is his loss percent? SSC CGL 09/12/2022 (Shift- 03)
di M

(a) 75% (b) 60%


(c) 59% (d) 80%
33. A man sells two cows for Rs.15,640 each,
gaining 15% on one and losing 15% on the
SSC CGL 08/12/2022 (Shift- 03)
other. Find his total gain or loss.
(a) 7% (b) 4%

(c) 6% (d) 5%
29. If the cost of 20 books is equal to the selling
price of 18 books, then the gain percentage
SSC CGL 09/12/2022 (Shift- 04)
is:
(a) Rs.720 loss (b) Rs.360 loss
A


(c) Rs.720 gain (d) Rs.360 gain
34. The ratio of the cost price and selling price
SSC CGL 08/12/2022 (Shift- 04) of an article is 10 : 11. The gain per cent is:

1 1
(a) 11 % (b) 11 %
9 3
SSC CGL 12/12/2022 (Shift- 01)
2 1 (a) 10% (b) 8%
(c) 11 % (d) 11 %
9 9 (c) 5% (d) 15%

Aditya Ranjan (Excise Inspector) Selected Selection 16


https://sscstudy.com/
Join Telegram- Maths by Aditya Ranjan PROFIT & LOSS

35. A Microwave oven is sold in Hyderabad for SSC CGL 12/12/2022 (Shift- 04)
Rs.M. A retailer, Elahi from Hyderabad went
(a) 25.65% (b) 12.82%
to Madras and bought it for 25% less (when
compared to the price in Hyderabad). He (c) 28.75% (d) 13.41%
spends Rs.1,000 on transport to bring it from 39. Ramesh purchases a table and a chair for
Madras to Hyderabad. He sold it in Hyderabad Rs.3,900. He sells the table at a profit of 8%
for Rs.M making a profit of 10%. Find the and the chair at a profit of 16%. He earns a
value of M (in Rs.). profit of Rs.540. What is the difference
Rs.M between the original price of the table and
the chair?




Rs.M
M SSC CGL 13/12/2022 (Shift- 01)

r
SSC CGL 12/12/2022 (Shift- 01) (a) Rs.2,00 (b) Rs.1,800

si
(a) 6,305.8 (b) 6,258.8 (c) Rs.1,900 (d) Rs.1,700
(c) 6,285.7
an by (d) 6,527.9
40. A girl purchases 9 mangoes for Rs.90 and sells
36. A dishonest dealer professes to sell his goods
10 mangoes for Rs.95. Find the gain or loss
at cost price but uses a false weight and thus

n
percentage.
gains 15%. For a kilogram, he uses a weight
of (rounded off to one digit after decimal).
ja
R s


a th

SSC CGL 13/12/2022 (Shift- 02)


(a) 2.5% loss (b) 5% loss
________ (c) 2.5% gain (d) 5% gain
ty a

41. Ram sold an item costing Rs.1,250 at a gain


SSC CGL 12/12/2022 (Shift- 02) of 18% to Ramesh. It was again sold by
di M

(a) 833.3 gm (b) 876.7 gm Ramesh to Shyam at a loss of 10%. Find the
(c) 869.6 gm (d) 898.33 gm selling price of Ramesh.
37. A watch is sold at a profit of 25%. Had it been
sold for Rs.120 less then, there would have

been a less then of 15%. What is the cost price
in rupees?
SSC CGL 13/12/2022 (Shift- 03)
(a) Rs.1,328.50 (b) Rs.1,428.50
(c) Rs.1,427.50 (d) Rs.1,327.50
SSC CGL 12/12/2022 (Shift- 03) 42. By selling an article for Rs.33,000 a man gains
A

(a) Rs.400 (b) Rs.350 10%. To get a profit of 20%, he has to sell it
(c) Rs.200 (d) Rs.300 for:
38. A dishonest dealer sells the goods at 7% loss
on cost price but uses 18% less weight. What

is his percentage of profit? (Correct to 2
decimal places)
SSC CGL 13/12/2022 (Shift- 04)
(a) Rs.30,000 (b) Rs.35,000
(c) Rs.36,000 (d) Rs.36,600

Aditya Ranjan (Excise Inspector) Selected Selection 17


https://sscstudy.com/
Join Telegram- Maths by Aditya Ranjan PROFIT & LOSS

43. A man buys a machine for Rs. 5,000. After 44. Products pass successively through the hands
one year, he sells it for Rs. 6000. After two of three traders and each of them sells his
years, again he buys the same machine at Rs. goods at a profit of 30% of his cost price. If
8,000 and sells it for Rs.10,000. Find his the last trader sold the products for Rs. 300,
overall profit percentage for both the then how much did the first trader pay for
transactions. them?
Rs.
Rs.
Rs.
Rs.

SSC CGL TIER - II 02/03/2023
SSC CGL TIER- II 03/03/2023
(a) 20.23%
(a) Rs. 330.55 (approx)
(b) 23.08% (b) Rs. 136.55 (approx)

r
(c) 18.75% (c) Rs. 240.55 (approx)
(d) Rs. 137.55 (approx)

si
(d) 15.23%

an by ANSWER KEY

n
1.(b) 2.(b)
ja 3.(c) 4.(b) 5.(b) 6.(a) 7.(a) 8.(b) 9.(a) 10.(d)
R s
11.(a) 12.(b) 13.(d) 14.(b) 15.(b) 16.(a) 17.(d) 18.(a) 19.(a) 20.(a)
a th

21.(d) 22.(b) 23.(c) 24.(d) 25.(d) 26.(a) 27.(d) 28.(d) 29.(a) 30.(c)
ty a

31.(a) 32.(d) 33.(a) 34.(a) 35.(c) 36.(c) 37.(d) 38.(d) 39.(b) 40.(b)
di M

41.(d) 42.(c) 43.(b) 44.(b)


A

Aditya Ranjan (Excise Inspector) Selected Selection 18


https://sscstudy.com/
Join Telegram- Maths by Aditya Ranjan PROFIT & LOSS

SOL U TION S
5. (b) Let C.P of pen be 100
1. (b) 166.44  114
Merchant Customer
114
Price 8 7 154.76   15476
16644
Quantity 28 36 = 106
 6% Profit
224 252 6. (a) I II
20% 25%
 1 25  20
12.5% = 
  Successive profit = 20 + 25 +
100

r
 8
= 50%

si
252 – 224 Let III. profit 60x%
 P/L =  100
224
an by 50x
 50 + x + = 65
100
28
=  100 = 12.5% Profit

n
224 3x
 = 15
2
ja
SP – CP  x = 10%
R s
2. (b) P% =  100
CP 7. (a) Let CP = 100
a th

 125  278000
1200
 15 =  100 278000
CP 118   118
125
 CP = 8000
ty a

= 262432
115
 SP =  8000 = 9200 8. (b)
di M

100 Trader Customer


3. (c) Let C.P of Kapil be 100 Price 100 115

Kapil Sachin Rohit Quantity 1000 g 1300 g

100 115 112 1000 1495


 115
100
= 128.80 495
 P% =  100 = 49.5%
1000
128.80  322
322 9. (a) CP SP
A

100   100  100


12880 72 100
= 250
4. (b) S.P = 60,000 100 – 72
P% =  100
P% = 20% 72
 120  60,000
28 700
=  100 =
60, 000 72 18
130   130
120 = 38.89%
= 65,000 Let the trader measure x gm. instead of l kg.

Aditya Ranjan (Excise Inspector) Selected Selection 19


https://sscstudy.com/
Join Telegram- Maths by Aditya Ranjan PROFIT & LOSS

10. (d) 350


Trader Customer 16. (a) C.P of 1 apple = = 3.5
100
Price 1 1
48
Quantity x 1000 S.P of 1 apple = =4
12
T-Rate 8 9
0.5 2
P% =  100 = 14 %
3.5 7
1000 9
=
x 8 17. (d) Let CP of mobile = 100
4675  85
8000
 x= = 888.8 g 85
9 6050   6050
11. (a) Let CP of plot = 100 4675
80  400000 = 110
 10% profit
400000
105   105 18. (a) By allegation

r
80
= 525000 20 12

si
12. (b)
an by Trader Customer
Price 100 89
18

n
Quantity 78 100
ja 78 89
R s
6 2
a th

11 1100
P% =  100 = :
78 78 3 1
= 14.1% gain. 4  2220
13. (d) Let, 2 investments be 200 and 300 1  555
ty a

Amount on I. investmet  200 + 20 = 220


Amount on II. investment  300 – 60 = 240 19. (a) Piyush Anuj Mayank
di M

500 – 460 132


Combined loss =  100 CP 100 116  116
500 100
= 8%
132  116
 3828
14. (b) CP MP SP 100
3828
100 120 80 100   100  100
 120 132  116
100
= 2500
 96 20. (a) ATQ,
28 SP = 17 CP
A

100 – 96
P/L   100 = 4% Loss SP 17
100 =
CP 28
15. (b) 12% cheating on buying
28 – 17
12% cheating on selling L/P% =  100
28
 overall profit:-
11
=  100
12  12 28
12  12 
100 2
= 39 % Loss
24 + 1.44 = 25.44% 7

Aditya Ranjan (Excise Inspector) Selected Selection 20


https://sscstudy.com/
Join Telegram- Maths by Aditya Ranjan PROFIT & LOSS

21. (d) Let Total CP = 400


26. (a)
ATQ, Shopkeeper Customer

3 Rate 100 93
 324
4
1 Quantity x cm 100 cm
 96
4
x 93
Total SP = 324 + 96 = 420
20  600
Price 100 124
600
400   400 = 12000
20
x 100
22. (b)  =
CP SP Profit 93 124

Initial 100 155 55


9300
 x= = 75 cm

r
New 124 155 31 124

si
31 27. (d) Let the seels remainder at by allegation:-
 P% =  100 = 25%
124
an by x% profit

1 27 5 x
23. (c) 13 %=

n
2 200

CP
ja SP
R s
25
a th

200 173
+ (1274)
200 222
x – 25 20
ty a

 222 – 173 = 49  1274


1274 1 : 1
200   200
di M

49  x – 25 = 20
 5200
24. (d) SP = 1500 x = 45
CP = 1200 28. (d) SP = 1197
P = 300
CP = 1260
300
P% =  100 = 25% L = 63
1200

25. (d) 63
Ramesh Customer L% =  100
1260

Rate 10 9 = 5%
A

29. (*) ATQ,


Quantity 4 5
20 CP = 18 SP
Price 40 45
CP 18 9
 = =
SP 20 10
45 – 40
 P% =  100
40
1 1
100  Gain% = = 11 %
= = 12.5% 9 9
8

Aditya Ranjan (Excise Inspector) Selected Selection 21


https://sscstudy.com/
Join Telegram- Maths by Aditya Ranjan PROFIT & LOSS

30. (c) CP 10
Dealer Customer 34. (a) =
SP 11
Gain = 11 – 10 = 1
Rate 5 6
1
Gain% = = 10%
Quantity 800 1000 10
35. (c) S.P in hydrabad = Rs. M
Price 2 3
3
S.P in Madras = M
4
3–2 ATQ,
Gain% =  100
2
33
= 50%  M  11000 = 10 M
4
31. (a) Let, CP = 100
 88  46068 4
 M = 11000
7
46068
118   118

r
44000
88  M=
7

si
= 61773 = 6285.7
32. (d) an by 36. (c) Let the ares x gm. for 1 kg.
CP SP CP for Dealer = x
SP for dealer = 1000

n
Initial 100 230 ATQ,
x 100 20
ja
New 128 230 = =
R s
1000 115 23
23x = 20000
a th

230 – 128
P% =  100 20000
128 x= = 869.6 gm.
23
102
=  100
ty a

128 37. (d)


CP SP
 80%
di M

I II Initial 100 125


85 – (120)
115
New 100 85

125 – 85 = 40  120
100  300
x
38. (d)
Dealer Customer

Rate 100 93
17 23
A

15640 Quantity 82 100


33. (a) C.P of I cow =  100 = 13600
115
Price 82 93
15640
C.P of II cow =  100 = 18400
85 93 – 82
P% =  100
 Total CP = 32000 82
Total SP = 31280 1100
= = 13.41%
 Loss = 720 82

Aditya Ranjan (Excise Inspector) Selected Selection 22


https://sscstudy.com/
Join Telegram- Maths by Aditya Ranjan PROFIT & LOSS

39. (b) CP = 3900


41. (d) Ram Ramesh Shyam
P = 540
90
100 118  118
540 540 100
P% =  100 =
3900 39 = 106.2

By allegation:- 100  1250


1250
8% 16% 106.2   106.2
100
= 1327.50
42. (c) Let CP = 100
540  110  33000
39% 33000
120   120 = 36000
110
43. (b)

r
84 228 C.P S.P Profit

si
7 : 19 5,000 6,000 1,000
an by 8,000 10,000 2,000
7 + 19 = 26  3900
13, 000 3, 000

n
3900 3, 000
19 – 7 = 12   12 Profit% = × 100 = 23.08%
26 13, 000
ja
R s
= 1800 44. (b)
C.P S.P
a th

90
40. (b) C.P of 1 Mango = = 10 I. Trader 10 13
9
II. Trader 10 13
95 III. Trader 10 13
S.P of 1 Mango = = 9.5
ty a

10
1000 2197
Loss = 0.5
2197  300
di M

0.5
L% =  100 = 5% 300
10 1000   1000 = Rs. 136.55
2197

......-------......
A

Aditya Ranjan (Excise Inspector) Selected Selection 23


https://sscstudy.com/
Join Telegram- Maths by Aditya Ranjan DISCOUNT

DISCOUNT
03

1. Riya could not decide between discount of SSC CGL 01/12/2022 (Shift- 04)
30% or two successive discounts of 25% and (a) Rs.7.80 (b) Rs.700
5%, both given on shopping of Rs.3,840. What (c) Rs.790 (d) Rs.607.2
is the difference between both the discounts? 5. What is the single percentage discount
equivalent to two successive discounts of 15%
and 5%?
15% 5%


r
SSC CGL 02/12/2022 (Shift- 01)
SSC CGL 01/12/2022 (Shift- 01)
(a) 19.00% (b) 19.25%

si
(a) Rs.44 (b) Rs.48
(c) 18.00% (d) 18.25%
(c) Rs.42 (d) Rs.46
an by 6. A laptop is sold for Rs.65,520 after a discount
2. A shopkeeper makes a net profit of 44% on
of 25%. What was the marked price of the
selling an article at successive discounts of
laptop?

n
10% and 20%. Find the net profit percentage,

if the shopkeeper sells the same article at a
ja
discount of 15%.
R s
SSC CGL 02/12/2022 (Shift- 02)
a th

(a) Rs.87,630 (b) Rs.87,360



(c) Rs.87,370 (d) Rs.83,760
7. Sunil purchased a fan at a 10% discount on
the labelled price. If he had purchased it at
ty a

SSC CGL 01/12/2022 (Shift- 02) a 15% discount, he would have saved Rs.400.
(a) 50% (b) 70% Find the labelled price of the fan.
di M

(c) 30% (d) 40%


3. The single discount equivalent to two
successive discounts of 15% and 12% on an
article is:

SSC CGL 02/12/2022 (Shift- 03)
(a) Rs.8,400 (b) Rs.7,500
SSC CGL 01/12/2022 (Shift- 03) (c) Rs.800 (d) Rs.8,000
(a) 3% (b) 25.2% 8. A shopkeeper gives two successive discounts
(c) 74.8% (d) 27% on a watch marked Rs.2,750. The first
4. A dealer buys an old cooler listed at Rs.650 discount given is 10%. If the customer pays
A

and gets successive discounts of 20% and Rs.2,103.75 for the watch, then what is the
10%. He spends Rs.38 on its repairs and sells second discount?
it at a profit of 20%. Find the selling price
of the cooler.


SSC CGL 02/12/2022 (Shift- 04)
(a) 15% (b) 30%
(c) 12% (d) 10%

Aditya Ranjan (Excise Inspector) Selected Selection 24


https://sscstudy.com/
Join Telegram- Maths by Aditya Ranjan DISCOUNT

9. The difference between a discount of 25% and 13. A hotel is giving a discount of 12% on the
two successive discounts of 15% and 10% on booking of 2 or more rooms. Additionally, the
a certain bill was Rs.25. Find the amount of hotel is offering a 5% discount only on
the bill. payment using any card of SBI. Rakesh booked
2 rooms in the hotel for a day at the rate of
Rs.1,500 per room per day. While checking
out, he paid the bill using SBI Silver Card.
How much amount did he have to pay?
SSC CGL 03/12/2022 (Shift- 01)
(a) Rs.3,333.33 (b) Rs.2,500
(c) Rs.833.33 (d) Rs.1,666.67
10. The marked price of an article is Rs.10,927.
Due festive season, a certain percentage of

discount is declared. Raju buys an article at
reduced price and sells it at Rs.109,27, and

makes a profit of 11.5%. What was the

r
percentage discount offered? SSC CGL 05/12/2022 (Shift- 01)

si
(a) Rs.2,498 (b) Rs.1,254
(c) Rs.2,508 (d) Rs.2,618

an by 14. A shopkeeper offers three types of discount
schemes for buyers. Which of them has the

n
maximum discount percentage?


ja
R s

SSC CGL 03/12/2022 (Shift- 02)
a th

(a) 10.3% (b) 11.3% I. Two successive discounts of 10% each.


(c) 11.5% (d) 10.9%
11. A man bought a watch for 12% discount. If II. Successive discounts of 15% and 5%.
ty a

he had bought it for 24% discount, he would


have got the watch for Rs.2400 less. The III. 20% discount.
di M

marked price of the watch is:


SSC CGL 05/12/2022 (Shift- 03)
(a) Only discount scheme I
I
(b) Only discount scheme II

SSC CGL 03/12/2022 (Shift- 03)
II
(c) Only discount scheme III
(a) 30,000 (b) Rs.22,500
III
(c) 20,000 (d) 25,000
(d) All provide equal discount
12. The marked price of a chair is Rs.2,400, which

is 20% above the cost price. If the chair is
A

15. A cap having marked price Rs.90 is sold for


sold at a discount of 10% on marked price,
Rs.68. What is the percent rate of discount
what is the profit percentage?
(correct up to two decimal places)?
90


SSC CGL 03/12/2022 (Shift- 04) SSC CGL 05/12/2022 (Shift- 04)
(a) 10% (b) 8% (a) 24.44% (b) 25.37%
(c) 9% (d) 26.2% (c) 22.00% (d) 23.22%

Aditya Ranjan (Excise Inspector) Selected Selection 25


https://sscstudy.com/
Join Telegram- Maths by Aditya Ranjan DISCOUNT

16. Successive discounts of 18% and 22% are SSC CGL 07/12/2022 (Shift- 01)
equal to a single discount of ______. (a) Rs.2,500 (b) Rs.2,225.5
______
(c) Rs.2,375.5 (d) Rs.2,137.5

21. A discount of 20% is given for the purchase
SSC CGL 06/12/2022 (Shift- 01) of two books by a bookseller and a discount
(a) 36.04% (b) 35.04% of 25% is offered if a customer buys more than
(c) 37.04% (d) 34.04% two books. A 10% discount is being offered
17. If 3 coconuts are offered free on purchase of
to all the customers on purchase of one book.
12 coconuts, priced Rs.25 each, what is the
An additional 5% discount will be given to
effective discount on each coconut?.
students. Sohan, a student of M.Com. in
college, bought a book for Rs.513. What was
the marked price of the book?


SSC CGL 06/12/2022 (Shift- 02)

(a) 20% (b) 24%

r
(c) 20.83% (d) 15%

si
18. A dealer buys an article listed at Rs.3,000 and
gets successive discounts of 15% and 15%.
an by
He spends Rs.250 on transportation and sells
it at a profit of 20%. Find the selling price

n
of the article
SSC CGL 07/12/2022 (Shift- 02)
ja
(a) Rs.650
R s
(b) Rs.540
a th

(c) Rs.600
SSC CGL 06/12/2022 (Shift- 03) (d) Rs.605
(a) Rs.3,300 (b) Rs.2,901
22. What will be the cost price of the goods if a
ty a

(c) Rs.3,250 (d) Rs.3,200


shopkeeper allows a discount of 10% on the
19. Rohan sold goods to Ankith worth Rs.55,000
marked price of Rs.100 and gains 8%(rounded
di M

at 15% trade discount. How much money did


off to the nearest integer)?
Ankith pay to Rohan?


SSC CGL 06/12/2022 (Shift- 04)
(a) Rs.8,250 (b) Rs.12,550 SSC CGL 07/12/2022 (Shift- 03)
(c) Rs.22,550 (d) Rs.46,750
(a) Rs.83 (b) Rs.86
20. In a shop, a discount of 5 percent is provided,
and if the total payable amount after discount (c) Rs.87 (d) Rs.81
is more than Rs.2,000, then an additional 23. At a Christmas sale, all the goods in a shop
discount of 10 percent is provided. Determine are on sale at 30% discount. If Jasmine buys
A

the final amount to be paid (in Rs.) by a a skirt marked at Rs.800, how much would
customer, if he buys five products each of she need to pay?
price Rs.500.




(a) Rs.476 (b) Rs.400
(c) Rs.770 (d) Rs.560

Aditya Ranjan (Excise Inspector) Selected Selection 26


https://sscstudy.com/
Join Telegram- Maths by Aditya Ranjan DISCOUNT

24. A dealer marks his goods at 15% above the 29. A shopkeeper offers a cash discount of Rs.85
cost price and allows a discount of 5% for cash first and then two successive discounts of 10%
payment. His profit percentage is: and 2% on the marked price of a fan. If the
marked price of the fan is Rs.1,285, what is
its selling price (in Rs.)?

SSC CGL 08/12/2022 (Shift- 01)
(a) 9.25% (b) 7.25%
(c) 11.25% (d) 10.25%
25. A shopkeeper listed the marked price of a

chair at a certain amount. If the shopkeeper SSC CGL 09/12/2022 (Shift- 02)
declares a 15% discount and sold it at (a) Rs.1058.4 (b) Rs.1000.4
Rs.1,445, then what is the marked price?
(c) Rs.958.4 (d) Rs.1100

30. A megastore is offering 20% discount on all

grocery items. Sakshi bought one grocery item

r
marked at Rs.400. What is its cost price if

si
SSC CGL 08/12/2022 (Shift- 02) the store earned a profit of 25% after giving
(a) Rs.1,750 (b) Rs.1,800 the discount?

26.
an by
(c) Rs.1,700 (d) Rs.1,850
A shopkeeper claims to sell his article at a

n
discount of 10%, but marks his articles by
increasing the cost of each by 20%. His gain
ja
percentage is:

R s

SSC CGL 09/12/2022 (Shift- 03)
a th


(a) Rs.256 (b) Rs.280
(c) Rs.380 (d) Rs.320
ty a

SSC CGL 08/12/2022 (Shift- 03) 31. A grocery shop is offering 10% discount on
(a) 18% (b) 12% the purchase of Rs.500 and above. A 5%
di M

(c) 8% (d) 16% discount is given on the purchase of value


27. A dealer offers 45% discount on a packet. She above Rs.250 but below Rs.500. A discount
offers a further discount of 25% on the of additional 1% is given if payment is made
reduced price on cash payment. What does instantly in cash. How much would a customer
a customer has to pay for the packet of have to pay by cash if he buys 25 packets of
Rs.2,800, in cash, to avail both discounts? biscuits and one packet is








A

SSC CGL 08/12/2022 (Shift- 04)



(a) Rs.1,480 (b) Rs.1,155
(c) Rs.1,155 (d) Rs.1,230
28. Three successive discounts of 10% is SSC CGL 09/12/2022 (Shift- 04)
equivalent to a single discount of:
(a) Rs.670.25

SSC CGL 09/12/2022 (Shift- 01) (b) Rs.668.25
(a) 27.1% (b) 19.0% (c) Rs.675
(c) 25.1% (d) 26.2% (d) Rs.667.05

Aditya Ranjan (Excise Inspector) Selected Selection 27


https://sscstudy.com/
Join Telegram- Maths by Aditya Ranjan DISCOUNT

32. A shopkeeper offers the following three 35. Raghav went to a shopping mall to purchase
schemes. clothes. He had two coupons with him, but
only one can be used on a single day. Using
the first coupon, a total discount of 30% could
Scheme-I: Two successive discounts of 15%
be obtained on the total amount. Using the
and 25%
second coupon, he will get 80% off on the
I price of the costliest shirt if he buys at least
Scheme-II: Buy 5, get 3 free 3 shirts. What is the price at which Raghav
can purchase 3 shirts at a minimum price if
II
the prices of the three shirts are Rs.1,250,
Scheme-III: Buy 4, get 6 Rs.1,540 and Rs.1,375?
III
Which scheme is the best for customers?

r
SSC CGL 12/12/2022 (Shift- 01)

si
(a) Scheme - I
- I
an by
(b) Scheme - III

n
- III

(c) Scheme - II
ja SSC CGL 12/12/2022 (Shift- 04)
R s
- II
(a) Rs.2,760
a th

(d) Any one-All are equal


(b) Rs.2,915.50

(c) Rs.2,775
33. The printed price of a TV set is Rs.14,500.
(d) Rs.2,933
ty a

It is sold for Rs.10,000 with two consecutive


discounts. If the first discount is 10%, then 36. An article costs Rs.4,000 to a shopkeeper, who
di M

what is the second discount? marks its price at Rs.8,400. The shopkeeper
sells it to a customer at a discount of 25%.
The customer gets a further discount of 15%
on the discounted price if the customer
redeems a coupon issued by the store
SSC CGL 12/12/2022 (Shift- 02) previously. What is the profit percentage (to
the nearest integer) earned by the shopkeeper
(a) 23.37% (b) 25.37%
in this transaction?
(c) 20.37% (d) 27.37%

34. Raghav purchased a shirt at Rs.1,500 and
marked up the price of the shirt by 40%. What
A

is the discount percentage he has to offer in


order to get a profit of Rs.75?



SSC CGL 12/12/2022 (Shift- 03) SSC CGL 13/12/2022 (Shift- 01)
(a) 25% (b) 15% (a) 34% (b) 51%
(c) 75% (d) 50% (c) 42% (d) 36%

Aditya Ranjan (Excise Inspector) Selected Selection 28


https://sscstudy.com/
Join Telegram- Maths by Aditya Ranjan DISCOUNT

37. Sita gets a discount of 20% on Rs.3,000 juicer 41. Find the single equivalent discount for
mixer machine. Since she pays cash, she gets successive discounts of 12%, 18% and 25% on
additional 5% discount too. How much does the marked price of a car.
she pay?




SSC CGL TIER- II 03/03/2023

(a) 35.28%
SSC CGL 13/12/2022 (Shift- 02) (b) 42.65%
(a) Rs.2,280 (b) Rs.2,280 (c) 40.25%
(c) Rs.2,282 (d) Rs.2,278
(d) 45.88%
38. The cost price of an article is Rs.800. After
allowing a discount of 10%, a gain of 12.5% 42. A dishonest trader marks up his goods by 50%
was made. The marked price of the article is: and then allows a discount of 20% on its

r
marked price. Additionally, he uses a faulty

si
scale which measures 900 gm. for 1 kg. shows.

What will be his net profit percentage (rounded
an by SSC CGL 13/12/2022 (Shift- 03)
off to the nearest integer)?
(a) Rs.1,300 (b) Rs.1,200

n
(c) Rs.1,000 (d) Rs.1,100

39. A shopkeeper allows a discount of 12% to his
ja
R s
customers and still gains 18%. Find the 900 gm. 1 kg.
marked price of an article which costs Rs.528
a th

to the shopkeeper.(Answer to the nearest


SSC CGL TIER- II 06/03/2023
rupees)
(a) 33 (b) 36

ty a

(c) 27 (d) 24

43. Two successive discounts of 10% and 10% will


di M

be equal to a single discount of _______.



10% 10%
SSC CGL 09/12/2022 (Shift- 04)

(a) Rs.728
SSC CGL TIER- II 06/03/2023
(b) Rs.708
(a) 20% (b) 18%
(c) Rs.695
(c) 19% (d) 21%
(d) Rs.798
44. At a clearance sale, a shopkeeper gives 45%
40. A bike is sold for Rs. 87,500 by allowing a discount
discount. If a customer paid Rs. 330 during the
of 44% on its marked price. The marked price
sale, then what is the marked price of that
(in Rs.) of the bike is:
A

shirt ?
%
Rs. 87,500
Rs.
SSC CGL TIER - II 02/03/2023 SSC CGL TIER- II 07/03/2023

(a) 1,56,100 (b) 1,58,225 (a) Rs. 550 (b) Rs. 600

(c) 1,56,250 (d) 1,55,500 (c) Rs. 500 (d) Rs. 650

Aditya Ranjan (Excise Inspector) Selected Selection 29


https://sscstudy.com/
Join Telegram- Maths by Aditya Ranjan DISCOUNT

ANSWER KEY
1.(b) 2.(b) 3.(b) 4.(d) 5.(b) 6.(b) 7.(d) 8.(a) 9.(d) 10.(a)

11.(c) 12.(b) 13.(c) 14.(c) 15.(a) 16.(a) 17.(a) 18.(b) 19.(d) 20.(d)

21.(c) 22.(a) 23.(d) 24.(a) 25.(c) 26.(c) 27.(c) 28.(a) 29.(a) 30.(a)

31.(b) 32.(c) 33.(a) 34.(a) 35.(b) 36.(a) 37.(a) 38.(c) 39.(b) 40.(c)

41.(d) 42.(a) 43.(c) 44.(b)

r
si
an by
n
ja
R s
a th
ty a
di M
A

Aditya Ranjan (Excise Inspector) Selected Selection 30


https://sscstudy.com/
Join Telegram- Maths by Aditya Ranjan DISCOUNT

SOL U T ION S
1. (b) 6. (b)
I. case: 30% Discount S.P
II. case: 25% and 5% Discount M.P =  100
(100 – D)
25  5
Eff. Discount = 25  5 – 65520  100
100 =
75
= 28.75%
= 87,360
Difference = 30 – 28.75 = 1.25%
7. (d)
1.25 5% = 400
 × 3840 = 48 Rs.
100 400

r
2. (b) 100% =  100
5

si
200 = 8000
Effective Discount = 10  20 –
100 8. (a)
= 28%
an by Discount = 2750 – 2103.75
= 646.25

n
CP 100 – D 72 1 100 Let, be the II discount
= = = = (say)
MP 100  P 144 2 200 10x 646.25
ja  10  x –   100
R s
100 – 15 85 100 2750
SP = × MP = × 200
100 100 9x
a th

= 170  10   23.5
10
170 – 100 9x 135
 Profit =  100 = 70% = 13.5 =
100 
ty a

10 10
3. (b) x = 15%
9. (d)
di M

15  12
Eff. Discount = 15  12 –
100 15 ×10
Eff. Discount = 15 +10 – = 23.5
= 27 – 1.8 100
= 25.2%  25 – 23.5 = 1.5%  25
4. (d) 25
100%   1000
15
200
Eff. Discount = 20  10 – = 1666.67
100
10. (a)
= 28% Let CP = 100
72 – x%  11.5%
C.P =  650 = 468 M.P  C.P   S.P = M.P
100
 SP  111.5 = MP
A

120 11.5
S.P = (468  38)
100  x% =  100
111.5
= 607.2 Rs. = 10.3%
5. (b) 11. (c)
15  5 24% – 12% = 12% = 2400
Eff. Discount = 15  5 –
100 2400
100%   100
= 20 – 0.75 12
= 19.25% = 20,000

Aditya Ranjan (Excise Inspector) Selected Selection 31


https://sscstudy.com/
Join Telegram- Maths by Aditya Ranjan DISCOUNT

12. (b) 19. (d)


MP CP SP 85
Amount paid =  55000
90 100
120 100  120  108
100 = 46750
 P% = 8% 20. (d)
13. (c)
50
12  5 Eff. Discount = 5  10 –
Eff. Discount = 12  5 – 100
100 = 14.5%
= 17 – 0.6
= 16.4% 85.5
 Amount paid =  2500
83.6 100
 Amount paid = × 3000 = 2137.5
100
= 1254 × 2 21. (c)
= 2508 50
14. (c) Eff. Discount = 10  5 –

r
100
100 = 14.5%

si
I. 10  10 – = 19%
100
513  100
an by
20  = M.P
II. 15  5 – = 19% 85.5
100
 M.P = Rs. 600

n
III. 20%  maximum.
22. (a)
15. (a) ja MP 100  P
R s
=
90 – 68 CP 100 – D
D=  100
a th

90 100 108
=
22 CP 90
=  100 = 24.44%
90
9000
ty a

16. (a) CP =  Rs. 83


108
18  22 23. (d)
di M

Eff. Discount = 18  22 –
100
= 40 – 3.96 70
Amount to pay =  800
= 36.04% 100
17. (a) = Rs.560
M.P = 15 24. (a)
S.P = 12
MP CP SP
3
 Discount =  100 95
15 115 100  115 = 109.25
100
= 20%
18. (b)  Profit = 9.25%
25. (c)
15 ×15
A

Eff. Discount = 15 + 15 – 1445


100 M. P =  100
Eff. Discount = 30 – 2.25 85
= 27.75% = 1700
 72.25  26. (c)
C.P    3000   250
 100  MP CP SP
= 2417.5 90
120 100  120 = 108
120 100
 S.P =  2417.5 = Rs. 2901  Gain = 8%
100

Aditya Ranjan (Excise Inspector) Selected Selection 32


https://sscstudy.com/
Join Telegram- Maths by Aditya Ranjan DISCOUNT

27. (c) 32. (c) Scheme- I:


Eff. Discount = 15 + 25 – 3.75
45 × 25
Eff. Discount = 45 + 25 – = 36.25%
100
Scheme- II:
Eff. Discount = 70 – 11.25
3
= 58.75% Discount =  100 = 60%
5
41.25 Scheme- III:
Amount to pay =  2800
100
2
= Rs. 1155 Discount =  100  33.33%
6
28. (a)  Scheme II is best for customers.

100 33. (a) Discount = 14500 – 10,000


10  10 – = 19% = 4500
100

r
4500 900
190 D% =  100 
19  10 – = 27.1% 14500 29

si
100
29. (c) an by 10x 900
Eff. Discount = 10  x – =
100 29
I. Step: 1285 – 85 = 1200

n
20 9 900
10  x=
II. Step: Eff. Discount = 10  2 – 10 29
ja 100
R s
= 11.8% 9x 610
=
a th

10 29
88.2
 S.P =  1200 610 10
100 x= 
29 9
ty a

= Rs. 1058.4
= 23.37%
M.P 100  P 34. (a) CP = 1500
di M

30. (a) =
C.P 100 – D
140
MP =  1500 = 2100
100
400 125
= SP = 1500 + 75 = 1575
C.P 80
2100 – 1575
400  80  D% =  100
 C.P = = Rs. 256 2100
125
31. (b) (Question incomplete) 525
= = 25%
21
Total purchase = Rs. 25 × 30
35. (b) I. Coupon:
A

= 750
70
S.P = × (1250 + 1540 + 1375)
10 100
Eff. Discount = 10  1 – = 11 – 0.1
100 II. Coupon:
= 10.9%
20
S.P = × 1540 + 308 + 1250 + 1375
89.1 100
Amount to pay =  750
100 = Rs. 2933
= Rs. 668.25  Coupon I gives minimum price.

Aditya Ranjan (Excise Inspector) Selected Selection 33


https://sscstudy.com/
Join Telegram- Maths by Aditya Ranjan DISCOUNT

36. (a) CP = 4000 41. (d)


M.P = 8400 Given,
Eff. Discount = 25 + 15 – 3.75
D1 = 12%, D2 = 18%, D3 = 25%
= 36.25%
We know,
C.P 100 – D
= D1  D2 12  18
M.P 100  P D1  D2 – = 12  18 –
100 100
4000 63.75
  = 27.84
8400 100  P
Again,
 100 + P  134
 P  34% 27.84  25
37. (b) Eff. Discount = 25 – 1 = 24% 27.84  25 –
100
76 = 45.88%
 Amount paid =  3000
100 42. (a) Dishonest
= Rs. 2280 C.P M.P C.P S.P

r
MP 100  P 100 150 90 120
=

si
38. (c)
CP 100 – D
120 – 90
 Profit% =  100
an by
 M.P =
800  112.5
90  33%
90

n
43. (c)
= Rs. 1000
ja We know,
R s
MP 100  P D1  D2
39. (b) = Eff. Dis = D1 + D2 –
a th

CP 100 – D 100

528  118 10  10
 MP = = 10  10 –
88 100
ty a

= 19%
= Rs. 708
44. (c)
di M

40. (c)
100 – D
S.P =  M.P
100 – D 100
S.P =  M.P
100
330  100
M.P =
87500  100 55
 M.P =
56
= 600
= 156250

......-------......
A

Aditya Ranjan (Excise Inspector) Selected Selection 34


https://sscstudy.com/
Join Telegram- Maths by Aditya Ranjan SIMPLE INTEREST

SIMPLE INTEREST
04

1. Rs.2,500, when invested for 8 years at a given
rate of simple interest per year, amounted to
Rs.3,725 on maturity. What was the rate of
simple interest that was paid per annum?







SSC CGL 01/12/2022 (Shift- 04)

r
(a) Rs.120 (b) Rs.100
SSC CGL 01/12/2022 (Shift- 01)
(c) Rs.132 (d) Rs.125

si
(a) 6% (b) 6.125% 5. A sum of Rs.6,000 is to be paid back in two

2.
(c) 6.25%an by (d) 5.875%
A person lent certain sum of money at the
equal annual instalments; each instalment is
to be paid at the end of every year. How much

n
annual rate of 25 percent on simple interest. is each instalment if the interest is
In 6 years the interest amounted to Rs.360 compounded annually at 2% p.a.? (Rounded
ja
more than the sum lent. What is the sum lent? off up to two decimal places)
R s

a th




ty a

SSC CGL 01/12/2022 (Shift- 02)


(a) Rs.600 (b) Rs.360 SSC CGL 02/12/2022 (Shift- 01)
di M

(c) Rs.720 (d) Rs.540 (a) Rs.2,092.29 (b) Rs.3,090.30


3. The simple interest on a certain sum for 3 (c) Rs.2,291.29 (d) Rs.3,589.30
years at 14% p.a. is Rs.4,200 less than the 6. What annual installment will discharge a debit
simple interest on the same sum for 5 years of Rs.5,664 in 4 years at 12% simple interest?
at the same rate. Find the sum.

SSC CGL 02/12/2022 (Shift- 03)
(a) Rs.1,230 (b) Rs.1,210
(c) Rs.1,200 (d) Rs.1,220
SSC CGL 01/12/2022 (Shift- 03) 7. On simple interest a sum of Rs.640 becomes
A

(a) Rs.16,000 (b) Rs.10,000 Rs.832 in 2 years. What will Rs.860 becomes
(c) Rs.15,000 (d) Rs.12,000 in 4 years at the same rate of simple interest?
4. Damani purchased an item costing Rs.7,500
and paid Rs.3,500 as a down payment for the
same. If the simple interest charged for the
remaining amount is 9% per annum and
SSC CGL 02/12/2022 (Shift- 04)
Damani cleared all dues after 4 months of the
purchase, how much did Damani pay after 4 (a) Rs.1,250 (b) Rs.1,376
months as interest? (c) Rs.1,426 (d) Rs.1,150

Aditya Ranjan (Excise Inspector) Selected Selection 35


https://sscstudy.com/
Join Telegram- Maths by Aditya Ranjan SIMPLE INTEREST

8. Rs.5,000 is divided into two parts such that


if one part is invested at 4% and the other
at 5%, then the whole annual interest from
SSC CGL 05/12/2022 (Shift- 01)
both the sums is Rs.223. How much was
(a) 8 years (b) 14 years
invested at 4%?
(c) 2 years (d) 15 years
13. A certain sum of money becomes triple of
itself in 26 years at simple interest. In how
many years it will becomes five times of itself?



SSC CGL 03/12/2022 (Shift- 01)
SSC CGL 05/12/2022 (Shift- 02)
(a) Rs.2,600 (b) Rs.2,700
(a) 64 years (b) 52 years
(c) Rs.2,400 (d) Rs.2,300 (c) 56 years (d) 60 years
9. Suman paid Rs.9,600 in interest on a loan she 14. A sum of money invested at simple interest

r
obtained 5 years ago with a simple interest 17

si
rate of 16%. What was the amount of the loan becomes of itself in 2 years and 6 months.
10
she had taken?
an by What is the rate of interest per annum?

n
17

10
ja SSC CGL 03/12/2022 (Shift- 02)
R s

(a) Rs.13,250 (b) Rs.12,500 SSC CGL 05/12/2022 (Shift- 03)
a th

(c) Rs.12,000 (d) Rs.11,750 (a) 22 percent (b) 16 percent


10. What would be the compound interest on (c) 28 percent (d) 34 percent
Rs.15,750 at 20% per annum, in two years, 15. A certain sum amounts to Rs.3,640 in 2 years
ty a

if the interest is compounded half yearly? and Rs.4,060 in 8 years at simple interest. Find
the approximate rate percentage per annum?

di M






SSC CGL 03/12/2022 (Shift- 03)
SSC CGL 05/12/2022 (Shift- 04)
(a) Rs.5,213.25 (b) Rs.3,307.5 (a) 4% (b) 2%
(c) Rs.7,305.975 (d) Rs.7,309.575 (c) 1% (d) 3%
11. A sum becomes Rs.15,500 in 7 years on simple 25
interest at the rate of 30 percent per annum. 16. The simple interest received on a sum is
36
What is the total interest for the 7 years?
of the sum. The number of years is equal to
the annual rate of interest. What is the annual
A

rate of interest?

SSC CGL 03/12/2022 (Shift- 04) 25

(a) Rs.12,200 (b) Rs.1,47,000 36
(c) Rs.10,500 (d) Rs.11,500
12. In how much time will a sum of Rs.5250 SSC CGL 06/12/2022 (Shift- 01)
amounts to Rs.9870 at the rate of 11 percent (a) 9.25 percent (b) 10.25 percent
per annum at simple interest? (c) 6.62 percent (d) 8.33 percent

Aditya Ranjan (Excise Inspector) Selected Selection 36


https://sscstudy.com/
Join Telegram- Maths by Aditya Ranjan SIMPLE INTEREST

17. A car with a price of Rs.6,50,000 is bought 21. A sum is deposited in a bank which gives simple
by making some down payment. On the interest. The sum becomes 1.25 times in 3
balance, a simple interest of 10% is charged years. If there is a requirement of Rs.7,60,000
in lump sum and the money is to be paid in after seven years, how much amount (in Rs.)
20 equal annual instalments of Rs.25,000. How should one deposit to fulfil the requirement?
much is the down payment?




SSC CGL 07/12/2022 (Shift- 02)
(a) 5,20,000 (b) 5,70,000
SSC CGL 06/12/2022 (Shift- 02) (c) 4,80,000 (d) 6,00,000
(a) Rs.1,55,945 (b) Rs.1,95,455 22. What annual instalment will discharge a debt of
(c) Rs.1,94,555 (d) Rs.1,45,955 Rs.9,600 due in 5 years at 10% simple interest?

r
18. A sum of money amounts to Rs.767 in 3 years,

si
and to Rs.806 in 4 years on simple interest
at 6% annum. What is the sum? SSC CGL 07/12/2022 (Shift- 03)
an by


(a) Rs.1450
(c) Rs.1500
(b) Rs.1450
(d) Rs.1600

n
23. Find the simple interest on Rs.2,700 for 8
SSC CGL 06/12/2022 (Shift- 03) months at 5 paisa per rupee per month?
ja
R s
(a) Rs.600 (b) Rs.560
(c) Rs.675 (d) Rs.650
a th

19. A person lent Rs. 23000 to B for 3 years and SSC CGL 07/12/2022 (Shift- 04)
Rs. 19000 to C for 4 years on simple interest (a) Rs.950 (b) Rs.720
at the same rate of interest and received Rs. (c) Rs.540 (d) Rs.1,080
ty a

3625 in all from both of them as interest. What 24. In a certain duration of time, a sum become 2 times
is the annual rate of interest? itself at the rate of 5% per annum simple interest.
di M

B C What will be the rate of interest if the same sum


becomes 5 times itself in the same duration?


SSC CGL 06/12/2022 (Shift- 04)
(a) 1.5 percent (b) 3 percent
(c) 2.5 percent (d) 4 percent SSC CGL 08/12/2022 (Shift- 01)
20. A person lent certain sum of money at the (a) 20 Percent (b) 16 Percent
annual rate of 7 percent on simple interest (c) 10 Percent (d) 18 Percent
and the interest received in 11 years is Rs. 25. Anuradha invests her money in a firm where
920 less than the sum lent. What is the sum
A

the principal amount becomes 3 times in 10


lent? years. What is the yearly rate of simple
interest offered by the firm?



SSC CGL 07/12/2022 (Shift- 01) SSC CGL 08/12/2022 (Shift- 02)
(a) Rs.41200 (b) Rs.4000 (a) 25% (b) 20%
(c) Rs.52000 (d) Rs.2400 (c) 22% (d) 18%

Aditya Ranjan (Excise Inspector) Selected Selection 37


https://sscstudy.com/
Join Telegram- Maths by Aditya Ranjan SIMPLE INTEREST

26. A certain amount will become six times in


20 years. How long does it take for the same
amount to become 5 times? Assume the same
rate of simple interest in each case.

SSC CGL 09/12/2022 (Shift- 03)
(a) Rs.2,200 (b) Rs.2,000
(c) Rs.3,000 (d) Rs.2,500
31. In how many least number of complete years
a sum of money become more than four times
SSC CGL 08/12/2022 (Shift- 03)
of itself at the rate of 50 percent per annum
(a) 16 years 8 months on simple interest?
(b) 15 years 8 months
(c) 15 years
(d) 16 years
SSC CGL 09/12/2022 (Shift- 04)
27. A certain sum of money is given at a certain

r
rate for 3 years. Had it been given at 5% (a) 9 years (b) 7 years
higher rate, it would have fetched Rs.600 (c) 6 years (d) 5 years

si
more. Find the sum. 32. A person borrows Rs.1,00,000 from a bank at
10% per annum simple interest and clears the
an by


debt in five years. If the instalment paid at
the end of the first, second, third and fourth

n
years to clear the debt are Rs.10,000,
Rs.20,000, Rs.30,000 and Rs.40,000,
ja SSC CGL 08/12/2022 (Shift- 04)
respectively, what amount should be paid at
R s
(a) Rs.8,000 (b) Rs.5,000 the end of the fifth year to clear the debt?
a th

(c) Rs.6,000 (d) Rs.4,000


28. In how much time will a sum of Rs. 10200
amounts to Rs. 19125 at the rate of 12.5
ty a

percent per annum at simple interest?



di M



SSC CGL 12/12/2022 (Shift- 01)
SSC CGL 09/12/2022 (Shift- 01)
(a) Rs.38,250 (b) Rs.39,490
(a) 6 years (b) 5 years (c) Rs.40,450 (d) Rs.36,450
(c) 8 years (d) 7 years 33. What is the rate of interest per annum for
simple interest at which Rs.880 amount to
29. What is the present value of Rs.10,000
received in 2 years, if the interest rate is 12% 1
Rs.913 in 1 years?
per year discounted semi-annually? 2

1
1
SSC CGL 09/12/2022 (Shift- 02) 2
SSC CGL 12/12/2022 (Shift- 02)
(a) Rs.7,020.94 (b) Rs.7,920.90
(c) Rs.7,920.94 (d) Rs.7,900.94 2 1
(a) 2 % (b) 2 %
3 4
30. Monty paid a simple interest of Rs.480 on a
particular sum after 2 years. The rate was 8% 1 1
(c) 2 % (d) 2 %
per annum. Find the sum. 2 3

Aditya Ranjan (Excise Inspector) Selected Selection 38


https://sscstudy.com/
Join Telegram- Maths by Aditya Ranjan SIMPLE INTEREST

34. A person deposited Rs.15,600 in a fixed


deposit at 10% per annum simple interest.
After every second year he adds his interest
earned to the principal. The interest at the
end of 4 years is: SSC CGL 13/12/2022 (Shift- 03)
10 9
(a) 730 (b) 840
63 61

11 10
(c) 640 (d) 250
63 63
SSC CGL 12/12/2022 (Shift- 03)
39. A certain sum of money lent at simple interest
(a) Rs.6,655 (b) Rs.6,864 amounts to Rs.1,200 in 2 years and Rs.1,600
(c) Rs.3,975 (d) Rs.3,744 in 4 years. The rate per cent per annum is:
35. A sum of money doubles itself in 7 years at

simple interest. In how much time will it
becomes 5 times of itself?

r

SSC CGL 13/12/2022 (Shift- 04)

si
SSC CGL 12/12/2022 (Shift- 04) (a) 20% (b) 30%
an by
(a) 25 years (b) 28 years (c) 25% (d) 16%
40. The difference of simple interest from two
(c) 23 years (d) 21 years
banks on Rs. 8,000 in 3 years is Rs. 800. If

n
36. The monthly income of Mr. Roy is Rs.18,000.
the rate of interest per annum in two banks
He took a loan of Rs.30,000 on simple interest
are R1 and R2, then what is the value of R1
ja
for 3 years at the rate of 5% per annum. The
- R2? (Where R1 > R2)
R s
amount that he will be paying as simple
interest in 3 years is what percent of his Rs.
a th

monthly salary? Rs.


R1 R2 R1 – R2
R1 > R2
ty a

SSC CGL TIER - II 02/03/2023


1 1
di M

(a) 5 % (b) 3 %
3 3
SSC CGL 13/12/2022 (Shift- 01)
1 1
(a) 30% (b) 35% (c) 1 % (d) 2 %
3 3
(c) 20% (d) 25%
37. A sum of Rs.10 is lent by a child to his friend to 41. What is the ratio of the simple interest earned
be returned in 11 monthly instalments of Rs.1 each, on a certain amount at the rate of 21% per
the interest being simple. The rate of interest is: annum for 8 years to that earned on the same
sum at the same rate for 21 years?


A

SSC CGL 13/12/2022 (Shift- 02)


9 9 SSC CGL TIER- II 03/03/2023
(a) 11 % (b) 21 %
11 11 (a) 8 : 21 (b) 21 : 5
2 1 (c) 5 : 21 (d) 21 : 8
(c) 10 % (d) 9 %
11 11 42. The simple interest on a sum of Rs X in 5
38. A person borrowed Rs.2,000 at 5% annual 2th
simple interest repayable in 3 equal annual years is of the principal. What is the
5
installments. What will be the annual
installment? annual rate of interest?

Aditya Ranjan (Excise Inspector) Selected Selection 39


https://sscstudy.com/
Join Telegram- Maths by Aditya Ranjan SIMPLE INTEREST

Rs. X (a) Rs. 10,123.20 (b) Rs. 9,824.00


2th (c) Rs. 10,520.00 (d) Rs. 9,956.86
5

46. If a sum of Rs. 6,500 is to be borrowed for 2
SSC CGL TIER- II 06/03/2023 years at 10% per annum compounded half
(a) 5% (b) 12% yearly, find the compound amount (integer
(c) 8% (d) 10% values only).
43. A sum triple itself in 9 years at simple interest.
Find the rate of interest per annum is.

SSC CGL TIER- II 06/03/2023

SSC CGL TIER- II 07/03/2023 (a) Rs. 7,900 (b) Rs. 8,250
(c) Rs. 7,650 (d) Rs. 8,150
2 1
(a) 38 % (b) 33 %
47. Rajnish borrowed Rs. 1,500 from a bank and
9 3
repaid the entire amount with interest in two
1 2

r
(c) 27 % (d) 22 % equal annual instalments, the first instalment
9 9 being paid a year after Rajnish borrowed from

si
44. If interest be compounded half-yearly, then the bank. If the rate of interest was 40% per
find the compound interest on Rs. 8,000 at annum, compounded annually, then what was
an by
the rate of 20% per annum for 1 year.
Rs.
the value (in Rs.) of each instalment paid by
Rajnish?

n
%

ja
R s
SSC CGL TIER - II 02/03/2023

a th

(a) RS. 1675 (b) Rs. 1690



(c) Rs. 1685 (d) Rs. 1680
45. What is the compound interest on a sum of ¹
SSC CGL TIER- II 07/03/2023
ty a

25,000 after three years at a rate of 12 per


cent per annum interest compounded yearly? (a) 1125
(b) 1470
di M


(c) 1225
(d) 1350
SSC CGL TIER- II 03/03/2023

ANSWER KEY
1.(b) 2.(c) 3.(c) 4.(a) 5.(b) 6.(c) 7.(b) 8.(b) 9.(c) 10.(d)
A

11.(c) 12.(a) 13.(b) 14.(c) 15.(b) 16.(d) 17.(b) 18.(d) 19.(c) 20.(b)

21.(c) 22.(d) 23.(d) 24.(a) 25.(b) 26.(d) 27.(d) 28.(d) 29.(c) 30.(c)

31.(b) 32.(b) 33.(c) 34.(b) 35.(b) 36.(d) 37.(b) 38.(a) 39.(c) 40.(b)

41.(a) 42.(c) 43.(d) 44.(d) 45.(a) 46.(a) 47.(c)

Aditya Ranjan (Excise Inspector) Selected Selection 40


https://sscstudy.com/
Join Telegram- Maths by Aditya Ranjan SIMPLE INTEREST

SOL U T ION S
1. (b) Interest for 8 yrs. = 3725 – 2500 6. (c) Amount = 5664
= 1225 T = 4 yrs.
R = 12%
1225
Interest for 1 yr. =
8 100  A
Installment =
R t (t– 1)
1225  100 100 t 
 Rate = 2
8  2500
100  5664
49 =
= = 6.125% 3
8 400  48 
2

r
2. (c) R = 25%
T = 6 yrs. 100  5664

si
=
I = P + 360 472
an by
P  360 =
P  25  6
100 7. (b)
= 1200
Interest for yrs. = 832 – 640

n
= 192
3 P = 2P + 720
Interest for 1 yr. = 96
P = 720 ja
R s
3. (c) T1 = 3 yrs. , T2 = 5 yrs. 96
Rate =  100 = 15%
R = 14% 640
a th

ATQ,
Now,
P  14  2
 4200 860 96
100 = 860    4  100
ty a

100 640
P = 15000
= 860 + 516  1376
di M

4. (a) Amount on interest = 7500 – 3500


= 4000 223
8. (b) Whole interest rate =  100
ATQ, 5000
9 4 = 4.46%
4000   = 120
100 12 By Allegation:
1 4 5
5. (b) 2% =
50
Principle Amount
5050 5151 4.46
2500 2601
A

 Principle Amount
2500 2601
2500 2601 0.54 0.46
 
5050 5202 27 : 23
5050  6000
27
6000  at 4%   5000
2601   2601 50
5050
 2700
 3090.30

Aditya Ranjan (Excise Inspector) Selected Selection 41


https://sscstudy.com/
Join Telegram- Maths by Aditya Ranjan SIMPLE INTEREST

4 A 17 5
9. (c) 16% = 14. (c) = in yrs.
25 P 10 2

4 I 7 5
P  5 = 9600  = in yrs.
25 P 10 2

4 7 2
 P  9600 Rate =   100
5 10 5
 P = 12000  28%
10. (d) R = 20% , R1 = 10%
15. (b) 6 yrs. interest = 4060 – 3640
Time = 2 yrs. , T1 = 4 yrs.
= 420
 Effective rate = 46.41%
2 yr. = 140
46.41 Principle = 3640 – 140
 × 15750 = 7309.575
100

r
= 3500

si
30 21
11. (c) R  7  (for 7 yrs .) 70  100
100 10 Rate =
an by 3500
 31  15500
= 2%
1  5000

n
 Interest = 15500 – 5000 I 25
16. (d) 
ja = 10500 P 36
R s
12. (a) Interest = 9870 – 5250 Time = Rate
a th

= 4620
Time  Rate I
 
4620  100 100 P
Time =
5250  11
ty a

(Rate)² 25
= 8 yrs.  
100 36
di M

I 2P 2
13. (b) In 26 yrs. = = Rate 5
P P 1  
10 6
Now,
5 time Principle = Amount 25
 Rate =  8.33%
3
I 4
 =
P 1 17. (b) Total Amount paid in installments

 26 yrs.  2 times Interest. = 20 × 250000

 for times interes  26 × 2 = 50,00,000


Also, Rate = 10%
A

= 52 yrs.
'OR'  Amount on which interest is liveied

Interest1 Interest2 500000


= =  10
Time1 Time2 11
= 4,54,545.455
2 4
 =  Down payment = 6,50,000 – 454545.455
26 x
 1,95,455
 x = 52 yrs.

Aditya Ranjan (Excise Inspector) Selected Selection 42


https://sscstudy.com/
Join Telegram- Maths by Aditya Ranjan SIMPLE INTEREST

18. (d) Interest of 1 yrs. = 806 – 767 23. (d) P = 2700


= 39 8 2
T= = yrs.
Interest of 3 yrs. = 39 × 3 12 3
= 117 5
R=  12 Per annum
P = 767 – 117 100
P = 650 2 5
S.I = 2700    12
19. (c) 3 100
ATQ, = Rs. 1080
23000 × 3 × R + 19000 × 4 × R = 3625 Rate1 Rate2
24. (a) = .........(1)
(69000 + 76000) R = 3625 Interest1 Interest2

3625 5 A 2 I 1
R= = = 2.5% =  
145000 23 P 1 P 1

r
A 5 I 4
P  7  11 =  

si
20. (b) = P – 920 P 1 P 1
100
an by From..(1)
 77 P = 100 P – 92,000
 23 P = 92, 000 5 Rate2
=

n

1 4
P = 4,000
ja  Rate2 = 5 × 4 = 20%
R s
Amount 1.25 5
21. (c) = = 25. (b) A = 3P
Principle 1 4
a th

 I = 2P
I 1 T = 10 yrs.
 =
P 4
2P 100
ty a

R= 
P 10
1
3 yrs. 
di M

4 = 20%
7
7 yrs.  I 5
12 26. (d) 
P 1
After 7 yrs.
I1 4

Amount 19 76, 0000 P 1
 = =
Principle 12 x
I I1
=
 x = 4,80,000 Time1 Time2

100  A 5 4
22. (d) Installment =  =
A

(t– 1) 20 Time2
100 t  R t
2
 Time2 = 16 yrs.
100  9600 27. (d) R%, Time = 3 yrs.
=
500  50  2 R1 = R + 5%

100  9600  For 3 yrs. 15% more


=
600  15  600
= 1600  100  4000

Aditya Ranjan (Excise Inspector) Selected Selection 43


https://sscstudy.com/
Join Telegram- Maths by Aditya Ranjan SIMPLE INTEREST

33. (c) Interest = 913 – 880


1
28. (d) 12.5% =
8 = 33

I = 19125 – 10200 33  100  2 5


Rate = = = 2.5%
= 8925 3  880 2

89258 10
Time = 34. (b) Interest for first 2 yrs. = 2  × 15600
10200  1 100
= 1560 × 2 = 3120
= 7 yrs.
Principle for 3rd yr. = 15600 + 3120 = 18720
Amount
29. (c) Present value = time
Principle for next 2 yrs. = 18720
 R 
1 
 

 100  18720
 Interest = 2× × 10 = 1872 × 2 = 3744
100
10, 000  Total interest after 4 yrs. = 3120 + 3744

r
= 4
 6 
1 
 
 = 6864

si
 100 
an by 35. (b) I 1 I1 4
7 yrs.   , =
10000  (50)4 P 1 P 1
=
(53)4

n
Time1 Time2
= 7920.94 =
I1 I2
30. (c) I = 480
ja
R s
R = 8% 7 Time2
a th

 =
1 4
T = 2 yrs.
 Time2 = 28 yrs.
480  100
P=
ty a

8 2 5
36. (d) S.I in 3 yrs. = 3  × 30,000 = 4500
100
= 3000
di M

1 4500
31. (b) Rate = 50% = Required%   100 = 25%
2 18000

37. (b) Let Rate = R%


1
P  1 year
2  Total effective payment

1  1 R 10   1 R  9   1  R 1 
6 P = 3P  6 years =
1  1 
 
   ...  
 1  1

2  100 12   100 12   12 100 

more than 3P  more than 6 years.


 10R   9R   R 
= 7 years  least = 1 +  + 1 +  + .... + 1 +  +1
 1200   1200   1200 
A

32. (b) Amount to be paid Amount paid Remaining


R(1  2  ....  10)
end of I. yr.  110000 10,000 10,0000 = 11 
1200
end of II. yr. 110000 20,000 90,000
 10 11 
end of III. yr. 99000 30,000 69,000 = 11  R   
 
1200 2
end of IV. yr. 75900 40,000 35,900

end of V. yr.  39490 11R


= 11  .....(1)
 Vth installment to year dues = 39490 240

Aditya Ranjan (Excise Inspector) Selected Selection 44


https://sscstudy.com/
Join Telegram- Maths by Aditya Ranjan SIMPLE INTEREST

Amount to be paid (if at the end of 11 months) 43. (d)


Given, Amount = 3 × Principle
 10 11  11R  S.I = 3P – P = 2P
= 10   R
    10  .....(2)

100 
12   120 A.T.Q,

P R9
From (1) and (2) = 2P
100
11R 11R 200 2
10   11   R= = 22 %
120 240 9 9
44. (d)
11R 240 9
 =1  R = = 21 % P = 8000
240 11 11
R = 20%, R1 = 10%
38. (a) 100  A Time = 1 yr. = 2 half yrs.
Installment 
Rt (t – 1)
100t  Eff. rate = 21%
2

r
2000  5  3 21
Amount = 2000 +  C.I =  8000 = 1680

si
100 100
= 2000 + 300 = 2300 45. (a)


an by
Installment =
2300  100
P = 25,000, t = 3 yrs. R = 12%
I yr. 3000

n
300  15
II yr. 3000 360
230000 10
ja = = 730 III yr. 3000 360 360 43.2
R s
315 63
 Total C.I = 10123.20
39. (c) Interest of 2 yrs. = 1600 – 1200
a th

'OR'
= 400
 Principle = 1200 – 400 Effective rate of 12% for 3 yrs. = 40.4928
= 800 40.4928
 C.I =  25000 = 10123.20
ty a

200 100
R=  100  25%
800 46. (a)
di M

40. (b) P = 6500., t = 2 yrs. = 4 half yrs.


r = 10%, r1 = 5%
PR T
S.I = Effective rate = 21.550625%
100
A.T.Q, 21.550625
 C.I =  6500  1400
100
8000  3
[R1 – R 2 ] = 800  Amount = 6500 + 1400 = 7900
100
47. (c)
10 1
 R1 – R 2 = =3 % 2
3 3 P = 1500, rate = 40% =
5
A

41. (a)
P A
P  21  8 8 5×7 7 × 7 = 49
A.T.Q, =
P  21  21 21
25
42. (c) 49
60
Let, Rate = R%
60  1500
X  5 R 2
 = X 1500
100 5 49   49 = 1225
60
 R = 8%

Aditya Ranjan (Excise Inspector) Selected Selection 45


https://sscstudy.com/
Join Telegram- Maths by Aditya Ranjan RATIO

RATIO
05

1. The ratio of expenditure to savings of a woman
is 5:1. If her income and expenditure are
increased by 10% and 20%, respectively, then
find the percentage change in her savings.


SSC CGL 02/12/2022 (Shift- 04)
(a) 212 : 117 : 47
(b) 37 : 47 : 83

r
SSC CGL 01/12/2022 (Shift- 03) (c) 176 : 105 : 189
(a) 55% (b) 60% (d) 189 : 115 : 117

si
(c) 50% (d) 40% 1
2. an by
By mistake, instead of dividing Rs.702 among 5. The tax on the salary of C is of the salary
4
Ram, Ramesh, and Naresh in the ratio
1

n
1 1 1 and savings are of the salary. the ratio of
: : , it was divided in the ratio of 3 : 4 3
3 4 6 the expenditures of the savings is.
ja
: 6. Who gained the most and by how much?
R s
1
C
4
a th

1 1 1 1
: :
3 4 6
3
SSC CGL 03/12/2022 (Shift- 04)
ty a

(a) 4 : 5 (b) 4 : 3
SSC CGL 02/12/2022 (Shift- 01) (c) 5 : 4 (d) 3 : 4
di M

(a) Ram Rs.158 6. Two numbers are in the ratio of 5 : 7. If 6 be


(b) Ramesh Rs.158 added to each, the ratio becomes 3 : 4. Find
the numbers.
(c) Naresh Rs.168
(d) Ram Rs.168
3. A, B, C and D share a property worth
Rs.93,100. If A : B = 1 : 2, B : C = 3 : 4 and
C : D = 5 : 6, then find the share of C. SSC CGL 05/12/2022 (Shift- 04)
A, B, C D 93,100 (a) 30, 42 (b) 55, 88
(c) 10, 16 (d) 25, 40
A : B = 1 : 2, B : C = 3 : 4 C : D
= 5 : 6 C 1 1 1
: : .
A

7. The sides of a triangle are in the ratio


SSC CGL 02/12/2022 (Shift- 02) 3 5 6
(a) Rs.34,000 (b) Rs.28,000 If the perimeter is 147 cm, then the length
of the smallest side is.
(c) Rs.25,000 (d) Rs.31,000
4. The number of books on Mathematics, Physics 1 1 1
: :
and Chemistry in a University library is in 3 5 6
the ratio 8 : 5 : 9. There is a proposal to
increase these books by 10%, 5% and 5% SSC CGL 06/12/2022 (Shift- 03)
respectively. What will be the ratio of the (a) 12 cm (b) 25 cm
number of books after increment?
(c) 30 cm (d) 35 cm

Aditya Ranjan (Excise Inspector) Selected Selection 46


https://sscstudy.com/
Join Telegram- Maths by Aditya Ranjan RATIO

8. Rohan scored twice as many marks in English


as he did in Science. His total marks in Rs.
English, Science and Mathematics are 126. If
the ratio of his marks in English and Rs.
Mathematics is 2 : 3, his marks in English are:
SSC CGL TIER - II 02/03/2023
(a) 21120 (b) 20120
(c) 21150 (d) 18150
12. The ratio of marks obtained by Rajesh, Rakesh
and Ramesh in an exam is 2 : 4 : 9.

SSC CGL 06/12/2022 (Shift- 04) What are the marks obtained by Rakesh and
Ramesh, if Rajesh scored 30 marks in the
(a) 63 (b) 20 exam?
(c) 21 (d) 42

r
9. Wheat worth Rs.80 per kg and Rs.50 per kg
is mixed with a third variety in the ratio 1 :

si

2 : 3 . If the mixture is worth Rs.75 per kg,
an by SSC CGL TIER- II 03/03/2023
then the price of the third variety per kg will
be equal to: (a) Rakesh = 40, Ramesh = 90

n
(b) Rakesh = 60, Ramesh = 135
kg kg
(c) Rakesh = 120, Ramesh = 180
ja

R s
(d) Rakesh = 90, Ramesh = 40
kg
13. Rewa has some hens and some goats. If the
a th

kg total number of animal heads is 100 and the


SSC CGL 07/12/2022 (Shift- 01) total number of animal feet is 348, then what
is the total number of goats with Rewa?
(a) Rs.95
ty a

(b) Rs.85

(c) Rs.80 (d) Rs.90

di M

10. Salaries of Rida and Riya are in the ratio of


3 : 5, respectively. If the salary of each one
is increased by Rs.5,000, then the new ratio SSC CGL TIER- II 03/03/2023

becomes 5 : 7. What is Riya’s present salary? (a) 76 (b) 74


(c) 55 (d) 80

14. Six years ago, the ratio of the ages of A and B was

7 : 5. 4 years from now, the ratio of their ages
will be 11 : 9. What is the age of A at present?
A B
SSC CGL 13/12/2022 (Shift- 01)
A

A
(a) Rs.12,500 (b) Rs.7,500
SSC CGL TIER- II 06/03/2023
(c) Rs.2,500 (d) Rs.10,000
11. The ratio of the incomes of two employees 1 1
(a) 22 yrs. (b) 23 yrs.
is 7 : 4, and the ratio of their expenditures 2 2
is 3 : 1. If each of them manages to save Rs.
1 1
4,800 per month, find the sum of their (c) 24 yrs. (d) 21 yrs.
2 2
monthly incomes (in Rs).

Aditya Ranjan (Excise Inspector) Selected Selection 47


https://sscstudy.com/
Join Telegram- Maths by Aditya Ranjan RATIO

15. The numbers of coins of 25 paise, 50 paise, 16. If two LED TVs and one mobile phone cost
Rs. 2 and Rs. 5 are in the ratio 5 : 4 : 3 : 1 Rs. 31,000, while two mobile phones and one
respectively. If the total amount of coins is Rs LED TV cost Rs. 35,000, then the value of one
285, then what will be the difference between mobile phone is:
the number of 25 paise and Rs. 5 coins?



SSC CGL TIER- II 07/03/2023
SSC CGL TIER- II 06/03/2023 (a) Rs. 13,500 (b) Rs. 12,500
(a) 60 (b) 30 (c) Rs. 13,000 (d) Rs. 12,000
(c) 80 (d) 40

ANSWER KEY

r
si
1.(d) 2.(c) 3.(b) 4.(c) 5.(c) 6.(a) 7.(d) 8.(d) 9.(d) 10.(a)

11.(a)
an by
12.(b) 13.(b) 14.(b) 15.(c) 16.(c)

n
ja
R s
a th
ty a
di M
A

Aditya Ranjan (Excise Inspector) Selected Selection 48


https://sscstudy.com/
Join Telegram- Maths by Aditya Ranjan RATIO

SOL U T ION S
1. (d) 5. (c)
Let the expenditure and savings be 500 and 100 Let, Salary = 300
Income Expenditure Savings Tax = 75
Old 600 500 100 Savings = 100
New 600 600 60 Expenditure = 300 – 75 – 100 = 125

100 – 60 Expenditure 125 5


Required % = ×100 = 40%  = =
100 Savings 100 4

2. (c) 6. (a)

r
Let nos. be 5x and 7x
Ram Ramesh Naresh

si
A.T.Q.
Mistaken 3 4 6
an by
1 1 1 5x + 6 3
Actual = .
3 4 6 7x + 6 4

n
 4 3 2 20x + 24 = 21x + 18
6=x
ja
Clearly, Naresh gained the most his Actual
R s
 nos. are = 30, 42
2
portion = × 702 = 156. 7. (d)
a th

9
a : b : c
6 1 1 1
Mistaken portion = × 702 = 324 : :
13 3 5 6
ty a

 324 – 156 = 168 10 : 6 : 5


NOTE: Since Naresh is in just one option, we
di M

A.T.Q.
need not find the value.
 10x + 6x + 5x = 147
3. (b)
21x = 147
A B C D
x=7
1 2
 Smallest side = 5x = 35
3 4 D
5 6 8. (d)
15 30 40 48 E = 2S
E + S + M = 126
40
 C's share = × 93100
133 E 2
=
A

= 28000 M 3
4. (c)
3
 M= E
Maths Physics Chemisty 2
8 5 9
E 3
Original 800 500 900  E+ + E = 126
2 2
New 880 525 945
 6E = 252
176 105 189
E = 42

Aditya Ranjan (Excise Inspector) Selected Selection 49


https://sscstudy.com/
Join Telegram- Maths by Aditya Ranjan RATIO

9. (d) 13. (b)


Let, hens = x
I II III goats = y
A.T.Q,
Quantity 1 : 2 : 3
x + y = 100 .........(1)
Rate 80 50 x 2x + 4y = 348 .........(2)
Solving (1) and (2)
 (80 × 1) + (50 × 2) + (3 × x) = 75 × 6 y = 74
14. (b)
 180 + 3x = 450
A B
 3x = 270

x = 90 – 6 yrs. 7 5
+ 10 +4
10. (a) + 4 yrs. 11 9

Rida Riya 4 10
Salary 3 5

r
5 55
+5000 +5000 11   11 =
2 2

si
Now 5 7
55 1
an by  Present age of A = – 4 = 23 yrs.
3x + 5000 5 2 2
 =
5x + 5000 7 15. (c)

n
21x + 35000 = 25x + 25000
1 1
10,000 = 4x Amt. 2 5
ja 4 2
R s
2500 = x Ratio 5x 4x 3x 1x
5x = 12500 5x
a th

T.Amt. 2x 6x 5x
11. (a) 4

A B A B A.T.Q,
ty a

Income 7 4 ×2 = 14 8 5 4 
Exp.    6  5
  x = 285
3 1 ×3 = 9 3  4 2 
di M

Saving 4800 4800 5 5


57
x = 285
5  48000 4
4800 x = 20
14  8 = 22   22  21120  5x – x = 4x = 4 × 20 = 80
5
16. (c)
12. (b) Let, LED TV = x
Rajesh Rakesh Rajesh Phone = y
2 4 9 A.T.Q,
2x + y = 31000 ..........(1)
2  30
x + 2y = 35000 ..........(2)
Rakesh 4  60 Solving (1) and (2)
A

Ramesh 9  135 y = 13000

......-------......

Aditya Ranjan (Excise Inspector) Selected Selection 50


https://sscstudy.com/
Join Telegram- Maths by Aditya Ranjan PROPORTION

PROPORTION
06
1. The mean proportion of 169 and 144 is: (6 + 8) (3 – 2)
169 144 SSC CGL 03/12/2022 (Shift- 02)
SSC CGL 01/12/2022 (Shift- 01) (a) 212 (b) 14
(a) 156 (b) 147 (c) (6 – 8) (d) 15 – 7
(c) 126 (d) 165 7. Which smallest positive number should be
2. What is the ratio of the mean proportional subtracted from each of 9 and 13 so that 18
between 1.6 and 3.6 and the third proportional is the third proportion to them?
of 5 and 8? 9 13

r
1.6 3.6 5 8
SSC CGL 03/12/2022 (Shift- 03)

si
SSC CGL 01/12/2022 (Shift- 02) (a) 2 (b) 4
an by
(a) 2 : 15
(c) 3 : 16
(b) 5 : 16
(d) 4 : 15 8.
(c) 3 (d) 1
A varies directly as the positive square root

n
3. What is the ratio of the fourth proportional of B, and inversely as the cube of C. If A =
of 2, 5, 6 and the fourth proportional of 6, 15, when B = 27 and C = 2, then find B when
8, 9? ja A = 9 and C = 2.
R s
2, 5, 6 6, 8, 9 A, B C
B = 27 C = 2
a th

SSC CGL 02/12/2022 (Shift- 03) A = 15 A = 9 C = 2 B


(a) 3 : 2 (b) 5 : 3
(c) 3 : 4 (d) 5 : 4
ty a

SSC CGL 05/12/2022 (Shift- 02)


4. If a : b = c : d, then which of the following
ratio is equal to a : c? 281 243
di M

(a) (b)
a : b = c : d 42 25
a : c 264 275
SSC CGL 03/12/2022 (Shift- 01) (c) (d)
37 51
(a) a : d (b) b : c 9. The fourth porportion to 12 , 24 and 27 is
(c) a + d : b + c (d) a + b : c + d the same as the third proportion to A and 36.
5. The cost of a piece of diamond varies with What is the value of A?
the square of its weight. A diamond of 12, 24 27 A 36
Rs.6,084 value is cut into 3 pieces whose
A
weights are in the ratio 3 : 2 : 1. Find the
loss involved in the cutting. SSC CGL 05/12/2022 (Shift- 03)
(a) 22 (b) 24
A


(c) 26 (d) 20

10. The mean proportional of a and b is c. What
is the mean proportion of a² c and b² c?
a b c a² c and b²
SSC CGL 03/12/2022 (Shift- 02) c
(a) Rs.3,768 (b) Rs.3,718 SSC CGL 06/12/2022 (Shift- 01)
(c) Rs.3,168 (d) Rs.3,518
(a) c (b) 3c
6. Find the mean proportion between (6 + 8)
(c) c³ (d) c²
and (3 – 2).

Aditya Ranjan (Excise Inspector) Selected Selection 51


https://sscstudy.com/
Join Telegram- Maths by Aditya Ranjan PROPORTION

11. If x is the mean proportional between 12.8 15. Find the fourth proportional to the numbers
and 64.8, then the value of x is: 24, 192 and 27.
x, 12.8 64.8 x 24, 192 27
SSC CGL 07/12/2022 (Shift- 03)
SSC CGL 06/12/2022 (Shift- 02) (a) 216 (b) 108
(a) 32 (b) 34 (c) 162 (d) 170.67
(c) 28.8 (d) 26.4 16. Find the mean proportion between 27 and 300.
27 300
a3 + b3
12. Find the mean proportion of and SSC CGL 07/12/2022 (Shift- 04)
a–b
(a) 90 (b) 45
a 2 – b2 (c) 135 (d) 180
.
a 2 – ab + b2 17. The fourth proportion to 16 and 26 and 32
is:
a3 + b3 a 2 – b2 16, 26 32

a–b a 2 – ab + b2 SSC CGL 08/12/2022 (Shift- 02)

r
SSC CGL 06/12/2022 (Shift- 03) (a) 16 (b) 52
(a) 1 (b) a + b

si
(c) 54 (d) 24
a+b 18. Find the fourth proportional to the number

13.
(c) an by
a–b
(d) a+b

The mean proportionl of 6 and 54 is more than


17, 272 and 19.
17, 272 19

n
15. SSC CGL 08/12/2022 (Shift- 03)
6 54 15 (a) 332 (b) 323
ja (c) 403 (d) 304
R s
SSC CGL 07/12/2022 (Shift- 01)
19. If a : b = 4 : 5 and b : c = 3 : 10, then a : c
(a) 3 (b) 6
a th

is:
(c) 4 (d) 5
14. If 14 : 30 :: 7 : x, then what is th value of x?
a : b = 4 : 5 b : c = 3 : 10, a : c
SSC CGL 08/12/2022 (Shift- 04)
14 : 30 :: 7 : x x
ty a

(a) 4 : 10 (b) 5 : 3
SSC CGL 07/12/2022 (Shift- 02)
(c) 25 : 6 (d) 6 : 25
(a) 15 (b) 21
di M

(c) 09 (d) 12

ANSWER KEY
1.(a) 2.(c) 3.(d) 4.(d) 5.(b) 6.(b) 7.(d) 8.(b) 9.(b) 10.(c)

11.(c) 12.(b) 13.(a) 14.(a) 15.(a) 16.(a) 17.(b) 18.(d) 19.(d)


A

Aditya Ranjan (Excise Inspector) Selected Selection 52


https://sscstudy.com/
Join Telegram- Maths by Aditya Ranjan PROPORTION

SOL U TION S
1. (a) Add (1) Both sides in (1)
If b is the mean proportion of a and c, then,
a c
1= 1
a b b d
=
b c
ab cd
or b² = ac =
b d
 b² = 169 × 144
ab b
b = 169  144  = .........(3)
cd d
= 13 × 12
From (2) and (3)

r
= 156
2. (c) Let, mean proportion = b a ab

si
=
c cd
4²  6²
an by
 b² = 1.6 × 3.6 =
(10)² 5. (b) cost k (weight)²
 C = kw²

n
4 × 6 24 6084 = k (6)²
b= =
10 10
ja 6084
R s
b = 2.4 k= = 169 ..........(1)
36
Let, third proportion be c
a th

 8² = 5 × c New cost:- C = C1 + C2 + C3
 kw12 + kw22 + kw32
64
c=  k (9 + 4 + 1)
5
ty a

 169 × 14 = 2366
02.4  Loss = 6084 – 2366
 Required ratio = 5
di M

64 = 3718 Rs.
3 6. (b) b² = 6+ 8 3 – 2
=
16

3. (d) =
2 6
=

= 2 3 2 3 – 2
5 a = 2 × 7 = 14
 a = 15
b = 14
Also,
7. (d) Let no. be x
6 9
= = 9–x 13 – x
8 a  
A

13 – x 18
 a = 12
From option:- (d)
15 5
Required ratio = =
12 4 9 – 1 13 – 1
=
13 – 1 18
a c
4. (d) = ........(1)
b d 8 12
=
12 18
a b
 = ..........(2) 1=1
c d

Aditya Ranjan (Excise Inspector) Selected Selection 53


https://sscstudy.com/
Join Telegram- Maths by Aditya Ranjan PROPORTION

11. (c) x² = 12.8 × 64.8


3
8. (b) A k
C³ 128  648
x² =
100
3
 A =k 256  324
C³ x² =
100
 x = 28.8
27
 15 = k
2 a³  b³ a² – b²
12. (b) c² = 
a–b a² – ab  b²
15  8
 k= (a ² – ab  b ²) (a– b) (a + b)
27  (a + b) 
(a– b) (a² – ab  b ²)
Now,
c² = (a + b)²
k 13 c = (a + b)

r
9=
2 13. (a) b² = 6 × 54

si
b = 18
9  2 27
an by  18 – 15 = 3
 B=
15  2
(Question incomplete)

n
243 14 7
 B= 14. (a) =
ja 25 30 x
R s
 x = 15
a th

243
 B=
25 24 27
15. (a) =
192 d
12 27
ty a

9. (b) =  d = 27 × 8
24 a
= 216
di M

A 36 16. (a) c² = 27 × 300


 a = 27 × 2 and =
36 b = 81 × 100
c = 90
36×36
 b= 16 32
A 17. (b) =
26 d
ATQ,
 d = 52
a = b7
17 19
36  36 18. (d) =
 27 × 2 = 272 d
A
A

 d = 16 × 19
 A = 24
 304
10. (c) c² = ab
Let, d be the mean proportion 19. (d) a : b : c
4 5
 d² = a²c × b²c
3 10
d² = a² b² c² = c4.c² 12 : 15 : 50
d = c³
 a : c = 6 : 25

Aditya Ranjan (Excise Inspector) Selected Selection 54


https://sscstudy.com/
Join Telegram- Maths by Aditya Ranjan AGE & PARTNERSHIP

AGE & PARTNERSHIP


07

1. Ratio between the present ages of A ais 2 : 3, 5. A and B invested their money in a business
respectively. After 5 years, the ratio between in the ratio of 9 : 5. If 10% of the total profit
their ages will be 3 : 4. What is B’s age at present? goes for charity and A’s share is Rs.29,840,
A B what is the total profit (in Rs., to the nearest
integer)?
B A B
SSC CGL 02/12/2022 (Shift- 04)
(a) 20 years (b) 15 years A

r
(c) 10 years (d) 25 years

si
2. The ratio of the present ages of Ram and SSC CGL 08/12/2022 (Shift- 02)
Ramesh is 3 : 5. After 7 years the ratio of (a) Rs.51,745 (b) Rs.55,715
an by
their ages will be 4 : 5. Find the present age
of Ramesh. 6.
(c) Rs.57,545 (d) Rs.51,575
Nihit starts a business with Rs.700. After 5

n
months Amit and Patel joined him with Rs.300

and Rs.400 respectively. At the end of the

ja year, the business gave a profit of Rs.627. Find
R s
the share of Patel in the profit.
SSC CGL 05/12/2022 (Shift- 01)
a th

(a) 5 years (b) 15 years


(c) 7 years (d) 12 years
ty a

3. The average age of a man and his son is 55


years. The ratio of their ages is 7 : 4,
respectively. What will be the ratio of their
di M

SSC CGL 08/12/2022 (Shift- 04)


ages after 6 years?
(a) Rs.127 (b) Rs.145
(c) Rs.132 (d) Rs.156
7. A, B and C did certain investments and the ratio
of their time periods is 3 : 2 : 7 respectively.
SSC CGL 12/12/2022 (Shift- 02) Ratio of the profits of A, B and C is 4 : 3 : 14
(a) 1 : 2 (b) 12 : 7 respectively. What is the ratio of the investments
of A, B and C?
(c) 25 : 17 (d) 38 : 23
A, B C
4. The ratio of the present age of father to that
of his son is 7 : 2. If after 10 years the ratio of A, B C
A

their ages will become 9 : 4, then the present A, B C


age of the father is:
SSC CGL TIER - II 02/03/2023

(a) 1 : 3 : 4 (b) 7 : 9 : 11

(c) 8 : 9 : 12 (d) 2 : 3 : 11

8. Suresh, Dinesh and Ramesh became partners
SSC CGL 13/12/2022 (Shift- 03) in a business by investing money in the ratio
(a) 43 years (b) 35 years of 3 : 6 : 8. If their investments is increased
by 5%, 15% and 20%, respectively, then what
(c) 37 years (d) 40 years
will be the ratio of their profits for one year?

Aditya Ranjan (Excise Inspector) Selected Selection 55


https://sscstudy.com/
Join Telegram- Maths by Aditya Ranjan AGE & PARTNERSHIP

(a) 16 : 18 : 15 (b) 13 : 18 : 15
(c) 16 : 21 : 18 (d) 15 : 16 : 13
10. A starts a business with Rs. 75,000 and B joins
the business 5 months later with an investment
of Rs. 80,000. After 1 year, they earn a profit of
SSC CGL TIER- II 03/03/2023
Rs. 4,08,800. Find the share of A and B (in Rs. ).
A
(a) 7 : 46 : 64 (b) 19 : 46 : 64
B
(c) 21 : 46 : 64 (d) 35 : 46 : 64

9. A, B and C invested capitals in the ratio 3 : 4 : 8.
A B
At the end of the business period, they
received profits in the ratio 2 : 3 : 5. What is
the ratio of their time invested? SSC CGL TIER- II 07/03/2023

A, B C (a) 2,52,000 and 1,56,800


(b) 2,50,000 and 1,58,800

r
(c) 2,48,000 and 1,60,800

si
SSC CGL TIER- II 06/03/2023 (d) 2,49,500 and 1,59,300
an by
n
ja ANSWER KEY
R s
1.(b) 2.(c) 3.(d) 4.(b) 5.(d) 6.(c) 7.(c) 8.(c) 9.(a) 10.(a)
a th
ty a
di M
A

Aditya Ranjan (Excise Inspector) Selected Selection 56


https://sscstudy.com/
Join Telegram- Maths by Aditya Ranjan AGE & PARTNERSHIP

SOL U TION S
1. (b) 6. (c) N A P
A B Investment 700 300 400
Time 12 7 7
Present 2 3
Difference = 1 Profit 8400 2100 2800
5 yrs. hence 3 4 12 : 3 : 4
19  627
1  5
627
4  4
3  15 19
= 33 × 4 = 132

r
2. (c) Ram Ramesh 7. (c)

si
Investment x time = Profit
Present 3 5×1
an by A B C
7 yrs. hence 4 5×2 Time 3 2 7
Profit 4 3 14

n
Present 3 5 4 3 14
Difference = 5 Investment
ja 3 2 7
7 yrs. hence 8 10
R s
= 8 9 12
57
8. (c)
a th

 Ramesh Present age = 7 yrs.


Given:- S D R
3. (d) M + S = 55 × 2 Let, Initial 300 600 800
ty a

Final 315 690 960


M 7
= 21 : 46 : 64
X 4
di M

9. (a)
 11  110
Man = 7  70 A B C
Son = 4  40 I. 3 4 8
P. 2 3 5
M 76 38
After 6 yrs.:- = = 2 3 5
X 46 23 T.
3 4 8
4. (b) Father Son = 16 : 18 : 15
Present 10. (a)
7 2
Difference = 2 A B
10 yrs. hence 9 4 Investment 75, 000 80,000
A

 2  10
Time 12 7
7  35 yrs.
Profit 9, 00, 000 5,60,000
5. (d) A : B
9:5 14,60,000  4,08,800
Let x = Total profit 7
1
ATQ, 25
90 9 A  2,52,000
x = 29840
100 14 B  1,56,800
x = 51575

Aditya Ranjan (Excise Inspector) Selected Selection 57


https://sscstudy.com/
Join Telegram- Maths by Aditya Ranjan MIXTURE

MIXTURE
08

1. A container contains 25 litre of milk. From 5. A can of water and milk mixture contains 60%
this container, 5 litre of milk is taken out milk. A part of this mixture is replaced by
and replaced by water. This process is further another mixture containing 50% milk and the
repeated two times. How much milk is there percentage of milk was found to be 52%. The
in the container now? quantity of mixture replaced is:



r
(a) 11.5 litre (b) 14.8 litre SSC CGL 13/12/2022 (Shift- 02)

si
(c) 13.5 litre (d) 12.8 litre
1 3
SSC CGL 01/12/2022 (Shift- 03) (a) (b)
an by 5 5
2. A mixture of milk and water measures 60
litres. It contains 10% water. How much water 4 2

n
should be added to it, so that the water may (c) (d)
5 5
be 25%? 6. P and Q are two alloys of aluminium and
ja
copper. The ratios of aluminium and copper
R s
in P and Q are 5 : 11 and 3 : 5, respectively.
a th

If a third alloy is formed by mixing alloys P


and Q in the ratio of 1 : 3, then aluminium
SSC CGL 02/12/2022 (Shift- 01)
is what percentage (rounded off to the nearest
(a) 18 litre (b) 14 litre
integer) of the copper in the third alloy ?
(c) 16 litre (d) 12 litre
ty a

3. A 100 ml solution of H 2 SO 4 having P Q


concentration of 20% is mixed with a 50% P Q
di M

concentrated x ml mixture such that the net P Q


mixture is 30% concentrated. Determine x.
20% H2SO4 100 ml
50% x ml
(net mixture) 30% x SSC CGL 13/12/2022 (Shift- 04)
(a) 63 Percent (b) 52 Percent
SSC CGL 08/12/2022 (Shift- 01) (c) 48 Percent (d) 56 Percent
(a) 70 ml (b) 80 ml 7. 40 litres of milk are kept in a container. 4
(c) 60 ml (d) 50 ml litres of milk were removed from this
4. A mixture contains milk and water in the ratio container and replaced with water. This
of 5 : 3, respectively. On adding 7 litres of procedure was performed two more times. How
A

water, the ratio of milk to water becomes much milk does the container now hold?
1 : 2. Find the quantity of milk in the
mixture.





SSC CGL TIER - II 02/03/2023
SSC CGL 08/12/2022 (Shift- 01)
(a) 7 litres (b) 10 litres (a) 30 litres (b) 34.23 litres
(c) 5 litres (d) 3 litres (c) 29.16 litres (d) 32 litres

Aditya Ranjan (Excise Inspector) Selected Selection 58


https://sscstudy.com/
Join Telegram- Maths by Aditya Ranjan MIXTURE

8. The cost of 3 kg of rice is Rs. 180. The cost


of 8 kg of rice is equal to that of 5 kg of
pulse.The cost of 15 kg of pulses is equal to
that of 2 kg of tea. The cost of 3 kg of tea is SSC CGL TIER- II 06/03/2023
equal to that of 6 kg of walnuts. What is the (a) 80 L (b) 60 L
cost (in Rs.) of 10 kg of walnuts? (c) 70 L (d) 50 L
10. How many kilogram of rice costing Rs. 60 per
kg. must be mixed with 24 kg. of rice costing
Rs. 42 per kg. so that there may be a gain of
12% by selling the mixture at Rs. 56 per kg.?

kg. kg.
kg. 24 kg.
SSC CGL TIER- II 03/03/2023

(a) 2400 (b) 3200 kg.
(c) 2800 (d) 3600
SSC CGL TIER- II 07/03/2023
9. If 80 liters of milk solution contains 60% milk,

r
how much milk should be added to make the (a) 22 kg. (b) 20 kg.

si
solution 80% milk? (c) 19.2 kg. (d) 21.2 kg.
an by ANSWER KEY

n
1.(d) 2.(d)
ja 3.(d) 4.(c) 5.(c) 6.(d) 7.(c) 8.(d) 9.(a) 10.(c)
R s
a th
ty a
di M
A

Aditya Ranjan (Excise Inspector) Selected Selection 59


https://sscstudy.com/
Join Telegram- Maths by Aditya Ranjan MIXTURE

SOL U TION S
1. (d) Final milk = Initial quantity Quantity of Milk is same in new solution
n
 
1 – Quantity out  Milk Water
 Total quantity 
 
 5 
3 Old 5 : 3
= 25 
1 – 
 +7
 25 
New 5 : 10
4 4 4 64
= 25    = = 12.8 ltr.
5 5 5 5 77
2. (d) By allegation 5  5 ltr.

r
10% 100% 5. (c) Using allegation:-

si
60 50

an by 25%

n
52
ja
R s
75 15
a th

5 : 1 2 8
5  60 1 : 4
1  12 ltr.
ty a

3. (d) Using allegation: 4


 Mixture replaced =
5
20 50
di M

6. (d)

Al Cu Total Al Cu

P 5 11 = 16 × 1 5 + 18 11 + 30
30
Q 3 5 =8  23 43
23
Q 6 10 = 16 × 3 Required% =  100  56%
43
20 10
7. (c)
2 : 1
We know,
2  100 Volume left
A

100  no. of times


x = 50 ml. Amount taken out 
2 = original amt. × 1 – 
 Original amount 
4. (c)
Milk Water
3
 4 
= 40 1 – 
Old 5 : 3 
 40 
+7
9 9 9
New 1 : 2 = 40    = 29.16 ltr.
10 10 10

Aditya Ranjan (Excise Inspector) Selected Selection 60


https://sscstudy.com/
Join Telegram- Maths by Aditya Ranjan MIXTURE

8. (d) 10. (c)


3 kg. Rice = Rs. 180 Final S.P = 56
1 kg. Rice = Rs. 60 Profit% = 12%
 5 kg. pulse = 8 × 60
56
= Rs. 480 C.P =  100
112
15 kg. Pulse = Rs.1440
= 50
2 kg. Tea = Rs.1440
By allegation:
3 kg. Tea = Rs.2160 = 6 kg. walnuts
60 42
2160
 10 kg. walnuts =  10 = 3600
6
9. (a)
Amt. of water remained same before and after 50
addition of milk.

r
A.T.Q,

si
40 20
 80 =  (80  Milk) 8 10
100 100

32 =
1
an by
(80 + Milk)
8 : 10

n
5 5  24
 Milk added = 80 ltr. 4  19.2 kg.
ja
R s
a th

......-------......
ty a
di M
A

Aditya Ranjan (Excise Inspector) Selected Selection 61


https://sscstudy.com/
Join Telegram- Maths by Aditya Ranjan AVERAGE

AVERAGE
09

1. The batting average for 27 innings of a cricket 4. The average weight of 49 students in a class
player is 47 runs. His highest score in an is 39 kg. Seven of them whose average weight
innings exceeds his lowest score by 157 runs. is 40 kg leave the class and other seven
If these two innings are excluded, the average students whose average weight is 54 kg join
score of the remaining 25 innings is 42 runs. the class. What is the new average weight (in
Find his highest score in an innings. kg) of the class?
39 kg

40 kg
54 kg

r

(kg)

si

(a) 176
an by SSC CGL 01/12/2022 (Shift- 01)
(b) 188
(a) 41
SSC CGL 02/12/2022 (Shift- 04)
(b) 39

n
(c) 42 (d) 40
(c) 186 (d) 174 5. The average marks of 60 students in a class
2. ja
The average of the marks of 25 students in are 62. The average marks of boys and girls
R s
a class, in an examination was calculated to in that subject are 60 and 65, respectively.
be 19. Later, the teacher realized that the The number of boys in the class is:
a th

marks of two students were taken as 18 and


19 respectively, instead of 14 and 15. Find
the new actual average marks of the class.
ty a

SSC CGL 03/12/2022 (Shift- 01)


(a) 36 (b) 25
di M

(c) 41 (d) 31
6. The average weight of 5 men decreases by 3

kg when one of them weighing 150 kg is
replaced by another person. Find the weight
SSC CGL 01/12/2022 (Shift- 02) of the new person.
(a) 17.43 (b) 16.56 kg
(c) 18.68 (d) 17.65 kg
3. Find the weighted arithmetic mean of the first
‘n’ natural numbers, the weights being the SSC CGL 05/12/2022 (Shift- 02)
corresponding numbers.
(a) 125 kg (b) 100 kg
A

'n' (c) 120 kg (d) 135 kg


7. Sonali applied for a job of Science teacher in
SSC CGL 02/12/2022 (Shift- 02) a school. In the test for job, she scored 8 in
Physics, 8 in Chemistry, 6 in Biology, and 6.5
(a)
n n +1 2n +1 (b)
n n +1 in the interview. For calculating the final
6 6 score, weightage of 2, 3, 3, and 4 were
assigned to Physics, Chemistry, Biology and
2n + 1 interview, respectively. What is the weighted
(c) (d) n
3 average score of Sonali?

Aditya Ranjan (Excise Inspector) Selected Selection 62


https://sscstudy.com/
Join Telegram- Maths by Aditya Ranjan AVERAGE

11. A student‘s marks were wrongly entered as


65 instead of 45. Due to this, the average

1
marks for the class got increased by . The
3
number of students in the class is:


1

SSC CGL 06/12/2022 (Shift- 01) 3

(a) 7 (b) 8.4


SSC CGL 08/12/2022 (Shift- 03)
(c) 7.2 (d) 21
(a) 40 (b) 20
8. The average weight of 12 persons increases
(c) 60 (d) 30
by 3.5 kg when a new person comes in place
12. While finding the average marks of a class,
of one of them weighing 58 kg. The weight
Vikas’s marks were wrongly entered as 98 in

r
of the new person is:
place of 89. Due to this error, the average

si
58 kg marks of the class were 0.25 more than the
actual average. What is the number of students
an by


3.5 kg in the class?

n
SSC CGL 07/12/2022 (Shift- 02)

ja
(a) 100 kg
R s

(b) 16 kg
SSC CGL 08/12/2022 (Shift- 04)
a th

(c) 70 kg
(a) 32 (b) 38
(d) 42 kg (c) 34 (d) 36
9. The average of the first 7 non-zero multiples 13. In a hotel, there are 120 staff members. Their
ty a

of 17 is: average weight is 62.5 kg. When one of the


staff members leaves the hotel, the average
di M


weight is reduced by 250 g. Find the weight
SSC CGL 07/12/2022 (Shift- 03) of the staff member who left the hotel.
(a) 67 (b) 68
(c) 69 (d) 66 kg
10. The average weight of apples in a basket is g
50 kg. 6 more apples with the average weight
of 60 kg are added in the basket. If the average

weight of the basket is increased by 5 kg, then
SSC CGL 09/12/2022 (Shift- 01)
find the number of apples in the basket
originally. (a) 64.25 kg (b) 78.75 kg
A

(c) 90.50 kg (d) 92.25 kg


50 kg
14. If the average of s number is r4 and the average
60 kg r number is s4, then find the average of all r
5 kg + s number?
s r4 r
s4 r + s
SSC CGL 07/12/2022 (Shift- 04) SSC CGL 09/12/2022 (Shift- 04)
(a) 4 (b) 6 (a) rs (r² + s² – rs) (b) rs
(c) 2 (d) 8 (c) r² + s² (d) rs (r² + s²)

Aditya Ranjan (Excise Inspector) Selected Selection 63


https://sscstudy.com/
Join Telegram- Maths by Aditya Ranjan AVERAGE

15. The average of six numbers is 3.52. The (a) 88 (b) 69


average of two of them is 3.7, while the (c) 79 (d) 100
average of other two is 2.5. What is the 19. The average of 20 numbers is 32. If two
average of the remaining two? numbers are 29 and 31, then what is the
average of the remaining numbers (correct up
to two decimals)?

SSC CGL 12/12/2022 (Shift- 01)
(a) 3.7 (b) 2.5
(c) 4.36 (d) 3.52 SSC CGL TIER- II 03/03/2023
16. The average age of six members of a family (a) 31.24 (b) 30.22
is 40 years. If the age of a guest is included, (c) 34.44 (d) 32.22
then the average age increases by 12.5%.What 20. The average weight of three men is increased
is the age (in years) of the guest? by 4 kg when one of them, whose weight is 100
kg, is replaced by another man. What is the
weight of the new man?

si
SSC CGL 12/12/2022 (Shift- 04)
(a) 59 an by (b) 65
(c) 75 (d) 69 SSC CGL TIER- II 03/03/2023
17. Ram, Shyam, Rohan, Reeta and Mukesh are (a) 107 kg. (b) 103 kg.

n
five members of a family who are weighed (c) 104 kg. (d) 112 kg
consecutively and their average weight is 21. The average marks obtained by Saloni in four
ja
calculated after each member is weighed. If papers is 51, and in the fifth paper she
R s
the average weight increases by 2 kg each obtained 56 marks. Find his new average in
time, how much heavier is Mukesh than Ram?
a th

all the five papers.





ty a

2 SSC CGL TIER- II 06/03/2023


kg (a) 52 (b) 49
di M

SSC CGL 13/12/2022 (Shift- 01) (c) 50 (d) 51


(a) 14 kg (b) 18 kg 22. Find the average of the following sets of
(c) 16 kg (d) 12 kg numbers.
18. In nine innings of cricket, the average of a 693, 456, 876, 532, 934, 691, 596, 398, 682
batsman is 58 runs. How many runs does he
need to make in the tenth innings to get the
693, 456, 876, 532, 934, 691, 596, 398, 682
average score 61?
SSC CGL TIER- II 07/03/2023
(a) 550.50 (b) 750.80

(c) 650.89 (d) 725.90

SSC CGL 13/12/2022 (Shift- 04)
A

ANSWER KEY
1.(b) 2.(c) 3.(c) 4.(a) 5.(a) 6.(d) 7.(a) 8.(a) 9.(b) 10.(b)

11.(c) 12.(d) 13.(d) 14.(a) 15.(c) 16.(c) 17.(c) 18.(a) 19.(d) 20.(d)

21.(a) 22.(c)

Aditya Ranjan (Excise Inspector) Selected Selection 64


https://sscstudy.com/
Join Telegram- Maths by Aditya Ranjan AVERAGE

SOL U TION S
1. (b)
14 × 7
H – L = 157  Average change = 39 +
49
A.T.Q.
= 41
x +H+L
= 47 ...(1) 5. (a)
27
Boys Girls
x 60 65
and = 42
25
x = 1050 ...(2) 62
put (2) in (1)

r
 H + L = 47 × 27 – 1050
3 2

si
H + L = 219 ...(3)
 H – L = 157 5x = 60
an by
H + L = 219
6.
3x = 36
(d)

n
376
 H= = 188 Let y be the weight of new person
2
2. (c) ja  Change in weight = +4 – 150
R s
Total error = (18 + 19) – (14 + 15) A.T.Q.
a th

=8
y – 150
 Change in average = = –3
8 5
 Actual Average = 19 –
25  y = 135
ty a

= 19 – 0.32
7. (a)
= 18.68
di M

3. (c) Weightage Marks


wi = weight , xi = numbers
Physics 2 8
 wixi Chemistry 3 8
Weighted average =
 wi Biology 3 6
1 × 1 + 2 × 2 + 3 × 3 + .... +  n × n  Interview 4 6.5
=
1 + 2 + 3 + .... + n
Weighted average 
12 + 22 + 32 + .... + n 2
= 2 × 8 + 3 × 8 + 6 × 3 + 6.5 × 4 
1 + 2 + 3 + .... + n =
12
A

n  n + 1 2n + 1 × 2
= 84
6  n  n +1 = = 7 marks.
12
2n +1 8. (a)
=
3 Change in weight = x – 58
4. (a)
Total change in weight of loss x – 58
Change in average = = 3.5
 54 × 7 – 40 × 7 12
 14 × 7  x = 100 kg

Aditya Ranjan (Excise Inspector) Selected Selection 65


https://sscstudy.com/
Join Telegram- Maths by Aditya Ranjan AVERAGE

9. (b) 14. (a)


Sum = 17 + 34 +....+ 119 Total of s numbers = s × r4
Sum = 17(1 + 2 +....+7) Total of r numbers = r × s4
rs  r 3 + s3 
17 × 7 × 8 Average (r + s) numbers =
Sum =
2
r + s 
= rs(r² – rs + s²)
17 × 7 × 8 15. (c)
Average = = 68
2×7 Total sum = 3.52 × 6
10. (b) Sum of I two = 3.7 × 2
Sum of II two = 2.5 × 2
Old New
50 60 Sum of remaining two = –2(3.7 + 2.5) + 6 × 3
– 52
= –2(6.2) + 6 × 3.52
55
= 8.72

r
8.72

si
5 : 5 Average = = 4.36
2
1 : 1
1
an by 6
16. (c)

n
11. (c) 12.5% =
8
Error = –45 + 65 = +20
ja Let, age of guest be 'x' years
R s
Let these be x students Change in average =
a th

20 1 9
 Change in average = = 6 × 40 + x = 7 × × 40
x 3 8
 x = 60 x = 315 – 240
ty a

12. (d) = 75 years


17. (c)
di M

Error = 98 – 89 = 9
Let no. of students = x Ram Shyam Rohan Reeta Mukesh
x x+2 x+4 x+6
9 1 Average increased upto Reeta is 6 kg
 Change in average = =
x 4 i.e., if Average of Ram = x
 x = 36 Average upto Reeta = x + 6
13. (d) Again 2 kg increase with Mukesh
 2kg per 5 person increase
1
Change in average = kg  10 kg more than x + 6
4
 x + 6 + 10 = x + 16
A

1  Mukesh – Ram:-
kg reduced from all 119 members
4  (x + 16) – x = 16 kg
18. (d)
1 Increase in average = 61 – 58 = 3
 Total weight reduced = 119 ×
4
Total inning = 10
= 29.75  Total sums increase = 3 × 10 = 30
 Weight of the person left  Runs in 10th inning = 58 + 30
= 62.5 + 29.75 = 92.25 kg. = 88

Aditya Ranjan (Excise Inspector) Selected Selection 66


https://sscstudy.com/
Join Telegram- Maths by Aditya Ranjan AVERAGE

19. (d) 21. (a)


S20 = 20 × 32 Extra marks than avg. = 56 – 51 = 5

20  32 – 29 – 31 5
S18 = New avg. = 51  = 52
18 5
= 32.22 22. (c)
20. (d)
Total Sum
Total increase in weight = 4 × 3 Avg. =
Total Numbers
= 12
 Weight of new man = 100 + 12 = 112 kg. 5858
= = 650.89
9

......-------......

r
si
an by
n
ja
R s
a th
ty a
di M
A

Aditya Ranjan (Excise Inspector) Selected Selection 67


https://sscstudy.com/
Join Telegram- Maths by Aditya Ranjan TIME & WORK

TIME & WORK


10

1. P and Q can complete a project in 15 days 4. A takes 4 times as much time as B or 5 times
and 10 days, respectively. They started doing as much time as C to finish a piece of work.
the work together, but after 2 days, Q had Working together, they can finish the work
to leave and P alone completed the remaining in 4 days. B can do the work alone in:
work. In how many days was the whole work
A B
completed?
C
P Q
B

r
Q
P SSC CGL 01/12/2022 (Shift- 03)

si
(a) 10 days (b) 15 days

(a) 11
an by SSC CGL 01/12/2022 (Shift- 01)
(b) 12
(c) 12 days (d) 20 days

n
5. 6 men and 8 women can do a piece of work
(c) 13 (d) 10 in 10 days, whereas 26 men and 48 women
2. ja
A, B and C can complete a piece of work can do the same work in 2 days. What will
R s
separately in 10, 20 and 40 days, respectively. be the time taken by 15 men and 20 women
a th

In how many days will the work be completed to do the same work?
if A is assisted by both B and C every third
day?
A, B C 10,
ty a

20 40 A

di M

B C
SSC CGL 01/12/2022 (Shift- 04)
SSC CGL 01/12/2022 (Shift- 02) (a) 6 days (b) 10 days

2 (c) 4 days (d) 8 days


(a) 8 (b) 9
7 6. If 6 women can complete a work in 3 days,
and 9 girls can complete the same work in 2
2
(c) 10 (d) 6 days, then find the time taken to complete
3
the same work by 8 women and 6 girls?
3. If 450 men can finish construction of an
apartment in 20 days, then how many men
A

are needed to complete the same work in 30


days?

SSC CGL 02/12/2022 (Shift- 01)

(a) 3 days (b) 4 days
SSC CGL 01/12/2022 (Shift- 03)
(a) 150 (b) 300 9
(c) 6 days (d) days
(c) 400 (d) 250 7

Aditya Ranjan (Excise Inspector) Selected Selection 68


https://sscstudy.com/
Join Telegram- Maths by Aditya Ranjan TIME & WORK

7. A takes twice as much time as B and thrice


as much time as C to finalise a task. Working
together, they can complete the task in 8

days. The time (in days) taken by A, B and

C, respectively, to complete the task is:
SSC CGL 03/12/2022 (Shift- 01)
A B C 550 85
(a) days (b) days
12 4
71 360
A, B C (c) days (d) days
2 17
11. A and B can do a certain work in 6 hours,
and A, B and C together take 4 hours to do
SSC CGL 02/12/2022 (Shift- 02)
the same. How long will it take for C alone
(a) 42, 21, 14 (b) 60, 30, 20 to accomplish the task?
A B

r
(c) 54, 27, 18 (d) 48, 24, 16
A, B C

si
8. If one man or two women or four boys or five C
girls can finish a work in 39 days, then how
an by
many days will one man, one woman, one boy
and one girl together take to finish the same (a) 12 hours
SSC CGL 03/12/2022 (Shift- 02)
(b) 4 hours

n
work? (c) 2 hours (d) 6 hours
12. 5 men and 8 women can complete a work in

ja 12 days working together, while 3 men and
R s
7 women together can complete the same
a th

work in 15 days. In how many days will 11


women complete the same work?

SSC CGL 02/12/2022 (Shift- 03)
ty a

(a) 40 (b) 10
di M


(c) 30 (d) 20

9. A and B together can do a piece of work in SSC CGL 03/12/2022 (Shift- 03)
50 days. If A is 40% less efficient than B, in (a) 12 (b) 8
how many days can A working alone complete (c) 6 (d) 16
60% of the work? 13. A and B working separately can complete a
piece of work in 10 and 16 days, respectively.
A B If they work for a day alternately, with A
A, B A beginning the work, in how many day(s) will
the work be completed?
A B
SSC CGL 02/12/2022 (Shift- 04)
A


(a) 70 (b) 110 A

(c) 80 (d) 105
SSC CGL 03/12/2022 (Shift- 04)
10. Ruchi, Khushi and Teju can do a piece of work 1 1
in 30, 40 and 60 days respectively. In how (a) 10 (b) 12
4 4
many days can Ruchi do the work, if she is
assisted by both Khushi and Teju on every 1 1
(c) 1 (d)
third day? 4 4

Aditya Ranjan (Excise Inspector) Selected Selection 69


https://sscstudy.com/
Join Telegram- Maths by Aditya Ranjan TIME & WORK

4 17. There are 3 taps A, B, and C in a tank. These


14. Numan does half the work as Gagan in time. can fill the tank in 10 hours, 20 hours and 25
5
If together they take 16 days to complete a hours, respectively. At first, all three taps are
piece of work, then how long will Gagan take opened simultaneously. After 2 hours, tap C is
closed and A and B keep running. After 4 hours
to complete the work?
from the beginning, tap B is also closed. The
4 remaining tank is filled by tap A alone. Find
the percentage of work done by tap A itself.
5
A, B C


SSC CGL 05/12/2022 (Shift- 01)
C A B
(a) 23 days (b) 24 days B
(c) 25 days (d) 26 days A A

r
15. A contractor decided to complete a work in
80 days and employed 60 men at the SSC CGL 05/12/2022 (Shift- 04)

si
beginning and 20 men additionally after 20 (a) 32% (b) 75%
an by
days and got the work completed as per
schedule. If he had not employed the
(c) 52% (d) 72%

n
additional men, how many extra days would 18. A, B and C can do a piece of work in 11 days,
he have needed to complete the work (round 20 days and 55 days respectively. If B works
ja
off to the nearest integer)? daily and is supported by A and C on alternate
R s
days beginning with A, then in how many days
will the work be finished?
a th

A, B C
B

ty a

A A
C
di M


SSC CGL 06/12/2022 (Shift- 01)
SSC CGL 05/12/2022 (Shift- 02)
1
(a) 10 (b) 9
(a) 32 days (b) 20 days 3
(c) 30 days (d) 26 days (c) 12 (d) 8
16. 40 men can complete a work in 15 days. Three 19. A group of men decided to do a job in 6 days,
days after they started working, 20 more men but 18 men dropped out every day. If the job
joined them. In how many days the total work was completed in 8 days, then how many men
will be completed? initially decided to do the job.
A





SSC CGL 05/12/2022 (Shift- 03)
SSC CGL 06/12/2022 (Shift- 02)
(a) 12 (b) 13
(a) 300 (b) 252
(c) 11 (d) 10 (c) 188 (d) 150

Aditya Ranjan (Excise Inspector) Selected Selection 70


https://sscstudy.com/
Join Telegram- Maths by Aditya Ranjan TIME & WORK

20. A and B can complete a piece of work in 10 23. A civil contractor planned to build an over
and 16 days respectively. A begins to do the bridge of 3 km distance in 600 days. For this
work and they work alternatively one at a time purpose, he employed 90 workers. After 200
for one day each. The whole work will be days of work, it was observed that only 0.5
completed in : km of the bridge was completed. What is the
number of extra workers required to complete
A B the work in time?
A




SSC CGL 06/12/2022 (Shift- 03)

1 1
(a) 12 days (b) 13 days
4 4
SSC CGL 07/12/2022 (Shift- 03)

r
1 1 (a) 100 (b) 140
(c) 15 days (d) 14 days

si
4 4
(c) 125 (d) 135
21. 1 man and 4 women can complete a work in
an by 24. 12 men and 12 women can complete a piece
65 of work in 9 days. 6 men and 15 women can
days, while 3 men and 4 women can

n
4 complete the same work in 12 days. How long
13 (in days) would it take 9 women to complete
ja
complete it in days. In who many days will half the work?
R s
2
be 13 women compelete the same.
a th

65
1
4
13
ty a


2
SSC CGL 07/12/2022 (Shift- 04)

di M

(a) 36 (b) 12

SSC CGL 06/12/2022 (Shift- 04) (c) 18 (d) 24

(a) 20 (b) 16 25. Ten men begin to do a work. But after some
days, four of them left the job. As a result,
(c) 14 (d) 18
the job that could have been completed in 40
22. 25 men and 45 women can complete a piece days is completed in 50 days. How many days
of work in 15 days, while 15 men and 60 after the commencement of the work did the
women can complete it in 20 days. In how four men leave?
many days can 69 men and 67 women
complete the work?
A






SSC CGL 07/12/2022 (Shift- 01) SSC CGL 08/12/2022 (Shift- 01)

(a) 10 (b) 5 (a) 30 (b) 20

(c) 8 (d) 6 (c) 35 (d) 25

Aditya Ranjan (Excise Inspector) Selected Selection 71


https://sscstudy.com/
Join Telegram- Maths by Aditya Ranjan TIME & WORK

26. In a factory, Ajay and Vijay work on the same 28. A and B can complete a piece of work in 13
machine to cut diamonds but on alternate and 17 days respectively. A begins to do the
hour basis. Ajay works for the first hour and work, and they work alternatively one at a
then Vijay works for the second hour and so time for one day each. The whole work will
on. Ajay can complete the work in 6 hours, be completed in:
while Vijay completes it in 16 hours if they
A B
work individually. In how much time can they
complete the work if they are using the A
machine on alternate basis?


SSC CGL 08/12/2022 (Shift- 04)

11 17
(a) 17 days (b) 17 days
17 19

r
11 11
(c) 14 days (d) 11 days

si
17 17

an by


29. Anshu, Swati and Rajni can finish a work in
4 days if they work together. However, Anshu

n
alone will take 9 days to complete the work,
SSC CGL 08/12/2022 (Shift- 02) and Swati alone will complete the work in 12
ja days. How many days will Rajni alone take
R s
(a) 8 hours 30 min
to complete the work?
a th

(b) 8 hours

(c) 9 hours
(d) 9 hours 30 min
ty a


27. Raju, Sunil and Vishal can separately finish
di M

a work in 20, 30 and 40 days, respectively.


In how many days Raju can finish the work,
if he is assisted by Sunil and Vishal on
SSC CGL 09/12/2022 (Shift- 01)
alternate days, starting with Sunil?
(a) 18 (b) 12
(c) 24 (d) 16

30. 20 men can finish a work in 220 days, but
at the end of 90 days, 20 additional men are
employed. In how many more days will the
work be completed?
A


SSC CGL 08/12/2022 (Shift- 03)

3 1
(a) 12 (b) 12
5 20
SSC CGL 09/12/2022 (Shift- 02)

1 3 (a) 50 (b) 60
(c) 11 (d) 11
20 5 (c) 65 (d) 55

Aditya Ranjan (Excise Inspector) Selected Selection 72


https://sscstudy.com/
Join Telegram- Maths by Aditya Ranjan TIME & WORK

31. Abha and Anuj working together completed 34. If Anju is 30% more efficient than Bitt, then
how much time will they take, working
40
a job in days. If Abha had worked twice together, to complete a job which Anju alone
9
could have done in 23 days?
as efficiently as she actually did and Anuj had

1
worked of his actual efficiency, then the
3

60 SSC CGL 12/12/2022 (Shift- 01)
work would have been completed in days.
17
Find the time Abha would take to complete 3
the work alone. (a) 13 days (b) 20 days
17
(c) 15 days (d) 11 days
40 35. A machine takes 10 h to cut 240 tools. How

9 many tools will it cut in 25 h?

si
1

3 SSC CGL 12/12/2022 (Shift- 01)
an by

60
17

(a) 600
(c) 550
(b) 480
(d) 360

n
36. Raju, Shobh and Mohan can do a work in 15
ja SSC CGL 09/12/2022 (Shift- 03) days, 20 days and 25 days respectively. In
R s
how many days,will the work be finished, if
(a) 10 days (b) 8 days
they do it on alternate days?
a th

(c) 12 days (d) 6 days


32. Two persons P and Q can do a job together
in 36 days. P is 3 times as efficient as Q. In
how many days can Q alone complete the
ty a

work?

P Q
di M

SSC CGL 12/12/2022 (Shift- 02)


P, Q Q
9 7
(a) 15 days (b) 18 days
SSC CGL 09/12/2022 (Shift- 04) 10 10
(a) 148 (b) 140
(c) 144 (d) 146 7 9
(c) 21 days (d) 18 days
33. A, B and C can separately do a work in 6, 10 10 10
and 15 days respectively. They started to work 37. A is 40% more efficient than B. How much
together but C left after 2 days. In how many time will both, working together, take to finish
days will the remaining work be finished? the work, which B alone can finish in 36 days?
A, B C A, B
A


C, 2 B
SSC CGL 12/12/2022 (Shift- 03)
SSC CGL 09/12/2022 (Shift- 04)
1
2 1 (a) 18 days (b) 9 days
(a) 1 (b) 1 3
4 4
5 7 2
(c) 1 (d) 2 (c) 15 days (d) 11 days
8 4 3

Aditya Ranjan (Excise Inspector) Selected Selection 73


https://sscstudy.com/
Join Telegram- Maths by Aditya Ranjan TIME & WORK

38. Aarif, Arun and Abraham can do a work in 12, 1 1


20 and 24 days, respectively. They all begin (a) 9 hours (b) 7 hours
3 3
together. Aarif leaves the work 1 day, Arun 3
days and Abraham 4 days before its completion. 1 1
(c) 6 hours (d) 8 hours
In how many days is the work finished? 3 3
42. Pawan can do a piece of work in 32 days. He
worked for 8 days and left the work. Thereafter
Sandeep finished the remaining work in 27
days. In how many days can Pawan and
Sandeep together do the whole work?

SSC CGL 12/12/2022 (Shift- 04)
(a) 8 (b) 10
(c) 11 (d) 9
39. A is 20% more efficient than B. How much SSC CGL 13/12/2022 (Shift- 04)
time will they working together take to 16 13

r
complete a job which A alone could have done (a) 16 days (b) 16 days
17 17

si
in 29 days?
15 14
A, B (c) 16 days (d) 16 days
an by 17 17
A 43. There are three taps of diameter 2 cm, 3 cm
and 4 cm, respectively. The ratio of the water

n
SSC CGL 13/12/2022 (Shift- 01) flowing through them is equal to the ratio of
the square of their diameters. The biggest tap
145 ja 116 can fill an empty tank alone in 81 min. If all
R s
(a) (b)
11 11 the taps are opened simultaneously, then how
long will the tank take (in min) to be filled?
a th

203 174
(c) (d)
11 11

40. A, B and C can all together do a piece of work

ty a

in 10 days, in which B takes 3 times as long


as A and C together to do the work. In how
di M

many days can B alone do the work?


A, B C
B A C SSC CGL TIER - II 02/03/2023
20 20
(a) 34 (b) 60
B 29 29
SSC CGL 13/12/2022 (Shift- 02) 20 20
(a) 50 days (b) 40 days (c) 54 (d) 44
29 29
(c) 10 days (d) 15 days 44. There are two inlets A and B connected to a
41. Two persons A and B working separately can tank. A and B can fill the tank in 32 h and 28
sanitise a building in 6 and 10 hours, h, respectively. If both the pipes are opened
A

respectively. They work in stretches of one alternately for 1 h, starting with A, then in
hour alternately. If A begins at 8 a.m., then how much time (in hours, to nearest integer)
when will the work be finished? will the tank be filled?
A B A B A B
32 h 28 h
A 1 h
8 a.m. A

SSC CGL 13/12/2022 (Shift- 03) SSC CGL TIER- II 03/03/2023

Aditya Ranjan (Excise Inspector) Selected Selection 74


https://sscstudy.com/
Join Telegram- Maths by Aditya Ranjan TIME & WORK

(a) 22 (b) 30 SSC CGL TIER- II 07/03/2023


(c) 36 (d) 24
45. A and B together can complete a piece of work 13 9
(a) (b)
in 48 days. A is 4 times as efficient as B. In 2 2
how many days B alone can complete the work?
A B 7 5
(c) (d)
A, B B 2 2
47. Working for 10 hours per day, 6 men can build
SSC CGL TIER- II 06/03/2023 a wall in a certain number of days. By working
for how many hours/minutes per day can 12
(a) 220 Day (b) 240 Day
men build the wall in the same days?
(c) 120 Day (d) 320 Day

46. P, Q and R can finish a work in 5 days, 10 days,

and 15 days, respectively, working alone. P and
Q works on first day, P and R works on second
day and P and Q works on third day and son

r
on till the work is completed. In how many SSC CGL TIER- II 07/03/2023
days the work will be completed?

si
(a) 7 hours 30 minutes/
P, Q R
an by

Q
P
P R
(b) 5 hours/
(c) 20 hours/

n
P Q (d) 10 hours/

ja
R s

a th

ANSWER KEY
ty a

1.(b) 2.(a) 3.(b) 4.(a) 5.(c) 6.(d) 7.(d) 8.(d) 9.(c) 10.(b)
di M

11.(a) 12.(a) 13.(b) 14.(d) 15.(b) 16.(c) 17.(d) 18.(b) 19.(b) 20.(a)

21.(a) 22.(d) 23.(d) 24.(c) 25.(d) 26.(a) 27.(a) 28.(c) 29.(a) 30.(c)

31.(b) 32.(c) 33.(b) 34.(a) 35.(a) 36.(d) 37.(c) 38.(a) 39.(d) 40.(b)

41.(b) 42.(a) 41.(d) 42.(a) 43.(d) 44.(b) 45.(b) 46.(c) 47.(b)


A

Aditya Ranjan (Excise Inspector) Selected Selection 75


https://sscstudy.com/
Join Telegram- Maths by Aditya Ranjan TIME & WORK

SOL U T ION S
1. (b) 5. (c)
Given,
P – 15 (2)
30 6M + 8W  10 days
Q – 10 (3)  3M + 4W  20 days
 15M + 20W  4 days
A.T.Q. 6. (d)
2(P + Q) + xP = 30 6W  3 days  1W  18 days
2(5) + 2x = 30 9G  2 days  1G  18 days
2x = 20 W 18 (1)
x = 10 18 units

r
 Whole work completed in:- G 18 (1)
10 + 2 = 12 days

si
18 18 9
2. (a) 8W + 6G = = =
8 + 6 14 7
an by
A – 10 (4)
7. (d)
A B C

n
B – 20 (2) 40 units Time 6 3 2
C – 40 (1) Efficiency 1 2 3
ja
R s
Total work = 8 × 6 = 48 units
(A,A, A + B + C)  Working fashion
48
a th

3 days  4 + 4 + 7 = 15 units A = 48 days


1
6 days  30 units
48
7 days by A  34 units B = 24 days
2
8 days by A  38 units
ty a

48
2 C = 16 days
9 days by (A + B + C)  3
di M

7 'or'
 38 + 2 = 40 units Directly check from option ratio of time
should be
2 (6 : 3 : 2)
Total days = 8 days. 8. (d)
7
1M + 2W = 4B = 5G = 39 days
3. (b) 1M = 39
M1 × D1 × H M2 × D2 × H2 1W = 39 × 2
= 39 × 20
W1 W2 1B = 39 × 4
 450 × 20 = x × 30 1G = 39 × 5
 x = 300 39 × 20
A

4. (a)  1M + 1W + 1B + 1G =
20 +10 + 5 + 4
= 20 days
A B C 9. (c)
days  20 5 4 A + B  50
A B
efficiency  1 4 5 3 5
Total work = 8 × 50 = 400 units
Let total work = 10 × 4 = 40 units
3 400
40 60% work by A = ×
B = 10 days. 5 3
4 = 80 days

Aditya Ranjan (Excise Inspector) Selected Selection 76


https://sscstudy.com/
Join Telegram- Maths by Aditya Ranjan TIME & WORK

10. (b) 14. (d)


R  30 (4) Naman Gagan
1
K  40 (3) 40 units  Work 1
2
T  60 (2) 1 : 2
 Time 4 : 5
Working fashion  R, R, R + K + T
1 2
3 days  4 + 4 + 9 = 17 units  Efficiency
21 days  119 units 4 5
5 : 8
th
1 Total work = 16 × 13
22 days by R  days 120 units
4 16 ×13
Gagan = = 26 days
1 8
 Total time = 21 days 15. (b)
4
60 Men  20 days
85 80 Men  60 days
= days

r
4  (60 × 20) + (80 × 60) = Total work

si
11. (a)
6000
A + B  6 (2)  Required days = = 100 days
an by 60
12 units
A + B + C  4 (3)  20 Additional days

n
16. (c)
C  (1)
(40 × 15) = Total work
ja 12 A.T.Q.
 C will take = = 12 hours
R s
1 (40 × 3) + (60 × x) = (40 × 15)
12. (a) 60x = 600 – 120
a th

5M + 8W  12 days 60x = 480


3M + 7W  15 days x=8
12(5M + 8W) = 15(3M + 7W)  Total days = 8 + 3 = 11 days
ty a

20M + 32W = 15M + 35W 17. (d)


5M = 3W A – 10 (10)
di M

M 3 B – 20 (5)
= 100 units
W 5
 Total work = 12(15 + 40) C – 25 (4)
= 660 units A.T.Q.
2 × (A + B + C) + 2(A + B) + xA = 100
660  38 + 30 + xA = 100
 11 women  ]
11 × 5 xA = 100 – 68 = 32
 12 days 32 16
13. (b) x= = days.
10 5
A  10 (8) 16
80 units Work  (2 × 10) + (2 ×10) + ×10
5
A

B  16 (5)  40 + 32 = 72
Working fashion = (A, B, A, B...) 72
2 days  13 units Required% = ×100 = 72%
100
12 days  78 units
'or'
1 C=4×2=8
13th days by A  days  80 units B = 5 × 4  20
4
100 – 28 = 72
1 72
 Total time = 12 days ×100 = 72%
4 100

Aditya Ranjan (Excise Inspector) Selected Selection 77


https://sscstudy.com/
Join Telegram- Maths by Aditya Ranjan TIME & WORK

18. (b)  5M + 20W = 6M + 8W


12W = M
A  11 (20)
W 1
B  20 (11) (55 × 4) units =
M 12
 220 units
C  55 (4) 13
Total work =
2
 36 + 4 
working fashion  (A = B , C + B)
2 days  31 + 15 = 46 units = 260 units
8 days  184 units 260
9 days  184 + 31 = 215  13 women  = 20 days
13 ×1
1
rd
22. (d)
10 days  days  220 units 25M + 45W  15 days
3
 5M + 9W  75 days
1 15 M + 60W  20 days
Total time = 9 dasy
3  1M + 4W  300 days

r
19. (b)  300(1M + 4W) = 75(5M + 9W)

si
Let there be 'x' men initial with efficiency '1'  4M + 16W = 5M + 9W
of each man.  7W = M
an by
 First day  x × 1 units
II day  (x – 18) units
W 1
M 7
=

n
VIII day  (x – 126) units
Total work = 300 × (7 + 4)
also, total work = 6x
ja = 3300 units
 x + (x – 18) +....+ (x – 126) = 6x
R s
3300
 8x – 18 – 36-------126 = 6x  69 man + 67 women =
69 × 7 + 67
a th

 8x – 18(1 + 2 +....+ 7) = 6x
= 6 days
 8x – 18 × 7 × 4 = 6x
23. (d)
 2x = 18 × 28
 x = 252. M1 × D1 × H1 M2 × D2 × H2
=
ty a

20. (a) W1 W2

A  10 (8)
di M

90 × 200 M 2 × 400
80 units  =
1 5
B  16 (5) 6 6
Working fashion  (A, B, A, B,....) M2 = 225
2 days  13 units Extra people = 225 – 90
12 days  78 units = 135
24. (c)
1 12M + 12W  9 days
13 days  days  80 units
4 1M + 1W  108 days
also, 6M + 15W  12
1
 Total time = 12 days 2M + 5W  36 days
4
 108 (1M + 1W) = 36(2M + 5W)
A

21. (a) 3M + 3W = 2M + 5W
65 M = 2W
1M + 4W = days
4 M 2
=
13 W 1
3M + 4W = days
2 Total work = 36 (4 + 5)
= 36 × 9 = 324
65 13
1M + 4W  = 3M + 4W  162
4 2 9 Women  = 18 days
9

Aditya Ranjan (Excise Inspector) Selected Selection 78


https://sscstudy.com/
Join Telegram- Maths by Aditya Ranjan TIME & WORK

25. (d)
36
Let 4 men leave after x days. = 18 days.
A.T.Q. 2
10x + 6(50 – x) = 40 × 10 30. (c)
 10x + 300 – 6x = 400 20 Men  220 days
 x = 25 days A.T.Q.
26. (a) (20 × 90) + (40 × x) = 220 × 20
40x = 4400 – 1800
A  6 (8)
40x = 2600
48 units
x = 65
V  16 (3)
31. (b)
Working fashion  (A, V, A, V....)
2 hours  11 units 40
Ab + An 
8 hours  44 units 9
1 1 60
9 hours  hours  48 units 2Ab + An 
2 3 17

1 40 60  1 

r
 Total time = 8 hours  (Ab + An) =  2Ab + An 
2 9 17 3

si
27. (a)
2 2 6 1
an by
R –20 (6) Ab + An = Ab + An
9 9 17 17
S – 30 (4) 120 units 2 1 
An
 6 2
Ab

n
 –  = –
 17 9 
V – 40 (3) 9 17 
ja
Working fashion  (R + S, R + V)
25An = 20Ab
R s
2 days  10 + 9 = 19 units An 4
=
a th

12 days  114 units Ab 5


6 40
13 days  days  120 units Total work = (9) = 40 units
10 9
ty a

3 40
Total time 12 days Abha  = 8 days
5 5
di M

28. (c) 32. (c)


A  13 (17) P + Q  36
13 × 17 = 221 units P:Q
B  17 (13) 3:1
Working fashion  (A, B, A, B,....) Let total work = 4 × 36
2 days  30 units 4 × 36
14 days  210 units Q=
1
11  144 days
15 days  day  221 units
17 33. (b)

11 A  6 (5)
Total time  14 days
A

17
B  10 (3) 30 units
29. (a)
C  15 (2)
A + S + R  4 (9)
A.T.Q.
A  9 (4) 36 2(A + B + C) + x(A + B) = 30
20 + 8x = 30
S  12 (3)
8x = 10
R  (2) 5 1
x= = 1 days
 Rajani will complete the work in 4 4

Aditya Ranjan (Excise Inspector) Selected Selection 79


https://sscstudy.com/
Join Telegram- Maths by Aditya Ranjan TIME & WORK

34. (a) 39. (d)


Anju : Bitt A : B
6 : 5
13 : 10
Total work = 29 × 6
Let, total work = 23 × 13
Together they can do work in:-
23 × 13
A+B = 13 days 29 × 6 174
23 = days
11 11
35. (a)
40. (b)
M1 × D1 M2 × D2 A + B + C  10
=
W1 W2 B : A+C
Efficiency 1 : 3
10 25
 240 = W Total work = 40 units
2
40
W2 = 24 × 25  B will do in  = 40 days
= 600 1

r
36. (d) 41. (b)

si
R  15 (20) A  6 (5)
an by 30
S  20 (15) 300 units B  10 (3)
Working fashion  (A, B, A, B,....)

n
M  25 (12)
2 hours  8 units
Working fashion  (R, S, M, R, S, M....)
ja
3 days  47 units
6 hours  24 units
R s
18 days  282 units 7 hours  29 units
a th

18 1
19 days  days  300 units 8 hours  hours  30 units
20 3

9 1
 Total time 18 days
ty a

Total time  7 hours  work will finish


10 3
37. (c ) 42. (a)
di M

A B P  32
14 10 8
In 8 days, Pawan can do =
 7 : 15 32
Let, total work = 36 × 5 th
1
36 × 5 = work
Both will take = = 15 days 4
12
th
38. (a) 3
 work by Sandeep in 27 days
4
A  12 (10)
Fill work by Sandeep in 36 days
A

B  20 (16) 120 units


P  32 (9)
C  24 (5) 72 × 4
S  36 (8) = 288 units
Let, they did not leave and do some extra work
 Extra work = 10 + 18 + 20
288
= 48 units Time =
 Total work by A, B and C = 120 + 48 9+8

168 288 16
 Time taken = = 8 days = = 16 days.
21 17 17

Aditya Ranjan (Excise Inspector) Selected Selection 80


https://sscstudy.com/
Join Telegram- Maths by Aditya Ranjan TIME & WORK

43. (d) 45. (b)


Ratio of efficiency of 3 taps:- A + B  48
(2)² : (3)² : (4)² A B
4 1
 4 : 9 : 16
Let, total work = 5 × 48
Total efficiency = 4 + 9 + 16 = 29
5  48
Given, tap with eff. 16 takes 81 min.  B = 240 days
1
Let, total work = 16 × 81
46. (c)
16  81 1296 20 P5 6
Total time = = = 44
29 29 29
Q  10 3 30 Units
44. (b)
R  15 2
A  32
P + Q, Q + R, P + Q, ..........
7

r
224 Units 2 days  17 units
8

si
B  28 3 days  26 units
an by
2 hr.  15 units
28 hrs.  210 units
1
2
days  30 units

n
29 hrs.  217 units Total time = 3 1 = 7 days
ja 2 2
R s
7 47. (b)
hrs.  224 units
8
a th

We know,

7 M1  D1  H1 M2  D2  H2
 Total time  29 hrs. =
8 W1 W2
ty a

 30 hrs. 10 × 6× D1 = 12 × H1 × D1
 5 = H1
di M

......-------......
A

Aditya Ranjan (Excise Inspector) Selected Selection 81


https://sscstudy.com/
Join Telegram- Maths by Aditya Ranjan TIME & DISTANCE

TIME & DISTANCE


11

1. A truck runs 492 km on 36 Litter of diesel. 97 m
How many kilometres can it run on 33 Litter
of diesel? 21 m/sec
492 km 23 m/sec
km
SSC CGL 01/12/2022 (Shift- 04) SSC CGL 02/12/2022 (Shift- 01)
(a) 40 sec (b) 45 sec
(a) 454 km (b) 453 km (c) 62.5 sec (d) 48.5 sec

r
(c) 451 km (d) 452 km 1
2. If Ram covers a certain journey at 60% of his 5. If a man travels at km/h on a journey and
x

si
usual speed, he reaches the destination late
1
by 36 min. His usual time (in min) to reach
an by returns at km/h, then his average speed
the destination is: x2
for the journey is:

n

1
km/h
ja x

R s
1
km/h
a th

x2
SSC CGL 01/12/2022 (Shift- 04)
(a) 60 (b) 72 SSC CGL 02/12/2022 (Shift- 03)
(c) 50 (d) 54
ty a

x +1 2
3. Two-thirds of a certain distance was covered (a) (b)
x2 x + x2
at the speed of 45 km/h, one-fourth at the
di M

1 2x 2
speed of 60 km/h and the rest at the speed (c) (d)
of 75 km/h. Find the average speed per hour x +x 2
x +1
for the whole journey. (correct to 2 decimal 6. A car starts from point A towards point B, travelling
places) 1
at the speed of 20 km/h 1 hours later, another
45 km/h 2
car starts from point A and travelling at the speed
60 km/h
1
75 km/h of 30 km/h and reaches 2 1hours before the
2
first car. Find the distance between A and B.
A B 20 km/h
A

SSC CGL 01/12/2022 (Shift- 04) 1


1
(a) 51.25 km/h (b) 45.75 km/h 2
(c) 58.25 km/h (d) 49.77 km/h A 30 km/h
4. A thief is noticed by a policeman from a 1
2 A
distance of 97 m. The thief starts running and 2
the policeman chases him. The thief and the B
policeman run at a speed of 21 m/sec and SSC CGL 03/12/2022 (Shift- 01)
23 m/sec respectively. What is the time taken (a) 300 km (b) 240 km
by the policeman to catch the thief? (c) 260 km (d) 280 km

Aditya Ranjan (Excise Inspector) Selected Selection 82


https://sscstudy.com/
Join Telegram- Maths by Aditya Ranjan TIME & DISTANCE

7. A thief steals a bike at 12:30p.m. and drives 450 m


it at 48km/h. But the theft is discovered after
half an hour. The bike owner starts to chase
him on another bike at 58km/h. The thief will

be caught at ________. (km
)
12:30p.m.
48km/h SSC CGL 05/12/2022 (Shift- 01)
58 km/ (a) 3.75 (b) 3.4
h (c) 3.6 (d) 3.15
11. R jogs at twice the speed of walking and runs
SSC CGL 03/12/2022 (Shift- 02) at twice the speed of jogging. From his home
(a) 3:40p.m. (b) 3:54p.m. to office, he covers half of the distance by
(c) 3:10p.m. (d) 3:24p.m. walking and the rest by jogging. From his
8. Vikas covered a certain distance by bike. If office to home, he covers half the distance
he covers 40% of the distance at 40 km/h, jogging and the rest by running. What is his

r
50% of the distance at 25 km/h and the average speed (in km/h) in a complete round
from his home to office and back home if the

si
remaining 10% distance at 10 km/h. Find his
average speed over the whole distance. distance between his office and home is 10
an by
km and he walks at the speed of 5 km/h?

40 km/h 25 R

n
km/h 10 km/h

ja
R s

SSC CGL 03/12/2022 (Shift- 02)
a th

(a) 25 km/h (b) 28 km/h


(c) 26 km/h (d) 30 km/h
9. A chain Snatcher appears at a distance of 500 (km/h)
ty a

u on a signal. He starts chasing the running


10 km 5 km/h
chain Snatcher with a speed of 5 m/s while

di M

Snatcher follows it with a speed of 15 m/s.


What is the distance covered by the chain SSC CGL 05/12/2022 (Shift- 01)
Snatcher when the Policeman of the chain
Snatcher is caught? 90 60
(a) (b)
500 m 8 8
5 m/s 60 80
15 m/ (c) (d)
9 9
s
12. A person travels from P to Q at a speed of
50 km/h and returns by increasing his speed
by 60%. What is his average speed for both
A

SSC CGL 03/12/2022 (Shift- 03) the trips?


(a) 200 m (b) 180 m P Q 50 km/h
(c) 150 m (d) 250 m 60%
10. A policeman saw a thief from a distance of

450 m. When the policeman started chasing
him, the thief also started running. The ratio
of speeds of the thief to the policeman is 7 : SSC CGL 05/12/2022 (Shift- 03)
8. After running how much distance (in km) (a) 36.61 km/h (b) 61.53 km/h
can the policeman catch the thief?
(c) 31.56 km/h (d) 16.53 km/h

Aditya Ranjan (Excise Inspector) Selected Selection 83


https://sscstudy.com/
Join Telegram- Maths by Aditya Ranjan TIME & DISTANCE

13. Which of the following is NOT a correct 280 m


statement? 10 km/h
24 km/h
SSC CGL 05/12/2022 (Shift- 03)

(a) The speed of 20 m/s is less than the speed
of 85 km/h. SSC CGL 06/12/2022 (Shift- 02)
20 m/s 85 km/h (a) 1.4 min (b) 1.5 min
(b) Time may be calculated by dividing the (c) 1.2 min (d) 1.3 min
distance by the speed. 17. A is chasing B in the same interval of time.
A jumps 8 times, while B jumps 6 times. But
the distance covered by A in 7 jumps is the
(c) Covering the same distance in lesser time same as that of B in 5 jumps. The ratio
implies a higher speed. between the speeds of A and B is ________.
A B A8
B 6 A

r
(d) The speed of 99 km/h is less than the 7 B 5

si
speed of 24 m/s.
A B
99 km/h 24 m/s
an by
14. Prasad goes 96 kilometres on a bike at a speed
of 16 km/h, 124 kilometres at 31 km/h in a SSC CGL 08/12/2022 (Shift- 01)

n
car, and 105 kilometres at 7 km/h in a horse (a) 32 : 24 (b) 30 : 56
cart. Find his average speed for the entire (c) 20 : 21 (d) 28 : 36
ja
distance travelled.
R s
18. John drives 250 km at 50 km/h and then he
16 km/h
drives 350km at 70km/h. Find his average
a th

31 km/h speed for the whole journey in km/h.


7 km/h 250 km 50 km/h
350 km
ty a

70 km/h
SSC CGL 05/12/2022 (Shift- 04)
( km/h )
di M

(a) 16 km/h (b) 13 km/h


(c) 17 km/h (d) 11 km/h SSC CGL 08/12/2022 (Shift- 02)
15. Two tourist buses start from the same point (a) 55 km/h (b) 60 km/h
and move along two roads at right angles at (c) 58 km/h (d) 65 km/h
speeds of 48 km/h and 36 km/h, respectively.
19. A thief stole jewellery from a shop at 8:15
The distance between the buses after 15
p.m. and left on a bike at a speed of 60 km/
seconds is ________.
h. The police were informed at 8:30 p.m. If
the police want to arrest the thief at 9:00 p.m.,
48 km/h what should be the minimum speed of the
36 km/h police jeep?
A

8:15 p.m.
SSC CGL 05/12/2022 (Shift- 04) 60 km/h
(a) 175m (b) 350m 8:30 p.m.
(c) 250m (d) 150m
9:00 p.m.
16. A dog saw a cat at a distance of 280 m. The
cat started running at the speed of 10 km/h
and the dog also ran to catch it with the speed SSC CGL 08/12/2022 (Shift- 04)
of 24 km/h. How much time will the dog take (a) 60 km/h (b) 75 km/h
to catch the cat?
(c) 80 km/h (d) 90 km/h

Aditya Ranjan (Excise Inspector) Selected Selection 84


https://sscstudy.com/
Join Telegram- Maths by Aditya Ranjan TIME & DISTANCE

20. Two cities A and B are 135 km apart on a straight 'x' km/h 10
track. One car starts from A at 8 a.m. and travels km/h
towards B at 25 km/h. Another car starts from

B at 9 a.m. and travels towards A at a speed of
30 km/h. At what time will they meet?

('x'
)
A B 135 km SSC CGL 09/12/2022 (Shift- 03)
A 8 a.m. (a) 4 km/h (b) 8 km/h
B 25 km/h (c) 10 km/h (d) 6 km/h
9 a.m. B 24. A person covers a certain distance by bus at 45
km/h and immediately returns to the starting
A 30 km/h
point by car at a speed of 80 km/h. What is his
average speed during the whole journey?
SSC CGL 09/12/2022 (Shift- 01) 45 km/h
(a) 11 a.m. (b) 10 a.m. 80 km/
h
(c) 10:45 a.m. (d) 11:30 a.m.

r

21. A person travels in a car at 40 km/h for 3

si
hours, on a bike at 30 km/h for 2 hours, and SSC CGL 09/12/2022 (Shift- 03)
(a) 57.6 km/h (b) 63.2 km/h
in a train at 80 km/h for 5 hours. What is
an by (c) 73.5 km/h (d) 45.5 km/h
the average speed at which he travelled?
25. A jeep travels to a place 300 km away at an
40 km/h

n
average speed of 60 km/h and returns at a
30 km/h 80 speed of 30 km/h. What is the average speed
ja (in km/h) for the whole journey?
km/h 5
R s
300 km 60 km/

h 30 km/h
a th

SSC CGL 09/12/2022 (Shift- 02)



(a) 56 km/h (b) 60 km/h (km/h )
(c) 62 km/h (d) 58 km/h SSC CGL 12/12/2022 (Shift- 02)
ty a

22. Menu and Daya travel from point A to B, a (a) 75 (b) 80


distance of 105 km, at speeds of 10 km/h and (c) 40 (d) 60
di M

25 km/h, respectively. Daya reaches point B 26. Some students went on a field trip. At a speed
first and returns immediately and meets Menu of 6 km/h, they travelled by bikes for 2 hours;
then at a speed of 2 km/h, they walked for another
at point C. Find the distance from point A to
hour; then they took rest for an hour and had
point C.
lunch for half an hour; then at a speed of 1 m/
10 km/h 25 km/h s, they walked for an hour to visit a garden; and
A B 105 km finally at a speed 4 km/h, they returned home
in 3 hours. Their average speed, in km/h, (rounded
B
off to two places of decimal) is:
C A C
6 km/h
2 km/
A

SSC CGL 09/12/2022 (Shift- 02) h


(a) 35 km (b) 60 km
(c) 45 km (d) 62 km 1 m/s
23. A thief running at speed of ‘x’ km/h is chased 4 km/h
by a policeman running at a speed of 10 km/ km/h
h. If the thief is ahead by 100 metres, the
policeman catches the thief after 3 minutes. SSC CGL 12/12/2022 (Shift- 03)
At what speed is the thief running (‘x’ being (a) 3.48 (b) 129.23
the unknown speed)? (c) 0.28 (d) 251.62

Aditya Ranjan (Excise Inspector) Selected Selection 85


https://sscstudy.com/
Join Telegram- Maths by Aditya Ranjan TIME & DISTANCE

27. Ram starts from point A at 8 a.m. and reaches


point B at 2 p.m. on the same day. On the same
day, Raju starts from point B at 8 a.m. and
reaches point A at 6 p.m. on the same day. Both

points A and B are separated by only a straight
line track. At what time they both meet?
A 8 a.m. SSC CGL 13/12/2022 (Shift- 02)
B 2 p.m. (a) 1.5 km/h (b) 3 km/h
(c) 2 km/h (d) 4 km/h
B 8 a.m.
30. A man can row 10 km/h in still water. When
A 6 p.m.
the river is running at a speed of 4.5 km/h,
A B then it takes him 2 h to row to a place and
comes back to the initial point . How far is
SSC CGL 12/12/2022 (Shift- 03) the place (in km) (rounded off to two decimal
places)?
(a) 11:45 a.m. (b) 9:42 a.m.
(c) 10:42 a.m. (d) 12:42 p.a.
10 km/h

r
4.5 km/h
28. A policeman spotted a thief 40 metres ahead

si
of him. The moment they both saw each other
they started running in the same direction
an by
on the same track. The thief was running at
10 m/sec and the policeman was chasing him
at the speed of 12 m/sec. How much distance
km

n
(in metres) the policeman needed to cover to SSC CGL TIER - II 02/03/2023
catch the thief?
ja (a) 5.50 (b) 8.98
R s

(c) 7.98 (d) 6.25

a th

10 m/sec 31. Reshma travels 45 km at 15 km/h by cycle,


80 km at 40 km/h by car and the remaining 6
12 m/sec
km by foot at 2 km/h. Find his average speed
for the whole journey.(Rounded off to 2
ty a

decimal places)
SSC CGL 12/12/2022 (Shift- 04) 45 km 15 km/h
di M

(a) 180 (b) 240 80 km 40 km/h


(c) 200 (d) 225
29. A student walked 6 km from his house to
6 km 2 km/h
reach the metro station, then boarded a metro
that has an average speed of 60km/h, and
reached the destination. It took 3 hours for SSC CGL TIER- II 06/03/2023
the entire journey. If the average speed of the
entire journey is 32 km/h, then the speed of (a) 15.25 km/h (b) 16.38 km/h
walking is ________. (c) 18.36 km/h (d) 43.50 km/h

ANSWER KEY
A

1.(c) 2.(d) 3.(d) 4.(d) 5.(b) 6.(b) 7.(d) 8.(a) 9.(d) 10.(c)

11.(d) 12.(b) 13.(d) 14.(b) 15.(c) 16.(c) 17.(c) 18.(b) 19.(d) 20.(a)

21.(d) 22.(b) 23.(b) 24.(a) 25.(c) 26.(a) 27.(a) 28.(b) 29.(d) 30.(c)

31.(b)

Aditya Ranjan (Excise Inspector) Selected Selection 86


https://sscstudy.com/
Join Telegram- Maths by Aditya Ranjan TIME & DISTANCE

SOL U TION S
1. (c) 36 ltr.  492 km. 2
492  x³
33 ltr.   33 x 1
36

= 41 × 11
2 x²
= 451 km.  
x³ (x  1)
1
2. (d) Speed  2
Time 
x (x  1)
Usual New 6. (b) A B

r
Speed 5 3 20 km/hr.

si
1
Let AB = x kmr. in 1 hrs. first can travel
Time an by3 5 2

2 3
20  = 30 km.

n
2  36 min. 2
ATQ,
3  54 min.
ja x – 30 x 5
R s
3. (a) Let, Total Distance = 12 × 900 – =
20 30 2
a th

12  900 2
 T1 =  = 160 3x – 90 – 2x 5
45 3 =
60 2
12  900 1  x = 240 km.
T2 =  = 45
ty a

60 4 48
7. (d) In how hover thief will cover = = 24 km.
2
12  900 1
di M

T3 =  = 12 Relative speed = (58 – 48) km/hr.


75 12
= 10 km/hr.
12  900 24
 Average Speed =  Time = = 2.4 hr.
160  45  12 10
= 2 hr. 24 min.
12  900
=  at 1 : 00 + 2 hr 24 min.
217  03 : 24 pm.
= 49.76 km/hr. 8. (a) Let, Total distance = 100 km.
4. (d) Relative speed = (23 – 21) m/sec. 40
 T1 = = 1hr.
= 2m/sec. 40
A

97 50
 Time = = 48.5 sec . T2 = = 2 hr.
2 25

2 a  b 10
5. (b) Average speed = T3 = = 1hr.
ab 10
100
1 1 Average speed =
2  T1  T2  T3
 x x²
1 1 100
 = = 25 km/hr.
x x² 4

Aditya Ranjan (Excise Inspector) Selected Selection 87


https://sscstudy.com/
Join Telegram- Maths by Aditya Ranjan TIME & DISTANCE

9. (d) Relative speed = (15 – 5) m/s


5 50
= 10 m/s 13. (d) (a) 20  =  85. km/hr.
18 9

 Snatcher will be caught after = 500 D


10 (b) T =
S
= 50 sec.
In 50 sec, distance covered by chain snatcher 1
(c) Time 
= (50 × 5) = 250 is. Speed
10. (d) Relative speed = (8 – 7) = 1x
18
450 450 (d) 99  24 m/s.
 Time = = sec . 5
1x x
Total distance
 Distance covered by thief in 450 sec. 14. (b) Avg. speed =
Total time
450
=  7x
x 96  124  105

r

= 3150 m 96 124 105
 

si
16 31 7
 3.15 km.
11. (d) Walking
an by Jogging Running 325 325
Speed 5 10 20  = = 13
6  4  15 25

n
10 km. 15. (c)
48 km/hr.
ja
R s
Home K Office
36 km/hr.
a th

Hk ko 1
T1 = = 1, T2 = =
5 10 2

ok 1 kh 1
T3 = = , T4 = =
ty a

10 2 20 4
di M

Total distance
Avg. speed =
Total time Let, Bus 1 travels D1 km. in 15 sec. and Bus 2
travels D2 km. in 15 sec.
20
=
1 1 1 5
1    D1 = 48 ×  15  200 m.
2 2 4 18

20 5
= D2 = 36   15 = 150 m.
1 18
2
4 Distance b/w D1 and D2 :-
80 = (200)²  (150)²
=
A

9
= 250
8 16. (c) Relative Speed = 24 – 10
12. (b) S1 = 50 S2 = × 50
5 = 14 km/hr.
= 80 280  18
Time = = 72 sec .
2  S1 S2 14  5
Avg. speed =
S1  S2 72
 = 1.2 min .
= 61.53 km/hr. 60

Aditya Ranjan (Excise Inspector) Selected Selection 88


https://sscstudy.com/
Join Telegram- Maths by Aditya Ranjan TIME & DISTANCE

A B 22. (a) 105


17. (c)
1 1
Time taken for 1 jump A B
8 6 C
x x x
Distance of 1 jump
7 5
ATQ,
Where, x is the distance covered in 7 jumps
and 5 jumps by A and B respectively Distance travelled by Meenu = 105 – x
Distance travelled by Daya = 105 + x
8 6
 Speed :  40 : 42
7 5 105 – x 105  x
 =
20 : 21 10 25
 105 × 5 – 5x = 105 × 2 + 2x
18. (b) Avg. speed = Total distance
Total time  105 × 3 = 7x
 x = 45
250  350

r
=  AC = 105 – 45
250 350
 = 60 km.

si
50 70
23. (b) Relative speed = (10 – x) km/hr.
600 an by
= = 60 km/hr. ATQ,
5 5
100  18

n
19. (d) Distance travelled by thief in 15 min. = 3  60
(10 – x)  5
1
 60 × ja= 15 km.
4  10 – x = 2
R s
ATQ,  x = 8 km/hr.
a th

1 15 2 × S1 × S2
= 24. (a) Avg. speed =
2 (P– 60) S1 + S2
30 = P – 60
ty a

2  45  80
P = 90 km/hr. =
45  80
di M

C = 57.6 km/hr.
20. (a) A B

25 Total distance
25. (c) Avg. speed =
Relative speed = 25 + 30 Total time
= 55 km/hr. 300  300 600
Distance = 135 – 25 = 110 km. = =
5  10 15
110 = 40 km/hr.
 Time = = 2 hrs.
55
Total distance
i.e. at 11 am. 26. (a) Avg. speed =
Total time
21. (d) Avg. speed = Total distance
A

Total time 18 


(6  2)  (2  1)  
  1
  (4  3)
 5 
(40  3)  (30  2)  (80  5) 
= 1
3 2 5 2 11 3 1
2
120  60  400
=
10 148  2 296
 =
5  17 85
580
=  58 km/hr. = 3.48 km/hr.
10

Aditya Ranjan (Excise Inspector) Selected Selection 89


https://sscstudy.com/
Join Telegram- Maths by Aditya Ranjan TIME & DISTANCE

27. (a)
3
A B  he walked of hrs.
2
A B
Time 6 10 62
 Walking speed =
Speed 10x : 6x 3
 Relative speed = 10 + 6 = 16x = 4 km/hr.
Distance = 10 × 6 = 60x 30. (c)
Time at which they meet:- x = 10 km/hr., y = 4.5 km/hr.

60 15 3 D D
= = = 3 hrs.  =2
16 4 4 10  4.5 10 – 4.5
 3 hrs. 45 min.  1 1 
 at 11 : 45 am. D  =2
14.5 5.5 

28. (b)

r
Relative speed = 12 – 10 2
D=  14.5  5.5
= 2 m/sec. 20

si
= 7.98
40
an by
Time = = 20 sec . 31. (b)
2
Total distance

n
 distance by police = 20 × 12 Average speed =
= 240 m. Total time
29. (d) ja
R s
45  80  6
=
6 km. x km. 45 80 6
 
a th

15 40 2
ATQ,
131 131
32 × 3 = 6 + 60 × t = = = 16.38
3 23 8
ty a

3
 t= = 1.5 hr.
di M

2
......-------......
A

Aditya Ranjan (Excise Inspector) Selected Selection 90


https://sscstudy.com/
Join Telegram- Maths by Aditya Ranjan TRAIN

TRAIN
12

1. The distance covered by a train in (5y – 1) H 6:30 K
hours is (125y³ – 1) km. The speed of the train

is:
7:30 K H
(5y – 1)
(125y³ – 1) km

SSC CGL 02/12/2022 (Shift- 02)
J 11:30
(a) (5y³ – 1) km/h HJ KJ
(b) (25y² – 5y + 1) km/h SSC CGL 06/12/2022 (Shift- 04)

r
(c) (5y + 1) km/h (a) 25 : 16 (b) 5 : 16
(d) (25y² + 5y + 1) km/h

si
(c) 36 : 25 (d) 31 : 19
2. Two trains P and Q start from stations S and
an by
T towards each other. Train P takes 4 hours
48 minutes and train Q takes 3 hours 20
5. Two trains are moving in the same direction
at the speed of 36 km/hr and 48 km/hr. The

n
minutes to reach T and S, respectively, after time taken by faster train to cross a man
they meet. If the speed of train P is 45 km/ sitting in the slower train is 33 seconds. What
h, what is the speed of train Q?
ja will be the length of the faster train?
R s
P Q, S T


a th


T S P

Q
P 45 km/h Q

ty a

SSC CGL 02/12/2022 (Shift- 04)



di M

(a) 48 km/h (b) 50 km/h SSC CGL 07/12/2022 (Shift- 04)


(c) 54 km/h (d) 55 km/h (a) 770 metres (b) 90 metres
3. With an average speed of 45 km/h, a train
(c) 110 metres (d) 180 metres
reaches its destination on time. If it goes with
an average speed of 30 km/h, it is late by 6. Two trains are moving in the opposite
15 minutes. The total journey is: direction at the speed of 48 km/hr and 60
45 km/h km/hr respectively. The time taken by the
30 km/h slower train to cross a man sitting in the faster
train is 12 seconds. What is the length of the

slower train?

SSC CGL 03/12/2022 (Shift- 04)
A

(a) 20.5 km (b) 40.5 km


(c) 22.5 km (d) 30.5 km
4. From point H, at 6 : 30 pm, a train starts
moving towards point K at the speed of 90
km/hr. Another train starts moving from point
K at 7:30 p.m. towards point H at the speed SSC CGL 08/12/2022 (Shift- 02)
of 72 km/hr. Both the train meet at 11:30
(a) 480 metres (b) 720 metres
p.m. at point J. What is the ratio of the
distance HJ and KJ? (c) 180 metres (d) 360 metres

Aditya Ranjan (Excise Inspector) Selected Selection 91


https://sscstudy.com/
Join Telegram- Maths by Aditya Ranjan TRAIN

7. A train travels at a speed of 18 m/s in the 9. Two stations R and S are 400 km apart from
first 5 minutes, covers 7.5 km in the next 10 each other. A train leaves from R to S and
minutes and covers 12 km in another 10 simultaneously another train leaves from S to
minutes. What is its average speed (in km/h, R. Both trains meet after 10 hours. If the speed
rounded off to 1 decimal place) for the entire of the first train is 4 km/hr more than the second
journey?
train, then what is the speed of the slower train?
18 m/s
R S
7.5 km
12 km R S
(km/h S R
) 10
SSC CGL 13/12/2022 (Shift- 03)
(a) 59.8 (b) 71.2

(c) 45.7 (d) 64.6 SSC CGL 13/12/2022 (Shift- 04)
8. Train A running at 81 km/h takes 72 sec to (a) 18 km/h (b) 26 km/h

r
overtake train B, when both the trains are (c) 16 km/h (d) 22 km/h

si
running in the same direction, but it takes
36 sec to cross each other if the trains are 10. A goods train 350 m long passes through a
an by
running in the opposite direction. If the length
of train B is 600 metres, then find the length
of train A. (in metres)
tunnel of length 1250 m in 80 seconds. What
is the speed of the train?

n
350 m 1250 m
81 km/h A, B
ja
72 sec
R s
SSC CGL TIER- II 07/03/2023
a th


(a) 64 km/h
36 sec B (b) 56 km/h
A
(c) 78 km/h
ty a

SSC CGL 13/12/2022 (Shift- 03)


(a) 600 (b) 480 (d) 72 km/h
di M

(c) 590 (d) 900

ANSWER KEY
1.(d) 2.(c) 3.(c) 4.(a) 5.(c) 6.(d) 7.(a) 8.(b) 9.(a) 10.(d)
A

Aditya Ranjan (Excise Inspector) Selected Selection 92


https://sscstudy.com/
Join Telegram- Maths by Aditya Ranjan TRAIN

SOL U T ION S
1. (d) I train travelled for 5 hours before meeting
Distance = (125y³ – 1³)  Distance by P = 90 × 5 = 450
Time = (5y –1) = HJ
II Train travelled for 4 hours before meeting
Distance
Speed =  Distance by  = 72 × 4
time
= 288 km = KJ
125y – 1
3 3
 HJ : KJ = 450 : 288
=
5y – 1 = 25 : 16
5. (c)

r
5y 3 – 13 Relative speed = 48 – 36
=

si
5y – 1
= 12 km/hr
= (25y² + 5y + 1) km/hr
an by Length of faster train
2. (c)  = time
Relative speed

n
We know,
5
=
ja
Speed of P time by Q after meet  Length = 33 × 12 ×
R s
Speed of Q time by p after meet 18
= 55 × 2 = 110 metre.
a th

48 6. (d)
4 10 × 5
45 60 Relative speed = 48 + 60
 = =
Q 20 3 × 24 = 108 km/hr
ty a

3
60
5
di M

Length = 12 × 108 ×
18
25
= = 360 metre.
36
7. (a)
45 5 Distance1 = 18 × 5 × 60
=
Q 6
= 5400 metre.
 Q = 54 km/hr = 5.4 km
3. (c) Distance2 = 7.5 km
We know, Distance 3
= 12 km

S1 × S2 Total Distance
A

× t = Distance  Average speed =


S1 – S2 Total Time

45 × 30 15 5.4 + 7.5 +12


 × = 22.5km. = × 60
45 – 30 60 5 +10 +10
4. (a)
24.9
= × 60
90 km/hr 72 km/hr 25
H J K = 59.9 km/hr

Aditya Ranjan (Excise Inspector) Selected Selection 93


https://sscstudy.com/
Join Telegram- Maths by Aditya Ranjan TRAIN

8. (b) 9. (a)
(A – B)× 72 = LA + 600
400 km
5
 (81 – B)× 72 = LA + 600 ...(1) R S
18
Let, speed of B = x
Also,
Speed o A = 4 + x
5 A.T.Q.
(A + B) × 36 = LA + 600 ...(2)
18
400
From (1) and (2)  (x + 4 + x) =
10
5 5 = 2x = 40 – 4
× (81 – B)72 = (81 + B) 36 ×
18 18 x = 18
 81 × 2 – 2B = 81 + B  Speed of slower train = 18 km/hr
3B = 81 10. (d)
81
B= = 27 Length of train + tunnel
3 Speed =

r
Time
 Put B in (1)

si
350  1250 1600
5 = =
 (81 – 27) × 72 ×
an by = LA + 600 80 80
18
 1080 – 600 = LA = 20 m/s.

n
 LA = 480 mtr. 18
d  20 × = 72 km/hr.
5
ja
R s
......-------......
a th
ty a
di M
A

Aditya Ranjan (Excise Inspector) Selected Selection 94


https://sscstudy.com/
Join Telegram- Maths by Aditya Ranjan RACE

RACE
13

1. In a 1500 m race, Anil beats Bakul by 150 SSC CGL 03/12/2022 (Shift- 01)
m and in the same race Bakul beats Charles
by 75 m. By what distance does Anil beat 17 26
(a) m / sec (b) m / sec
Charles? 3 9
1500 m 150 m
17 26
75 m (c) m / sec (d) m / sec
9 3

5. In a linear race of 1000 m, A beats B by 50
SSC CGL 01/12/2022 (Shift- 01)

r
m or 5 seconds. What is the difference between
(a) 217.50 m (b) 200.15 m the speeds (in m/s) of A and B?

si
(c) 293.50 m (d) 313.75 m
2. In a 400-metre race, A runs at a speed of 16
an by 1000 m A, B 50 m 5
m/sec. If A gives B a start of 15 metres and
still beats him by 10 sec, then what is the A B (m/s )

n
speed of B?
ja
A 16 SSC CGL 06/12/2022 (Shift- 03)
R s
A, B
10 10
a th

B (a) 1 (b)
19 19

SSC CGL 02/12/2022 (Shift- 02) 9 9
(c) (d)
ty a

(a) 11 m/sec (b) 10 m/sec 19 10


(c) 13 m/sec (d) 9 m/sec 6. In a race, there are 4 members in a team. Each
di M

3. In a 100-m race, A beats B by 20 m and B member has to cover 5 km one after another.
beats C by 20 m. By how much distance does If the total time taken is 30 minutes, then
A beat C. what would have been the average speed?
100 m A, B 20 m 4
B, C 20 m A, C 5 km
30
SSC CGL 02/12/2022 (Shift- 03) SSC CGL 06/12/2022 (Shift- 04)
(a) 64 m (b) 24 m
(c) 25 m (d) 36 m km km
(a) 50 (b) 40
hr hr
4. In a 1200 m race, bike A beats bike B by 100
A

m. Bike B beats bike C by 100 m in a 600 m


m m
race. If bike A beats bike C by 30 sec in a (c) 50 (d) 40
sec sec
720 m race, then what is the speed of bike C?
1200 m A, B 100 m 7. In a 1500 m race, if vehicle P gives vehicle
Q a start of 200 m, then vehicle P wins the
600 m B, C
race by 8 sec. Alternatively, if vehicle P gives
100 m A 720 m vehicle Q a start of 400 m, the race ends in
C 30 a dead heat. How long does vehicle P take to
C run 1500 m?

Aditya Ranjan (Excise Inspector) Selected Selection 95


https://sscstudy.com/
Join Telegram- Maths by Aditya Ranjan RACE

1500 m P Q 200 9. In a 1000 m race, Arjun, Balaji and Charan


m P, 8 are running. Arjun beats Balaji by 100 m, and
Balaji beats Charan by 100 m. In the next
P Q 400
1000 m race (the speeds are the same as in
m
the previous), Balaji gives Charan a head start
P 1500 m of 100 m, and Arjun gives Balaji a head start
SSC CGL 07/12/2022 (Shift- 01) of 100 m. Find the distance by which the
(a) 44 sec (b) 45 sec winner is ahead of the person just behind him.
(c) 40 sec (d) 60 sec
1000 m
8. In a linear race of 500 m, A can beat B by
50 m and in a race of 600 m, B can beat C 100 m
by 60 m. By how many metres will A beat C 100 m 1000 m
in a race of 400 m? 100
500 m A, B 50 m
m 100
600 m B, C 60
m

r
m 400 m A, C

si
SSC CGL 13/12/2022 (Shift- 01)
SSC CGL 08/12/2022 (Shift- 03)
(a) 70 an by (b) 68 (a) 100 m (b) 40 m
(c) 76 (d) 72 (c) 30 m (d) 20 m

n
ja ANSWER KEY
R s
a th

1.(a) 2.(a) 3.(d) 4.(a) 5.(b) 6.(b) 7.(a) 8.(c) 9.(d)


ty a
di M
A

Aditya Ranjan (Excise Inspector) Selected Selection 96


https://sscstudy.com/
Join Telegram- Maths by Aditya Ranjan RACE

SOL U TION S
1. (a) Anil Bakul Charles Total distance
6. (b)  Avg. speed
15001350 Total time
1500 1425 45
 = 20 × 2 = 40 km/hr.
1500 : 1350 : 1282.5 30
 Anil beats charles by (1500 – 1282.5) 60
 217.50m 7. (a) P Q
2. (a) A's speed = 16 m/sec. Distance 1500 1100
A B Time 1 1
Distance 400 385 Speed 15x 11x
Also,
Speed 16 m/s –

r
Time 25 sec. 35 sec. 1100 1500
– =8

si
11x 15x
385
B's speed = = 11 m/s 1300 – 1100
an by 35 =8
11x
3. (d) A B C 200 = 88x

n
100 80 200
x=
100 80 88
ja
R s
25 : 20 : 16 200
 Speed of P = 15 
  88
a th

100 64 1500
 Time =  88
 A beat C by 36 m 15  200
= 44 sec.
ty a

4. (a) A B C
8. (c) A B C
1200 1100
500 450
di M

600 500
600 540
72 66 55
100 90 81
 
 
720 550
400 324
C cover (720 – 550)m in
 A beat C by 76 m
30 sec.
170 17 9. (d) Arjun Balay Charan
 C's Speed = = m/sec. 1000 900
30 3
5. (b) A B 1000 900
100 950 100 90 81
A

B cover 50m in 5sec.


Also,
 B's speed = 10m/sec. In next race. –
A's speed  Arjun Balay Charan
1000 950 Distance 1000 900 800
= = Time Speed 100 90 81
A B
1000 950 800
= Time 10 10
A 10 81
Time 810 810 800
200 10 81 : 81 : 80
 A= =
19 19  In 1 sec, Arjun can travel 100m and charan 80m

Aditya Ranjan (Excise Inspector) Selected Selection 97


https://sscstudy.com/
Join Telegram- Maths by Aditya Ranjan BOAT & STREAM

BOAT & STREAM


14

1. A man rows a boat a certain distance SSC CGL 05/12/2022 (Shift- 02)
downstream in 9 hours, while it takes 18 hours
to row the same distance upstream. How many 23 22
(a) m/sec (b) m/sec
hours will it take him to row three-fifth of 27 27
the same distance in still water?
20 19
(c) m/sec (d) m/sec
27 27

3
4. A boat racer can row 21 km/h in still water.

r
2

si
If the speed of river is 12.5 km/h, it takes him
40 minutes to row to a place and back, how

(a) 9.5
an by SSC CGL 01/12/2022 (Shift- 02)
(b) 7.2
far off is the place (consider up to two decimals)?

n
3
(c) 10 (d) 12 21
2
2. A swimmer swims from a point P against the
ja 12.5
R s
current for 6 min and then swims back along
the current for next 6 min and reaches at a
a th

point Q. If the distance between P and Q is 40


120 m then the speed of the current (in km/

h) is:

P
ty a

SSC CGL 06/12/2022 (Shift- 01)



di M

Q P 5 2
(a) 5 km (b) 5 km
Q 120 m (km/ 27 5
h )
SSC CGL 01/12/2022 (Shift- 03) 5 5
(c) 4 km (d) 3 km
27 27
(a) 0.4 (b) 0.2
(c) 1 (d) 0.6 5. A man rows to a place 48 km distant and
comes back in 14 hours. He finds that he can
1
3. The speed of a boat in still water is 5 km/ row 4 km with the stream in the same time
3
as 3 km against the stream. The speed of the
h. It is found that the boat takes thrice as stream is:
much time to row up than it does to row down
A

the same distance in the river stream. Find 48 km


the speed of the river stream.
1 4 km
5 km/h
3 km
3

SSC CGL 07/12/2022 (Shift- 02)
(a) 1.5 km/h (b) 3.5 km/h
(c) 1.8 km/h (d) 1 km/h

Aditya Ranjan (Excise Inspector) Selected Selection 98


https://sscstudy.com/
Join Telegram- Maths by Aditya Ranjan BOAT & STREAM

6. A boat moves 25km upstream and 39km


downstream in 8 hours. It travels 35km
upstream and 52km downstream in 11 hours.
What is the speed of the stream if it travels

at a uniform speed?
25 km
39 km SSC CGL 12/12/2022 (Shift- 02)
35 km (a) 13 : 3 (b) 9 : 2
(c) 11 : 6 (d) 8 : 5
52 km
8. A boat covers 35 km downstream in 2 h and
covers the same distance upstream in 7 h. Find
SSC CGL 12/12/2022 (Shift- 01) the speed (in km/h) of the boat in still water.
(a) 4 km/h
(b) 5 km/h

(c) 6 km/h
(d) 3 km/h

r
7. A boat covers a certain distance against the

si
stream in 9 hours 36 min and it covers the SSC CGL TIER- II 03/03/2023
same distance along the stream in 6 hours.
an by (a) 11.25 (b) 28.75
What is the ratio of the speed of the boat in
still water to that of the stream? (c) 18.25 (d) 10.55

n
ja ANSWER KEY
R s
a th

1.(b) 2.(d) 3.(c) 4.(a) 5.(d) 6.(a) 7.(a) 8.(a)


ty a
di M
A

Aditya Ranjan (Excise Inspector) Selected Selection 99


https://sscstudy.com/
Join Telegram- Maths by Aditya Ranjan BOAT & STREAM

SOL U TION S
1. (b) 45
Let, x = speed of boat 4. (a) x= km / hr.
2
y = speed of stream
y = 12.5 km/hr.
ATQ,
9 (x + y) = (x – y) 18 D D 40
 =
27y = 9x x – y x  y 60
y 1 1 1  2
= D  =
x 3 10 35  3
D = 9 (x + y) 2 35  10 35  4 5
D=  = =5 km.
= 9 (4y) 3 45 27 27

r
3 3 xy 4
 D × 9x 4y 5. (d) 
5 x–y 3
 5

si
x 3y  x = 7y
ATQ,
an by
=
36
5
= 7.2
48
xy x–y

48
= 14

n
2. (d) P 48 48
  = 14
ja 8y 6y
Q
R s
6 8
P   = 14
y y
a th

PQ = 120m
 y = 1 km/hr.
x = speed of swimmer
y = speed of stream 25 39
6. (a)  =8
ATQ, x–y xy
ty a

6 6 120 35 52
 x + y –  x – y =  = 11
60 60 1000 x–y xy
di M

 x–y=5
20  6 6 x + y = 13
  x  y –  x – y  
100 5 y = 4 km/hr.
3
6 9
xy 48 8
2y =
5 7. (a) = 5 = =
x–y 6 5 6 5
y = 0.6 km/hr.  5x + 5y = 8x – 8y
 3x = 13y
16
3. (c) x km / hr. x 13
3 =
y 3
x–y 1 8. (a)
 Let, Speed of boat = x
xy 3
Speed of stream = y
A

 x = 2a, y = a A.T.Q,
16 35 35
2a  = 2, =7
3 xy x–y
x – y = 5
16 8
 a= = km / hr. 35
6 3 xy=
2
8 5 20 45
  m / sec . x= = 11.25
3 18 27 4

Aditya Ranjan (Excise Inspector) Selected Selection 100


https://sscstudy.com/
Join Telegram- Maths by Aditya Ranjan NUMBER SYSTEM

NUMBER SYSTEM
15

1. Which of the following numbers is a divisor 5. If the 4-digit number x67y is exactly divisible
of (4915–1)? by 9, then the least value of (x + y) is _____.

x67y
(x +y) _________
SSC CGL 01/12/2022 (Shift- 01) SSC CGL 02/12/2022 (Shift- 01)
(a) 46 (b) 14
(a) 9 (b) 0
(c) 8 (d) 50
(c) 5 (d) 3
2. The table given below shos the number of

r
spoon manufactured by five factories. 6. In a class of students, the first student has
2 toffees, second has 4 toffees, third has 6

si
Factory Spoon toffees and so on. If the number of students
an by P 100 in the class is 25, then the total number of
Q 200 toffees are divisible by ________.

n
R 150
S 50
ja T 250
R s
What are the ratio of number of spoon
a th

manufactured by P to the number of spoon _______


manufactured by R? SSC CGL 02/12/2022 (Shift- 02)
(a) 5 and 7 (b) 5 and 13
ty a

(c) 11 and 13 (d) 7 and 11


di M

P R 7. What will be the least number which when


doubled will be exactly divisible by 15, 18,
SSC CGL 01/12/2022 (Shift- 02) 25 and 32?
(a) 3 : 2 (b) 2 : 3
(c) 14 : 1 (d) 1 : 3
3. What will be the remainder when 2727 + 27 SSC CGL 02/12/2022 (Shift- 02)
is divided by 28?
(a) 3600 (b) 7200

(c) 6400 (d) 3200

8. The difference of two numbers is 1564. After
SSC CGL 01/12/2022 (Shift- 03)
dividing the larger number by the smaller, we
A

(a) 28 (b) 27 get 6 as quotient and 19 as remainder. What


(c) 25 (d) 26 is the smaller number?
4. If the 5-digit number 750PQ is divisible by 3,
7 and 11, then what is the value of P + 2Q?

750 Q
P + 2Q
SSC CGL 01/12/2022 (Shift- 04) SSC CGL 02/12/2022 (Shift- 03)
(a) 17 (b) 15 (a) 456 (b) 287
(c) 18 (d) 16 (c) 623 (d) 309

Aditya Ranjan (Excise Inspector) Selected Selection 101


https://sscstudy.com/
Join Telegram- Maths by Aditya Ranjan NUMBER SYSTEM

9. Which of the following numbers are divisible 14. Find the largest number of 3 digits divisible
by 2, 3 and 5? by 4 and 7.

SSC CGL Pre 3/12/2022 (Shift- 04)
SSC CGL 02/12/2022 (Shift- 04) (a) 960 (b) 980
(a) 5467760 (b) 1345678 (c) 990 (d) 970
15. If the 8-digit number 123456xy is divisible
(c) 2345760 (d) 2456732
by 8, then the total possible pairs of (x,y) are:
10. Which of the following statement is correct?
123456xy, 8
I. The value of 100²–99²+98²–97²+96²–95²+94²–
93²+.......+2²–1² is 5050. (x,y)
SSC CGL 03/12/2022 *Shift- 04)
8
II. If 8x + = –16 and x < 0, then the value (a) 8 (b) 13
x (c) 10 (d) 11
of x197+x–197 is 2. 16. Which of the following pairs of non-zero values
of p and q make 6-digit number 674pq0

r
I. 100²–99²+98²–97²+96²–95²+94²– divisible by both 3 and 11?

si
93²+.......+2²–1² 5050 p q
674pq0
an by
8
II. 8x + = –16 x < 0 x197+x –197
x

SSC CGL 05/12/2022 (Shift- 01)

n
(a) p = 2 and q = 2 (b) p = 5 and q = 4
SSC CGL 02/12/2022 (Shift- 04) (c) p = 4 and q = 2 (d) p = 5 and q = 2
ja
R s
(a) Only I (b) Only II 17. On dividing a certain number by 363, we get
(c) Both I and II (d) Neither I nor II 17 as the remainder. What will be the
a th

11. The nearest number which is greater to 87501, remainder when the same number is divided
and is completely divisible by 765 is : by 11?

ty a


SSC CGL 03/12/2022 (Shift- 01)
di M

(a) 88975 (b) 87975 SSC CGL 05/12/2022 (Shift- 02)


(c) 87966 (d) 87775 (a) 7 (b) 8
(c) 6 (d) 9
12. What will be the remainder when 742 is divided
18. The largest five-digit number which when
by 48?
divided by 7, 9 and 11, leaves the same
remainder as 3 in each case, is:

SSC CGL 03/12/2022 (Shift- 02)
(a) 2 (b) 3
(c) 1 (d) 0
SSC CGL 05/12/2022 (Shift- 02)
13. A number when divided by 7 leaves remainder
A

(a) 95840 (b) 98685


of 4. If the square of the same number is
(c) 96720 (d) 99795
divided by 7, then what is the remainder?
19. Find the greatest 5-digit number which is
divisible by 11, 33, 99 and 121.


SSC CGL 03/12/2022 (Shift- 03) SSC CGL 05/12/2022 (Shift- 03)
(a) 3 (b) 1 (a) 90099 (b) 99990
(c) 4 (d) 2 (c) 99099 (d) 90909

Aditya Ranjan (Excise Inspector) Selected Selection 102


https://sscstudy.com/
Join Telegram- Maths by Aditya Ranjan NUMBER SYSTEM

20. What is the smallest number that should be 26. The average of eight consecutive odd numbers
added to 4567 so that the sum is divisible is 48. Find the sum of the fifth and seventh
by 7? numbers.


SSC CGL 05/12/2022 (Shift- 03) SSC CGL 06/12/2022 (Shift- 03)
(a) 7 (b) 5
(a) 102 (b) 86
(c) 6 (d) 4
(c) 89 (d) 90
21. If the 9–digit number 83p93678Q is divisible
by 72, then what is the value of 27. What is the smallest perfect square which is
completely divisible by each of 16, 18 and 36?
P 2 + Q 2 + 12 ?

83p93678Q, 72
P 2 + Q2 + 12 SSC CGL 06/12/2022 (Shift- 03)

r
SSC CGL 05/12/2022 (Shift- 04) (a) 144 (b) 81
(a) 6 (b) 7 (c) 196 (d) 169

si
(c) 8 (d) 9 28. Numbers divisible by 9 between 43 and 481
22. an by
The value of 97 × 103 is _________.
97 × 103 _____
are:

n
SSC CGL 06/12/2022 (Shift- 01)
(a) 7999 (b) 9991 SSC CGL 06/12/2022 (Shift- 03)
(c) 8991
ja (d) 9981
R s
(a) 51 (b) 49
23. Find the greatest number that will divide 49,
a th

147 and 322 to leave the same remainder in (c) 48 (d) 50


each case. 29. What should be subtracted from 246837 to
make it divisible by 13?
ty a



di M

SSC CGL 06/12/2022 (Shift- 01) SSC CGL 06/12/2022 (Shift- 04)
(a) 9 (b) 5 (a) 4 (b) 5
(c) 7 (d) 8
(c) 3 (d) 6
24. A number n when divided by 6 leaves a
30. If a number K = 42 × 25 × 54 × 135 is divisible
remainder 3. What will be the remainder when
by 3, then find the maximum value of a.
(n² + 5n + 8) is divided by 6?
K = 42 × 25 × 54 × 135, 3
(n² + 5n + 8) a
SSC CGL 07/12/2022 (Shift- 01)
SSC CGL 06/12/2022 (Shift- 01) (a) 6 (b) 7
A

(a) 1 (b) 3 (c) 4 (d) 5


(c) 5 (d) 2 31. How many numbers from 1 to 430 are divisible
25. 3 + 9 + 27 + 9 + 929 is divisible by which
50 26 18 28
by 7 and 11 both?
of the following integers?

350 + 926 + 2718 + 928 + 929


SSC CGL 07/12/2022 (Shift- 02)
SSC CGL 06/12/2022 (Shift- 02)
(a) 11 (b) 5 (a) 5 (b) 11
(c) 7 (d) 2 (c) 9 (d) 7

Aditya Ranjan (Excise Inspector) Selected Selection 103


https://sscstudy.com/
Join Telegram- Maths by Aditya Ranjan NUMBER SYSTEM

32. 9435 is added to 7593, then 2607 is 37. If 7-digit number 678p37q is divisible by 75
subtracted from the sum. The result is and p is not a composite, then the values of
divisible by: p and q are:
678p37q, 75
p p q

SSC CGL 07/12/2022 (Shift- 03) SSC CGL 08/12/2022 (Shift- 04)
(a) 4 (b) 10 (a) p = 5, q = 5 (b) p = 3, q = 0
(c) 3 (d) 5 (c) p = 3, q = 5 (d) p = 2, q = 5
33. Find the remainder when we divide 3x4 – 2x² 38. The largest 5-digit number that is exactly
+4x – 1 by 2x – 1. divisible by 88 is:
3x4– 2x² +4x – 1 2x – 1 ______

SSC CGL 07/12/2022 (Shift- 04)
SSC CGL 09/12/2022 (Shift- 01)
(a) 2 (b) 3

r
(a) 99968 (b) 99689
11 15

si
(c) 68999 (d) 66698
(c) (d)
16 16 39. If the number 123456789 is divided by 9, then
34.
an by
A number when divided by 221, leaves a
remainder 30. If the same number is divided
the remainder is:

n
by 13, the remainder will be:
SSC CGL 09/12/2022 (Shift- 02)
ja

R s
(a) 0 (b) 1
(c) 2 (d) 3
a th

SSC CGL 08/12/2022 (Shift- 01) 40. In a 7-digit number 89476*2, what is the
(a) 4 (b) 3 smallest possible value of * such that the
(c) 2 (d) 1 number is divisible by 8?
ty a

35. In a division sum, the divisor is 10 times the 89476*2 *


quotient and four times the remainder. What

di M

is the dividend if the remainder is 45?


SSC CGL 09/12/2022 (Shift- 03)

(a) 2 (b) 1

(c) 4 (d) 3
41. Three positive numbers are in the ratio of 4
SSC CGL 08/12/2022 (Shift- 02) : 5 : 7, and the sum of their squares is 15,210.
(a) 4123 (b) 3285 Find the sum of the three numbers.
(c) 2895 (d) 5412 4 : 5 : 7
36. 7 is added to a certain number and the sum

is multiplied by 5. The product is then divided
by 3 and 4 is subtracted from the quotient.
If the result comes to 16, then what is the SSC CGL 09/12/2022 (Shift- 03)
A

original number? (a) 148 (b) 156


(c) 126 (d) 208
42. The remainder when 1919+20 is divided by 18 is:
1919 + 20

SSC CGL 08/12/2022 (Shift- 03) SSC CGL 09/12/2022 (Shift- 04)
(a) 3 (b) 1 (a) 3 (b) 2
(c) 5 (d) 4 (c) 1 (d) 0

Aditya Ranjan (Excise Inspector) Selected Selection 104


https://sscstudy.com/
Join Telegram- Maths by Aditya Ranjan NUMBER SYSTEM

43. The least number that should be added to 48. Choose the option in which the numbers are
35460 so that the sum is exactly divisible by in correct ascending order.
3, 4, 5 and 7 is:



SSC CGL 13/12/2022 (Shift- 02)
SSC CGL 12/12/2022 (Shift- 01)
(a) 84 (b) 420 4 2 1 2 1 2 2 4
(a) , , and (b) , , and
5 3 11 9 11 9 3 5
(c) 240 (d) 180
44. Any six-digit number that is formed by
2 1 4 2 2 4 1 2
repeating a three-digit number, is always (c) , , and (d) , , and
9 11 5 3 3 5 11 9
divisible by:

49. A four-digit pin, say abcd, of a lock has


different non-zero digits. The digits satisfy

r
b = 2a, c = 2b, d = 2c. The pin is divisible
SSC CGL 12/12/2022 (Shift- 02) by.

si
(a) 111 (b) 1001 abcd
(c) 19
an by (d) 101 b = 2a, c =

n
45. If the seven-digit number 52A6B7C is divisible 2b, d = 2c
by 33, and A, B, C are primes, then the
ja SSC CGL 13/12/2022 (Shift- 02)
maximum value of 2A+3B+C is:
R s
(a) 2, 3, 5
52A6B7C, 33
a th

A, B, C 2A + 3B + C (b) 2, 3, 7
(c) 2, 3, 13
ty a

SSC CGL 12/12/2022 (Shift- 03) (d) 2, 3, 11


(a) 32 (b) 23
50. If the number 6788934a4 is divisible by
di M

(c) 27 (d) 34 11,then find the smallest whole number in


46. If a 7-digit number 54p3987 is divisible by the place of a.
11, then p is equal to: 6788934a4, 11 a
54p3987, 11
p SSC CGL 13/12/2022 (Shift- 03)
SSC CGL 12/12/2022 (Shift- 04)
(a) 3 (b) 4
(a) 5 (b) 9
(c) 5 (d) 2
(c) 4 (d) 1
A

51. If the nine-digit number 3422213AB is divisible


47. When m 12 – 1 is divided by m + 1, the by 99, then what is the value of 2A + B?
remainder is:
3422213AB, 99
m12–1 m + 1
2A + B

SSC CGL 13/12/2022 (Shift- 01) SSC CGL 05/12/2022 (Shift- 01)

(a) 1 (b) 2 (a) 11 (b) 12

(c) 0 (d) –1 (c) 10 (d) 13

Aditya Ranjan (Excise Inspector) Selected Selection 105


https://sscstudy.com/
Join Telegram- Maths by Aditya Ranjan NUMBER SYSTEM

52. 8 + 1681 7 1
(a) (b)
18 9

8 + 1681 2 5
(c) (d)
9 18
SSC CGL TIER - II 02/03/2023
(a) 5 (b) 6 58. The number 1254216 is divisible by which of
(c) 4 (d) 7 the following numbers?

53. Find the average of the cubes of the first five


natural numbers. SSC CGL TIER- II 06/03/2023
(a) 16 (b) 5

(c) 8 (d) 11
59. The largest three-digit number that gives the
SSC CGL TIER - II 02/03/2023 same remainder 2 when divided by 3, 5 and 9
(a) 35 (b) 40 is ________.

r
(c) 45 (d) 50
54. The number 5769116 is divisible by which of

si
the following numbers? SSC CGL TIER- II 07/03/2023
an by


(a) 999
(c) 998
(b) 984
(d) 992

n
SSC CGL TIER - II 02/03/2023 60. If x = 64  121 – 169 , then find the value
(a) 4 (b) 5 of x².
(c) 12
ja (d) 8
R s
x = 64  121 – 169 x²
55. Find the value of the given expression.
a th


SSC CGL TIER- II 07/03/2023

(a) 16 (b) 25
20 – 20 – 20 – 20 – ......
ty a

(c) 36 (d) 49
(a)
61. If the four-digit number 463y is divisible by 7,
di M

SSC CGL TIER- II 03/03/2023 then what is the value of y?


(a) 4 (b) 6 463y, 7 y
(c) 5 (d) 2
56. The number 2918245 is divisible by which of SSC CGL TIER- II 07/03/2023
the following numbers?
(a) 4 (b) 6
(c) 3 (d) 5
62. If 2 + 3 = 17 and 2
x y x+2
– 3y + 1 = 5, then:
SSC CGL TIER- II 03/03/2023 2x + 3y = 17 2x + 2 – 3y + 1 = 5
(a) 3 (b) 11 SSC CGL TIER- II 07/03/2023
A

(c) 12 (d) 9 (a) x = 1, y = 3 (b) x = 3, y = 3


57. Simplify (solve) the following./
(c) x = 3, y = 2 (d) x = 1, y = 2

63. Which of the following numbers is divisible by 11?

10  25  108  154  225
SSC CGL TIER- II 07/03/2023
16  19.25  4²
(a) 5214341 (b) 5648741
SSC CGL TIER- II 06/03/2023
(c) 6598321 (d) 2378965

Aditya Ranjan (Excise Inspector) Selected Selection 106


https://sscstudy.com/
Join Telegram- Maths by Aditya Ranjan NUMBER SYSTEM

ANSWER KEY
1.(c) 2.(b) 3.(d) 4.(a) 5.(c) 6.(b) 7.(a) 8.(d) 9.(c) 10.(a)

11.(b) 12.(c) 13.(d) 14.(b) 15.(b) 16.(d) 17.(c) 18.(d) 19.(c) 20.(d)

21.(c) 22.(b) 23.(c) 24.(d) 25.(a) 26.(a) 27.(a) 28.(b) 29.(d) 30.(b)

31.(a) 32.(c) 33.(c) 34.(a) 35.(b) 36.(c) 37.(c) 38.(a) 39.(a) 40.(d)

41.(d) 42.(a) 43.(c) 44.(b) 45.(b) 46.(a) 47.(c) 48.(b) 49.(c) 50.(d)

51.(a) 52.(d) 53.(c) 54.(a) 55.(a) 56.(b) 57.(c) 58.(c) 59.(d) 50.(c)

r
si
61.(a) 62.(c) 63.(a)
an by SOL U T ION S

n
ja
1. (c) 4915 – 1 is divisible by 5. (c) x 67y
R s
(49 – 1) = 48 and 8 is a factor of 48  x + 6 + 7 + y  9a
a th

 x + y = 5 (least value)
P 100 2
2. (b) = = 6. (b) 2, 4, 6..........
R 150 3
= (2 + 4 + 6 + 8 + ....... + 50)
ty a

27  27 (–i)  (–i)
27 27
= 2 (1 + 2 + 3 + .......... + 25)
3. (d) 
28 28 25  26
di M

= 2 Divisible by 13 and 5
2
–1 – 1
= = –2
28 7. (a) LCM (15, 18, 25, 32) = 7200

= 26 7200
 Required no. = = 3600
4. (a) 750 PQ 2
Divisible by 3:- 8. (d) x – y = 1564 ......(1)
7 + 5 + P + Q = 12 + P + Q x = y × 6 + 19 .....(2)
 P + Q = 3a ..........(1) But (2) in (1)
(7 + Q) – (5 + P) = 0 or 116 6y – y = 1564 – 19
 2+Q–P=0 5y = 1545
A

 Q–P=–2 y = 309
 P–Q=2 ..........(2) 9. (c) From options:-
From (1) and (2) (b) and (d) are eliminated by unit digit addivig
digits of (a)  not divisible by 3
(P, Q) = (4, 2) or (7, 5)
 (c) is divisible by 2, 3, 5
But 75042 is not divisible by 7
10. (a) I. 100² – 99² + 98² – 97² + 96² – 95² + 94² –
 (P, Q) = (7, 5) 93² + ..... + 2² – 1²
 P + 2Q = 17  (100 – 99) (100 + 99) + (98 – 97) (98 + 97) +
(96 – 95) (96 + 95) + ...... + (2 – 1) (2 + 1)

Aditya Ranjan (Excise Inspector) Selected Selection 107


https://sscstudy.com/
Join Telegram- Maths by Aditya Ranjan NUMBER SYSTEM

 100 + 99 + 98 + 97 + 96 + 95 + ..... + (2 + 1) 18. (d) LCM (7, 9, 11) = 693


99999 = 693 × 144 + 207
n  (n 1) 100  101
 = = 5050 Required no. = 99999 – 207 + 3
2 2
= 99795
it is tone
19. (c) LCM (11, 33, 99, 121) = 1089
 1 99999 = 1089 × 91 + 900
II. 8 
x  
 = –16
 x  Required no. = 99999 – 900
= 99099
1 20. (d) 4567 = 7 × 652 + 3
 x+ = –2  x = – 1
x  4 should be added
21. (c) 83p93678Q ÷ (72 = 8 × 9)
1
x197  =–1–1=–2 78Q is divisible by 8
x197
Q=4
it is incorrect
Now,
11. (b) 87501 = 765 × 114 + 291
8+3+P+9+3+6+7+8+4

r
 Required no. = 87501 + (765 – 291)
= 48 + P

si
= 87975 P=6

12. (c)
48
=
an by
742 (7²)21 (49)21
48
=
48
=1 
=8
P²  Q²  12  36  16  12

n
13. (d) N = 7a + 4 22. (b) 97 × 103
N² = 49a² + 16 + 56a
ja  (100 – 3) (100 + 3)
R s
= 7a (7a + 8) + 14 + 2 = 100² – 9  10000 – 9
 7 [7a² + 8a + 2] + 2 = 9991
a th

 Remainder = 2 23. (c) HCF (147 – 49, 322 – 147)


14. (b) LCM (4, 7) = 28  HCF (98, 175)
999 = 28 × 35 + 19 =7
ty a

 Required no. = 999 – 19 24. (d) n = 6a + 3


n² = 36a² + 9 + 36a
di M

= 980
n² + 5n = 36a² + 9 + 36a + 30a + 15
15. (c) 1 2 3 4 5 6 xy
= 36a² + 96a + 24
 6xy is divisible by 8
(n² + 5n + 8) = 36a² + 96a + 32
(x, y) = (0, 0), (1, 6) (2, 4)
= 6 (6a² + 16a + 5) + 2
(3, 2), (4, 0) (5, 6)
 Remainder = 2
(7, 2), (8, 0) (8, 8), (9, 6) 25. (a) 350 + 926 + 2718 + 928 + 929
 10 pairs  925 + 926 + 927 + 928 + 929
16. (d) 674pq0  925 [1 + 9 + 9² + 9³ + 94]
 6 + 7 + 4 + p + q = 17 + p + q  925 [1 + 9 + 81 + 729 + 6561]
 p + q = 4 or 7 or 10 and 6 + 4 + q – (7 + p)  925 × 7381
A

3+q–p=0 = 925 × 671 × 11 is a multiple of 11


q–p=–3 26. (a) let nos. be:-
or p – q = 3 n–6, n–4, n–2, n, n+2, n+4, n+6, n+8
From option 8n  8
Avg. = = 48
(d) P = 5, q = 2 satisfies. 8
17. (c) N = 363a + 17  n + 1 = 48
= 11 (33a + 1) + 6  n = 47
 Remainder = 6  Required sum = n + 2 + n + 6 = 2n + 8 = 102

Aditya Ranjan (Excise Inspector) Selected Selection 108


https://sscstudy.com/
Join Telegram- Maths by Aditya Ranjan NUMBER SYSTEM

27. (a) LCM (16, 18, 36) = 24 × 32 = 288 36. (c) ATQ,
only 144 is divisible by 288 {[(7 + a) × 5] ÷ 3} – 4 = 16
28. (b) A = 45, L = 477, D = 9
35  5a
 L = A + (N – I) D  – 4 = 16
3
432  35 + 5a = 60
  N–I
9  5a = 25
 N = 48 + 1 = 49 a=5
29. (d) 246837 = 13 × 18987 + 6 37. (c) 678p37q is divisible by (25 and 3) = 75
6 must be sulo tracted  7q is divisible by 25
30. (b) K = 42 × 25 × 54 × 135 q=5
= 3 × 14 × 25 × 3³ × 2 × 3³ × 5 Now,
6 + 7 + 8 + p + 3 + 7 + q = 36 + p, p = prime
 37 × 2 × 14 × 25 × 5
 p = 3.
(question incorrect)
 p = 3, q = 5

r
31. (a) LCM (7, 11) = 77
38. (a) 99999 = 88 × 1136 + 31

si
430  Required no. = 99999 – 31
 =5
77 = 99968
an by
Required no. = 5 39. (a) 1 2 3 4 5 6 7 8 9  is divisible by 9

n
32. (c) (9435 + 7593) – 2607  Remainder = 0
= 14421 40. (d) 89476  2
ja
by unit digit option
R s
 6  2 is divisible by 8
(a), (b) and (d) are
  =3
a th

Eliminated
41. (d) Let nos. be 4x, 5x, 7x.
33. (c) Put 2x – 1 = 0
16x² + 25x² + 49x² = 15210
1 x² = 169
ty a

x=
2 x = 13
f (x) = 3x4 – 2x² + 4x – 1  4x +5x +7x = 16x
di M

= 16 × 13 = 208
1  3 2 4
f
  = –  –1
 2  16 4 2 1919  20
42. (a)
18
3 – 8  32 – 16
=
16 (1)19  2
 =3
18
11
= = Remainder 43. (c) LCM (3, 4, 5, 7) = 420
16
35460 = (420 × 84) + 180
34. (a) N = 221a + 30
 Required no. = 420 – 180
= 17 × 13a + 26 + 4
A

= 240
= 13 (17a + 2) + 4 44. (b) xyzxyz is divisible by 1001
Remainder = 4  xyzxyz = xyz × 1001
35. (b) Divisor, d = 10q and d = 4r 45. (b) 5 2 A 6 B 7 C is divisible by 33 = 3 × 11
r = 45  33 = 3 × 11
 dividevd = dq + r 5+2+A+6+B+7+C
45  20 + A + B + C  3a
= 4 × 45 × × 4 + 45 = 3240 + 45 = 3285 A + B + C = 4, 7, 10
10

Aditya Ranjan (Excise Inspector) Selected Selection 109


https://sscstudy.com/
Join Telegram- Maths by Aditya Ranjan NUMBER SYSTEM

Also, 52. (d)


(5 + A + B + C) – (2 + 6 + 7) = A + B + C – 10
8  1681 = 8  41 = 7
= 11a or 0 for maximum
53. (c)
Value of 2A + 3B + C  (3 × 2) + (5 × 3) + 2
 23. n (n 1)² = 5 (6)² = 45
Avg. of cubes =
46. (a) 54p3987 4 4

 (5 + p + 9 + 7) – (4 + 3 + 8) = 11a 54. (a)


Last 2 digits of the given number are divisible
 21 + p – 15 = 11a
by 4.
 6 + p = 11a
 It is divisible by 4.
p=5
55. (a)
47. (c) (m12 – 1) ÷ (m + 1)
put m + 1 = 0 x = 20 – 20 – 20.....
m=–1 We know, 4 × 5 = 20

r
 f(m) = m 12
–1 x=4

si
f(–1) = (–1) 12
–1=1–1=0 56. (b)
 Remainder = 0
an by Since the given number is not divisible by '3'
or '9' [sum rule]
4 2 1 2 only option left is (b) 11.
, , ,

n
48. (b)
5 3 11 9
57. (c)
ja 1
R s
clearly, is smallest of all
11 10  25  108  154  225 x
=
a th

 (b) is correct option. 16  19.25  4² y


49. (c) a b c d
b = 2a, c = 2b, d = 2c. x = 10  25  108  169
ty a

1248
it is divisible by = 10  25  11
di M

1248 = 2 × 3 × 13 × 16 = 10  6 = 4
50. (d) 6 78 8 9 3 4a 4
y = 16  19.25  16 = 16  308
 (6 + 8 + 9 + 4 + 4) – (7 + 8 + 3 + a)
 31 – 18 – a = 11 = 324
 13 – a = 11 = 18
a=2 x 4 2
 = =
51. (a) 3422213AB is divisible by 99 = 9 × 11 y 18 9
3+4+2+2+2+1+3+A+B
58. (c)
 17 + A + B  A + B = 10, 19, ....
A

Since last there digits of the given number is


and (3 + 2 + 2 + 3 + B) – (4 + 2 + 1 + A) divisible by 8.
 10 + B – 7 – A  It is divisible '8'.
 3 + (B – A) 59. (d)
 B – A = – 3, 8 largest 3 digit no. = 999
B–A=8 LCM (3, 5, 9) = 45
 9 – 1 = 8 (A, B) = (1, 9) 999 = 45 × 22 + 9
 2A + B = 2 + 9 = 11  Given no. is 999 – 9 + 2 = 992

Aditya Ranjan (Excise Inspector) Selected Selection 110


https://sscstudy.com/
Join Telegram- Maths by Aditya Ranjan NUMBER SYSTEM

60. (c) Then (1) and (2) become.


a + b = 17
x = 64  121 – 169
4a – 3b = 5
= 8 + 11 – 13
solving above eq, we get
=6
a = 8, b=9
 x² = 36
 2x = 8 = 2³
61. (a)
463y is divisible by 7.  x = 3 and 3y = 9 = 3²
y=4 y=2
62. (c) 63. (a)

2x + 3y = 17 and Checking from options, we find that


........(11) (a) 5 + 1 + 3 + 1 = 2 + 4 + 4
2x+2 – 3y+1 = 5  It is divisible by 11
4 × 2x – 3 × 3y = 5 ..........(2)

r
Let, 2x = a 3 = b
y

si
an by ......-------......

n
ja
R s
a th
ty a
di M
A

Aditya Ranjan (Excise Inspector) Selected Selection 111


https://sscstudy.com/
Join Telegram- Maths by Aditya Ranjan LCM & HCF

LCM & HCF


16

1. The HCF of two numbers is 12. Which one 6. The HCF of two numbers 2040 and 391 is:
of the following can never be their LCM? (HCF)
(HCF) 12
(LCM) SSC CGL 02/12/2022 (Shift- 03)
(a) 17 (b) 21
SSC CGL 01/12/2022 (Shift- 01) (c) 16 (d) 18
(a) 72 (b) 60 7. Three numbers are in the ratio of 2 : 3 : 5
(c) 90 (d) 84 and their LCM is 90. Find their HCF.

r
2. What is the ratio between the HCF and LCM
of the numbers whose LCM is 48 and the

si
(LCM) 90 (HCF)
product of the numbers is 384?
an by
(HCF)
SSC CGL 02/12/2022 (Shift- 04)
(LCM)
(a) 9 (b) 1

n
(LCM) (c) 6 (d) 3
ja 12 14 16
R s
SSC CGL 01/12/2022 (Shift- 02) 8. Claculate the HCF of , and .
(a) 1 : 4 (b) 1 : 6 5 15 17
a th

(c) 1 : 3 (d) 2 : 5 12 14 16
3. Two numbers are in the ratio of 6 : 5. If their , (HCF)
5 15 17
HCF is 3, then what is the LCM of the two

ty a

numbers?
SSC CGL 03/12/2022 (Shift-s 01)
di M

(HCF) 3 4 3
(a) (b)
(LCM) 255 255
SSC CGL 01/12/2022 (Shift- 03)
2 1
(a) 64 (b) 110 (c) (d)
225 255
(c) 90 (d) 80
4. The ratio of two numbers is 5 : 4 and their 9. What is the HCF of 36 and 198?
HCF is 4. What is their LCM? (HCF)
SSC CGL 03/12/2022 (Shift- 02)
(HCF) 4 (LCM) (a) 36 (b) 22
SSC CGL 01/12/2022 (Shift- 04) (c) 18 (d) 9
A

(a) 80 (b) 48 10. The LCM of two numbers is 120 and the
(c) 36 (d) 60 numbers are in the ratio 3 : 8. The sum of
5. What is the largest common divisor of the the numbers will be:
numbers 1026, 2268 and 2430? (LCM) 120
( 3 : 8
largest common divisor)
SSC CGL 02/12/2022 (Shift- 01) SSC CGL 03/12/2022 (Shift- 03)
(a) 108 (b) 54 (a) 48 (b) 55
(c) 81 (d) 27 (c) 45 (d) 60

Aditya Ranjan (Excise Inspector) Selected Selection 112


https://sscstudy.com/
Join Telegram- Maths by Aditya Ranjan LCM & HCF

11. The HCF of two numbers 110 and 1980 is: 15. What is the LCM of (8x³ + 80x2 + 200x) and
(HCF) (4x4+16x³ – 20x²)?
(8x³ + 80x² + 200x) (4x4 + 16x³ – 20x²)
SSC CGL 05/12/2022 (Shift- 01) (LCM)
(a) 140 (b) 110 SSC CGL 07/12/2022 (Shift 01)
(c) 120 (d) 180 (a) 8x2 (x+5)2 (x–1) (b) 8x2 (x–1)2 (x+5)
12. The LCM of the two numbers is 4104 and the (c) 4x2 (x–1)2 (x+5) (d) 4x2 (x+5)2 (x–1)
HCF is 9. If one of the numbers is 171, find 16. Find the LCM of 186.6 and 373.2.
the other.
(LCM)
(LCM) 4104 (HCF)

9
SSC CGL 07/12/2022 (Shift- 03)

(a) 373.2 (b) 398.2
SSC CGL 05/12/2022 (Shift- 04)
(c) 186.6 (d) 276.6
(a) 218 (b) 215
17. If the HCF of two numbers is 8, then which
(c) 220 (d) 216

r
of the following can NEVER be their LCM?
13. Choose the correct statement from the
(HCF) 8

si
following.
(LCM)
an by
SSC CGL 06/12/2022 (Shift- 02)
(a) HCF is the least common multiple of the

SSC CGL 07/12/2022 (Shift- 04)

n
given numbers. (a) 56 (b) 48

ja (c) 42 (d) 40
R s
18. The HCF of three numbers 105, 335 and 465
(b) HCF of two or more numbers is the highest will be:
a th

number which perfectly divides all the


given numbers. (HCF)
SSC CGL 08/12/2022 (Shift- 01)
ty a

(a) 11 (b) 5
(c) 7 (d) 3
di M

(c) HCF is also called the least common divisor. 19. The HCF of 222, 642 and 1062 is ____.
HCF (HCF) _____
(d) In prime factorisation method of HCF, the
multiples of all the given numbers are
SSC CGL 08/12/2022 (Shift- 02)
listed.
(a) 6 (b) 8

(c) 4 (d) 2

20. Find the HCF of 4.08 and 6.63.
14. The sum of two numbers is 18 and their HCF
and LCM are 3 and 54 respectively. What will (HCF)
be the sum of their reciprocals? SSC CGL 08/12/2022 (Shift- 03)
(a) 0.50 (b) 0.52
A


(HCF) (LCM) (c) 0.51 (d) 0.53
21. If the HCF of xy , x y and x³y4 is xy, then their
3 2

LCM is _____.
SSC CGL 06/12/2022 (Shift- 04)
xy3, x²y x³y4 (HCF),
1 1
(a) (b) xy (LCM) _____
7 11
SSC CGL 08/12/2022 (Shift- 04)
1 1 (a) x y3 4
(b) x3y3
(c) (d)
6 9 (c) x4y3 (d) x4y4

Aditya Ranjan (Excise Inspector) Selected Selection 113


https://sscstudy.com/
Join Telegram- Maths by Aditya Ranjan LCM & HCF

22. The LCM of 96, 136 and 504 is: 28. The HCF of three numbers 98, 175 and 210
(LCM) will be:

SSC CGL 09/12/2022 (Shift- 01) (HCF)
(a) 34272 (b) 36548 SSC CGL 12/12/2022 (Shift- 03)
(c) 25872 (d) 28564 (a) 6 (b) 3
23. The LCM of 144, 360 and 450 is: (c) 5 (d) 7
(LCM) 29. Find the HCF of 60, 148 and 382.

(HCF)
SSC CGL 09/12/2022 (Shift- 02)

(a) 4800 (b) 3600
SSC CGL 12/12/2022 (Shift- 04)
(c) 7200 (d) 2400
(a) 2 (b) 6
1 3 5 7
24. The HCF of , , and is. (c) 4 (d) 24

r
2 4 6 8
30. Determine the LCM of two numbers if their

si
1 3 5 7 HCF is 9 and their ratio is 14 : 19.
, , (HCF)
2 4 6 8 (LCM)

an by SSC CGL 09/12/2022 (Shift- 03)


(HCF) 9

n
105 1 SSC CGL 13/12/2022 (Shift- 01)
(a)
2
ja (b)
24 (a) 2394 (b) 3990
R s
(c) 1596 (d) 3192
7 1
a th

(c) (d) 31. The HCF of two numbers is 17 and the other
24 48
two factors of their LCM are 11 and 19. The
3 5 7 smaller of the two numbers is:
25. The LCM of , and is
ty a

8 16 2 HCF 17 LCM
3 5 7
di M

, (LCM)
8 16 2 SSC CGL 13/12/2022 (Shift- 02)
SSC CGL 09/12/2022 (Shift- 04) (a) 209 (b) 187
(a) 101½ (b) 52½ (c) 323 (d) 306
(c) 28½ (d) 25¼ 32. The HCF of three numbers 72, 108 and 2010
26. The HCF of two numbers is 21 and their LCM is:
is 840. If one of the numbers is 49, then the

other number is:
(HCF)
(HCF) 21
SSC CGL 13/12/2022 (Shift- 03)
(LCM) 840
(a) 18 (b) 6

A

(c) 12 (d) 5
SSC CGL 12/12/2022 (Shift- 01)
33. The ratio of two numbers is 6 : 7 and their
(a) 650 (b) 540
HCF is 3. Their LCM is _____.
(c) 810 (d) 360
27. The LCM of 1.2 and 2.7 is:
(LCM) (HCF), 3 (LCM)____
SSC CGL 12/12/2022 (Shift- 02) SSC CGL 13/12/2022 (Shift- 04)
(a) 5.4 (b) 10.8 (a) 124 (b) 128
(c) 1.08 (d) 32.4 (c) 122 (d) 126

Aditya Ranjan (Excise Inspector) Selected Selection 114


https://sscstudy.com/
Join Telegram- Maths by Aditya Ranjan LCM & HCF

34. The product of the two numbers is 1500 and 36. If the greatest common factor (HCF) of x and
their HCF is 10. The number of such possible y is 15, then the HCF of 36x² – 81y² and 81x²
pairs is/are: – 9y² is divisible by _________?
x y (HCF) 15
HCF _____ 36x² – 81y² 81x² – 9y² HCF _________

SSC CGL TIER - II 02/03/2023
SSC CGL TIER- II 06/03/2023
(a) 1 (b) 3
(a) 135
(c) 4 (d) 2
(b) 120
35. The LCM of x² – 8x + 15 and x² – 5x + 6 is:
(c) 90
SSC CGL TIER- II 03/03/2023
(d) 180
(a) (x – 2) (x – 3) (x – 5)
(b) (x – 6)² (x + 1) (x – 3)
(c) (x – 6) (x + 1) (x – 3)

r
(d) (x + 6) (x + 1) (x – 3)

si
an by ANSWER KEY

n
1.(c) 2.(b)
ja 3.(c) 4.(a) 5.(b) 6.(a) 7.(d) 8.(c) 9.(c) 10.(b)
R s
11.(b) 12.(d) 13.(b) 14.(d) 15.(a) 16.(a) 17.(c) 18.(b) 19.(a) 20.(c)
a th

21.(a) 22.(a) 23.(b) 24.(b) 25.(b) 26.(d) 27.(b) 28.(d) 29.(a) 30.(a)
ty a

31.(b) 32.(b) 33.(d) 34.(d) 35.(a) 36.(a)


di M
A

Aditya Ranjan (Excise Inspector) Selected Selection 115


https://sscstudy.com/
Join Telegram- Maths by Aditya Ranjan LCM & HCF

SOL U T ION S
1. (c) LCM = HCF x a 11. (b) 110 = 2 × 5 × 11
But, 90 is not a multiple of 12 1980 = 110 × 18
2. (b) N1 × N2 = HCF × LCM  HCF = 110
384 = HCF × 48 12. (d) N1 × N2 = LCM × HCF
171 × N2 = 4104 × 9
384
HCF = =8
48 4104  9
N2 =
171
HCF 8 1
 = = = 216
LCM 48 6
13. (b) HCF = Highest common factor

r
3. (c) N1 × N2 = LCM × HCF
Which perfectly divides all given mumbers

si
6 × 3 × 5 × 3 = LCM × 3
14. (d) N1 + N2 = 18
LCM = 90
an by N1 + N2 = 3 × 54
4. (a) N1 × N2 = LCM × HCF
= 162
5 × 4 × 4 × 4 = LCM × 4

n
 LCM = 80 1 1 N  N2
 = 1
ja
5. (b) HCF (1026, 2268, 2430) N1 N2 N1N2
R s
1026 = 2 × 3 × 9 × 19 = 2 + 3³ × 19
18
=
a th

2268 = 2² × 34 × 7
162
2430 = 2 × 35 × 5
 HCF = 2 × 3³ = 54 1
=
9
ty a

6. (a) HCF (2040, 391)


2040 = 2³ × 3 × 5 × 17 15. (a) 8x³ + 80x² + 200x = 8x(x² + 10x + 25)
di M

391 = 17 × 23 = 8x (x + 5) (x + 5)
HCF = 17 4x + 16x³ – 20x² = 4x² (x² + 4x – 5)
4

7. (d) 2 × 3 × 5 × x = HCF × 90 = 4x² (x – 1) (x + 5)


x=3  LCM = 8x² (x + 5)² (x – 1)

12 14 16  1866 3732 


8. (c) HCF 
 , , 
 16. (a) LCM 
 , 

 5 15 17   10 10 

HCF (12, 14, 16) 2 LCM (1866, 3732) 3732


= = = =
LCM (5, 15, 17) 225 HCF (10, 10) 10

9. (c) 36 = 2² × 3² = 373.2
A

198 = 2 × 3² × 11 17. (c) LCM = HCF × a


HCF = 2 × 3² = 18 = 8a
10. (b) N1 × N2 = LCM × HCF 42 is not a multiple of 8.
3x × 8x = 120 × x 18. (b) HCF (105, 335, 465)
 x=5 105 = 3 × 5 × 7
N1 + N2 = 5(3 + 8) 335 = 5 × 67
= 55 465 = 3 × 5 × 31
 HCF = 5

Aditya Ranjan (Excise Inspector) Selected Selection 116


https://sscstudy.com/
Join Telegram- Maths by Aditya Ranjan LCM & HCF

19. (a) HCF (222, 642, 1062) 29. (a) HCF (60, 148, 382)
222 = 2 × 3 × 37 60 = 2² × 3 × 5
642 = 2 × 3 × 107 148 = 2² × 37
1062 = 2 × 3² × 59 382 = 2 × 191
 HCF = 2 × 3 = 6 HCF = 2
 408 663  30. (a) N1 × N2 = LCM × HCF
20. (c) HCF 
 , 

100 100  14 × 9 × 19 × 9 = LCM × 9
LCM = 2394
HCF (408, 663) 3  17
 = 31. (b) 11 × 19 × 17 × 17 = LCM × 17
LCM (100, 100) 100
 LCM = 323
408 = 2³ × 3 × 17 32. (b) 72 = 2³ × 3²
663 = 3 × 17 × 13 108 = 2² × 3³
 HCF = 0.51 2010 = 2 × 3 × 5 × 67
21. (a) HCF (xy³, x²y, x³y4) = xy
HCF = 2 × 3 = 6
LCM = x³y4

r
33. (d) N1 × N2 = HCF × LCM
22. (a) LCM (96, 136, 504)
6 × 3 × 7 × 3 = 3 × LCM

si
 96 = 25 × 3
 LCM = 126
136 = 2³ × 17


an by
504 = 2³ × 3² × 7
LCM = 25 × 3² × 7 × 17
34. (d)
N1 × N2 = 1500 = HCF × LCM

n
= 34272  LCM = 150
23. (b) LCM (144, 360, 450) 10a × 106 = 1500
ja  ab = 15
R s
144 = 24 × 3²
360 = 2³ × 3² × 5 3 × 5 = 15
a th

450 = 2 × 3² × 5² 1 × 15 = 15
LCM = 24 × 3² × 5²  2 possible values.
= 3600 35. (a)
ty a

1 3 5 7  x² – 8x + 15 = 0
24. (b) HCF 
 , , , 
 x² – 5x – 3x + 15 = 0
2 4 6 8 
di M

x (x – 5) – 3 (x – 5) = 0
HCF (1, 3, 5, 7) 1 (x – 3) (x – 5) = 0 ..........(1)
= =
LCM (2, 4, 6, 8) 24 Also x² – 5x + 6 = 0
x² – 2x – 3x + 6 = 0
3 5 7  = 105 = 52 1
25. (b) LCM 
 , , 
 x(x – 2) – 3(x – 2) = 0
 8 16 2  2 2
(x – 3)(x – 2) = 0 ..........(2)
26. (d) N1 × N2 = LCM × HCF
 LCM = (x – 3) (x – 2) (x – 5)
49 × N2 = 840 × 21
36. (a)
N2 = 360
HCF (x, y) = 15
12 27   x = 15a, y = 156
27. (b) LCM (1.2, 2.7) = LCM 
 , 

A

10 10  36x² – 81y² = (6x)² – (9y)²


= (6 × 15a)² – (9 × 15b)²
LCM (12, 27) 108
= = = 10.8 = (45)² [4a² – 9a²]
HCF (10, 10) 10
= (45)² (–5a²)
28. (d) HCF (98, 175, 210)
81x² – 9y² = 9 [9x² – y²]
98 = 2 × 7²
= (45)² [9a² – a²]
175 = 5² × 7
= (45)² (8a²)
210 = 2 × 3 × 5 × 7
HCF = (45)², which is divisible by 135
HCF = 7

Aditya Ranjan (Excise Inspector) Selected Selection 117


https://sscstudy.com/
Join Telegram- Maths by Aditya Ranjan SIMPLIFICATION

SIMPLIFICATION
17

1. Simplify x9 × x5 × x–4 × x0 × x–6. 6. The value of 95 × 105 is:
x9 × x5 × x–4 × x0 × x–6 95 × 105
SSC CGL 01/12/2022 (Shift- 02) SSC CGL 03/12/2022 (Shift- 03)
(a) x4 (b) x–4
(a) 9981 (b) 9935
(c) x–6 (d) x6
(c) 9965 (d) 9975
428 × 428 × 428 + 348 × 348 × 348 7. Simplify the following 25³ – 75³ + 50³
2. The value of
428 × 428 – 428 × 348 + 348 × 348

is:

r
25³ – 75³ + 50³
428 × 428 × 428 + 348 × 348 × 348

si
SSC CGL 05/12/2022 (Shift- 03)
428 × 428 – 428 × 348 + 348 × 348
(a) –281250 (b) 281350
an by (c) 271250 (d) –281450
SSC CGL 01/12/2022 (Shift- 04)
8. Simplify the following equation. What is the

n
(a) 776 (b) 62080
difference between the two value of x?
(c) 80 (d) 40
3. ja
Simplify x + 3(y + x – 2) – (x + y).  1 
R s
x + 3(y + x – 2) – (x + y). 7x+4[x² ÷ (5x ÷ 10)] – 3 5 – x 3 ÷ 3x 2 ÷ x  = 0
 3 
a th

SSC CGL 02/12/2022 (Shift- 01)


(a) x + 2y – 6 (b) 2x + y – 6
x
(c) 3x + 2y – 6 (d) 5x + 1y – 6
ty a

x 2 + 2x + y 2 y2  1 
4. Simplify if x + = 5. 7x+4[x² ÷ (5x ÷ 10)] – 3 5 – x 3 ÷ 3x 2 ÷ x  = 0
x – 5x
3 2
x  3 
di M

y2 x 2 + 2x + y 2 SSC CGL 07/12/2022 (Shift- 01)


x + = 5
x x 3 – 5x 2 (a) 8 (b) 16
(c) 5 (d) 17
SSC CGL 02/12/2022 (Shift- 02) 9. Simplify the given expression.
5 7 (5x + 8y) (25x² + 64y² – 40xy)
(a) (b)
y2 y2
5 7 (5x + 8y) (25x² + 64y² – 40xy)
(c) – 2 (d) – 2
y y SSC CGL 07/12/2022 (Shift- 03)
5. Simplify the following. (a) 125x³ – 512y³ (b) 25x³ + 64y³
A

762 × 762 × 762 + 316 × 316 × 316 (c) 125x³ + 512y³ (d) 25x³ – 64y³
762 × 762 – 762 × 316 + 316 × 316 10. Simplify x – 15x³ + 15x² – 15x + 40; given x
4

= 14.

762 × 762 × 762 + 316 × 316 × 316 x4 – 15x³ + 15x² – 15x + 40


762 × 762 – 762 × 316 + 316 × 316 x = 14
SSC CGL 02/12/2022 (Shift- 03) SSC CGL 09/12/2022 (Shift- 03)
(a) 1064 (b) 1056 (a) 0 (b) 40
(c) 1042 (d) 1078 (c) 14 (d) 26

Aditya Ranjan (E xcise Inspector) Selected Selection 118



https://sscstudy.com/
Join Telegram- Maths by Aditya Ranjan SIMPLIFICATION

14. Find the value of given expression.


146 ×146 ×146 – 143 ×143 ×143
11. The value of
146 ×146 +143 ×143 +146 ×143 [76 – {90 ÷ 5 × (24 – 36 ÷ 3) ÷ 3}]
is:

146 ×146 ×146 – 143 ×143 ×143
[76 – {90 ÷ 5 × (24 – 36 ÷ 3) ÷ 3}]
146 ×146 +143 ×143 +146 ×143
SSC CGL TIER- II 03/03/2023
SSC CGL 09/12/2022 (Shift- 03) (a) 71.5 (b) 75.5
(a) 0 (b) 3
(c) 289 (d) 1 (c) 4 (d) 77.5
15. Find the value of the given expression.
1.6 ×1.6 ×1.6 – 0.6 × 0.6 × 0.6
12. The value of is: 3 – (– 6) {– 2 – 9 – 3} ÷ 7 {1 + (– 2) (– 1)}
1.6 ×1.6 +1.6 × 0.6 + 0.6 × 0.6

1.6 ×1.6 ×1.6 – 0.6 × 0.6 × 0.6
3 – (– 6) {– 2 – 9 – 3} ÷ 7 {1 + (– 2) (– 1)}
1.6 ×1.6 +1.6 × 0.6 + 0.6 × 0.6

r
SSC CGL TIER- II 06/03/2023
SSC CGL 12/12/2022 (Shift- 03) (a) 7 (b) 15

si
(a) 0
(c) – 1 (d) 1
(b) 1
(c) –1
(d) 2
an by 16. The value of
0.7  0.7  0.7  0.3  0.3  0.3
0.7  0.7 – 0.7  0.3  0.3  0.3
is

n
13. Find the value of given expression. __________.
30 – [40 – {56 – (25 – 13 – 12)}]
ja
0.7  0.7  0.7  0.3  0.3  0.3
R s
0.7  0.7 – 0.7  0.3  0.3  0.3

30 – [40 – {56 – (25 – 13 – 12)}]
a th

SSC CGL TIER- II 07/03/2023


SSC CGL TIER - II 02/03/2023
(a) 38 (b) 22 (a) 2 (b) 1
(c) 14 (d) 46 (c) 3 (d) – 1
ty a
di M

ANSWER KEY
1.(a) 2.(a) 3.(c) 4.(d) 5.(d) 6.(d) 7.(a) 8.(d) 9.(c) 10.(d)

11.(b) 12.(b) 13.(d) 14.(c) 15.(c) 16.(b)


A

Aditya Ranjan (E xcise Inspector) Selected Selection 119



https://sscstudy.com/
Join Telegram- Maths by Aditya Ranjan SIMPLIFICATION

SOL U TION S
1. (a) 7. (a)
x9 × x5 × x–4 × x0 × x–6 25³ – 75³ + 50³
9+5–4+0–6 a = 25, b = –75, c = 50
 x4 a+b+c=0
2. (a)  a³ + b³ + c³ = +3abc
–3 × 25 × 75 × 50
428 × 428 × 428 + 348 × 348 × 348
 –281250
428 × 428 – 428 × 348 + 348 × 348
8. (d)
a +b
3 3

= = a + b  1 
7x + 4 [x² ÷ (5x ÷ 10)] –3 5 – x  3x  x  = 0

r
3 2
a 2 – ab + b2
 3 

si
= 428 + 348 = 776
3. (c)  2 1  16 
an by  7x + 4  x ÷ x  – 3  – x 3 ÷ 3x  = 0
x + 3(y + x – 2) – (x + y)  2  3 
 x + 3y + 3x – 6 – x – y

n
 7x + 4 × 2x – (16 – x²) = 0
 3x + 2y – 6  7x + 8x – 16 + x² = 0
4. (d) ja x² + 15x – 16 = 0
R s
x + 2x + y
2 2
y
2
x² – x + 16x – 16 = 0
a th

if x + =5
x 3 – 5x 2 x x = 1 , x = –16
Difference = 17
y2
x+ = 5  x² + y² = 5x ...(1) 9. (c)
ty a

x
 x² – 5x = –y² (5x + 8y) (25x² + 64y² – 40xy)
di M

 x³ – 5x² = –y²x ...(2) = (5x + 8y) ((5x)² + (8y)² – 5 × 8xy)


 Put (1) and (2) in original equation  a³ + b³ = (a + b) (a² + b² – ab)
 (5x)³ + (8y)³
5x – y2 + 2x + y2 7x 7
 = =– 2 = 125x³ + 512y³
–y2 x –y2 x y
10. (d)
5. (d) x4 – 15x³ + 15x² – 15x + 40 ...(1)
762 × 762 × 762 + 316 × 316 × 316 x = 14
762 × 762 – 762 × 316 + 316 × 316  x + 1 = 15 , put in equation (1)
= x4 – (x + 1)x³ + (x + 1)² – (x + 1)x + 40
a 3 + b3
= = (a + b) = x4 – x4 – x³ + x³ + x² – x² – x + 40
A

a 2 – ab + b2
= 40 – x = 40 – 14 = 26
= (762 + 316) = 1078
11. (b)
6. (d)
146 ×146 ×146 – 143 ×143 ×143
95 × 105
146 ×146 +143 ×143 +146 +143
= (100 – 5) (100 + 5)
= (100)² – (5)² a 3 – b3
= = a – b
= 10000 – 25 a + b2 + ab
2

= 9975  146 – 143 = 3

Aditya Ranjan (E xcise Inspector) Selected Selection 120



https://sscstudy.com/
Join Telegram- Maths by Aditya Ranjan SIMPLIFICATION

12. (b) 15. (c)

1.6 ×1.6 ×1.6 – 0.6 × 0.6 × 0.6 3 – (– 6) {– 2 – 9 – 3} ÷ 7 {1 + (– 2) (– 1)}


1.6 ×1.6 +1.6 × 0.5 + 0.6 × 0.6  3 – (– 6) {– 14} ÷ 7 {3}
 3 – 84 ÷ 21
a 3 – b3
 2 = a – b 3–4=–1
a + b2 + ab
16. (b)
 1.6 – 0.6 = 1
13. (d) 0.7  0.7  0.7  0.3  0.3  0.3
30 – [40 – {56 – (25 – 13 – 12)}] 0.7  0.7 – 0.7  0.3  0.3  0.3
= 30 – [40 – 56] = 30 + 16 = 46 a³  b³
= = (a b)³
14. (c) a² – ab  b³
[76 – {90 ÷ 5× (24 – 36 ÷ 3) ÷ 3}] = (0.7 + 0.3)³ = 1
= [76 – {18 × (12) ÷ 3}]
= [76 – 72] = 4

r
......-------......

si
an by
n
ja
R s
a th
ty a
di M
A

Aditya Ranjan (E xcise Inspector) Selected Selection 121



https://sscstudy.com/
Join Telegram- Maths by Aditya Ranjan ALGEBRA

ALGEBRA
18

1. If x² – 5x + 1 = 0, then the value of
 1
5. If  x +  = 5 2, then what is the value of
x6 + x4 + x2 +1  x
=? = ?
5x 3
(x4 + x –4) ?
x6 + x4 + x2 +1 1
x² – 5x + 1 = 0
5x 3
=  x +  = 5 2 (x4 + x–4)
 x


SSC CGL 01/12/2022 (Shift- 01)
SSC CGL 01/12/2022 (Shift- 02)

r
(a) 30 (b) 25
(c) 23 (d) 28 (a) 2542 (b) 2650

si
(c) 2452 (d) 2302
173 + 73
an by 6. If a + b = 11 and ab = 35, then what is the
2. If = 24, then what is the value
17 2
+ 7 – k
2
value of (a4 + b4)?

n
of k?
a + b = 11 ab = 35 (a4 + b4)

ja
173 + 73 SSC CGL 01/12/2022 (Shift- 03)
R s
= 24 k
17 2
+ 7 – k
2
(a) 151 (b) 261
a th

SSC CGL 01/12/2022 (Shift- 01) (c) 124 (d) 102


(a) 119 (b) 128 7. If x + y = 36, then find (x – 27)³ + (y – 9)³.
(c) 24 (d) 109 x + y = 36 (x – 27)³ + (y – 9)³
ty a


4  17 –  7 
3 3

3. If = 40, then what is the value SSC CGL 01/12/2022 (Shift- 03)
di M

172 + 72 + p (a) 1 (b) 81


of p? (c) 2y (d) 0

1 1
4  17 –  7  
3 3
8. If y + = 3, then what is the value of 3 +
= 40 p y y
172 + 72 + p y³ + 2 ?
SSC CGL 01/12/2022 (Shift- 01)
1 1
(a) –119 (b) –129 y + = 3, 3 + y³ + 2
y y
(c) 119 (d) 129
4. x is a negative number such that k + k–1 = –2, SSC CGL 01/12/2022 (Shift- 03)
(a) 24 (b) 18
A

k 2 + 4k – 2
then what is the value of ? (c) 20 (d) 29
k2 + k – 5
9. If p + q = 6 and pq = 4, then what is the value
x k + k–1 = –2 of p³ + q³ ?
k 2 + 4k – 2 p + q = 6 pq = 4 p³ + q³

k2 + k – 5
SSC CGL 01/12/2022 (Shift- 02) SSC CGL 01/12/2022 (Shift- 04)
(a) 7 (b) 1 (a) 81 (b) 64
(c) –7 (d) –4 (c) 144 (d) 256

Aditya Ranjan (Excise Inspector) Selected Selection 122


https://sscstudy.com/
Join Telegram- Maths by Aditya Ranjan ALGEBRA

5–2 1 5
10. If x = , then the value of x² + x–2 is : 15. If x + = 8, then find the value of 2 .
5 +2 x x – 8x +2

1 5
5–2 x + = 8 2
x= x² + x
–2
x x – 8x + 2
5 +2
SSC CGL 02/12/2022 (Shift- 04)
SSC CGL 02/12/2022 (Shift- 01)
(a) 350 (b) 345 (a) 3 (b) 4
(c) 284 (d) 322 (c) 0 (d) 5

1 1 1
11. If x > 0, and x 4 + = 2207, what is the value 16. If K + = 3, then what is the value of 3 + K 3?
x4 K K
1 1 1
of x 7 + ? K + = 3 3 + K
3
x7 K K
1 1 SSC CGL 02/12/2022 (Shift- 04)

r
x > 0 x 4 + = 2207 x 7 + 7
x 4
x (a) 36 (b) 10

si
(c) 18 (d) 54
an by SSC CGL 02/12/2022 (Shift- 01)
(a) 710649 (b) 710647  5 
17. If  4a + + 5 = 14, what is the value of
 

n
(c) 710654 (d) 710661 a
12. If a + b = 5 and ab = 6, then find 3(a² + b²). 25 
ja 
16a + 2  ?
2
a + b = 5 ab = 6 3(a² + b²)
R s
a

a th

SSC CGL 02/12/2022 (Shift- 02) 5 25


 4a + + 5 = 14 16a2 + 2 
(a) 39 (b) 48  a   a 
(c) 26 (d) 13
ty a

13. If a + b + c = 6, a² + b² + c² = 14 and ab + bc SSC CGL 03/12/2022 (Shift- 01)


+ ca = 11, then what is the value of a³ + b³ +
(a) 25 (b) 36
di M

c³ –3abc ?
a + b + c = 6, a² + b² + c² = 14 ab + (c) 41 (d) 40
bc + ca = 11 a³ + b³ + c³ –3abc 1 1
18. If x – = 13, what will be the valuek of x 4 + 4 ?
x x
SSC CGL 02/12/2022 (Shift- 03)
(a) 31 (b) 12 1 1
x – = 13 x + 4
4

(c) 18 (d) 42 x x
SSC CGL 03/12/2022 (Shift- 01)
1
14. If k 4 + = 194, then what is the value of (a) 28561 (b) 29243
k4
(c) 27887 (d) 29239
A

1
k3 + ? 19. the factors of x + x² + 25 are:
4
k3
x4 + x² + 25
1 1
k + 4 = 194 k 3 + 3
4
SSC CGL 03/12/2022 (Shift- 01)
k k
(a) (x² + 3x – 5) (x² – 3x + 5)
SSC CGL 02/12/2022 (Shift- 03) (b) (x² + 3x + 5) (x² – 3x + 5)
(a) 42 (b) 52 (c) (x² – 3x + 5) (x² – 3x + 5)
(c) 36 (d) 18 (d) (x² + 3x + 5) (x² + 3x + 5)

Aditya Ranjan (Excise Inspector) Selected Selection 123


https://sscstudy.com/
Join Telegram- Maths by Aditya Ranjan ALGEBRA

25. If 8a³ + 27b³ = 16 and 2a + 3b = 4, then find


1 1
= 2 cos , then x + 3 = ?
3
20. If x + the value of 16a4 + 81b4.
x x
8a³ + 27b³ = 16 2a + 3b = 4
1 1 16a4 + 81b4
= 2 cos x + 3 = ?
3
x +
x x
SSC CGL 03/12/2022 (Shift- 03)
SSC CGL 03/12/2022 (Shift- 02)
(a) 26 (b) 30
(a) 2 cos2 (b) cos 3 (c) 28 (d) 32
(c) 2 cos 3 (d) cos 2
26. Find the value of the following expression.
1 12³ + (–8)³ + (–4)³
21. If x = 3 + 22, x > 0, then the value of x –
x
is: 12³ + (–8)³ + (–4)³
SSC CGL 03/12/2022 (Shift- 03)
1
x = 3 + 22, x > 0 x – (a) 952 (b) 1152
x
(c) 1052 (d) 852
27. (mx + n) is a foctor of:

r
SSC CGL 03/12/2022 (Shift- 02) (mx + n)

si
(a) 1 (b) 2 SSC CGL 03/12/2022 (Shift- 04)
(c) 2 (d) 22 (a) m² x² + 2nx + n² (b) m² x² + 2mnx + n²

22. If x +
1
x
an by
= –2 then what is the value of x17 + 28.
(c) m² x² + 2mx + n² (d) m² x² + 2mn + n²
If x + y + z = 0, then what is the value of

n
x–17 + x¹² + x–12 ? (x < 0) x2 y2 z2
+ + ?
1 ja  yz   xz   xy 
R s
x + = –2 x17 + x–17 + x¹² + x–12
x
x2 y2 z2
a th

(x < 0) x + y + z = 0  yz  +  xz  +  xy 
SSC CGL 03/12/2022 (Shift- 02)
(a) –2 (b) –1
ty a

(c) 1 (d) 0 SSC CGL 03/12/2022 (Shift- 04)


23. If x – y = 1 and x² + y² = 41 where x, y  0, (a) 1 (b) 0
di M

then the value of x + y will be: (c) 2 (d) 3


29. What is the possible value of (a + b + c) –3,
x – y = 1 x² + y² = 41 x, y  0
if a² + b² + c² = 9 and ab + bc + ca = 8?
x + y a² + b² + c² = 9 ab + bc + ca = 8
SSC CGL 03/12/2022 (Shift- 02)
(a + b + c) –3
(a) 9 (b) 8 SSC CGL 03/12/2022 (Shift- 04)
(c) 6 (d) 7 (a) 5 (b) 3
(c) 9 (d) 2
k – kcot 30º
2

24. If = sin²60º + 4tan²45º – cosec²60º,


1 + cot2 30º 1
30. If x + = –14, and x < –1, what will be the
then the value of k (correct to two decimal x
A

places) is:
1
value of x –
2
?
k – kcot 30º
2 x2
= sin²60º + 4tan²45º – cosec²60º
1 + cot2 30º 1
k x + 1 = –14 x < –1 x 2 –
x x2

SSC CGL 03/12/2022 (Shift- 03) SSC CGL 05/12/2022 (Shift- 01)
(a) 5.55 (b) –6.83 (a) –1123 (b) 1123
(c) –5.58 (d) 6.83 (c) –1402 (d) 1402

Aditya Ranjan (Excise Inspector) Selected Selection 124


https://sscstudy.com/
Join Telegram- Maths by Aditya Ranjan ALGEBRA

II. The value of


1
31. If k 4 + = 47, then what is the value of
k4  2 1  1  4 1  1  4 1 
 k + 2   k –   k + 4   k +   k – 4 
k k k k k
1
k3 + ?
k3 1
is k16 –
k16
1 1
k 4 + 4
= 47 k 3 + 3  2 1  1  4 1   1  4 1 
k k  k + 2   k –   k + 4   k +   k – 4 
k k k k k
SSC CGL 05/12/2022 (Shift- 01)
1
(a) 4.5 (b) 54 k16 –
k16
(c) 18 (d) 9 SSC CGL 05/12/2022 (Shift- 02)
32. The simplified form of (x + 2y)³ + (x – 2y)³ is: (a) Neither I nor II (b) Only II
(c) Only I (d) Both I and II
(x + 2y)³ + (x – 2y)³ 36. Select the correct algebraic expression.

r
SSC CGL 05/12/2022 (Shift- 01)

si
(a) 2x³ + 24xy² (b) 2x³ – 24xy² SSC CGL 05/12/2022 (Shift- 03)
an by (a) ab – a – b + 1 = (a – 1) (b – 1)
(c) x³ – 8y³ (d) x³ + 8y³
(b) ab + a – b + 1 = (1 – a) (a – b) (1 + a) (a + b)
33. If a – b = 2 and a³ – b³ = 80, then what will (c) ab – a – b + 1 = (1 – a) (b – 1)

n
be the value of ab? (d) ab – a – b + 1 = (a – 1) (1 – b)
ja 37. What is the value of
a – b = 2 a³ – b³ = 80 ab
R s
100² – 99² + 98² – 97² + 96² – 95² + 94² – 93²
+ ..... + 12² – 11²?
a th

SSC CGL 05/12/2022 (Shift- 02) 100² – 99² + 98² – 97² + 96² – 95² + 94² – 93²
(a) 12 (b) –24 + ..... + 12² – 11²
SSC CGL 05/12/2022 (Shift- 03)
ty a

(c) 24 (d) –12


(a) 5050 (b) 4985
(c) 4995 (d) 4950
di M

1
34. If = x 2 – a 2 , then the value of x is:
x + a2
2 38. If x² + y² + z² = xy + yz + zx and x = 1, then

10x 4 + 5y 4 + 7z 4
1 find the value of
2 2 = x 2 – a2 x 13x 2 y 2 + 6y 2 z 2 + 3z 2 x 2
x +a
x² + y² + z² = xy + yz + zx x = 1
SSC CGL 05/12/2022 (Shift- 02)
10x 4 + 5y 4 + 7z 4
1
13x 2 y 2 + 6y 2 z 2 + 3z 2 x 2

(a) 1 – a 4  4 (b) a
SSC CGL 05/12/2022 (Shift- 04)
1 1 (a) 2 (b) 0
a – 1 4 (d)  a 4 +1 4
A

(c) 4
(c) –1 (d) 1
35. Which of the following statement is correct? 1 1
= 3, then exaluate 8x  3 .
3
39. If x +
2x x

1 1
I. The value of 100² – 99² + 98² – 97² + 96² x + = 3, 8x 3 + 3
– 95² + 94² – 93² + ..... + 22² – 21² is 4840. 2x x
SSC CGL 05/12/2022 (Shift- 04)
100² – 99² + 98² – 97² + 96² – 95² + 94² –
(a) 212 (b) 216
93² + ..... + 22² – 21² 4840 (c) 180 (d) 196

Aditya Ranjan (Excise Inspector) Selected Selection 125


https://sscstudy.com/
Join Telegram- Maths by Aditya Ranjan ALGEBRA

40. If x² – 2xy = 84 and x – y = –10, then the 45. If 2a + 3b = 10 and ab = 3, then find the value
value of y is: of 4a² + 9b².
x² – 2xy = 84 x – y = –10 y 2a + 3b = 10 ab = 3 4a² + 9b²

SSC CGL 05/12/2022 (Shift- 04) SSC CGL 06/12/2022 (Shift- 03)
(a) 2 (b) 1 (a) 60 (b) 62
(c) 4 (d) 3 (c) 64 (d) 66
46. If (4x – 7y) = 11 and xy = 8, what is the value
1 of 16x² + 49y² given that x and y are positive
41. If x + = 1, then the value of x¹² + x9 + x6 +
x numbers?
x³ + 1 is: (4x – 7y) = 11 xy = 8 x y
1 16x² + 49y²
x + = 1 x¹² + x9 + x6 + x³ + 1
x SSC CGL 06/12/2022 (Shift- 03)
(a) 596 (b) 484

r
SSC CGL 06/12/2022 (Shift- 01) (c) 569 (d) 448

si
(a) 1 (b) –1 47. What is the value of

(c) 0

a b
an by (d) –2
 1   2 1   4 1   8 1   16 1 
 k –   k + 2   k + 4   k + 8   k + 16 

n
42. If + = 1 and a + b = 2, then the value of k k k k k
b a
a³ + b³ is:ja  32 1 
 k + 32  ?
R s
k
a b
+ = 1 a + b = 2 a³ + b³
a th

b a
 1   2 1   4 1   8 1   16 1 
 k –   k + 2   k + 4   k + 8   k + 16 
SSC CGL 06/12/2022 (Shift- 02) k k k k k
ty a

(a) 0 (b) 3  32 1 
 k + 32 
(c) 1 (d) 2 k
di M

43. If 4x² + y² = 40 and xy = 6, find the positive SSC CGL 06/12/2022 (Shift- 03)
value of 2x + y.
4x² + y² = 40 xy = 6 2x + y 1 1
k 64 – k 32 –
(a) k 64 (b) k 32
1 1
k+ k–
SSC CGL 06/12/2022 (Shift- 02) k k
(a) 8 (b) 6
(c) 5 (d) 4 1 1
k 32 – k 32 +
(c) k 32 (d) k 32
44. If p = 7 + 43, then what is the value of 1 1
k+ k+
k k
A

p6 + p4 + p2 +1
?
p3
1 1
48. If x + = 2, then the value of x 57 + 57 is:
p6 + p4 + p2 +1 x x
p = 7 + 43 p3

1 1
x + = 2 x 57 + 57
SSC CGL 06/12/2022 (Shift- 02) x x
SSC CGL 06/12/2022 (Shift- 04)
(a) 2617 (b) 2167
(a) 1 (b) –2
(c) 2716 (d) 2176
(c) 0 (d) 2

Aditya Ranjan (Excise Inspector) Selected Selection 126


https://sscstudy.com/
Join Telegram- Maths by Aditya Ranjan ALGEBRA

49. If mxm – nxn = 0, then what is the value of


 x – y 3 +  y – z 3 + z – x 3
1 1
6  x – y   y – z  z – x 
x
+ m terms of xn is:
x +x
m n
x – xn
 y  z
Where x, m, n are > 0
SSC CGL 07/12/2022 (Shift- 01)
mx m – nx n = 0 x n
1 1
1 1 (a) (b)
+ m 4 2
x +x
m n
x – xn
1 1
x, m, n > 0 (c) (d)
3 9
SSC CGL 06/12/2022 (Shift- 04)
1
53. If K + + 2 = 0 and K < 0, then what is the
2mn 2mn K
(a) (b)
 n
x n 2
+ m2    x n  m2 – n 2   value of K10
1
?
K11

r
2mn 2mn
(c) (d) 1 1

si
 x m
n 2
+n 2
  x n
n 2
– m2   K +
K
+ 2 = 0 K < 0 K10 11
K
50.
an by
If, for a non-zero x, 5x² + 7x + 5 = 0, then
1

SSC CGL 07/12/2022 (Shift- 02)

n
the value of x 3 + is: (a) 1 (b) 0
x3
(c) – (d) 2
ja
x 5x² + 7x + 5 = 0
R s
1 1
54. If x + = 2, then x 3 + 3 = ?
1 x x
a th

x 3 + 3
x
1 1
SSC CGL 06/12/2022 (Shift- 04) x + = 2 x 3 + 3 = ?
x x
ty a

496 532 SSC CGL 07/12/2022 (Shift- 02)


(a) (b)
125 343 (a) 1 (b) 8
di M

(c) 2 (d) 0
125 182
(c) (d) 3
532 125  1  1
55. 3 a –  + a –  = ?
 a  a
x+y
51. If x² + y² + 2y + 4x + 5 = 0, then . SSC CGL 07/12/2022 (Shift- 02)
x–y
1 3 1
(b) a –
2
x+y (a) a –
a3 a3
x² + y² + 2y + 4x + 5 = 0
x–y
3 1 1
(c) a +
2
(d) a –
a3 a2
A

SSC CGL 07/12/2022 (Shift- 01)


1 1
If x +
4
56. = 194 and x 3 + 3 = ?
1 x4 x
(a) –3 (b)
3
1 1
(c) –1 (d) 3 x 4 + = 194 x 3 + 3 = ?
x 4
x
 x – y 3 +  y – z 3 + z – x 3 SSC CGL 07/12/2022 (Shift- 03)
52. The value of , where
6  x – y   y – z  z – x  (a) 52 (b) 58
x  y  z, is equal to: (c) 76 (d) 67

Aditya Ranjan (Excise Inspector) Selected Selection 127


https://sscstudy.com/
Join Telegram- Maths by Aditya Ranjan ALGEBRA

61. If m and n are two positive real numbers such


57. Simplify 36x 2 – 108x + 81. that 9m² + n² = 40 and mn = 4, then the value
of 3m + n is:
36x 2 – 108x + 81
m n
SSC CGL 07/12/2022 (Shift- 03)
9m² + n² = 40 mn = 4 3m + n
(a) 6x – 9 (b) 2x – 9
(c) 5x – 9 (d) 3x – 9 SSC CGL 08/12/2022 (Shift- 01)
1 1 (a) 160 (b) 64
58. If x + = 7, find the value of x 3 + 3 .
x x (c) 10 (d) 8

1 1 62. The foctors of x² + 4y² + 4y – 4xy – 2x – 8 are:


x + = 7 x + 3
3

x x x² + 4y² + 4y – 4xy – 2x – 8
SSC CGL 07/12/2022 (Shift- 04) SSC CGL 08/12/2022 (Shift- 01)

r
(a) 343 (b) 322
(a) (x – 2y – 4) (x – 2y + 2)

si
(c) 340 (d) 332
(b) (x² – 2y – 4) (x² – 2y + 2)
59. an by
Which of the following statement is correct?


(c) (x + 2y – 4) (x + 2y + 2)

n
(d) (x² – 2y – 4) (x² + 2y + 2)
1 1 63. If y = 1 +3 + 4, then the value of 2y4 – 8y³
I. If K +ja = 12, then K 2 + 2 = 142
– 6y² + 28y – 84 is:
R s
K K
y = 1 +3 + 4 2y4 – 8y³ – 6y² +
a th

1 1
K + = 12 K 2 + 2 = 142 28y – 84
K K
SSC CGL 08/12/2022 (Shift- 02)
 2 1  1  4 1 
ty a

II. The value of  k + 2   k –   k + 4  (a) 403 (b) 803


k k k
di M

(c) 203 (d) 603


 1 1
 k +  is k – 16 .
16

k k 5– 4 5+ 4
64. If x = and y = then the value
5+ 4 5– 4
 2 1  1  4 1   1
 k + 2   k –   k + 4   k +  x 2 – xy + y 2
k k k k of =?
x 2 + xy + y 2
1
k16 –
k16 5– 4 5+ 4
x= y =
SSC CGL 07/12/2022 (Shift- 04) 5+ 4 5– 4
A

(a) Only I (b) Neither I nor II x 2 – xy + y 2


x 2 + xy + y 2

(c) Both I and II (d) Only II
60. What is the value of (a + b)² – (a – b)²? SSC CGL 08/12/2022 (Shift- 02)
(a + b)² – (a – b)² 361 341
(a) (b)
SSC CGL 08/12/2022 (Shift- 01) 363 343

(a) 2(a² + b²) (b) 4(a² + b²) 384 321


(c) (d)
(c) 8ab (d) 4ab 387 323

Aditya Ranjan (Excise Inspector) Selected Selection 128


https://sscstudy.com/
Join Telegram- Maths by Aditya Ranjan ALGEBRA

2p 1 1 1
65. If = , p  0, then the value of K + = 3 k 2 + 2
p – 5p +1 10
2
K k
SSC CGL 08/12/2022 (Shift- 04)
 1
 p + p  is: (a) 9 (b) 6
(c) 7 (d) 5
70. What is the value of a² + b² + c², if a + b + c
2p 1
= , p  0  p + 1  = 9 and ab + bc + ca = 23?
p – 5p +1 10
2  p 
a² + b² + c² a + b + c = 9
ab + bc + ca = 23
SSC CGL 08/12/2022 (Shift- 02) SSC CGL 08/12/2022 (Shift- 04)
(a) 10 (b) 25
(a) 22 (b) 32
(c) 1 (d) 15
(c) 49 (d) 35
66. If abc = 5, what is the value of
1
  71. If x + = 2 5 where x > 1, then the value of
1 1 1 x

r
 + + 
1 + a + b 1 + b + 5c c –1  ?
–1 –1
1+ + a 1
 

si
 5  x3 – is:
x3


an by
abc = 5

x +
1
x
1
= 2 5 x > 1 x – 3
3

n
1 1 1
 + + 
1 + a + b
–1
1 + b + 5c –1 c
ja 1 + + a –1  SSC CGL 08/12/2022 (Shift- 04)
 5 
R s
  (a) 82 (b) 76
(c) 86 (d) 78

a th

SSC CGL 08/12/2022 (Shift- 03) x 2 y2 z 2


72. If x + y + z = 0, then the value of + + is:
(a) 5 (b) 1 yz zx xy
(c) 0 (d) (a + b + c)
ty a

67. If 4(z + 7) (2z – 1) = Az² + Bz + C, then the x2 y2 z2


x + y + z = 0 yz + zx + xy
value of A + B + C is:
di M

4(z + 7) (2z – 1) = Az² + Bz + C A +


B + C SSC CGL 09/12/2022 (Shift- 01)
SSC CGL 08/12/2022 (Shift- 03) (a) 3 (b) 1
(a) 28 (b) 32 (c) 0 (d) 2
(c) 74 (d) 81
68. Factorize the following expression:  p – q 3 +  q – r  3 +  r – p  3
73. The value of , where
p³ + 27 12  p – q   q – r   r – p 
p  q  r, is equal to:
p³ + 27
SSC CGL 08/12/2022 (Shift- 03) p – q 3 + q – r 3 + r – p3

A

(a) (p – 3) (p² – 3p + 9) 12  p – q   q – r  r – p 
(b) (p + 3) (p² + 3p + 9)
p  q  r
(c) (p + 3) (p² – 3p + 9)
(d) (p + 3) (p² – 3p – 9) SSC CGL 09/12/2022 (Shift- 01)

1 1 1
69. If K + = 3, then what is the value of (a) (b)
K 9 3

1 1 1
k2 + ? (c) (d)
k2 4 2

Aditya Ranjan (Excise Inspector) Selected Selection 129


https://sscstudy.com/
Join Telegram- Maths by Aditya Ranjan ALGEBRA

74. What is the value of (a + b)³ – a³ + b³ ? 1 1


79. If y + = 11, then the value of y3 – 3 is:
(a + b)³ – a³ + b³ y y
SSC CGL 09/12/2022 (Shift- 01) 1
y + = 11 y3 – 1
(a) –3ab (a – b) + 2b³ y y3
(b) 3ab(a + b) + 2b³ SSC CGL 09/12/2022 (Shift- 04)

(c) 3ab(a – b)b³ (a) 345 13 (b) 360 13

(d) –3ab(a + b) + b³ (c) 352 13 (d) 368 13


2
1 1  1
75. If x 2 + = 66, then value of x – is____ 80. If  x –  = 12, what is the value of
x2 x x

1 1  2 1 
x 2 + = 66 x – ____  x – 2  , given that x > 0?
x 2
x x
2

r
SSC CGL 09/12/2022 (Shift- 02)   1
 x –  = 12 x > 0
(a) 10 (b) 8  x

si
(c) 9 (d) 6  2 1 
an by  x – 2 
x
1
76. If k + = 4, then what is the value of SSC CGL 09/12/2022 (Shift- 04)

n
k
(a) 62 (b) 83
1 (c) 63 (d) 82
k4 + 4 ? ja 81. a³ + b³ + c³ = 3abc if____
R s
k
a³ + b³ + c³ = 3abc ____
a th

1 1 SSC CGL 09/12/2022 (Shift- 04)


k + = 4 k 4 + 4
k k (a) a = b =c (b) a + b + c = 0
(c) a + b +c=0 (d) a + b + c = –1
SSC CGL 09/12/2022 (Shift- 02)
ty a

1
(a) 410 (b) 192 82. If m + = 4, then find the value of (m – 2)²
m–2
(c) 212 (d) 194
di M

1
77. If p + q = 6 and pq = 4, then what is the value + .
of 3(p³ + q³)?
 m – 2 2

p + q = 6 pq = 4 3(p³ + q³) 1 1
m + = 4 (m – 2)² +
m–2  m – 22
SSC CGL 09/12/2022 (Shift- 02)
SSC CGL 12/12/2022 (Shift- 01)
(a) 512 (b) 144 (a) –2 (b) 4
(c) 288 (d) 432 (c) 0 (d) 2
5 1
1 1 83. If 5x – + 6 = 0, then x ² + 2 is:
A

78. If p – = 6, the what is the value of p4 + 4 ? x x


p p
5 1
5x – + 6 = 0 x ² + 2 = ?
1 1 x x
p – = 6 p4 + 4 SSC CGL 12/12/2022 (Shift- 01)
p p
86 43
SSC CGL 09/12/2022 (Shift- 03) (a) (b)
25 12
(a) 1562 (b) 1432
81 86
(c) 1442 (d) 1444 (c) (d)
10 11

Aditya Ranjan (Excise Inspector) Selected Selection 130


https://sscstudy.com/
Join Telegram- Maths by Aditya Ranjan ALGEBRA

84. If a = 26 and b = 22, then the value of


16
89. If x 4 + = 15617, x > 0, then find the value
a –b
3
3ab
3
x4
– is.
a 2 – b2 a + b
2
of x + .
a 3 – b3 3ab x
a = 26 b = 22 –
a 2 – b2 a + b
16 2
x 4 + = 15617, x > 0 x +
x 4
x
SSC CGL 12/12/2022 (Shift- 01)

5 13
(a) (b) SSC CGL 12/12/2022 (Shift- 03)
3 11
(a) 121 (b) 129
1 11
(c) (d)
3 13
(c) 123 (d) 127

r
1 1
85. If a + = 5, then what is the value of a 3 + 3 ? 90. If x = 222, y = 223 and z = 224, then find

si
a a
the value of x³ + y³ + z³ – 3xyz.

a +
an by 1
a
1
= 5 a + 3
3

a
x = 222, y = 223 z = 224 x³ + y³ +
z³ – 3xyz

n
SSC CGL 12/12/2022 (Shift- 02)
SSC CGL 12/12/2022 (Shift- 04)
(a) 110 ja (b) 15
R s
(c) 105 (d) –10 (a) 2007 (b) 2004
a th

86. What is the value of a² + b² + c² – 2ab – 2bc (c) 2006 (d) 2005
+ 2ca?
91. If ,  are the roots of 6x² + 13x + 7 = 0, then
a² + b² + c² – 2ab – 2bc + 2ca the equation whose roots are ², ² is:
ty a

SSC CGL 12/12/2022 (Shift- 02)


6x² + 13x + 7 = 0 , 
(a) (2a + b + c)² (b) (a – b + c)²
di M

², ²
(c) (a – b – 2c)² (d) (a + 2b – c)²
SSC CGL 12/12/2022 (Shift- 04)
x 8
87. If + = 1, then the value of x³ is:
8 x (a) 36x² – 87x 49 = 0

x 8 (b) 36x² – 85x + 49 = 0


+ = 1 x³
8 x (c) 36x² – 85x – 49 = 0
SSC CGL 12/12/2022 (Shift- 02)
(d) 36x² + 87x – 49 = 0
(a) –256 (b) –512
92. If x + y + z = 0 and x² + y² + z² = 40, then
(c) 256 (d) 512
A

what is the value of xy + yz + zx?


88. If p = 8.15, q = 9.06 and r = –17.21, then the
value of p³ + q³ + r³ – 3pqr is: x + y + z = 0 x² + y² + z² = 40
p = 8.15, q = 9.06 r = –17.21 xy + yz + zx
p³ + q³ + r³ – 3pqr SSC CGL 12/12/2022 (Shift- 04)
SSC CGL 12/12/2022 (Shift- 03)
(a) –20 (b) 5
(a) –3.81 (b) –5.62
(c) –5 (d) –10
(c) 4.75 (d) 0

Aditya Ranjan (Excise Inspector) Selected Selection 131


https://sscstudy.com/
Join Telegram- Maths by Aditya Ranjan ALGEBRA

93. Which of the following statement is correct? 97. If 2p + q = 19 and 8p³ + q³ = 361, then find
the value of pq.
I. If x = 12, y = –2 and z = –10, then x³ + y³ 2p + q = 19 8p³ + q³ = 361 pq
+ z³ = 360.
x = 12, y = –2 z = –10 x³ +
SSC CGL 13/12/2022 (Shift- 02)
y³ + z³ = 360
II. If x + y = 48 and 4xy = 128, then 4x² + (a) 56 (b) 59
4y² = 4480. (c) 58 (d) 57
x + y = 48 4xy = 128 4x² + 98. Which of the following statement is correct?
4y² = 4480

SSC CGL 13/12/2022 (Shift- 01)
I. If x = 12, y = –2 and z = –10, then x³ + y³
(a) Neither I nor II (b) Only I
+ z³ = 720
(c) Both I and II (d) Only II
x = 12, y = –2 z = –10 x³ +
1

r
94. If K + + 2 = 0 and K < 0, then what is the y³ + z³ = 720
K

si
II. If x + y = 48 and 4xy = 128, then s the
1 value of 4x² + 4y² is 8960
value of K an by 11
+ 4?
K
x + y = 48 4xy = 128 4x² +
1 1 4y²

n
K + + 2 = 0 K < 0 K11 + 4
K K SSC CGL 13/12/2022 (Shift- 02)
ja

R s
(a) Neither I nor II (b) Only I
SSC CGL 13/12/2022 (Shift- 01)
(c) Both I and II (d) Only II
a th

(a) 0 (b) –2
(c) –1 (d) –17 r 13
99. If + = 1 then the value of r³ is:
1 13 r
ty a

95. If x+ = 6, then find the value of


x
r 13
+ = 1 r³
di M

3x 13 r
.
2x – 5x + 2
2

1 3x SSC CGL 13/12/2022 (Shift- 03)


x + = 6
x 2x – 5x + 2
2

(a) –2157 (b) 2197



(c) 2157 (d) –2197
SSC CGL 13/12/2022 (Shift- 01)

3 a b
(a) 1 (b) 100. If + = –1 and a – b = 2 then the value a³ –
7 b a
b³ is:
2
A

(c) (d) 0 a b
3 + = –1 a – b = 2 a³ – b³

b a
96. If a – b = 8 and ab = 9, then the value of a +
b is.
a – b = 8 ab = 9 a + b SSC CGL 13/12/2022 (Shift- 03)

1
SSC CGL 13/12/2022 (Shift- 02) (a) 0 (b)
2
(a) ±9 (b) ±7
(c) ±8 (d) ±10 (c) 1 (d) –1

Aditya Ranjan (Excise Inspector) Selected Selection 132


https://sscstudy.com/
Join Telegram- Maths by Aditya Ranjan ALGEBRA

1 2 SSC CGL 13/12/2022 (Shift- 04)


101. If x = 2 – 2 3 + 2 3 , then find the value of x³ – (a) 554 (b) 228
6x² + 18x. (c) –288 (d) 144
1 2 1 1
x = 2 – 2 3 + 2 3 , x³ – 6x² + 18x 104. If + x = 4, then find 2 + x 2 .
x x
1 1
SSC CGL 13/12/2022 (Shift- 04) + x = 4, 2 + x 2 .
(a) 40 (b) 33 x x
SSC CGL TIER - II 02/03/2023
(c) 45 (d) 22
(a) 14 (b) 5
102. If a + b + c = 5, a³ + b³ + c³ = 85 and abc =
(c) 7 (d) 15
25, then find the value of a² + b² + c² – ab –
105. What is the simplifie form of the expression
bc – ca.
(1 + x) (1 + x²) (1 + x4) (1 + x8) (1 – x)?
a + b + c = 5, a³ + b³ + c³ = 85 abc = (1 + x) (1 + x²) (1 + x4) (1 + x8) (1 – x)
25 a² + b² + c² – ab – bc – ca


SSC CGL TIER- II 03/03/2023
SSC CGL 13/12/2022 (Shift- 04)
(a) 1 – x40 (b) 1 – x32
(a) 2 (b) 4

r
(c) 1 – x 16
(d) 1 – x42
(c) 6 (d) 8
106. If 4x² + y² = 40 and xy = 6, then find the value
103. If a = –12, b = –6 and c = 18, then what is

si
of 2x + y.
2abc 4x² + y² = 40 xy = 6 2x + y
the value of
an by 9
2abc SSC CGL TIER- II 06/03/2023

n
a = –12, b = –6 c = 18
9 (a) 8 (b) 6
(c) 4 (d) 5
ja
R s

ANSWER KEY
a th

1.(c) 2.(a) 3.(c) 4.(b) 5.(d) 6.(a) 7.(d) 8.(c) 9.(c) 10.(d)
ty a

11.(b) 12.(a) 13.(c) 14.(b) 15.(d) 16.(c) 17.(c) 18.(d) 19.(b) 20.(c)
di M

21.(c) 22.(d) 23.(a) 24.(b) 25.(d) 26.(b) 27.(b) 28.(d) 29.(d) 30.(b)

31.(c) 32.(a) 33.(a) 34.(d) 35.(c) 36.(a) 37.(c) 38.(d) 39.(c) 40.(c)

41.(a) 42.(a) 43.(a) 44.(c) 45.(c) 46.(c) 47.(a) 48.(d) 49.(d) 50.(d)

51.(d) 52.(b) 53.(b) 54.(c) 55.(b) 56.(a) 57.(a) 58.(b) 59.(a) 60.(d)

61.(d) 62.(a) 63.(a) 64.(d) 65.(b) 66.(b) 67.(b) 68.(c) 69.(c) 70.(d)
A

71.(b) 72.(a) 73.(c) 74.(b) 75.(b) 76.(d) 77.(d) 78.(c) 79.(b) 80.(b)

81.(b) 82.(d) 83.(a) 84.(c) 85.(a) 86.(b) 87.(b) 88.(d) 89.(b) 90.(a)

91.(b) 92.(a) 93.(a) 94.(a) 95.(b) 96.(d) 97.(d) 98.(c) 99.(d) 100.(a)

101.(d) 102.(a) 103.(b) 104.(a) 105.(c) 106.(a)

Aditya Ranjan (Excise Inspector) Selected Selection 133


https://sscstudy.com/
Join Telegram- Maths by Aditya Ranjan ALGEBRA

SOL U TION S
1. (c) 5. (d)
x² – 5x + 1 = 0 ..........(1) 1
x =5 2
Consider, x
1
 
2
x 6  x 4  x²  1 1  1 1 x²  = 5 2 –2
= x³  x    x²
5x³ 5 
 x x³  = 48
1
1  1  1  x4  = (48)² – 2
= 
 x 
 
 x³  
 x4
5 x  x ³  
= 2302
6. (a)

r
From (1)
a + b = 11, ab = 35

si
1 We know,
x – 5 =0
x a4 + b4 = (a² + b²)² – 2a²b²

 x+
1
an by
= 5  x³ 
1
= 5³ – 3  5 = 110
a² + b² = 121 – 70 = 51
 a4 + 64 = 2601 – 2450

n
x x³ = 151
7. (d)

ja
x 6  x 4  x²  1 1
= [5  110] = 23 x + y = 36  x – 27 + y – 9 = 0
R s
5x³ 5 Consider,
a th

2. (a) (x – 27)³ + (y – 9)³ = (x – 27° + y – 9) (x – 27)° +


(y – 9)³ – (x – 27) (y – 9)
(17)³  7³ =0
= 24
17²  7² – k 8. (c)
ty a

 a³ + b³ = (a + b) (a² + b² – ab) 1 1
y = 3  y³  = 3³ – 3  3 = 18
y y³
di M

(17  7) (17²  7² – 17  7)
 = 24 Consider,
(17²  7² – k)
1
 k = 17 × 7 = 119 y³ + y³ + 2 = 18 + 2 = 20
3. (c) 9. (c)
p + q = 6, pq = 4
4 [17³ – 7³]
= 40 p³ + q³ = (p + q)³ – 3pq (p + q)
17²  7²  P
= 6³ – 12 (6)
= 144
4 (17 – 7) (17²  7²  17  7)
 = 40 10. (d)
17²  7²  P
A

5–2 1 52
 P = 119 x= , =
52 x 5–2
4. (b)
1 5–2 52
1 x = 
k = –2 x 52 5–2
k
 k=–1 1 18
x =
x 1
k²  4k – 2 1 – 4 – 2
 = =1 1
k²  k – 5 1–1– 5 x = 18² – 2 = 322

Aditya Ranjan (Excise Inspector) Selected Selection 134


https://sscstudy.com/
Join Telegram- Maths by Aditya Ranjan ALGEBRA

11. (b) 16. (c)

1 1
x4  = 2207 k =3
x4 k
1
1 k³  = 3³ – 3  3 = 18
x²  = 2207  2 = 2209 = 47 k³

17. (c)
1
x  = 47  2 = 7 5
x 4a   5 = 14
a
1 5
 x³ + = 7³ – 3 × 7 = 322 4a  =9
x³ a

1  1  4 1   1 25
 x7 + =
 x³   x  4 
 –
x  
  16a² + = 9² – 2  4  5
x 7
 x³   x   x a²
= 41
= 322 × 2207 – 7

r
18. (d)
= 710647

si
1
12. (a) x– = 13
an by x
a + b = 5, ab = 6
1
3 (a² + b²) = 3 [(a + b)² – 2ab]  x²+ = 13²  2 = 171

n

= 3 [25 – 12]
1
= 39 ja  x4 + = 171² – 2 = 29239
R s
x4
13. (c) 19 (b)
a th

a³ + b³ + c³ – 3 abc = (a + b + c) [a² + b² + c² – x4 + x² + 25, Let x² = a


(ab + bc + ac)]  a² + a + 25 = a² + 10a + 25 – 9a
= (6) [14 – 11]
 a
2
 (a + 5)² –
ty a

= 18
14. (b)  a  5 – 3 a a  5  3 a 
di M

1  (x² + 5 – 3x) (x² + 5 + 3x)


k4  = 194
k4 20. (c)

1 1
k2  = 196 = 14 x = 2 cos
k2 x
1
1 x³  = 8 cos ³ – 6 cos
k = 16 = 4 x³
k
= 2 [4cos³ – 3cos]
1 = 2 cos3
 k³ + = 4³ – 12
k³ 21. (c)
A

= 52 x = 32 2
15. (d)
1
=3–2 2
1 x
x = 8  x² + 1 = 8x
x 1
x =6
Consider, x

5 5 1
= =5  x – = 6–2=2
x² – 8x  2 –1  2 x

Aditya Ranjan (Excise Inspector) Selected Selection 135


https://sscstudy.com/
Join Telegram- Maths by Aditya Ranjan ALGEBRA

22. (d) 26. (b)


12³ + (–8)³ + (–4)³
1
x = –2 We know,
x
a³ + b³ + c³ = 3abc, if a + b + c = 0
 x=–1 Here,
 x17 + x–17 + x12 + x–12 12 – 8 – 4 = 0
=–1–1+ 1+1=0  (12)³ + (–8)³ + (–4)³ = 3× 12 × 8 × 4
23. (a) = 1152
27. (b)
(x  y) = (x– y)²  4xy .........(1)
Squaring, we get
Also, (mx + n)² = m² x² + n² + 2 mnx
(x – y)² = x² + y² – 2xy 28. (d)
1 = 41 – 2xy Consider,
 xy = 20 .........(2) x² y² z²
 
Put (2) in (1) yz xz xy

r
(x  y) = 1  80 x³ y³ z³
=  

si
= 81 = 9 xyz xyz xyz
24. (b) an by x³  y³  z³ 3xyz
= = =3
k – k cot²30 xyz xyz
= sin²60 + 4 tan²45 – cosec²60°

n
1  cot ²30 [ x + y + z = o  x³ + y³ + z³ = 3 xyz]
LHS:- 29. (d)
ja (a + b + c)² = a² + b² + c² + 2 (ab + bc + ca)
R s
k – 3k –k
= =9+2×8
1 3 2
a th

= 25
RHS:-  a+b+c=5
3 4 9  48 – 16  (a + b + c) – 3 = 2
4– = 30. (d)
4 3 12
ty a

1
41 x = –14
= x
di M

12
1
–k 41 – 41 x²  = 196 + 2 = 198
 =  = = – 6.83 x²
2 12 6
25. (d) 1
x² – = (198)² – 4
8a³ + 27b³ = 16, 2a + 3b = 4 x²
We know, = 39200
(2a + 3b)³ = 8a³ + 27b³ + 3 × 2a × 3b (2a + 3b)
= 2  19600
 64 = 16 + 18 ab (4) = 140 2
48 2 31. (c)
A

 = ab =
42 3 1
k4  = 47
 (2a)² + (3b)² = (2a + 3b)² – 12ab k4

2 1
= 16 – 12 =8 x²  = 47  2 = 7
3 k²
 16a4 + 81b4 = (8)² – 2 × 4a² × 96² 1
k = 72 =3
4 k
= 64 – 72 
9 1
 k³ + = 3³ – 3 × 3 = 18
= 32 k³

Aditya Ranjan (Excise Inspector) Selected Selection 136


https://sscstudy.com/
Join Telegram- Maths by Aditya Ranjan ALGEBRA

32. (a) 36. (a)


(x + 2y)³ + (x – 2y)³ (a – 1) (b – 1) = ab – a – b + 1
a³ + b³ = (a + b) (a² + b² – ab)  option (a) is correct.
= (2x) [x² + 4y² + 4xy + x² + 4y² – 4xy – x² + 37. (c)
4y²]
100² – 99² + 98² – 97² +96² – 95² +.......+ 12² – 11²
= (2x) [x² + 12y²]  (100 – 99) (100 + 99) + (98 – 97) (97 + 97) +
 2x³ + 24 xy² (96 – 95) (96 + 95) +........ + (12 – 11)
(12 + 11)
33. (d)
 100 + 99 + 98 + 97 + 96 + 95 +.......12 + 11
a³ – b³ = (a – b)³ – 3ab (a – b)
80 = 8 – 3ab (2) 100×101 10  11
 –
2 2
72  5050 – 55
= – ab  ab = – 12
6

r
 4995
34. (d) 38. (d)

si
1 x² + y² + z² = xy + yz + zx
x ²  a²
an by
= x² – a²
 x=y=z=1

n
 x4 – a4 = 1 10x 4  5y 4  7z 4 10  5  7
 =
 x4 = 1 + a4 13x²y²  6y²z²  3z²x² 13  6  3
ja
R s
1 =1
 x = (1 + a 4 ) 4
a th

39. (c)
35. (c)
1
x =3
I. 100² – 99² + 98² – 97² +96² – 95² +94² – 2x
ty a

93² +...........+22² – 21²


 (100 – 99) (100 + 99) + (98 – 97) (98 + 97) 1
di M

 2x + =6
+ ........ + (22 – 21) (22 + 21) x

= 100 + 99 + 98 + 97 +........ + 22 + 21  1 3
 (2 x)³  
   = 6³ – 3  6  2
100  101 20  21 x 
= –
2 2 = 180

= 5050 – 210 = 4840. 40. (c)

 It is true x² – 2 xy = 84
 x (x – 2y) = 84 .........(1)
 1  1  4 1  1  4 1 
II. 
k²  k – 
 k  4 
k  
k – 4 
 x – y = – 10
A

 k²   k  k  k  k 
 x = y – 10 .......(2)
 1  1  4 1  4 1  Put (2) in (1)
=
k²  k² –
 k  4 
 k – 4 

 k²   k²   k  k  (y–10) (y–10–2y) = 84
– (y–10) (y+10) = 84
 4 1  8 1   16 1 
=
k – 4 
k – 8  k – 16 
  y² – 100 = – 84
 k  k   k 
y² = 16
 It is true y=4

Aditya Ranjan (Excise Inspector) Selected Selection 137


https://sscstudy.com/
Join Telegram- Maths by Aditya Ranjan ALGEBRA

41. (a) 45. (c)


1 4a² + 9b² = (2a + 3b)² – 2 × 2a × 3b
x =1
x = 100 – 12 × 3
Consider, = 64
x12 + x9 + x6 + x³ + 1
46. (c)
6 1 1 16x² + 49y² = (4x – 7y)² + 2 × 4x × 7y
 x + x³ + 1 + 
x³ x 6 = 121 + 448
 6 1   1 = 569
x  6 
   x³ 
  1
 .......(1)
 x   x³  47. (a)
Now,
 1  1  4 1  8 1 
1 k – 
 k²  k  4 
 k  8 

x  =1  k  k²   k  k 
x
 16 1  32 1 
1 k  16 
 k  32 

 x³ + =1– 3 = –2  k  k 

r
1  1  1  1  4 1  
 x6  =4–2=2 
k –  k² 
 k  4   
k 
 

si
x6    
x x x²   x 
=   
1 1  1  8 1   16 1   1 

42.
 x6 +

(a)
an by
x6
 x³ 

2–2+1=1 k 




x 
k  8 

 x 
k  16 

x 

32
k  32  
x  

n
1
a b k 64 –
 =1 k 64
b a ja =
1 [Using (a – b) (a + b) = a² – b²]
R s
 a² + b² = ab, a + b = 2 k
k
(a + b)² = a² + b² + 2ab
a th

48. (d)
 4 = 3ab
1 1
4 x = 2, x 57  57  1  1  2
ab = x x
3
ty a

=x=1
Consider,
a³ + b³ = (a + b) (a² + b² – ab) 49. (d)
di M

= 2 (0) = 0 mxm – nxn = 0


43. (a) nx n
4x² + y² = 40, xy = 6 xm =
m
(2 x  y) = 4x²  y²  2  2xy Consider,

= 40  24 1 1

xm  xn xm – xn
= 64 = 8
44. (c) 1 1
 
1 n n x
P = 74 3  =7–4 3 x  xn xn – xn
P m m

P6  P4  P2  1 1 1 m m
A

= P3  P   3  
P3 P P nx n  mx n nx n – mx n

1 m m
P = 14  
P x n (n  m) x n (n – m)
1 m (n– m)  m (n m)
 P³ + = 14³ – 14  3 
P³ x n (n² – m ²)
P 6  P 4  P²  1
 = 14³ – 14  3  14 2mn
P³  n
x (n ² – m ²)
= 2716

Aditya Ranjan (Excise Inspector) Selected Selection 138


https://sscstudy.com/
Join Telegram- Maths by Aditya Ranjan ALGEBRA

50. (d) 55. (b)


5x² + 7x + 5 = 0  1  1
3

3 a –   a – 
5 
 a  
 a 
5x  7  =0
x  1  1  1  

 3 a –    a³ – – 3 a –  
1 –7 
 a  
 a³ 
 a  

 x+ =
x 5 1
 a³ –

1 – 343 21
 x³ + =  56. (a)
x³ 125 5
1
x4  = 194
182 x4
=
125
1
x²  = 194  2 = 14
51. (d) x²
x² + y² + 2y + 4x + 5 = 0 1

r
x = 14  2 = 4
x² + 4x + 4 + y² + 2y + 1 = 0 x

si
(x + 2)² + (y + 1)² = 0
1
 x = – 2, y = – 1  x³ + = 4³ – 12
an by x³
x  y –3 = 52
 = =3 57. (a)

n
x – y –1
36x² – 108x  81
52. (b) ja = 9(4 x ² – 12 x  9)
R s
(x – y)³  (y– z)³  (z – x)³
(1)
= 3 (2 x– 3)²
a th

6(x – y) (y – z) (z – x)
= 3 (2x – 3)
Consider,
= 6x – 9
(x – y) + (y – z) + (z – x) = 0 58. (b)
ty a

 (x – y)³ + (y – z)³ + (z – z)³ = 3 (x – y) (y – z) 1


(z – x) x =7
di M

x
So, equation (1) we come:-
1
x³  = 343 – 7  3
3(x– y) (y– z) (3 – x) 1 x³
=
6(x– y) (y– z) (z– x) 2 = 322
59. (a)
53. (b)
1
I. k = 12
1 k
k  = –2
k
1
 k=–1 x²  = 144 – 2

1 = 142
A

 k10 – = –1  1 = 0  It is true
k11
 1  1  4 1  1
(Typing error in question) k² 
II.  k – 
 k  4 
k  

 k²   k k  k
54. (c)
 1  1  4 1 
1 k² 
  k² –
 k  4 
 
x = 2  x =1  k²   k²   k 
x
 4 1  4 1  1 1
k – 4 k  4 
 = k – 8  k – 16
8 16
1  
 x³ + =2  k  k  k k

 It is not true

Aditya Ranjan (Excise Inspector) Selected Selection 139


https://sscstudy.com/
Join Telegram- Maths by Aditya Ranjan ALGEBRA

60. (d) 64. (d)


(a + b)² – (a – b)²
5– 4 5 4
= 2ab + 2ab x= = 9 – 4 5, y = = 9 4 5
5 4 5– 4
= 4ab
We know,
61. (d) x² + y² = (x + y)² – 2xy = 18² – 2xy = 3zy – 2xy
9m² + n² = 40, mn = 4 = 322
Consider,
 3m + n = (9 m ²  n ²  6 mn)
x² – xy  y² 322 – 1 321
= =
= 40  24 x²  xy  y² 322  1 323

= 64 = 8 65. (b)
62. (a) 2P 1
=
x² + 4y² + 4y – 4xy – 2x – 8 P² – 5P  1 10

r
We will check from options.
2 1 1
 =  20 = P + – 5

si
Option (a) satisfies the equation 1
P – 5 10 P
an by P
Coefficient of x Coefficient of y Constant
1
 25 = P +

n
1 –2 –4 P

1 ja –2 2 66. (b)
R s
Consider,
x²  1, y² = 4, constant = – 8, xy = – 4
a th

1 1 1
y = + 4, x = – 2  
1 5 c 1
1 a  1 b  1 
63. (a) b c 5 a
ty a

y = 1 3  4 = 1 3  2 = 3  3 abc = 5 Let, a = 1, b = 1, c = 5

1 1 1
di M

y–3= 3    =1
3 3 3
Squaring both sides 67. (b)
y² + 9 – 6y = 3 4 (z + 7) (2z – 1) = Az² + Bz + C
 y² = 6y – 6 .........(1)  8z² – 4z + 56z – 28 = Az² + Bz + C
Squaring both sides again:-  8z² + 52z – 28 = Az² + Bz + C
 y4 = 36 [y² + 1 – 2y] .........(2)  A = 8, B = 52, C = – 28
 A + B + C = 8 + 52 – 28
y³ = y.y² = 6 (y² – y) .........(3)
= 32
Putting (1), (2) and (3) in given equation
68. (c)
 2y4 – 8y³ – 6y² + 28y – 84 = 72y² + 72 –
A

P³ + 27 = P³ + 3³
144y – 48y² + 48y – 6y² + 28y – 84
= (P + 3) (P² + 9 – 3P)
= 18y² – 68y – 12
[ a³ + b³ = (a + b) (a² + b² – ab)]
= 108y – 108 – 68y – 12
69. (c)
= 40y – 120
1
k =3
= 40 [y – 3] = 40  
3  3 – 3 

k
1
k²  =9–2=7
= 40 3 k²

Aditya Ranjan (Excise Inspector) Selected Selection 140


https://sscstudy.com/
Join Telegram- Maths by Aditya Ranjan ALGEBRA

70. (d) 77. (d)


a² + b² + c² = (a + b + c)² – 2 (ab + bc + ca) p² + q² = (p + q)² – 2pq
= 81 – 2 × 23 = 36 – 8
= 35 = 28
71. (b)  3 (p³ + q³) = 3[(p + q) (p² + q² – pq)]
= 3 [6[28 – 4]]
1
x =2 5 = 432
x
78. (c)
1
2 5 
2
 x– = – 4 = 16 = 4 1
x p– =6
p
1 2
x³ – = 4³  3  4 1   1

x³ p²  = p–  2
p²  p
= 76
= 38
72. (a)

r
1
x + y + z = o  x³ + y³ + z³ = 3xyz p4  = 1444 – 2

si
p4
Consider,
= 1442

 
yz zx xy
an by
x² y² z² x³  y³  z³
=
xyz
79. (b)
1 1

n
y = 11  y – = 121 – 4
=3 y y
73. (c) ja 1
 
3
R s
y³ – = 3 13 – 3  3 13
Consider, y³
a th

(p – q) + (q – r) + (r – p) = 0
= 351 13  9 13
 (p – q)³ + (q – r)³ + (r – p)³ = 3 (p – q) (q – r) (r – p)
= 360 13
(p– q)³ (q– r)³  (r– p)³ 3 1 80. (b)
ty a

 = =
12 (p– q) (q– r) (r– p) 12 4  2
1 1

 x –  = 12  x – = 2 3

di M

74. (b)  x x
(a + b)³ – a³ + b³ Now,
a³ + b³ + 3ab (a + b) – a³ + b³ 1  1  1
x² – =
 x  
 x – 

2b³ + 3ab (a + b) x²  x x
75. (b)
= 42 3
1 =8 3
x²  = 66
x² 81. (b)
a³ + b³ + c³ = 3abc
 1 if a + b + c = o
x – 
  = 66 – 2 = 8
 x 82. (d)
A

76. (d) 1
m =4
m–2
1
k =4
k 1
 (m – 2)+ =2
(m – 2)
1
x²  = 14  (m – 2) = 1
k²  m=3
1 1
k4  = 194  (m – 2)² + =2
k4 (m – 2)²

Aditya Ranjan (Excise Inspector) Selected Selection 141


https://sscstudy.com/
Join Telegram- Maths by Aditya Ranjan ALGEBRA

83. (a) 89. (b)

 1  2 4
5
x – 
= –6 x4  
 x    = 15617
x 
2
1  1  2 2
x²  =
 x –  2
 x²  
   = 15617  8 = 15625 = 125
x²  x x 

36 2
= 2 x = 125  4 = 129
25 x
86 90. (a)
=
25 We know that,
84. (c)
1
a³ + b³ + c³ – 3abc = (a + b + c) [(a – b)² + (b – c)²
a³ – b³ 3ab 2

r
a² – b² a  b + (c – a)²]

si
a²  b²  ab 3ab 1
 – = (222 + 223 + 224) [1 + 1 + 4]
ab
an by ab 2

a²  b² – 2ab (a – b)² = 3 [669]

n
 =
ab a b  2007

ja 91. (b) 6x² + 13x + 7 = 0


(4)² 16 1
R s
= = =
48 48 3 –13 7
 = ,  =
a th

85. (a) 6 6

1  ² + ² = [ + ]² – 2
a =5
a
ty a

169 7
= –
1 36 3
a³  = 5³ – 3  5 = 110
di M


169 – 84
=
86. (b) 36
a² + b² + c² – 2ab – 2bc + 2ca
= (a – b + c)² 85
=
87. (b) 36

x 8 x 49
 =1 Let =y ²² =
8 x 8 36
1
y =1  y³ = – 1  x² – (² + ²) x + ² ² = o is therequired
y equation
A

 x 3 85 49
 
   = –1  x² – x =0
8 36 36
 x³ = – 512
 36x² – 85x + 49 = 0
88. (d)
Consider, 92. (a)
p + q + r = 8.15 + 9.06 – 17.21 (x + y + z)² = x² + y² + z² + 2 (xy + yz + xz)
=0 0 = 40 + 2 (xy + yz + xz)
 p³ + q³ + r³ – 3 pqr = 0  (xy + yz + xz) = – 20

Aditya Ranjan (Excise Inspector) Selected Selection 142


https://sscstudy.com/
Join Telegram- Maths by Aditya Ranjan ALGEBRA

93. (a) 99. (d)


I. x + y + z = 12 – 2 – 10 = 0
 x³ + y³ + z³ = 3xyz r 13
 =1
= 3 × 12 × (–2) (–10) 13 r
= 720
II. 4 (x² + y²) = 4 [(x + y)² – 2xy] 1
If x  =1
= 4 [(48)² – 64] x
= 8960
 both statements are false  x³ = – 1
94. (a)
 r 3
1  
   = –1
k = –2  k = – 1 13 
x
1  r³ = – 2197
k4  = –1  1 = 0
k4 100. (a)

r
95. (b)
a b

si
1  = –1
x =6 b a
x
Consider,

3x
an by 3
 a² + b² + ab = 0

n
=  a³ – b³ = (a – b) (a² + b² + ab)
2x² – 5x  2 2
2x – 5  =0
ja x
R s
101. (d)
3
=
a th

 1 1 2
2
x  –5 x = 2 – 2 3  23
 x

3 3  2 1 3
ty a

= =  (x – 2)³ = 2 – 2 3 
 3
12 – 5 7 
 
96. (d)
di M

(a b)   (a – b)²  4 ab  x³ – 8 – 3x × 2 (x – 2) = 2² – 2 –
2 1  2 1
=  64  36
3  23. 23 
 2 3
– 2 3

=  10  
97. (d)
8p³ + q³ = (2p + q)³ – 6pq (p + q) x³ – 6x² + 12x = 2 – 3× 2 (x – 2)
361 = 19³ – 6pq (19)
x³ – 6x² + 12x – 8 = 2 – 6x + 12
361 = 19 (361 – 6pq)
19 = 361 – 6pq  x³ – 6x² + 18x = 22
6pq = 342 102. (a)
A

114 a + b + c = 5, a³ + b³ + c³ = 85, abc = 25


pq = = 57
2
We know,
98. (c)
I. x + y + z = 0 a³ + b³ + c³ – 3abc = (a + b + c) (a² + b² + c² –
 x³ + y³ + z³ = 3xyz = 3 × 12 × (– 2) × (– 10) ab – bc – ca)
II. 4 (x² +y²) = 4 [(x + y)² – 2xy] 85 – 75 = 5 (a² + b² + c² – ab – bc – ca)
= 4 [(48)² – 64]
= 8960 2 = a² + b² + c² – ab – bc – ca
 Both I and II are correct.

Aditya Ranjan (Excise Inspector) Selected Selection 143


https://sscstudy.com/
Join Telegram- Maths by Aditya Ranjan ALGEBRA

103. (b) 105. (c)


a = – 12 , b = –6 , c = 18 (1 + x) (1 + x²) (1 + x4) (1 + x8) (1 – x)
abc  = (1 – x²) (1 + x²) (1 + x4) (1 + x8)
2
a
= (1 – x4) (1 + x4) (1 + x8)
  –12 ×  –6  ×18 = (1 – x8) (1 + x8)
2
9 = (1 – x16)
2 × 1296 106. (a)
9
4x² + y² = 40, xy = 6
2592 (2x + y)² = 4x² + y² + 4xy
 288
9
= 40 + 24 = 64
104. (a)
2x + y = 8
1

r
x=4
x

si
1
x²  = 4² – 2 = 14
x² an by ......-------......

n
ja
R s
a th
ty a
di M
A

Aditya Ranjan (Excise Inspector) Selected Selection 144


https://sscstudy.com/
Join Telegram- Maths by Aditya Ranjan TRIGONOMETRY

TRIGONOMETRY
19

cos45º sin30º sin40º sin50º sin60º
1. Simplify
sec30º +cosec30º cos30º cos40º cos50º cos60º
SSC CGL 01/12/2022 (Shift- 02)
cos45º
1
sec30º +cosec30º (a) (b) 3
2
SSC CGL 01/12/2022 (Shift- 01)
1
(c) 1 (d)
3 2+ 6 3 3

r
(a) (b) 5. The value of (sin30º cos60º – cos30º sin60º)
8 2 2–2 6
is equal to:

si
3 2– 6 3 (sin30º cos60º – cos30º sin60º)
(c) an by
8
(d)
2 6–2 2
SSC CGL 01/12/2022 (Shift- 02)

n
2. In a right-angled triangle PQR, right-angled
at Q, the length of the side PR is 17 units, (a) –cos30º (b) –sin30º
length of the base QR is 8 units, and length (c) cos30º (d) sin30º
ja 6. What is sin – sin ?
R s
of the side PQ is 15 units. If RPQ = , then
sin + cos is: sin – sin
a th

SSC CGL 01/12/2022 (Shift- 02)


PQRQ
PR 17 QR 8  + –
(a) 2 cos sin
PQ 15 2 2
ty a

RPQ =  sin + cos  + –


(b) 2 sin sin
di M

SSC CGL 01/12/2022 (Shift- 01) 2 2

–  +
18 23 (c) 2 cos sin
(a) (b) 2 2
17 17
 + –
21 15 (d) 2 cos cos
(c) (d) 2 2
17 17
2tan60º
3. The value of tan² + cot² – sec² cosec² is: 7. The value of is:
1 + tan2 60º
tan² + cot² – sec² cosec² 2tan60º

SSC CGL 01/12/2022 (Shift- 01) 1 + tan2 60º
A

(a) 2 (b) –2 SSC CGL 01/12/2022 (Shift- 03)


(a) cos60º (b) tan60º
1 (c) sin60º (d) sin30º
(c) 0 (d)
2 8. What is the value of tan240º?
4. What will be the value of tan240º
SSC CGL 01/12/2022 (Shift- 03)
sin30º sin40º sin50º sin60º (a) 2 (b) (–)3
?
cos30º cos40º cos50º cos60º (c) 3 (d) 3

Aditya Ranjan (Excise Inspector) Selected Selection 145


https://sscstudy.com/
Join Telegram- Maths by Aditya Ranjan TRIGONOMETRY

9. If tan + sec = 7, being acute, then the value


3
of 5sin is: 14. If cotx – tanx = , then what will be the value
2
of cotx + tanx?
tan  + sec = 7  5sin
3
cotx – tanx = , cotx + tanx
SSC CGL 01/12/2022 (Shift- 03) 2

25 24
(a) (b) SSC CGL 02/12/2022 (Shift- 01)
24 25
(a) 3 (b) 2
1 24
(c) (d) 5 7
24 5 (c) (d)
2 2
 
10. 16cos³ – 12 cos = . 4
6 6 15. I f sec²A + t an²A = , then sec4A – tan4A is
17
SSC CGL 01/12/2022 (Shift- 04)
equal to:

r
(a) 0 (b) 2
(c) 1 (d) –1 4

si
sec²A + tan²A = sec4A – tan4A
17
a 2 + b2 + c2 – 1024
11. If an by = –2 and a + b = 5c, where
ab – bc – ca
SSC CGL 02/12/2022 (Shift- 01)
c > 0, then the value of c is.

n
13 4
a 2 + b2 + c2 – 1024 (a) (b)
ja = –2 a + b = 5c 17 13
ab – bc – ca
R s
c > 0 c 4 5
(c) (d)
a th

SSC CGL 01/12/2022 (Shift- 04) 17 17


(a) 8 (b) 4 16. If sin(A + B) = cos(A + B), what is the value
(c) 12 (d) 5 of tanA?
ty a

9 sin(A + B) = cos(A + B) tanA


12. If cosA = , find cotA.
41
di M

9 SSC CGL 02/12/2022 (Shift- 01)


cosA = cotA
41
1 – tanB 1 + tanB
SSC CGL 01/12/2022 (Shift- 04) (a) (b)
1 + tanB 1 – tanB
9 41
(a) (b) 1 + secB 1 – cosecB
40 40 (c) (d)
1 – secB 1 + cosecB
40 9
(c) (d) 17. In ABC, right-angled at B, AB = 7 cm and
9 41
AC – BC = 1 cm. Find the value of sin C.
13. If tan² = 1 – a², then the value of sec + tan³
ABC, B
AB = 7 cm AC –
A

cosec is:
tan² = 1 – a² sec + tan³ cosec BC = 1 cm sinC
SSC CGL 02/12/2022 (Shift- 02)
SSC CGL 01/12/2022 (Shift- 04)
3 12
3 (a) (b)
7 13
a – 1
3
(a) 2 – a  2 (b)
2 2

3 3 3 7
(c) 2 – a 
2 2
(d) a 2 (c)
25
(d)
25

Aditya Ranjan (Excise Inspector) Selected Selection 146


https://sscstudy.com/
Join Telegram- Maths by Aditya Ranjan TRIGONOMETRY

18. If sin + cos = 5 sin(90 – ), find the value


1 – cos  1
of cot. 23. If = , then what will be the value
sin 5
sin + cos = 5 sin(90 – ) cot
1 + cos  
of ?
sin 
SSC CGL 02/12/2022 (Shift- 02)
1 – cos  1 1 + cos 
5 –1 5 +1 = ,
(a) (b) sin 5 sin
5 4

5 +1 5 –1 SSC CGL 02/12/2022 (Shift- 04)
(c) (d)
3 4 2
(a) 5 (b)
1 5
19. If cosecA – cotA = , then the value of tanA is:
4 4 1
(c) (d)
1 5 5
cosecA – cotA = tanA

r
4 4 15
24. If sinA = and sin B = , what is the value
SSC CGL 02/12/2022 (Shift- 02) 5 17

si
of sin(A – B)?
8 8
(a)
15
an by (b)
17 sinA =
4
5
sin B =
15
17
sin(A – B)

n
15 17
(c) (d)
17 15
SSC CGL 02/12/2022 (Shift- 04)
20.
ja
The value of cot15º cot25º cot45º cot75º
R s
32 13
cot65º is: (a) – (b) –
45 85
a th

cot15º cot25º cot45º cot75º cot65º


13 32
(c) (d)
85 45
SSC CGL 02/12/2022 (Shift- 03)
ty a

(a) 1 (b) 3 cosA sinA


25. Simplify: –
(c) 2 (d) 2 1 + tanA 1 + cotA
di M

21. In triangle PQR, right angled at Q, if cotP = cosA sinA


3, then the value of sinP is: –
1 + tanA 1 + cotA
PQR, Q cotP = SSC CGL 03/12/2022 (Shift- 01)
3 sinP (a) tanA (b) cosA – sinA
SSC CGL 02/12/2022 (Shift- 03) (c) cosA. sinA (d) cosA + sinA
1 1
1 26. + =
(a) 1 (b) 1 + cos  90º –   1 – cos 90º –   ____.
3
SSC CGL 03/12/2022 (Shift- 01)
3 1 (a) 2sec² (b) 1
(c) (d)
A

2 2 (c) 0 (d) 2tan²


22. If tanA + cotA = 2, then the value of 2(tan²A 27. sin4 + cos4 in terms of sin can be written as:
+ cot²A) is: sin4 + cos4 sin
tanA + cotA = 2 2(tan²A + cot²A)
SSC CGL 03/12/2022 (Shift- 02)
(a) 2sin4 + 2sin² – 1
SSC CGL 02/12/2022 (Shift- 04)
(b) 2sin4 – 2sin²
(a) 1 (b) 4 (c) 2sin4 – 2sin² – 1
(c) 2 (d) 3 (d) 2sin4 – 2sin² + 1

Aditya Ranjan (Excise Inspector) Selected Selection 147


https://sscstudy.com/
Join Telegram- Maths by Aditya Ranjan TRIGONOMETRY

28. tan( – 14) is equal to: 33. ABC is a right-angle triagle at B. If tanA =
tan( – 14) 5
, then sinA + sinB + sinC will be equal to:
SSC CGL 03/12/2022 (Shift- 02) 12
(a) tan (b) –cot ABC, B tanA =
(c) cot (d) –tan 5
sinA + sinB + sinC
k – kcot²30º 12
29. If = sin²60º + 4tan²45º – cosec²60º,
1 + cot²30º SSC CGL 03/12/2022 (Shift- 04)
then the value of k (correct to two decimal
5 4
places) is: (a) 1 (b) 2
13 13
k – kcot²30º
= sin²60º + 4tan²45º – cosec²60º 1 1
1 + cot²30º (c) 3 (d) 2
13 13
k
3 –1
34. If sin + cos = , then what is the value

r
SSC CGL 03/12/2022 (Shift- 03) 2 2

si
(a) 5.55 (b) –6.83 of tan + cot?
(c) –5.58 an by (d) 6.83 3 –1
30. If cot B = 9, then what will be the value of
sin + cos = tan + cot
2 2
sec2 B?

n

cotB = 9 sec²B
SSC CGL 05/12/2022 (Shift- 01)
ja SSC CGL 03/12/2022 (Shift- 03)
(a) 8(3 – 2) (b) 12(3 – 2)
R s
98 92 (c) 12(3 + 2) (d) 8(3 + 2)
a th

(a) (b)
81 81 8
35. If sin = , then find the value of tan.
17
42 82
(c) (d)
81 81 8
ty a

sin = tan
17
cosx – 3sinx
di M

31. Simplify the following: SSC CGL 05/12/2022 (Shift- 01)


2
15 8
cosx – 3sinx (a) (b)
17 15
2
15 17
SSC CGL 03/12/2022 (Shift- 03) (c) (d)
8 15
    36. If sec2 = cosec ( – 36º), where 2 is an acute
(a) cot  – x  (b) sin  + x 
3 3 angle, find the value of .

    sec2 = cosec ( – 36º) 2


(c) cos  + x  (d) sin  – x  
3 3
A

SSC CGL 05/12/2022 (Shift- 01)


1 + cosA
32. The value of is: (a) 32º (b) 46º
1 – cosA
(c) 20º (d) 42º
1 + cosA
tan34º
1 – cosA 37. cos²35º + cos55º . sin35º + =.
cot56º
SSC CGL 03/12/2022 (Shift- 04) SSC CGL 05/12/2022 (Shift- 02)
(a) secA – tanA (b) cosecA + cotA (a) 2 (b) 3
(c) secA + tanA (d) cosecA + cotA (c) 4 (d) 1

Aditya Ranjan (Excise Inspector) Selected Selection 148


https://sscstudy.com/
Join Telegram- Maths by Aditya Ranjan TRIGONOMETRY

1 + cos 1 – cos sin + cos


38. + =. 44. If 8cot = 6, then the value of is :
1 – cos 1 + cos sin – cos

SSC CGL 05/12/2022 (Shift- 02) sin + cos


(a) 2sin (b) 2cos 8cot = 6
sin – cos
(c) 2cosec (d) 2sec SSC CGL 05/12/2022 (Shift- 04)
39. What will be the value of cosx cosecx – sinx
(a) 12 (b) 7
secx?
(c) 2 (d) 5
cosx cosecx – sinx secx
SSC CGL 05/12/2022 (Shift- 02) 5
45. If tanA = , then the value of cosA =.
12
2x
(a) cot (b) tan2x
2 5
tanA = cosA
(c) cot2x (d) 2cot 2x 12

2 SSC CGL 05/12/2022 (Shift- 04)

r
40. If tanA = , then what is the value of the
3 12 13

si
following? (a) (b)
5 5
(5 sin²A – 2 cos²A) ÷ (15 sin²A + 3 cos²A)
an by 5 12
2 (c) (d)
tanA = 13 13

n
3
46. If bsin = a, then sec + tan = ?
(5 sin²A – 2 cos²A) ÷ (15 sin²A + 3 cos²A)
ja bsin = a sec + tan = ?
R s
SSC CGL 05/12/2022 (Shift- 03)
SSC CGL 06/12/2022 (Shift- 01)
a th

21 2
(a) (b) b+a 1
47 87 (a) (b)
b–a b+a
2 3
ty a

(c) (d) 1 b–a


5 77 (c) (d)
b–a b+a
41. If  is an acute angle and tan + cot = 2,
di M

then the value of tan² + cot² + 2 tan5 cot4 cos20º cos


is: 47. + =.
sin70º sin  90º –  

tan + cot = 2
SSC CGL 06/12/2022 (Shift- 01)
tan² + cot² + 2tan  cot4
5

1
SSC CGL 05/12/2022 (Shift- 03) (a) –2 (b)
2
(a) 3 (b) 4
(c) 1 (d) 2 1
(c) – (d) 2
42. cot²A – cos²A is equal to: 2
cot²A – cos²A 4
48. What will be the value of sin10º – sin³ 10º?
3
A

SSC CGL 05/12/2022 (Shift- 03)


(a) cos²A . sin²A (b) cos²A . cot²A 4
sin10º – sin³ 10º
(c) 1 (d) 0 3
SSC CGL 06/12/2022 (Shift- 01)
1 + sinA
43. =.
1 – sinA 1 1
(a) (b)
SSC CGL 05/12/2022 (Shift- 04) 3 3 6
(a) cosecA – cotA (b) secA – tanA 1 3
(c) (d)
(c) secA + tanA (d) secA 2 3 6

Aditya Ranjan (Excise Inspector) Selected Selection 149


https://sscstudy.com/
Join Telegram- Maths by Aditya Ranjan TRIGONOMETRY

15 1
49. If cotA = , then what will be the value of tan2A? 54. If sin = , then the value of (3cos – 4cos³) is:
8 2
15 1
cotA = tan2A sin = (3cos – 4cos³)
8 2
SSC CGL 06/12/2022 (Shift- 02) SSC CGL 06/12/2022 (Shift- 03)
(a) 0 (b) 1
200 240 (c) 2 (d) –1
(a) (b)
161 161 7
55. If sec – 2cos = , where  is a positive acute
240 220 2
(c) (d)
173 171 angle, then the value of sec is:
cos37º 7
50. What is the value of ? sec – 2cos = 
sin53º 2
cos37º sec
SSC CGL 06/12/2022 (Shift- 04)
sin53º
(a) 6 (b) 8

r
SSC CGL 06/12/2022 (Shift- 02)
(c) 5 (d) 4

si
1 1 56. Find the value of tan3 if sec 3 = cosec (4 – 15º).
(a) (b) sec 3 = cosec (4 – 15º) tan3
2 an by 2
(c) 0 (d) 1

SSC CGL 06/12/2022 (Shift- 04)

n
 17   1
51. What is the value of cos  –
 3  (a) (b) 3
ja 3
R s
 17   (c) –1 (d) 1
cos  –
 3 
57. What is the value of cos²15º?
a th

cos²15º
SSC CGL 06/12/2022 (Shift- 02) SSC CGL 06/12/2022 (Shift- 04)

(a) 1 (b)
3 2 + 3 
ty a

2 (a) (2 + 3) (b)


4
1
2 + 3  1 + 3 
di M

(c) (d) 0
2 (c) (d)
52. Simplify the follwing: 2 2
cot13º cot27º cot45º cot63º cot77º = ? 58. In XYZ, X = 90º, YZ = 15 cm, and XZ = 12
cm. Then find CosY.

XYZXYZ, X = 90º, YZ = 15 cm XZ
cot13º cot27º cot45º cot63º cot77º = ?
= 12 cm CosY
SSC CGL 06/12/2022 (Shift- 03)
SSC CGL 07/12/2022 (Shift- 01)
(a) 1 (b) 3
(c) 2 (d) 2 2 4
(a) (b)
53. Solve the following to find its value in terms 5 5
of trigonometric ratios. 3 3
(c) (d)
A

(sin A + cos A) (1 – sin A cosA) 5 4


59. If tan A – tan B – tan C = tan A tan B tan C,
what is the value of A in terms of B and C?
(sin A + cos A) (1 – sin A cosA) tan A – tan B – tan C = tan A tan B tan C
SSC CGL 06/12/2022 (Shift- 03) B CA
SSC CGL 07/12/2022 (Shift- 01)
(a) sin³A + cos³A]
(b) sin²A – cos²A (a) A = B + C (b) A = 2B – 2C
(c) [cosA – sinA] [sin²A + cos²A] B–C
(d) sin³A – cos³A (c) A = B – C (d) A =
2

Aditya Ranjan (Excise Inspector) Selected Selection 150


https://sscstudy.com/
Join Telegram- Maths by Aditya Ranjan TRIGONOMETRY

60. If tan² = 3 + Q², then sec + tan³ cosec = ?


3
tan² = 3 + Q² sec + tan³ cosec = ? 66. If sin = , then what is tan?
5
SSC CGL 07/12/2022 (Shift- 01)
3
3 3 sin = tan
5
(a) 3 + Q  2 2 (b) 7 + Q 
2 2

SSC CGL 07/12/2022 (Shift- 03)


3 3
(c) 5 – Q  2 2 (d) 4 + Q 
2 2
3 4
(a) (b)
61. What is the value of cos45º sin15º? 5 3
cos45º sin15º
4 3
SSC CGL 07/12/2022 (Shift- 02) (c) (d)
5 4

(a)
 3 –1  (b)
 3 –1  4sin – cos +1
2 4 67. If 4tan = 3, then =.
4sin + cos – 1
(c) (3 + 1) (d) 23 – 1

r
62. If 1 + sin = mcos, then what is the value of sin? 4sin – cos +1
4tan = 3 =
1 + sin = mcos sin 4sin + cos – 1

si
SSC CGL 07/12/2022 (Shift- 02) SSC CGL 07/12/2022 (Shift- 04)

(a)
an by
2m 2 – 1
m2 +1
(b)
m2 – 1
m2 + 1 (a)
14
(b)
12

n
11 11
m2 +1 m2 + 1 10 13
(c) ja (d) (c) (d)
2m 2 – 1 m2 – 1
R s
11 11
63. cos(30º + ) – sin(60º – ) = .
68. If tan + cot = 12, then tan² + cot² is:
a th

SSC CGL 07/12/2022 (Shift- 03)


tan + cot = 12 tan² + cot²
3
(a) (b) 0
2
ty a

SSC CGL 07/12/2022 (Shift- 04)


1 1 (a) 66 (b) 128
di M

(c) (d)
2 2 (c) 142 (d) 132
64. Find the value of the following expression. 69. If sinY = x, then what will be the value of
5(sin4 + cos4) + 3(sin6 + cos6) + 19sin² cos² cos2Y (where 0  Y  90º)?
sinY = x cos2Y ( 0  Y  90º)
5(sin4 + cos4) + 3(sin6 + cos6) + 19sin² cos²
SSC CGL 07/12/2022 (Shift- 03) SSC CGL 08/12/2022 (Shift- 01)
(a) 8 (b) 5
(c) 6 (d) 7 (a) (2 – 1)x (b) 2x
(c) 1 – 2x (d) 1 – 2x²
tan45º –tan15º
65. What will be the value of ? 70. If A + B = C, then tanA tanB tanC = ?
1 + tan45º tan15º
A

tan45º –tan15º A + B = C tanA tanB tanC



1 + tan45º tan15º
SSC CGL 07/12/2022 (Shift- 03) SSC CGL 08/12/2022 (Shift- 01)
1 (a) tanC + tanA – tanB
(a) 3 (b)
2 (b) tanC + tanA + tanB

1 (c) tanA – tanB – tanC


(c) (d) 2 (d) tanC – tanA – tanB
3

Aditya Ranjan (Excise Inspector) Selected Selection 151


https://sscstudy.com/
Join Telegram- Maths by Aditya Ranjan TRIGONOMETRY

71. If x = 8(sin + cos) and y = 9(sin – cos), 76. Which of the following will satisfy a² = b² +
(ab)² for the values a and b?
x 2
y 2

then the value of + is: a² = b² + (ab)²a b


82 92
x = 8(sin + cos) y = 9(sin – cos)
SSC CGL 08/12/2022 (Shift- 03)
x 2 y2
2 + 2 (a) a = sinx, b = cotx
8 9
(b) a = cosx, b = tanx
SSC CGL 08/12/2022 (Shift- 01)
(c) a = cotx, b = cosx
(a) 4 (b) 6
(c) 8 (d) 2 (d) a = sinx, b = tanx
77.  = 135º; y = 15º
12
72. If cotA = , then the value of sinA = ? What is the value of 2cos() sin(y)?
5
 = 135º; y = 15º
12 2cos() sin(y)
cotA = sinA =?
5
SSC CGL 08/12/2022 (Shift- 03)

r
SSC CGL 08/12/2022 (Shift- 02)

si
3 –1 1– 3
5 12 (a) (b)
(a) (b) 2 2
13 an by 13
(c) 2 – 3 (d) 3 – 2
5 13 78. If secx – cosx = 4, then what will be the value

n
(c) (d)
12 12
73. What is the value of tan6º × tan45º × tan84º? of
1 + cos x 
2

?
ja cosx
R s
tan6º × tan45º × tan84º
SSC CGL 08/12/2022 (Shift- 03) 1 + cos x 
2
a th

(a) 1 seccx – cosx = 4


cosx
(b) tan6º × tan90º

(c) 3
SSC CGL 08/12/2022 (Shift- 03)
ty a

(d) tan6º + tan45º + tan84º


4 9 1
di M

74. If sin – cos = , then find the value of sin (a) (b)
5 4 4
+ cos. (c) 25 (d) 5
4 sin 2 63º + sin2 27º
sin – cos = sin + cos 79. Evaluate the expression .
5 cos217º + cos2 73º

SSC CGL 08/12/2022 (Shift- 02) sin2 63º + sin2 27º

cos217º + cos 2 73º
5 5
(a) (b) SSC CGL 08/12/2022 (Shift- 04)
34 24
(a) 0 (b) 3
A

34 24 (c) 1 (d) 2
(c) (d)
5 5
80. If  is an acute angle, tan(4 – 50º) = cot(50º–
75. If tan( + ) = 3, tan ( –) = 1 where ( + ) ), then find the value of  (in degreees).
and ( – ) are acute angles, then what is tan (6)?
 tan(4 – 50º) = cot(50º– )
tan( + ) = 3, tan ( –) = 1 ( + )

( – ) tan(6)
SSC CGL 08/12/2022 (Shift- 04)
SSC CGL 08/12/2022 (Shift- 02)
(a) 60º (b) 45º
(a) –1 (b) 0
(c) 1 (d) 2 – 1 (c) 30º (d) 90º

Aditya Ranjan (Excise Inspector) Selected Selection 152


https://sscstudy.com/
Join Telegram- Maths by Aditya Ranjan TRIGONOMETRY

cosx 2
81. If tanA tanB + = 1, then x = ? 86. If tanA = , then find sinA.
cosA cosA 3
2
cosx tanA = sinA
tanA tanB + = 1 x = ? 3
cosA cosA
SSC CGL 09/12/2022 (Shift- 02)
SSC CGL 08/12/2022 (Shift- 04)
1 2
(a) A (b) B (a) (b)
3 13
(c) A + B (d) A – B
2 3
82. cos³60 – cos³240 – cos³360 = . (c) (d)
3 13
SSC CGL 09/12/2022 (Shift- 01)
sin + cos 3
–3 –7 87. If = , then the value of sin4 –
(a) (b) sin – cos 2
4 5 cos4 is:

r
–3 –9 sin + cos 3
(c) (d) = sin4 – cos4
5 7 sin – cos 2

si
83. If 5sin²A + 3cos²A = 4, 0 < A < 90º, then what

an by
is the value of tanA? SSC CGL 09/12/2022 (Shift- 02)

5sin²A + 3cos²A = 4, 0 < A < 90º 5 12

n
(a) (b)
tanA 12 13

ja SSC CGL 09/12/2022 (Shift- 01) 11 5


(c) (d)
R s
(a) 0 (b) 3 12 13
a th

(c) 1 (d) 2 8
88. If sin = where  is an acute angle, then
84. What is the value of sin28º sin35º sin45º 17
sec62º sec55º? what is the value of tan + cot?
ty a

sin28º sin35º sin45º sec62º sec55º 8


sin =  tan + cot
SSC CGL 09/12/2022 (Shift- 01) 17
di M


1 SSC CGL 09/12/2022 (Shift- 03)
(a) 2 (b)
2 217 281
(a) (b)
110 190
1
(c) 2 (d)
2 289 512
(c) (d)
120 312
3
85. If cos(A – B) = , and cos(A + B) = 0, where 2
2 89. If sin2  cos2  =
, then what will be the value
9
A and B positive acut angles and A  B, then
of cosec² + sec² ?
the measures of A and B are:
A

2
3 sin2  cos 2  = cosec² + sec²
9
cos(A – B) = cos(A + B) = 0
2
A B A  B SSC CGL 09/12/2022 (Shift- 03)
A B 7 5
(a) (b)
SSC CGL 09/12/2022 (Shift- 02) 2 2
(a) 80º and 10º (b) 60º and 30º 9
(c) (d) 92
(c) 70º and 20º (d) 50º and 40º 2

Aditya Ranjan (Excise Inspector) Selected Selection 153


https://sscstudy.com/
Join Telegram- Maths by Aditya Ranjan TRIGONOMETRY

94. If b cos = a, then cosec + cot = ___.


cos 2 x – sec2 x
90. If 2 = a + b cos2x then a, b = ?
tan 2 x b cos = a cosec + cot =
cos 2 x – sec2 x SSC CGL 09/12/2022 (Shift- 04)
2 = a + b cos2x a, b = ?
tan 2 x
SSC CGL 09/12/2022 (Shift- 04) 1 b–a
(a) (b)
b+a b+a
–3 –1 3 1
(a) , (b) ,
2 2 2 2
(c) –3, –1 (d) 3, 1 b+a 1
(c) (d)
91. What is the value of sin75º + sin15º? b–a b–a
sin75º + sin15º
SSC CGL 09/12/2022 (Shift- 04) p2 – 1
95. If cosec + cot = p, then the value of 2 is:
1 p +1
3
(a) (b)
2 2

r
p2 – 1
3 3 cosec + cot = p p2 +1

si
(c) (d)
2 2
92. If cosec + cot = 2, then what is the value
an by SSC CGL 12/12/2022 (Shift- 01)
of cosec ? (a) cos (b) sin
cosec + cot = 2 cosec

n
SSC CGL 09/12/2022 (Shift- 04) (c) cot (d) cosec

5 ja 96. What is the value of the expression cos2A


R s
(a) (b) 2 cos2B + sin²(A – B) – sin²(A + B)?
4
a th

3 cos2A cos2B + sin²(A – B) – sin²(A + B)


(c) 5 (d)
2
93. In the given figure, ABC is an isosceles triangle SSC CGL 12/12/2022 (Shift- 01)
ty a

with BC = 8 cm and AB = AC = 5 cm. The value


of tanC – cotB is. (a) sin (2A – 2B) (b) sin (2A + 2B)
di M

ABC (c) cos (2A + 2B) (d) cos (2A – 2B)


BC = 8 cm AB = AC = 5 cm tanC –
97. Find the value of tan27º tan34º + tan34º
cotB
tan29º + tan29º tan27º.
A
tan27º tan34º + tan34º tan29º + tan29º tan27º

SSC CGL 12/12/2022 (Shift- 01)
(a) 0 (b) –1
(c) 3 (d) 1
A

98. If sin² – 3 sin + 2 = 0, then find the value


B C of (0º    90º).
8 cm
sin² – 3 sin + 2 = 0 (0º    90º)
SSC CGL 09/12/2022 (Shift- 04)

5 7
(a) – (b) – SSC CGL 12/12/2022 (Shift- 02)
12 12
(a) 45º (b) 0º
7 5
(c) (d)
12 12 (c) 60º (d) 90º

Aditya Ranjan (Excise Inspector) Selected Selection 154


https://sscstudy.com/
Join Telegram- Maths by Aditya Ranjan TRIGONOMETRY

1 + sin 4
99. is equal to which of the following 104. If sinA = , then what is the value of sin²A?
cos 5

  4
 where   ? sinA = sin²A
2 5
SSC CGL 12/12/2022 (Shift- 04)
1 + sin  
cos
    ? 26 36
2 (a) (b)
25 25
SSC CGL 12/12/2022 (Shift- 02) 48 16
(c) (d)
1 + cos tan +1 25 25
(a) (b) 105. If sec²A + tan²A = 3, then what is the value
sin tan – 1
of cotA?
tan – 1 cos sec²A + tan²A = 3 cotA
(c) (d)
tan +1 1 – sin SSC CGL 13/12/2022 (Shift- 01)

r
100. If tan3. tan7 = 1, where 7 is an acute angle,
1
then find the value of cot15. (a) (b) 0

si
3
tan3. tan7 = 1 7
(c) 1 (d) 3
cot15
an by 106. If  is an acute angle and tan + cot = 2,
SSC CGL 12/12/2022 (Shift- 03) then the value of tan200 + cot200 is:

n
(a) 1 (b) –1
 tan + cot = 2
(c) –3 (d) 3
ja tan  + cot 
200 200
101. Find the value of sec – tan, if sec + tan
R s
SSC CGL 13/12/2022 (Shift- 01)
= 5.
a th

(a) 1 (b) 2
sec – tan sec + tan = 5
(c) –1 (d) 0

107. If a = 45º and b = 15º, what is the value of
SSC CGL 12/12/2022 (Shift- 03)
ty a

cos  a – b  – cos  a + b 
1 ?
(a) 5 (b) 5 cos  a – b  + cos  a + b 
5
di M

a = 45º b = 15º
5
(c) (d) 5 cos  a – b  – cos  a + b 
5
cos  a – b  + cos  a + b 
102. If 2 sin + 2 sin² = 2, then the value of 2
cos4 + 2 cos² is: SSC CGL 13/12/2022 (Shift- 02)
2 sin + 2 sin² = 2 2 cos4 + 2 cos² (a) 2 – 22 (b) 3 – 6
(c) 3 – 2 (d) 2 – 3
SSC CGL 12/12/2022 (Shift- 03) 108. What is the value of cosec15º sec15º
(a) 4 (b) 2
cosec15º sec15º
(c) 1 (d) 0
A

SSC CGL 13/12/2022 (Shift- 02)


5 tanA sinA
103. If secA = , then the value of – is: (a) 0.5 (b) 4
4 1 + tan2 A secA
(c) 2 (d) 1
5 tanA sinA
secA = – cosA sinA
4 1 + tan2 A secA 109. + =
1 – tanA 1 – cotA
SSC CGL 12/12/2022 (Shift- 04) SSC CGL 13/12/2022 (Shift- 02)
(a) 2 (b) 1 (a) tanA – cotA (b) tanA + cotA
(c) 0 (d) 3 (c) sinA – cosA (d) sinA + cosA

Aditya Ranjan (Excise Inspector) Selected Selection 155


https://sscstudy.com/
Join Telegram- Maths by Aditya Ranjan TRIGONOMETRY

1 (a) Cos²A (b) SinA


110. If sec + = 2. find the value of (c) Sin²A (d) CosA
cos
115. Evaluate the following.
55 1
sec  + .
sec55  sin 25° sin 65° – cos25° cos 65°.
1 1
sec + = 2 sec55  +
cos sec55 
sin 25° sin 65° – cos25° cos 65°.
SSC CGL 13/12/2022 (Shift- 03) SSC CGL TIER - II 02/03/2023
(a) 2 (b) 0
(c) 1 (d) 55 (a) 4 (b) 1
tan320º –tan310º (c) 0 (d) 40
111. If tan40º = , then find .
1 + tan320º .tan310º
1
tan320º –tan310º 116. If tan(A + B) = 3 and tan (A – B) = ; 0º <
tan40º =  3
1 + tan320º .tan310º

r
(A + B) < 90º; A > B, then the values of A and

B are _____.respectively.

si
SSC CGL 13/12/2022 (Shift- 03)
1 – 2 1 + 2 1
(a) an by (b) tan(A + B) = 3 tan (A – B) = ; 0º
  3
< (A + B) < 90º; A > B, A B

n
1 – 2 1 + 2
(c) (d) _____
2 
ja
112. If tan² + tan4 = 1, then: SSC CGL TIER - II 02/03/2023
R s
tan² + tan4 = 1 (a) 45º and 15º (b) 15º and 45º
SSC CGL 13/12/2022 (Shift- 03)
a th

(a) cot² + cot4 = 1 (b) cos² + cos4 = 1 (c) 30º and 30º (d) 60º and 30º
(c) sin² + sin4 = 1 (d) cosec2 + sec4 = 1 117. Select the INCORRECT formula from the
113. If tan( + ) = a, tan( – ) = b, then the value following options.
ty a

of tan2 is:
tan( + ) = a, tan( – ) = b tan2
SSC CGL TIER- II 03/03/2023
di M


SSC CGL 13/12/2022 (Shift- 04) (a) cosec² – cot²= 1
a+b a+b (b) sin² + cos²= 1
(a) (b)
1 – ab 1 + ab (c) sec² + cos² = 1
a–b a–b (d) sec²q – tan² = 1
(c) (d)
1 + ab 1 – ab 118. Find the exact value of 120°.
114. If A is an acute angle, the simplified from of cos 120°
  SSC CGL TIER- II 06/03/2023
cos   – A  . cot  + A  cos  –A 
2  (a) – 0.5 (b) 0
is :
 3 
tan   + A  tan  + A  sin 2  – A 
A

(c) 1 (d) 0.5


 2 
119. If a cot + b cosec = p and b cot + a cosec =
A q then p² – q² is equal to _______.

a cot + b cosec = p b cot + a cosec = q
 
cos   – A  . cot  + A  cos  –A  p² – q²
2 
is : SSC CGL TIER- II 07/03/2023
 3 
tan   + A  tan  + A  sin 2  – A 
 2  (a) b² – a² (b) a² – b²
SSC CGL 13/12/2022 (Shift- 04) (c) b – a (d) a² + b²

Aditya Ranjan (Excise Inspector) Selected Selection 156


https://sscstudy.com/
Join Telegram- Maths by Aditya Ranjan TRIGONOMETRY

120. (sin + cosec)² + (cos + sc)² = ? m


SSC CGL TIER- II 07/03/2023 60°
m
(a) 5 + tan² + cot²
m
(b) 7 + tan² – cot²
SSC CGL TIER- II 03/03/2023
(c) 7 + tan² + cot²
(d) 5 + tan² – cot² 151 3
121. Two ships are on the opposite of a light house such (a) 40 3 (b)
6
that all three of them are colline Zar. The
angles of depression of the two ships from the top 77 40 3
of the light house 30º and 60º. If the ships (c) 3 (d)
4 3
are 230 3 m apart, then find the height of the light 124. The angle of elevation of the top of an upright
house (in m).
tower from the top of a straight pole 24 3 feet
high is 60°. If the base of the pillar was at a

r
distance of 60 feet from the base of the tower,

si
30º 60º what was the height (in feet) of the tower?

230 3 m
an by m 24 3
60°

n

SSC CGL TIER - II 02/03/2023

(a) 175.4 ja SSC CGL TIER- II 06/03/2023
R s
(b) 165.2
(c) 172.5 (a) 44 3
a th

(d) 180.5 (b) 36 3


122. The angle of elevation of the top of a building
(c) 60 3
at a distance of 70 m from its foot on a
ty a

horizontal plane is found to be 60°. Find the (d) 84 3


height of the building.
di M

125. The angle of elevation of the top of a tower from


70 m the top of a building whose height is 680 m is
60° 45° and the angle of elevation of the top of same
tower from the foot of the same building is 60°.
What is the height (in m) of the tower?
SSC CGL TIER- II 03/03/2023

(a) 70 3 m 45°
60°
(b) 60 3 m
SSC CGL TIER- II 07/03/2023
(c) 50 3 m
 
A

(a) 340 3  3
(d) 70 2 m
(b) 310 3 – 3
123. From the top of an upright pole 17.75 m high,
the angle of elevation of the top of an upright
(c) 310 3  3
tower was 60° . If the tower was 57.75 m tall,
how far away (in m) from the foot of the pole
was the foot of the tower?
(d) 340 3 – 3

Aditya Ranjan (Excise Inspector) Selected Selection 157


https://sscstudy.com/
Join Telegram- Maths by Aditya Ranjan TRIGONOMETRY

ANSWER KEY
1.(c) 2.(b) 3.(b) 4.(c) 5.(b) 6.(a) 7.(c) 8.(c) 9.(d) 10.(a)
11.(a) 12.(a) 13.(c) 14.(c) 15.(c) 16.(a) 17.(d) 18.(b) 19.(a) 20.(a)
21.(d) 22.(b) 23.(a) 24.(b) 25.(b) 26.(a) 27.(d) 28.(a) 29.(b) 30.(d)
31.(c) 32.(b) 33.(b) 34.(a) 35.(b) 36.(d) 37.(a) 38.(c) 39.(d) 40.(b)
41.(b) 42.(b) 43.(c) 44.(b) 45.(d) 46.(a) 47.(d) 48.(b) 49.(b) 50.(d)
51.(c) 52.(a) 53.(a) 54.(a) 55.(d) 56.(d) 57.(b) 58.(c) 59.(a) 60.(d)
61.(b) 62.(b) 63.(b) 64.(a) 65.(c) 66.(d) 67.(d) 68.(c) 69.(d) 70.(d)
71.(d) 72.(a) 73.(a) 74.(c) 75.(a) 76.(c) 77.(b) 78.(c) 79.(c) 80.(c)

r
81.(c) 82.(a) 83.(c) 84.(b) 85.(b) 86.(b) 87.(b) 88.(c) 89.(c) 90.(c)

si
91.(c) 92.(a) 93.(b) 94.(c) 95.(a) 96.(c) 97.(d) 98.(d) 99.(d) 100.(b)
101.(c)
an by
102.(b) 103.(c) 104.(d) 105.(c) 106.(b) 107.(d) 108.(b) 109.(d) 110.(a)

n
111.(c) 112.(b) 113.(a) 114.(d) 115.(c) 116.(a) 117.(c) 118.(a) 119.(a) 120.(c)
121.(c) 122.(a)
ja 123.(d) 124.(d) 125.(a)
R s
a th
ty a
di M
A

Aditya Ranjan (Excise Inspector) Selected Selection 158


https://sscstudy.com/
Join Telegram- Maths by Aditya Ranjan TRIGONOMETRY

SOL U TION S
cos 45 sin 30º sin 40º sin 50º sin 60º
1. (c) 4. (c)
sec 30  cosec30 cos 30º cos 40º cos 50º cos 60º
We know, sin = cos (90º – )
1
2 cos 60º cos 50º cos 40º cos 30º
 =
2 2 cos 30º cos 40º cos 50º cos 60º

3 1
=1
5. (b) sin30º cos60º – cos30º sin60º
1
2  sin30º sin30º – cos30º cos30º

r
22 3 = sin² 30º – cos²30º
= – cos60º

si
3
= – sin30º

1
2
an by

3
22 3 6. (a) sin – sin = 2cos

2
sin
 –
2

n
3 by direct formula.

ja
2 22 6
R s
2 tan 60º 2 3
7. (c) =

3 2 6–2 2  1 tan ²60º 13
a th


16
3
=
6 2–2 6 2

ty a

16 = sin60º
8. (c) tan240º
di M

3 2– 6

8  tan(180º + 60º)
 tan60º = 3
2. (b) P
9. (d) tan + sec = 7

1
17 sec– tan =
15 7

50
 2 sec =
7
Q 8 R
25
sec =
A

8 15 7
sin + cos = 
17 17
7
23 cos =
= 25
17
3. (b) tan² + cot² – sec² cosec². 24
 sin =
25
Let  = 45°
 1 + 1 – (2)² (2)² 24
 5 sin =
 2 – 4 = –2 5

Aditya Ranjan (Excise Inspector) Selected Selection 159


https://sscstudy.com/
Join Telegram- Maths by Aditya Ranjan TRIGONOMETRY

  3 1
10. (a) 16 cos ³ – 12 cos 14. (c) cot x – tan x = =2–
6 6 2 2
1
 3 3  cot x = 2, tan x =
 16×   – 12  3 2
 2  2
  1 5
 cot x + tan x = 2  =
 6 3–6 3=0 2 2
4
a²  b²  c² – 1024 15. (c) sec²A + tan²A =
11. (a) = –2 17
ab – bc – ca
sec4A – tan4A = (sec²A – tan²A) (sec²A + tan²A)
a² + b² + c² – 1024 = 2bc + 2ca – 2ab = 1 (sec²A + tan²A)
 a² + b² + c² + 2ab – 2ca – 2bc = 1024
4
 (a + b – c)² = 1024 =
17
 a + b – c = 32 .........(1)
16. (a) sin (A + B) = cos (A + B)

r
Given,
sin (A  B)
a + b – 5c = 0 .....(2)  =1

si
cos (A  B)
From (1) and (2)
4C = 32
C=8
an by  tan (A + B) = 1
tan A  tan B
=1

n

1 – tan A tan B
9 40
12. (a) cos A =  sin A =  tan A + tan B = 1 – tan A tan B
ja 41 41
R s
tan A + tan A + tan B = 1 – tan B
cos A 9  tan A (1 + tan B) = 1 – tan B
 cot A = =
a th

sin A 40
1 – tan B
 tan A =
1 1  tan B
1 – a² (1 – a ²) 2
13. (c) tan²=  tan =
ty a

1 1 17. (d) A
di M

1
1 (2 – a²)2 25
 sec= = 7
cos 1
3

tan³ = (1 – a ²) 2
B 24 C
1
 2 – a²  2
cosec 
  AC – BC = 1
2 
1 – a   AC = 25, BC = 24
 sec + tan³. cosec 7
 sin C =
1 25
1 3
(2 – a²) 2
A

= (2 – a²)  (1 – a²) 
2 2
1
18. (b) sin + cos = 5 sin (90 – )
(1 – a²) 2
sin = 5 cos – cos
1
(2 – a²) 2 [1 + (1 – a²)]
sin = cos  5 –1 
1
1
 cot =
 (2 – a²) 2 (2 – a²) 5 –1
3 5 1
= (2 – a²) 2 cot =
4

Aditya Ranjan (Excise Inspector) Selected Selection 160


https://sscstudy.com/
Join Telegram- Maths by Aditya Ranjan TRIGONOMETRY

1 4 15
19. (a) cosec A – cot A = 24. (b) sin A = , sin B =
4 5 17
 cosec A + cot A = 4
1 15 3 8
 2 cot A = 4 – =  cos A = , cos B =
4 4 5 17

15  sin (A – B) = sin A cos B – cos A sin B


 cot A =
8 4 8 3 15
=  – 
8 5 17 5 17
 tan A =
15
20. (a) 32 45
= –
cot15º cot25º cot45º cot75º cot65º 85 85
 tan75º tan65º cot45º cot75º cot65º
13
 cot45º = 1 =–
85
21. (d) P

r
cos A sin A
25. (b) 1  tan A – 1  cot A

si
an by 
cos A
1
sin A

sin A
1
cos A

n
Q R cos A sin A

cot P =
ja
base
=
3 
cos ²A

sin² A
R s
perp. 1 cos A  sin A sin A  cos A
a th

 Hypotenus = 2 cos ²A – sin ² A



1 cos A  sin A
 sin P =
2  cos A – sin A
ty a

'OR'
1 1
cot P = 3  P = 30º 26. (a) 1  cos(90 – )  1 – cos(90 –)
di M

1
 sin 30º =
2 1 1
= 
22. (b) tan A + cot A = 2 1  sin 1 – sin
Squaring Both sides:-
2 2
tan²A + cot²A + 2 = 4  =
1 – sin² cos²
2 (tan²A + cot²A) = 4
 2 sec²
1 – cos 1
23. (a) = 27. (d)
sin 5
We know,
Multiply and divide by (1 + cos)
sin4 + cos4 = 1 – 2 sin² cos²
1 – cos1  cos 1
A

 = = 1 – 2 sin² (1 – sin²)
sin 1  cos 5
= 1 – 2 sin² + 2 sin4
sin 
2
1 = 2 sin4 – 2 sin² + 1
 =
sin(1  cos) 5 28. (a)
sin 1 tan ( – 14)
 =
1  cos 5 – tan (14 – )
1  cos – tan (7 × 2 – )
 =5
sin = tan

Aditya Ranjan (Excise Inspector) Selected Selection 161


https://sscstudy.com/
Join Telegram- Maths by Aditya Ranjan TRIGONOMETRY

29. (b) 5
tan A =
k – k cot² 30º 12
= sin²60º + 4 tan²45º – cosec²60º sin A + sin B + sin C
1 cot² 30º
5 12
k (1 – cot² 30º )  1
 13 13
1 cot² 30º
30 4
= sin²60º + 4 tan²45º – cosec²60º  =2
13 13
k (– 2) 3 4 34. (a)
 = 4–
4 4 3
3 –1
9  48 – 16  sin cos = .......(1)
 k=   (– 2) 2 2

 12 
Consider, tan + cot
= – 6.83
30. (d) sin cos 1
=  
cot B = 9 cos sin sincos

r
1 squaring both sides in (1)

si
 tan B =
9
4– 3–8
an by  1 + 2 sin cos =
1 8
 tan²B =
81
4  2 3 

n
 sec²B = 1 + tan²B =–
8
ja 82
=
2  3 
R s
81
31. (c) =–
4
a th

cos x – 3 sin x – 4  2 
 tan  cot =  
2  2  3 

ty a


1
2
cos x –
3
2
sin x =
– 8
  3
di M

 cos 60º cos x – sin 60º sin x  8  3 – 2


 cos (60º + x)
35. (b)
 
 cos 
  x
 8
3  sin =
17
32. (b)
8 8
tan = =
1 + cosA  1 + cosA  17² – 8² 15
1 – cosA  1 + cosA  36. (d)
sec² = cosec ( – 36º)
1 + cosA cosec (90º – 2) = cosec ( – 36º)
= cosecA + cotA
sinA
A

 90º – 2 =  – 36º
33. (b)  3 = 126º
A  = 42º
37. (a)
tan34º
13 cos²35º + cos55º sin35º +
cot56º
12
cot56º
 cos²35º + sin²35º +
cot56º
B C  1+1=2

Aditya Ranjan (Excise Inspector) Selected Selection 162


https://sscstudy.com/
Join Telegram- Maths by Aditya Ranjan TRIGONOMETRY

38. (c) 41. (b)


tan + cot = 2 ........(1)
1  cos 1 – cos
Let,  =x Consider,
1 – cos 1  cos
tan² + cot² + 2 tan5 cot4
Squaring both sides
 tan² + cot² + 2 tan
1  cos 1 – cos Squaring (1)
  2 = x²
1 – cos 1  cos tan² + cot² + 2 = 4
 tan² + cot² = 2 ......(2)
(1  cos)²  (1 – cos)²  2 sin²
 = x² From (1) and (2)
sin²
tan = 1 = cot
(1  cos 1 – cos)²   = 45°
 = x²
sin²  tan² + cot² + 2 tan5 cot4
 2+2=4
 2 2
 
  = x²
 42. (b)

r
 sin
cot²A – cos²A

si
 x = 2 cosec
 cos²A [cosec²A – 1]
39. (d)
an by  cos²A × cot²A
cos x cosec x – sin x sec x
43. (c)

n
cos x sin x
 – 1  sin A
sin x cos x
ja 1 – sin A
R s
cos² x – sin² x Rationalizing we, get

cos x sin x
a th

(1  sinA)²
cos 2x 
= 2 cos ²A
2 cos x sin x
ty a

1  sin A
cos 2x 
  2 = 2 cot 2x cos A
sin 2x
di M

 sec A + tan A
40. (b)
44. (b)
A

13 5
3 4

B C
2 3
8 cot = 6
A

2
tan A =
3 3
cot =
 (5 sin²A – 2 cos²A) ÷ (15 sin²A + 3 cos²A) 4

 4 9  4 9  4 3
5  
  – 2 ÷
 15   3 
 sin cos 5 5
 13 13   13 13   =
sin– cos 4 – 3
5 5
2 13 2
  =
13 87 87 7

1

Aditya Ranjan (Excise Inspector) Selected Selection 163


https://sscstudy.com/
Join Telegram- Maths by Aditya Ranjan TRIGONOMETRY

45. (d) 49. (b)


15 8
cot A =  tan A 
8 15
13
5 2 tan A
tan 2 A =
1 – tan ²A

A 8 16
2
12 = 15 = 15
64 161
5 12 1–
tan A =  cos A = 225 225
12 13
46. (a) 240
=
161
50. (d)
b cos37º sin(90º –37º )
=
a

r
sin53º sin53º
 1

si
51. (c)

b sin = a
an by
b² – a²  17
cos 
–
 3 

n
a 17
sin =
b ja  cos 
 
 [  cos (– x) = cos x]
 3 
R s
1  sin
sin tan =  
cos
a th

= cos 
6 – 

a  3
1
= b  1
b² – a² = cos 
 =
ty a

3  2
b
52. (a)
ba ba
di M

  1 cot13º cot27º cot45º cot63º cot77º


b² – a²
(b a) (b– c)
2
 tan77º tan63º cot45º cot63º cot77º
ba 1
 53. (a)
b–a
47. (d) (sinA + cosA) (1 – sinA cosA)
cos 20 cos  (sinA + cosA) (sin²A + cos²A – sinA cosA)
  sin³A + cos³A
sin 70 sin(90 –)
54. (a)
sin70 cos
  =2
sin 70 cos 1
sin =
48. (b) 2
A

4 Consider. (3 cos – 4 cos³) = – cos³


sin10º – sin³10º
3  cos 90º
1 =0
 [3 sin10º – 4 sin³10º] 55. (d)
3
1 7 2 7
 [sin30º ] sec – 2 cos =  sec – =
3 2 sec 2
1 Directly from options, we can see that option

6 (d) sec = 4 satisfies the given equation.

Aditya Ranjan (Excise Inspector) Selected Selection 164


https://sscstudy.com/
Join Telegram- Maths by Aditya Ranjan TRIGONOMETRY

56. (d) 61. (b)


tan3 = ? sec3 = cosec (4 – 15º) cos45° sin15°
 cosec (90º – 3) = cosec (4 – 15º)  cos 45º × sin (45º – 30º)
 90º – 3 = 4 – 15º
 105º = 7 1
= [sin45º cos30º – cos45º sin30º]
  = 15º 2
 tan3 = tan45º = 1
1  
 1  3 – 1  1
57. (b) =

2  2 2 2 2 
cos²15°
We know, cos² = 2cos² – 1
1 
 3 – 1 

=
cos ² 2  
 cos² = 2 2 2 2 
2
cos 30º1 3 –1
=
 = 4
2
62. (b)

r
2 3 1 + sin = m cos
cos ²15º =
4

si
58. (c) 1  sin (1  sin)²
 m= 
4 an by cos 1 – sin ²

(1  sin) (1  sin) 1  sin

n
15 = =
(1 – sin) (1  sin) 1 – sin
9
ja m² 1  sin
R s
 =
1 1 – sin
a th

x Using Component and dividend.


12 Z
yx = 225 – 144 = 81 = 9 m²  1 1 m² – 1
 =  sin 
m² – 1 sin m²  1
ty a

9 3
 cos y = = 63. (b)
15 5
cos (30º + ) – sin (60º – )
di M

59. (a)
 cos30º cos – sin30º sin – sin60º cos +
tanA – tanB – tanC = tanA tanB tanC cos60º sin
 tanA + tan(–B) + tan(–C) = tanA tanB tanC
 3 3 1 1 
We know, tanA + tanB + tanC = tanA tanB tanC   –  cos sin – =0
 2 
2  2 2 

If A + B + C = n, where n = 0, 1, 2, 3 
for A + B + C = 0, it is true 64. (a)
 A + (–B) + (–C) = 0 5 (sin4 + cos4) + 3 (sin6 + cos6) + 19 sin²
 A=B+C cos²
60. (d) = 5 [1 – 2 sin² cos²] + 3 [1 – 3 sin² cos²] +
tan² = 3 + Q² 19 sin²cos²
 sec² = 1 + 3 + Q² = 4 + Q² = 5 – 10 sin² cos² + 3 – 9 sin² cos² + 19
A

1 cos² sin²
 sec= (4 Q²) 2
=8
sin³ 65. (c)
Consider, sec  + tan³ cosec  = sec  +
cos ³
tan45º –tan15º
sin ²  tan (45 – 15) = tan30
cosec  = sec  + 1+tan45º tan15º
cos ³
1
cos ² sin ²
3
=
 = sec³  (4 Q²) 2 3
cos ³

Aditya Ranjan (Excise Inspector) Selected Selection 165


https://sscstudy.com/
Join Telegram- Maths by Aditya Ranjan TRIGONOMETRY

66. (d) 72. (a)


C

5 13
3 5

0
4 B
12 A
3
sin = 12
5 cot A =
5
3
tan = 5
4 sin A =
67. (d) 13
73. (a)
4 sin – cos 1

r
4 tan = 3, then tan6º tan45º tan84º
4 sin cos – 1
Consider, tan6º tan84º

si
A + B = 6º + 84º = 90º
an by If (A + B) = 90º
5
 tanA tanB = 1
3
 tan6º tan84º tan45º = 1

n
74. (c)
ja 4
R s
4 sin – cos =
5
3
a th

 tan =
4 4²
sin + cos = 1  1 –
3 4 13 5²
4 – 1
4 sin – cos 1 5 5 13
= = 5 = 16
ty a

 = 2–
4 sin cos – 1 4  3  4 – 1 11 11
25
5 5 5
di M

68. (c) 34
=
tan + cot = 12 5
 tan² + cot² + 2 = 144 75. (a)
 tan² + cot² = 142 tan ( + ) = 3   +  = 60º
69. (d) tan ( + ) = 1  –  = 45º
sin y = x  2 = 105º
cos 2y = 1 – 2 sin²y
6 = 315º
= 1 – 2x²
70. (d) tan6 = tan (315º) = tan (360º – 45º)
A+B=CA+B–C=0 = – tan 45º
 tanA tanB tanC = tanA + tanB – tanC =–1
'or' 76. (c)
A

C–A–B=O a² = b² + (ab)²
 tanA tanB tanC = tanC – tanA – tanB  a² – a²b² = b²
71. (d)  a² (1 – b²) = b²
x = 8 (sin + cos)
y = 9 (sin – cos) b²
 a² =
x² = 64 [1 + 2 sin cos] 1 – b²
x ² y² From option, (c) a = cot x
Consider,  = 1 + 2sin cos + 1 – 2 sin b = cos x
8² 9²
cos= 2 Satisfies the condition.

Aditya Ranjan (Excise Inspector) Selected Selection 166


https://sscstudy.com/
Join Telegram- Maths by Aditya Ranjan TRIGONOMETRY

77. (b) 82. (a)


 = 135º , y = 15º cos³60º – cos³240º – cos³360º
 2cos() sin(y) = cos³60º + cos³60º – cos³0°
cos135º = cos(45º + 90º) = – cos45º We know,
1 cos60º + cos60º – cos0º
=–
2
1 1
sin15º = sin(45º – 30º) = sin45º cos30º – cos45º =  –1= 0
2 2
sin30º
 cos³60º + cos³60º – cos³0º
3 –1
  – 3 × cos60º × cos60º × cos0º
2 2
 2 cos 135º sin y 1 1 3
 –3×  1 = –
2 2 4
 1   3 – 1
 2×    
–  83. (c)
 2 2 2 

5 sin²A + 3cos² A = 4

r
1– 3 2 sin²A + 3 = 4
=

si
2 sin²A = 1
78. (c) an by 1
sin A =
1  cos ²x 2
Consider, = sec x  cos x

n
cos x
 A = 45º
sec x – cos x = 4
tanA = tan45º = 1
ja
R s
 sec x + cos x = (secx– cosx)²  4 84. (b)
sin28º sin35º sin45º sec62º sec55º
a th

= 20 = 2 5
 cos62º cos55º sin45º sec62º sec55º
 1 1 
Using, if x – = a x  = a²  4  1

 x x  
2
ty a

79. (c)
85. (b)
sin² 63  sin² 27
di M

cos ²17  cos² 73 3


cos (A– B) = , cos(A + B) = 0
2
sin² 63  sin² 63
  A – B = 30º
cos ²17  cos² 17
A + B = 90º
 1  A = 60º, B = 30º
80. (c) 86. (b)
tan(4 – 50) = cot (50 – )
 cot [90 – 4 + 50] = cot (50 – ) C
 90 – 4 + 50 = 50 – 
 90 = 3 13
  = 30° 2
A

81. (c)
cos x B
tan A tan B  =1 3 A
cos A cos B

sin A sin B cos x 2


 =1 tan A =
cos A cos B cos A cos B 3
 sinA sinB + cosx = cosA cosB 2
 cosx = cosA cosB – sinA sinB sin A =
13
 cosx = cos (A + B)

Aditya Ranjan (Excise Inspector) Selected Selection 167


https://sscstudy.com/
Join Telegram- Maths by Aditya Ranjan TRIGONOMETRY

87. (b) 2(cos 4 x – 1)


 = a  b cos 2x
sin cos 3 sin ² x
= .........(1)
sin – cos 2 ((cos ² x– 1) (cos ² x  1))
 = a  b cos 2x
sin4 – cos4 = (sin² – cos²) (sin² + cos²) sin ² x
= sin² – cos² .........(2)  – 2 (cos²x + 1) = a + b cos2x
Using Compounds and dividend in (1)  – (2 cos²x – 1) – 3 = a + b cos2x
 – cos2x – 3 = a + b cos2x
sin 5 5
 = tan =  a = – 3, b = – 1
cos 1 1
91. (c)
sin75º + sin15º
We know,
26  sin (A + B) + sin (A – B) = 2 sinA cosB
5  sin (45º + 30º) + sin (45º – 30º) = 2 sin 45º
cos 30º
1 3
= 2 

r
2 2
1

si
 Form (2) 3
=
2
an by 25 1 24 12
sin4 – cos 4 = – = = 92. (a)
26 26 26 13
cosec + cot = 2, cosec = ?
88. (c)

n
1
 cosec – cot =
2
ja
R s
17 1 5
 2 cosec = 2  =
2 2
a th

8
5
 cosec =
4
93. (b)
ty a

15
A
8
sin =
di M

17
tan + cot
8 15 64  225 5 5
  
15 8 120
289
=
120
89. (c) B D C
2 8cm.
sin² cos² =
9 Since, ABC is an is scales triangle.
1 1 1 9 AD is the perpendicular bisector.
A

cosec² sec² =  = =  BD = DC = 4
sin² cos ² sin²cos ² 2
90. (c)  AD = 25 – 16 = 3
 cos ²x – sec ²x  3 4
2
  = a  b cos 2x
  tan C – cot B = –
 tan ² x  4 3

 9 – 16
1  =
cos ²x –
 
 12
 cos ²x 
 2 = a  b cos 2x 7
sin² x =–
cos ² x 12

Aditya Ranjan (Excise Inspector) Selected Selection 168


https://sscstudy.com/
Join Telegram- Maths by Aditya Ranjan TRIGONOMETRY

94. (c) 31 3 –1


= ,
2 2
b² a² = 2. 1
b But O <  < 90°
 sin = 1 = sin 90°
99. (d)
1  sin (1 – sin)

cos (1 – sin)
a
b cos = a 1 – sin ² cos
 =
a cos(1 – sin) 1 – sin
 cos =
b 100. (b)
 cosec + cot tan3 tan7= 1
 3 + 7 = 90
a  10 = 90

r
b b = 9
 
b² – a² b² – a²  cot 15 = cot 135º

si
b = cot(180º – 45º)
an by = – cot45º
b+a b+a =–1
 =
b² – a² b–a 101. (c)

n
95. (c)
sec tan = 5
cosec + cot = p
ja 1
R s
1 sec– tan =
cosec – cot = p 5
a th

p²  1 p² – 1 5
 cosec = and cot = =
2p 2p 5
102. (b)
ty a

Consider,
2sin + 2sin² = 2
p² – 1 cot  sin + sin² = 1
di M

= = cos
p²  1 cosec sin = 1 – sin²
sin = cos²
96. (b)
sin² = cos4
cos2A cos2B + sin² (A – B) – sin² (A + B)  2cos4 + 2 cos² = 2 [sin² + sin]
Let, A = B =2
 cos²2A + sin² (0) – sin² (2A) 103. (c)
 cos²2A – sin²2A
 cos4A = cos (2A + 2B) = cos (2A + 2B)
5
97. (d)
3
We know,
A

If A + B + C = 90°
 tanA tanB + tanB tanC + tanA tanC
4
 tan27º tan34º + tan34º tan29º + tan29º tan27º
=1 5
sec A =
4
98. (d)
sin² – 3sin + 2 = 0 tan A sin A

1  tan ²A sec A
3 9 – 8 tan A – sinA secA
 sin =  =0
2 sec ²A

Aditya Ranjan (Excise Inspector) Selected Selection 169


https://sscstudy.com/
Join Telegram- Maths by Aditya Ranjan TRIGONOMETRY

104. (d) 111. (c)


4 16 tan320º –tan310º
sin A = , sin² A = = tan10º
5 25 1 + tan320º tan310º
105. (c) tan10º = tan (50º – 40º)
sec²A + tan²A = 3
tan50º –tan40º
sec²A – tan²A = 1 =
1 + tan50º tan40º
tan²A = 1
 A = 45 cot40º –tan40º
=
 cot A = 1 1 + cot40º tan40º
106. (b)
1
tan + cot = 2 –
1 – ²
tan2n + cot2n = 2 = 
2 2
tan = cot = 1
112. (b)
 tan200+ cot200 = 2
tan² + tan4 = 1

r
107. (d)
tan² [1 + tan²] = 1

si
cos (a– b) – cos (a b)
tan². sec² = 1
cos (a– b)  cos (a b)
an by  tan² = cos²
cos 30º – cos 60º 3 –1  – 1 + sec² = cos²
 =

n
cos 30º cos 60º 3 1 1
 –1+ = cos ²
cos ²
ja =
4–2 3
R s
2  cos4 + cos²= 1
113. (a)
a th

=2– 3
tan ( + ) = a
108. (b)
cosec15º sec15º tan ( – ) = b
ty a

1 tan 2 = tan ( – –)



sin15º cos15º let  +  = A
di M

2 –=B

sin (215º )
tan A  tan B
 tan (A+B) =
2 1 – tan A tan B
 =4
sin 30º
109. (d) ab
tan2 =
1 – ab
cos A sin A
 114. (d)
1 – tan A 1 – cot A
cos ²A sin ²A  
  cos( – A) cot 
cos A – sin A sin A – cosA   A cos (– A)
2 
 3 
A

cos ²A – sin ²A tan( A) tan   A


 = cosA + sinA   sin(2 – A)
cos A – sin A 2 
110. (a)
1 cos A(– tan A) cos A
sec =2  –
cos tanA (– cot A) (– sin A)
 2 sec = 2
sin A cos A
 sec = 1 
sin A
1
 sec55 =2 = cos A
sec55

Aditya Ranjan (Excise Inspector) Selected Selection 170


https://sscstudy.com/
Join Telegram- Maths by Aditya Ranjan TRIGONOMETRY

115. (c) = 5 + cose² + sec²


sin25° sin65° – cos25° cos65° = 5 + 1 + cot² + 1 + tan²
= – cos (25° + 65) = 7 + cot² + tan²
=0 121. (c)
116. (a)
C
tan (A + B) = 3

1
tan (A – B) =
3
h
 A + B = 60°
A – B = 30°
30° 60º
 A = 45°, B = 15° A B
D
117. (c)

r
230 3
We know,

si
sec² + cos²  1
AD 3
118. (a) an by We know, =
DB 1
cos 120° = cos (180° – 60°)

n
= cos180º cos60º – sin180º sin60º  4  230 3
230 3
= –1
1 ja 1
– 0 = – = – 0.5
DB 1 
R s
4
2 2
a th

119. (a) CD
tan 60 =
a cot + bcosec = P DB

b cot + a cosec = q 230 3


ty a

 CD = 3 
Let  = 30° 4
Then,
di M

= 172.5

3a  2b = P 122. (a)

2a  3b = q

p–q=   
3 –2 a 2– 3 b 
p q =   
3 2 a 2 3 b  H
 p² – q² = – a² + b²
120. (c)
A

2 2 60°
 1   1 

 sin 
    cos 

 sin  cos 70 m

1 1
 sin²   2  cos ² 2
sin² cos ² H
tan 60 =
70
1 1
= 5   H = 70 3
sin² cos ²

Aditya Ranjan (Excise Inspector) Selected Selection 171


https://sscstudy.com/
Join Telegram- Maths by Aditya Ranjan TRIGONOMETRY

123. (d) 125. (a)

A h 45°
60°
D
B H
680 680
57.75 17.75

60°
C E x
x
h  680
tan 60 =
AB = 57.75 – 17.75 = 40 m x

AB AB h  680

r
 tan60º = = x= ..........(1)
BD CE 3

si
40 h
3=
an by tan 45 =
CE x
x=h ..........(2)

n
40 40 3
CE = = put (2) in (1)
3 3
ja h  680
R s
124. (d) h=
3
A
a th

60°
 3 h – h = 680
B D
680
 h= = 340  3 1
ty a

3 –1
24 3
 
di M

H = 680 + 340 3 1

= 340  
 3  1  2

C E
60
= 340  
 3  3 

AB AB
tan 60 = =
CE 60

AB = 60 3

AC = 60 3  24 3 = 84 3
A

......-------......

Aditya Ranjan (Excise Inspector) Selected Selection 172


https://sscstudy.com/
Join Telegram- Maths by Aditya Ranjan TRIANGLE

TRIANGLE
20

1. In the triangle ABC, AB = 12cm and AC = 4. In the figure, AB = AD = 7cm and AC = AE
10cm, and BAC = 60º What is the value of and BC = 11cm, then find the length of ED.
the length of the side BC?
AB = AD = 7 AC = AE BC =
ABC AB = 12 cm AC = 10 cm, 11 ED
BAC = 60º BC
E D

r
A

si
60º

an by
n
B C
B C
ja SSC CGL 01/12/2022 (Shift- 03)
R s
SSC CGL 01/12/2022 (Shift- 01) (a) 12 (b) 10
(a) 10 cm (b) 7.13 cm
a th

(c) 11 (d) 2
(c) 13.20 cm (d) 11.13 cm 5. From the following figure find x + y + z.
2. If ares of similar triangles ABC and DEF are x+y+z
x² cm² nad y² cm² respectively, and EF = a
ty a

cm, then BC (in cm) is: A


di M

ABC DEF
x y
x² cm² y² cm² EF = a cm BC
_____ 11
z 0º 120º
SSC CGL 01/12/2022 (Shift- 02)
B C D
y 2
y
(a) (b) SSC CGL 01/12/2022 (Shift III
a2 x 2 ax
(a) 100º (b) 130º
ax a2 x 2 (c) 120º (d) 110º
(c) (d)
y y2 6. In triangle ABC, the bisector of angle BAC cuts
the line BC at D. If BD = 6 and BC = 14 then
A

3. If the angles of a triangle are (x – 46) degrees,


what is the value of AB : AC?
(x + 96) degrees and 8x degrees, then what
is the value of 2x? ABC BAC BC
(x – 46) (x + 96) D BD = 6 BC = 14
8x 2x AB : AC

SSC CGL 01/12/2022 (Shift- 02) SSC CGL 01/12/2022 (Shift- 03)
(a) 3 : 4 (b) 7 : 3
(a) 15 degrees (b) 24 degrees
(c) 3 : 10 (d) 3 : 7
(c) 26 degrees (d) 13 degrees

Aditya Ranjan (Excise Inspector) Selected Selection 173


https://sscstudy.com/
Join Telegram- Maths by Aditya Ranjan TRIANGLE

7. In the given triangle, CD is the bisector of 11. In the given figure, AB=DB and AC = DC. If
BCA. CD = DA. If BDC = 76º, what is the ABD = 58º and DBC = (2x + 4)º, ACB = (y
degree measure of CBD? + 15)º and DCB = 63º, then the value of 2x
CD BCA CD + 5y is:
= DA BDC = 76º CBD AB = DB AC = DC
ABD = 58º DBC = (2x – 4)º,
ACB = (y + 15)º DCB = 63º
C 2x + 5y

B D A

r
SSC CGL 01/12/2022 (Shift- 04)

si
(a) 32º (b) 76º
an by SSC CGL 02/12/2022 (Shift- 04)
(c) 80º (d) 66º
(a) 325 (b) 273
8. In ABC, AB = AC, O is a point on BC such

n
that BO = CO and OD is perpendicular to AB (c) 259 (d) 268
and OE is perpendicular to AC. If BOD = 60º, 12. O is the incentre of the triangle PQR. If angle
ja
then measure of AOE is: POR = 140 degree, then what is the angle
R s
PQR?
ABC, AB = AC O, BC
a th

BO = CO OD AB O PQR POR = 140

OE AC BOD =
PQR
60º AOE SSC CGL 02/12/2022 (Shift- 04)
ty a

SSC CGL 02/12/2022 (Shift- 02) (a) 40 degree (b) 140 degree
(c) 100 degree (d) 70 degree
di M

(a) 120º (b) 60º


13. From the circumcentre L of XYZ, perpendicular
(c) 30º (d) 90º
LM is drawn on side YZ. If XYZ = 60º, then
9. The sides of a triangle are in the ratio 4 : 6 : the measure of YLM is:
8. The triangle is a/an:
XYZ L YZ LM
XYZ = 60º, YLM
SSC CGL 02/12/2022 (Shift- 04)
SSC CGL 02/12/2022 (Shift- 02)
(a) 60º (b) 120º
(a) isosceles triangle (b) obtuse-angled (c) 180º (d) 90º
(c) acute-angled (d) right-angled 14. In triangle ABC, the bisector of angle BAC cuts
A

10. Side of an equilateral triangle is 24 cm. What the side BC at D. If AB = 10 cm, and AC = 14
will be the radius of in circle of this equilateral cm, then what is BD : BC ?
triangle? ABC BAC BC
D AB = 10 AC =
14 BD : DC
SSC CGL 02/12/2022 (Shift- 03) SSC CGL 03/12/2022 (Shift- 01)
(a) 6 cm (b) 12 cm (a) 10 : 7 (b) 5 : 7
(c) 8 cm (d) 3 cm (c) 7 : 5 (d) 7 : 10

Aditya Ranjan (Excise Inspector) Selected Selection 174


https://sscstudy.com/
Join Telegram- Maths by Aditya Ranjan TRIANGLE

15. In an equilateral triangle ABC, D is the 19. Suppose ABC be a right-angled triangle where
midpoint of side BC. If the length of BC is 8 A = 90º and AD  BC. If area (ABC) = 80
cm, then the height of the triangle is: cm² and BC = 16 cm, then the length of AD
ABC D BC is:

BC 8 cm ABC A =

_____ 90º AD  BC (ABC) = 80


cm² BC = 16 cm AD
SSC CGL 03/12/2022 (Shift- 02)
SSC CGL 03/12/2022 (Shift- 03)
(a) 5.5 cm (b) 4.5 cm (a) 10 cm (b) 24 cm
(c) 636 cm (d) 43 cm (c) 18 cm (d) 12 cm
16. If ABC ~ FDE such that AB = 9 cm, AC = 20. In a ABC, the median BE intersects AC at
11 cm, DF = 16 cm and DE = 12 cm, then E. If BG = 12 cm, where G is the centroid,
the length of BC is: then BE is equal to:

ABC ~ FDE AB = 9 cm,


ABC AC BE E
AC = 11 cm, DF = 16 cm DE = 12 cm,
BG = 12 cm G BE

r
BC

si
SSC CGL 03/12/2022 (Shift- 04)
SSC CGL 03/12/2022 (Shift- 02)
(a) 16 cm (b) 18 cm

(a) 5
3
4
an by
cm (b) 4
3
cm 21.
(c) 15 cm (d) 13 cm
Study the given triangle and find the length

n
5
of BC.
5 3 BC
(c) 3 cmja (d) 6 cm
R s
7 4
17. XYZ ~ GST and XY: GS = 2:3, XV is the
a th

median to the side YZ, and GD is the median B


2
 YV 
to the side ST. the value of  is _____.
 SD 
ty a

10
x
XYZ ~ GST XY : GS = 2 : 3, XV YZ
di M

GD ST
2 30º
 YV 
 
SD 
A y C

SSC CGL 03/12/2022 (Shift- 02) SSC CGL 03/12/2022 (Shift- 04)
4 3 5
(a) (b) (a) (b) 6
9 9 2
(c) 5 (d) 3
1 2
(c) (d) 1
4 3
A

22. If BPQ ASR and A = R =S then find


3
18. A,B,C are three angles of a triangle. If A – B
Q. (All angles are in degrees).
= 45º and B – C = 15º then A = ?
1
A, B, C A – B = 45º BPQ ASR A = R = S
3
B – C = 15º A = ?
Q
SSC CGL 03/12/2022 (Shift- 03)
SSC CGL 03/12/2022 (Shift- 04)
(a) 83º (b) 85º (a) 108º (b) 36º
(c) 95º (d) 75º (c) 72º (d) 118º

Aditya Ranjan (Excise Inspector) Selected Selection 175


https://sscstudy.com/
Join Telegram- Maths by Aditya Ranjan TRIANGLE

23. In triangle ABC, AD is the angle bisector of 28. The perimeter of a triangle with sides of
angle A. If AB = 8.4 cm and AC = 5.6 cm and integer values is equal to 13. How many such
DC = 2.8 cm, then the length of side BC will triangles are possible?
be:
ABC AD A
AB = 8.4 cm AC = 5.6 cm DC = SSC CGL 05/12/2022 (Shift- 03)
2.8 cm BC (a) 5 (b) 8
SSC CGL 05/12/2022 (Shift- 01) (c) 7 (d) 6
29. In a ABC, the internal bisectors of the angle
(a) 4.2 cm (b) 5.6 cm
B and the angle C intersect at an angle of 105º.
(c) 7 cm (d) 2.8 cm What is the value of the angle A?
24. I ABC B C
m 105º A
SSC CGL 05/12/2022 (Shift- 01)

r
(a) A (b) A SSC CGL 05/12/2022 (Shift- 03)
(a) 52.5º (b) 75º

si
(c) A (d) A
(c) 15º (d) 30º
25. ABC and DEF are congruent respectively.
an by
If AB = 6 = DE, BC = 8 = EF and m B = 30º,
then m D + m C =_________.
30. If ABC ~ QRP,
ar  ABC  9
= , AB = 18 cm,
ar  QRP  4

n
ABC DEF AB = 6 BC = 15 cm, then the length of PR is:
ja
= DE, BC = 8 = EF m B = 30º m
ar  ABC  9
R s
D + m C ABC ~ QRP, = , AB = 18 cm,
ar  QRP  4
a th

SSC CGL 05/12/2022 (Shift- 02)


BC = 15 cm PR
(a) 160º (b) 120º
SSC CGL 05/12/2022 (Shift- 04)
(c) 130º (d) 150º
(a) 16 cm (b) 14 cm
ty a

26. If an angle of a right-angled triangle is 35º, (c) 10 cm (d) 12 cm


then find the other angles.
31. If ABC ~ DEF, and BC = 4 cm, EF = 5 cm
di M

35º and the area of triangle ABC = 80 cm², then


the area of the triangle DEF is:

SSC CGL 05/12/2022 (Shift- 02) ABC ~ DEF BC = 4 cm, EF = 5


cm ABC 80 cm²
(a) 90º and 75º (b) 90º and 35º
DEF
(c) 90º and 55º (d) 90º and 45º
SSC CGL 05/12/2022 (Shift- 04)
27. The lengths (in cm) of three sides of a triangle
(a) 169 cm² (b) 80 cm²
are, respectively, 48, 55 and 73. What is the
length (in cm) of the median joining the mid- (c) 144 cm² (d) 125 cm²
point of the longest side to its opposite 32. In a ABC, AB = BC, and P, Q and R are the
mid points of the sides AB, BC and AC,
A

vertex?
respectively. If PR = 5 cm, then find the length
(cm ) of QR.
ABC AB = BC, P, Q R
AB, BC AC PR = 5 cm
(cm ) QR
SSC CGL 05/12/2022 (Shift- 03) SSC CGL 05/12/2022 (Shift- 04)
(a) 27.5 (b) 73 (a) 5 cm (b) 2 cm
(c) 24 (d) 36.5 (c) 7 cm (d) 4 cm

Aditya Ranjan (Excise Inspector) Selected Selection 176


https://sscstudy.com/
Join Telegram- Maths by Aditya Ranjan TRIANGLE

33. The mid points of AB and AC of a ABC are 37. If the angles of a triangle are in the ratio of
X and Y, respectively. If BC + XY = 24 units, 1 : 2 : 3, what is the type of such triangle?
then the value of BC – XY is:
ABC AB AC X
Y BC + XY = 24 BC – XY SSC CGL 07/12/2022 (Shift- 01)
(a) Isosceles triangle
(b) Equilateral triangle
SSC CGL 06/12/2022 (Shift- 01)
(c) Right-angle triangle
(a) 5 cm (b) 4 cm (d) Obtuse-angle triangle
(c) 6 cm (d) 8 cm 38. In triangle XYZ, P is the incentre of the
triangle XYZ. If angle XYZ = 50º, then what
34. AD is the median of triangle ABC. P is the
is the value of angle XPZ?
centroid of triangle ABC. If AP = 14 cm, then
what is the length of PD? XYZ P XYZ
XYZ = 50º XPZ

r
AD ABC P ABC
SSC CGL 07/12/2022 (Shift- 02)
AP = 14 PD

si
(a) 100º (b) 115º
an by SSC CGL 06/12/2022 (Shift- 01) (c) 140º (d) 130º
(a) 14 cm (b) 28 cm 39. Two concentric circles are drawn with radii

n
20 cm and 16 cm. What will be the length of
(c) 21 cm (d) 7 cm a chord of the larger circle which is tangent
35. ja
PQR is an isosceles triangle and PQ = PR = to the smaller circle?
R s
2a unit, QR = a unit. Draw PX  QR, and find
a th

the length of PX.


PQR PQ = PR = 2a
QR = a PX  QR PX SSC CGL 07/12/2022 (Shift- 02)
ty a

(a) 34 cm (b) 24 cm
(c) 48 cm (d) 12 cm
di M

SSC CGL 06/12/2022 (Shift- 04)


40. If ABC ~ DEF such that 2AB = DE and BC
= 8 cm, then the length of EF is:
5a
(a) 5a (b) ABC ~ DEF 2AB = DE
2
BC = 8 cm, EF
SSC CGL 07/12/2022 (Shift- 02)
10a 15a
(c) (d) (a) 16 cm (b) 18 cm
2 2
(c) 22 cm (d) 20 cm
36. ABC ~ DEF such that AB = 9.1 cm and DE
= 6.5 cm. If the perimeter of DEF = 25 cm, 1
41. In a XYZ, XO is the median and XO = YZ.
then the perimeter of ABC is: 2
A

If YXO = 30º, then what is the value of


ABC ~ DEF AB = 9.1 cm XYZ?
DE = 6.5 cm DEF = 25 cm,
1
ABC XYZ XO XO = YZ
2
SSC CGL 06/12/2022 (Shift- 04) YXO = 30º, XYZ
(a) 40 cm (b) 30 cm SSC CGL 07/12/2022 (Shift- 03)

(c) 35 cm (d) 45 cm (a) 15º (b) 90º


(c) 30º (d) 60º

Aditya Ranjan (Excise Inspector) Selected Selection 177


https://sscstudy.com/
Join Telegram- Maths by Aditya Ranjan TRIANGLE

42. If in a triangle ABC as drawn in the figure, AB 45. In ABC and PQR, AB = PQ and B = Q.
= AC and ACD = 130º, then BAC is equal to: The two triangles are congruent by SAS
ABC criteria if:
AB = AC ACD = 130º ABC PQR AB = PQ B = Q
BAC _____ (SAS)
A
SSC CGL 08/12/2022 (Shift- 02)
(a) BC = QR (b) AC = PR
(c) AC = QR (d) BC = PQ
13 46. In ABC and DEF, A = 55º, AB = DE, AC =

DF, E = 85º and F = 40º. By which property
B C D are ABC and DEF congruent?
SSC CGL 07/12/2022 (Shift- 04)
DEF A = 55º, AB = DE, AC =

r
ABC
(a) 60º (b) 50º
DF, E = 85º F = 40º

si
(c) 70º (d) 80º
ABC DEF
43. In ABC, P and Q are points on AB and BC,
an by
respectively, such that PQ || AC. Given that SSC CGL 08/12/2022 (Shift- 02)
AB = 26, PQ = 7 and AC = 10 find the value (a) SAS property

n
of AP.
(b) ASA property
ABC ja
P Q AB BC
R s
(c) RHS property
PQ || AC AB = 26, PQ =
a th

7 AC = 10 AP (d) SSS property

SSC CGL 08/12/2022 (Shift- 01) 47. If the angles of a triangle are (x – 6) degree,
(x + 26) degree and 8x degree, then what is
(a) 7.1 (b) 7.8
the value of 2x?
ty a

(c) 18.2 (d) 16.4


(x – 6) (x + 26)
44. ABCD is a square and MAB is an equilateral
di M

triangle. MC and MD are joined. What is the 8x 2x


degree measure of MDC? SSC CGL 08/12/2022 (Shift- 03)
ABCD MAB MC (a) 12 degrees (b) 16 degrees
MD MDC
(c) 48 degrees (d) 32 degrees
SSC CGL 08/12/2022 (Shift- 01)
48. In a ABC, AD, BE and CF are the medians
M from vertices A, B and C, respectively. The
point of intersection of AD, BE and CF is called.

ABC AD, BE CF A, B
A

C AD, BE CF
A B
___
SSC CGL 08/12/2022 (Shift- 03)
(a) median point
(b) orthocentre
D C
(c) centroid
(a) 78º (b) 60º
(c) 65º (d) 75º (d) incentre

Aditya Ranjan (Excise Inspector) Selected Selection 178


https://sscstudy.com/
Join Telegram- Maths by Aditya Ranjan TRIANGLE

49. In the follwing figure, if l || m, then find the 53. Triangle ABC and DEF are similar. If AB = 92
measures of angles marked by a and b. cm, BC = 48 cm, AC =120 cm, and the length
if l || m a b of the smallest side of DEF is 200 cm, then
find the length of the longest side of triangle
DEF?
l m
ABC DEF AB = 92 cm,
BC = 48 cm, AC = 120 cm DEF
0º 200 cm DEF
c 11
a
40º
SSC CGL 09/12/2022 (Shift- 02)
b
(a) 400 cm (b) 225 cm
(c) 350 cm (d) 500 cm
54. Two angles of a triangle are equal and the third
angle measures 78º. What is the measure of
each of the unknown angles?
SSC CGL 08/12/2022 (Shift- 04)

r

(a) a = 90º and b = 90º

si
78º
(b) a = 55º and b = 125º
SSC CGL 09/12/2022 (Shift- 02)
an by
(c) a = 70º and b = 110º
(d) a = 60º and b = 120º
(a) 50º
(c) 49º
(b) 51º
(d) 52º

n
50. In a PRQ, AB is drawn parallel to QR, cutting 55. If it is given that for two right angled triangles
sides at A and B where length of PQ = 6cm, ABC and DFE, A = 25º, E = 25º, B = F =
ja
length of QR = 8cm and length of QA = 3cm. 90º and AC = ED, then which one of the
R s
What is the length of AB?
following is TRUE?
a th

PQR AB, QR ABC


A B PQ DFE A = 25º, E = 25º, B = F
= 6 cm, QR = 8 cm QA
= 90º AC = ED
ty a

= 3 cm AB

SSC CGL 08/12/2022 (Shift- 04)
SSC CGL 09/12/2022 (Shift- 03)
di M

(a) 4.0 cm (b) 4.2 cm (a) ABC FED (b) ABC  DEF
(c) 5.0 cm (d) 2.4 cm (c) ABC  EFD (d) ABC  DEF
51. In triangle ABC, B = 90º, and C = 45º. If 56. If ABC  PQR, BC = 6 cm, and A = 75º,
AC = 22 cm, then the length of BC is: then which one of the follwoing is true?
ABC B = 90º, C = 45º ABC  PQR, BC = 6 cm A = 75º
AC = 22 cm BC
SSC CGL 09/12/2022 (Shift- 01) SSC CGL 09/12/2022 (Shift- 03)
(a) 3 cm (b) 2 cm (a) QR = 6 cm, R = 75º
(b) QR = 6 cm, Q = 75º
(c) 1 cm (d) 4 cm
(c) QR = 6 cm, P = 75
52. ABC is an isosceles triangle such that AB =
A

(d) PR = 6 cm, P = 75º


AC, ABC = 55º, and AD is the median to the
57. The side of an equilateral triangle is 36 cm.
base BC. Find the measure of BAD.
What is the radius of the circle circumscribing
ABC AB = AC, ABC this equilateral triangle?
= 55º, AD, BC BAD

SSC CGL 09/12/2022 (Shift- 01) SSC CGL 09/12/2022 (Shift- 03)
(a) 50º (b) 55º (a) 133 cm (b) 103 cm
(c) 35º (d) 90º (c) 123 cm (d) 93 cm

Aditya Ranjan (Excise Inspector) Selected Selection 179


https://sscstudy.com/
Join Telegram- Maths by Aditya Ranjan TRIANGLE

58. If the ratio of corresponding sides of two


similar triangles is 5 : 7 then what is the

ratio of the area of the two triangles?

(c) Two triangles are said to be congruent if
5 : 7 2 sides and the included angle of one
triangle are equal to 2 sides and the
SSC CGL 09/12/2022 (Shift- 04) included angle of the other triangle.

(a) 3
5 : 7 (b) 25 : 49

(c) 5 : 7 (d) 5 : 7

59. In ABC, D and E are points on sides AB and (d) Two triangles are said to be congruent if
AC, such that DE || BC. If AD = x + 3, DB = any pair of 2 angles and any 1 pair of sides
2x – 3, AE = x + 1 and EC = 2x – 2, then the
of both the triangles are equal.
value of x is:

ABC D E AB AC

r
DE || BC AD = x +

si
3, DB = 2x – 3, AE = x + 1 EC = 2x – 2,
62. If the areas of two isosceles triangles are in
x the ratio of x²: y², then the ratio of their
an by SSC CGL 12/12/2022 (Shift- 01) corresponding heights is:
x² : y²

n
4 1
(a) (b)
5 2
ja SSC CGL 12/12/2022 (Shift- 04)
R s
3 1 (a) x : y (b) x : y
(c) (d) (c) x³ : y³ (d) x² : y²
5 5
a th

63. ABC ~ DEF and the perimeters of these


60. If ABC ~ EDF such that AB = 6 cm, DF = 16 triangles are 32 cm and 12 cm, respectively. If
cm and DE = 8 cm, then the length of BC is: DE = 6 cm, then what will be the length of AB?
ABC ~ EDF AB = 6 cm, ABC ~ DEF
ty a

DF = 16 cm DE = 8 cm BC 32 cm 12 cm DE = 6 cm AB

di M

SSC CGL 12/12/2022 (Shift- 01) SSC CGL 13/12/2022 (Shift- 01)
(a) 12 cm (b) 10 cm (a) 16 cm (b) 14 cm
(c) 14 cm (d) 8 cm (c) 12 cm (d) 18
64. In the figure BCDE is a square and ABC is
61. What is the ASA congruence rule of triangles,
equilateral triangle then ADC is:
where A and S represents angle and side of
BCDE ABC
triangle respectively?
ADC
(ASA)
A S A

SSC CGL 12/12/2022 (Shift- 03)
A

B C
(a) Two triangles are said to be congruent if
all three sides of both the triangles are equal.


E D
(b) Two triangles are said to be congruent if
2 angles and the included side of one SSC CGL 13/12/2022 (Shift- 01)
triangle are equal to 2 angles and the (a) 45º (b) 30º
included side of the other triangle. (c) 60º (d) 15º

Aditya Ranjan (Excise Inspector) Selected Selection 180


https://sscstudy.com/
Join Telegram- Maths by Aditya Ranjan TRIANGLE

65. In PQR, Q = 90º, PQ = 8 cm and PRQ = 70. In the given figure, if KI = IT and EK = ET,
45º. Find the length of QR. then TEI = ____.
PQR Q = 90º, PQ = 8 cm PRQ = KI = IT EK = ET
45º QR TEI =
SSC CGL 13/12/2022 (Shift- 01) K
(a) 6 cm (b) 3 cm
(c) 5 cm (d) 8 cm
66. ABC and DEF are similar triangles and their

150º
areas are 49 cm² and 144 cm² respectively. E I
If EF = 16.80 cm, then find BC.
ABC DEF
EF = 16.80
BC T
SSC CGL 13/12/2022 (Shift- 02) SSC CGL 13/12/2022 (Shift- 04)

r
(a) 7.5 cm (b) 9.8 cm (a) 75º (b) 125º

si
(c) 8.7 cm (d) 11.4 cm
(c) 105º (d) 150º
67. The side of an equilateral triangle is 9 cm.
an by 71. In a ABC, B + C = 110º, then find the
What is the radius of the circle circumscribing
measure of A.
this equilateral triangle?

n
ABCB + C = 110º A
ja

R s
SSC CGL 13/12/2022 (Shift- 03) SSC CGL 13/12/2022 (Shift- 04)
a th

(a) 23 cm (b) 53 cm (a) 90º (b) 70º


(c) 43 cm (d) 33 cm (c) 80º (d) 60º
68. The area of two similar triangles are 324 cm² 72. PQR is right-angled at Q. The length of PQ
ty a

and 289 cm², respectively. What is the ratio is 5 cm and PRQ = 30º. Determine the length
of their corresponding altitudes? of side QR.
di M

324 cm² PQR, Q PQ 5 cm


289 cm² PRQ = 30º QR
SSC CGL 13/12/2022 (Shift- 04)
SSC CGL 13/12/2022 (Shift- 03)
(a) 53 cm (b) 33 cm
17 17
(a) (b) 1 5
18 19 (c) cm (d) cm
3 3
19 18
(c) (d) 73. 'O' is a point in the interior of an equilateral
17 17
triangle. The perpendicular distance from 'O'
69. If ABC  PQR and ABC = (x + 60)º, PQR to the sides are 3 cm, 23 cm, 53 cm. The
A

= (85 – 4x)º, and RPQ = (3x + 65)º, then the perimeter of the triangle is:
value of ABC in degree is:
'O' 'O'
ABC  PQR ABC = (x + 60)º,
3 cm, 23 cm, 53
PQR = (85 – 4x)º RPQ = (3x + 65)º
cm
ABC
SSC CGL 13/12/2022 (Shift- 04)
SSC CGL 13/12/2022 (Shift- 03)
(a) 48 cm (b) 32 cm
(a) 15 (b) 5
(c) 45 (d) 65 (c) 24 cm (d) 64 cm

Aditya Ranjan (Excise Inspector) Selected Selection 181


https://sscstudy.com/
Join Telegram- Maths by Aditya Ranjan TRIANGLE

74. A,B, C are three points such that AB = 9 cm, BC ABCD BC


= 11 cm and AC = 20 cm. The number of circles A D BCA = 19°
passing through points A,B,C is:
CAD = 32° ACD
A, B, C AB = 9 cm, BC = SSC CGL TIER- II 03/03/2023
11 cm AC = 20 cm A, B, C (a) 41° (b) 38°
(c) 40° (d) 39°
SSC CGL TIER - II 02/03/2023 78. If the external angle of a regular polygon is
18°, then the number of diagonals in this
(a) 2 (b) 0 polygon is:

(c) 1 (d) 3 18°



75. A circle touches all four sides of a
SSC CGL TIER- II 03/03/2023
quadrilateral ABCD. If AB = 18 cm, BC = 21
(a) 180 (b) 150
cm and AD = 15 cm, then length CD is:
(c) 170 (d) 140

r
ABCD 79. If the areas of two similar triangles are in the
AB = 18 cm, BC = 21 cm AD ratio 196 : 625, what would be the ratio of the

si
= 15 cm CD corresponding sides?

(a) 16 cm
an by SSC CGL TIER - II 02/03/2023
(b) 14 cm

n
SSC CGL TIER- II 03/03/2023
(c) 12 cm (d) 18 cm (a) 14 : 25 (b) 13 : 20
76.
ja
In PQR, PQ = QR and O is an interior point
R s
(c) 14 : 20 (d) 13 : 25
of PQR such that OPR = ORP.
80. If ABC is right angled at B, AB = 12 cm. and
a th

Consider the following statements: CAB = 60°, then find the length of BC.
(i) POR is an isosceles triangle.  ABC, B AB = 12 cm
CAB = 60° BC
ty a

(ii) O is the centroid of PQR.


SSC CGL TIER- II 06/03/2023
(iii)PQO is congruent to RQO.
di M

(a) 24 3 cm (b) 12 cm
Which of the above statements are correct?

PQR PQ = QR PQR O (c) 12 3 cm (d) 12 2 cm


OPR = ORP 81. L is the centroid of an equilateral triangle PQR.
If PQ = 6 cm, then what is the length of PL?

PQR L PQ = 6
(i) POR cm, PL

(ii) O PQR SSC CGL TIER- II 06/03/2023

(iii) PQO RQO (a) 5 3 cm (b) 4 3 cm


A

(c) 3 3 cm (d) 2 3 cm
SSC CGL TIER - II 02/03/2023 82. Find the number of diagonals of a regular
polygon whose interior angles sum to 2700°.
(a) Only (i) and (ii) (b) Only (i) and (iii)

(c) Only (ii) and (iii) (d) Only (ii)
2700°
77. ABCD is a cyclic quadrilateral and BC is a
SSC CGL TIER- II 06/03/2023
diameter of the related circle on which A and
(a) 127 (b) 121
D also lie. BCA = 19° and CAD = 32°. What
is the measure of ACD? (c) 119 (d) 117

Aditya Ranjan (Excise Inspector) Selected Selection 182


https://sscstudy.com/
Join Telegram- Maths by Aditya Ranjan TRIANGLE

83. If one of the interior angles of a regular polygon 86. For what value of m will the system of
equations 17x + my + 102 = 0 and
15
is times of one of the interior angles of a 23x + 299y + 138 = 0 have infinite number
16
of solutions?
regular decagon, then find the number of
diagonals of the polygon. m
17x + my + 102 = 0 23x + 299y + 138 =
0
15
16
SSC CGL TIER - II 02/03/2023
(a) 221
SSC CGL TIER- II 07/03/2023 (b) 223
(a) 20 (c) 220
(b) 14 (d) 219

r
(c) 2 87. For which of the following values of the system

si
of equations 18x – 72y + 13 = 0 and
(d) 35
7x – my – 17 = 0 will have no solution?
an by
84. PQRS is a cyclic quadrilateral. IfP is three
times of R and S is four times of Q, then
m
18x – 72y + 13 = 0 7x – my – 17 = 0

n
the sum of S + R will be:
PQRS ja
P, R SSC CGL TIER- II 06/03/2023
R s
(a) 9
S, Q S + R
(b) 12
a th

(c) 24
SSC CGL TIER- II 07/03/2023 (d) 28
ty a

(a) 169° 88. Find the coordinates of the points where the
(b) 171° graph 57x – 19y = 399 cuts the coordinate
di M

axes.
(c) 187°

(d) 189°
57x – 19y = 399
85. It is given that ABC  PQR, AB = 5 cm,
SSC CGL TIER- II 07/03/2023
B = 40°, and A = 80°. Which of the following
options is true? (a) x – axis at (– 7,0) and y – axis at (0, – 21)

 ABC   PQR, AB = 5 cm, x– (– 7,0) y – (0, – 21)


B = 40°, A = 80°
(b) x – axis at (– 7,0) and y – axis at (0,21)

x– (– 7,0) y – (0,21)
A

SSC CGL TIER- II 07/03/2023


(c) x – axis at (7,0) and y – axis at (0, – 21)
(a) PQ = 5 cm and R = 60°
(b) QR = 5 cm and R = 60° x– (7,0) y – (0, – 21)

(c) QR = 5 cm and Q = 60° (d) x – axis at (7,0) and y – axis at (0,21)

(d) PQ = 5 cm and P = 60° x– (7,0) y – (0,21)

Aditya Ranjan (Excise Inspector) Selected Selection 183


https://sscstudy.com/
Join Telegram- Maths by Aditya Ranjan TRIANGLE

ANSWER KEY
1.(d) 2.(c) 3.(c) 4.(c) 5.(c) 6.(a) 7.(d) 8.(c) 9.(b) 10.(c)

11.(b) 12.(b) 13.(a) 14.(b) 15.(d) 16.(d) 17.(a) 18.(c) 19.(a) 20.(b)

21.(c) 22.(a) 23.(c) 24.(d) 25.(d) 26.(c) 27.(d) 28.(a) 29.(d) 30.(c)

31.(d) 32.(a) 33.(d) 34.(d) 35.(d) 36.(c) 37.(c) 38.(b) 39.(b) 40.(a)

41.(c) 42.(d) 43.(b) 44.(d) 45.(a) 46.(a) 47.(d) 48.(c) 49.(c) 50.(a)

51.(b) 52.(c) 53.(d) 54.(b) 55.(c) 56.(c) 57.(c) 58.(d) 59.(c) 60.(a)

r
si
61.(b) 62.(a) 63.(a) 64.(d) 65.(d) 66.(b) 67.(d) 68.(d) 69.(d) 70.(c)

71.(b)
an by
72.(a) 73.(a) 74.(b) 75.(d) 76.(b) 77.(d) 78.(c) 79.(a) 80.(c)

n
81.(d) 82.(c) 83.(a) 84.(d) 85.(a) 86.(a) 87.(d) 88.(c)
ja
R s
a th
ty a
di M
A

Aditya Ranjan (Excise Inspector) Selected Selection 184


https://sscstudy.com/
Join Telegram- Maths by Aditya Ranjan TRIANGLE

SOL U TION S
1. (d) A AB = AD (given)
AE = AC
BAC = EAD (VOA)
60°
 ABC  ADE
C = 12 10 = b
 BC = ED = 11 cm.
5. (c) A
B C
a x y
We know,

r
a² = b² + c² – 2bc cosA 120°
z 110°

si
= 100 + 144 – 2 × 120 × cos60º
B C D F
1
an by A + B = ADF [Exterior angle]
= 244 – 240 
2
 x + y + z = 120°

n
a² = 124 6. (a)
A
a = 124  11ja
R s
= 11.13 cm.
a th

2
ar ABC (x)²  BC 
2. (c) = =
 

ar DEF (y)²  EF 

Given, EF = a
ty a

B C
6 D 8
BC² x² By angle bisector theorem:-
di M

 =
a² y²
AB BD 6 3
= = =
ax AC DC 8 4
 BC =
y 7. (d) C

3. (c) By angle sum property:-


x x
(x – 46º) + (x + 96º) + 8x = 180º
 10x = 180º – 50º = 130º
y 76°
x = 13º x
2x = 26º B D A
4. (c) E BCD = DCA = x (given)
A

D
DCA = DAC = x [ CD = AD]
7 Also,
A 2x = 76º [Exterior angle]
x = 38º
By angle sum property:-
B C y + 3x = 180º.
11 y = 180º – 38º × 3
In ABC, ADE y = 66º.

Aditya Ranjan (Excise Inspector) Selected Selection 185


https://sscstudy.com/
Join Telegram- Maths by Aditya Ranjan TRIANGLE

8. (c) A AB = BD
AC = CD
BC = BC
 ABC  DBC (Bysss)
D E
 ABC = DBC
58
 2x – 4 = = 29
2
60° 60°
 2x = 33
B O C
Also,
AB = AC y + 15 = 63
 ABC = ACB = x (say)  y = 48  2x + 5y = 273
In ODB, OEC 12. (c) P
D = E
B = C

r
 BOD = EOC = 60°

si
Since, ABC is an isosceles triangle. 140°
 AO is the perpendicular bisector
an by O
 AOC = 90º

n
 AOE = 90º – 60º Q R
= 30º
ja 1
9. (b) Given:- POR = 90 + PQR
R s
Sides ratio = 4 : 6 : 8 2
a th

=2:3:4 1
Consider, 140º = 90º + PQR
2
(2x)² + (3x)² = 4x² + 9x²
 PQR = 100º
= 13x²
ty a

13. (a) X
13x² < (4x)² = 16x²
(Incorrect question)
 a² + b² < C²
di M

 It is an obtuse angled triaugle. 60°

10. (c) In radius = Side


2 3 L

12
=
3 Y Z
M
=4 3 YXZ = 60°
A  YLZ = 2 × YXZ
= 120°
A

58° In YLM, ZLM


(y+15)º YM = MZ
11. (b) B C
(2x+4)º 63° YML = LMZ
LY = LZ (Radius of circumcircle)
 YLM  ZLM (By RHS)
D
(Question printing mistake) 120º
 YLM = ZLM = = 60º
2
In ABC, DBC

Aditya Ranjan (Excise Inspector) Selected Selection 186


https://sscstudy.com/
Join Telegram- Maths by Aditya Ranjan TRIANGLE

14. (b) 17. (a) X G


A

10 14

Y V Z S D T
XYZ ~ GST
B C 1
D YZ
XY 2 YZ 2
By angle bisector theorem:- = = =
GS 3 ST 1 ST
AB BD 10 2
= =
AC DC 14 YV
=
5 BD SD
 =

r
7 DC
 YV 2 4

si
15. (d) A
 
  =

 SD  9
an by 18. (c) A – B = 45º
B – C = 15º

n
A + B + C = 180º (Angle sum prop.)
let c = x
ja
R s
60º + x + 15º + x + x = 180º
3x = 180º – 75º
a th

B C 105º
D x= = 35º
3
Side = 8 cm
ty a

 A = 60 + x = 95º
3
Height =  side
2 19. (a) B
di M

3
= 8
2 D
= 4 3 cm.
16. (d) A F

A
9 11 16 C
Area AB C = 80
B C D E
A

12 1
 × base × height = 80
ABC ~ FDE 2

AB BC 1
 =  × BC × AD = 80
FD DE 2
9 BC
 = 1
16 12  × 16 × AD = 80
2
27 3
 BC = = 6 cm.  AD = 10 cm.
4 4

Aditya Ranjan (Excise Inspector) Selected Selection 187


https://sscstudy.com/
Join Telegram- Maths by Aditya Ranjan TRIANGLE

20. (b) A 23. (c) A

8.4 5.6
E

12 G B C
D 2.8
By Angle Bisector theorem:-
B C
We know, AB BD
=
AC CD
BG 2
=
GE 1 8.4 BD
 =  BD = 4.2
12 2 5.6 2.8
 =

r
GE 1  BC = 4.2 + 2.8
 GE = 6 = 7 cm.

si
 BE = 12 + 6 = 18 24. (*)

21. (c)
an by B d
25. (d) A D

n
ja 10 6 6
R s
X
a th

30°
B C E F
8 8
30°
A ABC  DEF
Y C
ty a

 B = E
BC x
Sin 30 = = A = D
AB 10
di M

1
C = F
 x=  10 = 5
2 A + B + C = 180º
22. (a) A + C = 150º
B A
 D + C = 150º
x
26. (c) A

3x x 35°
Q P R S
1
A = R =  S
A

3
BPQ  ASR 90°
B C
 Q = R
In ASR:- A + B + C = 180º
x + 3x + x = 180º.
35º + 90º + C = 180º
5x = 180º
x = 36º C = 55º
 3x =108º  90º and 55º

Aditya Ranjan (Excise Inspector) Selected Selection 188


https://sscstudy.com/
Join Telegram- Maths by Aditya Ranjan TRIANGLE

27. (d) B ar ABC = 80


2
ar ABC  BC  16
=
  =

D 73 ar DEF  EF  25
48
80 16
 =
ar DEF 25

A C  ar DEF = 125 cm²


55
Given, sides are triplets of a right angle triangle 32. (a) A
BC 73
 AD = = = 36.5
2 2
28. (a) a + b + c = 13
P + 32 5
No. of possible s = P R
18
16  16
=

r
48
 5 B C

si
29. (d) Q
A
an by In PQR, ACB
PQ QR PR 1
= = =

n
AC AB BC 2
O
 PQR ~ ACB
ja 105°
R s
1 PR QR
= =
B C 2 BC AB
a th

1  BC = 10 = AB
BOC = 90º + BAC
2  QR = 5 cm.
(105º – 90º) = BAC
ty a

 30º = BAC 33. (d) A


30. (c) A Q
di M

18 X Y

B C R P
15
ar ( ABC)  AB 
2
9 3 
2 B C
=  = =  
 
ar (QRP)  
 QR  4 2  BC + × y = 24
Also, Also,
AB BC 3 1
= = BC = XY
QR PR 2 2
A

15 3 [Since, X and Y are mid pts. of AB & AC By


=
PR 2 1
similarity, XY = BC]
 PR = 10 2
31. (d) A D BC 1
=
XY 2
 3  24
18
B C E F  BC – XY = 8
4 5

Aditya Ranjan (Excise Inspector) Selected Selection 189


https://sscstudy.com/
Join Telegram- Maths by Aditya Ranjan TRIANGLE

34. (d) A Perimeter of DEF DE


=
Perimeter of ABC AB

M 25 6.5
 =
P of ABC 9.1
P
 Perimeter of ABC = 35 cm.
37. (c) a = x b = 2x c = 3x
B C x + 2 x + 3x = 180°
D
We know, 6x = 180°
x = 30°
AP 2
=  a = 30°, b = 60°, c = 90°
PD 1
 PD = 7cm  Right angled 

r
35. (d) P 38. (b) X

si
an by
2a 2a
P

n
ja 50°
R s
Q Y Z
a x a R
XYZ = 50º
a th

2 2
PQ = PR = 2a 1
 XPZ = 90º + × XYZ
In an is os cales  2
ty a

mediam = perpendicular bisector = 90º + 25º


di M

a = 115º
 QX =
2 39. (b)
By Pythagoras theorem in PXQ
 PQ² = QX² + PX²
a² 20
 PX² = –  4a² 16
4

15a²

4
A

a 15 Radius  Tangent
 PX =
2
 Tangent 2
36. (c) A D  (20)² = (16)² + 
 

 2 

9.1 6.5  Tangent 2



  = (12)²

 2 

B C E F  Tangent = 24 cm.

Aditya Ranjan (Excise Inspector) Selected Selection 190


https://sscstudy.com/
Join Telegram- Maths by Aditya Ranjan TRIANGLE

40. (a) A D 43. (b) A

x 2x

26 P 10
B C E F
8
AB DE 7
=
BC EF
B Q C
x 2x PQ || AC
 =
8 EF
 BQP ~ BCA
 EF = 16 cm.
PQ PB 7
41. (c)  = =
X AC AB 10

r
30°
PB 7

si
 =
26 10
an by  PB =
91
5

n
Y Z 91
ja O
AP = AB – PB = 26 – = 7.8 cm.
R s
5
In  XYO:-
44. (d) M
a th

1
OX = OY = YZ
2
60°
ty a

 YXO = XYO
A B
[Angles opposite to equal sides are equal]
di M

90°
 XYO = 30°
42. (d)
A

80° D C
ABCD is a square
 AB = AD, AB = AM
50° 50° 130°  AD = AM
B C D In ADM:-
A

ACB = 180º – ACD ADM = AMD


[Linear angles] DAB + BAM + ADM + AMD = 180º
 ACB = 50º = ABC 90º + 60º + 2 ADM = 180º
[Angles opp. to equal side] ADM = 15º
 A + B+ ACB = 180º ADC = 90º
[Angle sum property]  MDC = 90º – 15º
 A = 180º – 100º = 80º = 75º

Aditya Ranjan (Excise Inspector) Selected Selection 191


https://sscstudy.com/
Join Telegram- Maths by Aditya Ranjan TRIANGLE

45. (a) A P l 11 m
 C = 110º
 a = 180º – 110º [Linear angle]
b = 40º + a [Exterior angle]
= 40º + 70º
= 110º
B C Q R 50. (a) P
SAS  Angle includes in 2 equal sides
 BC = QR and AB = PQ
46. (a) A D
B A 6
55° 3

85° 40° R 8 Q
B C E F AB || QR

r
D = 180º – 85º – 40º  PBA ~ PRQ

si
AB PA 3
= 55º = A  = =
RQ PQ 6
an by
AB = DE & AC = DF
 ABC  DEF 
AB 3
8
=
6

n
By SAS.  AB = 4 cm.
47. (d) (x – 6) + (x + 26) + 8x = 180º 51. (b) A
ja
R s
By Angle sum Property
45°
 10x = 160º
a th

2 2
x = 16º
2x = 32º
48. (c)
ty a

A 45°
B
C
di M

BCA = CAB
 AB = BC = x, (say)
F E
O
 
2
 2x² = 2 2 =8
 x² = 4
 x = 2 = BC
52. (c) A
B D C
The point 'O' is called 'Centroid' where all
medians meet.
49. (c) l m
A

55° 55°
110°
B D C
40° a c ABD  ACD [SAS]
b 1
 BAD = CAD = BAC
2
BAC = 180º – 110º
= 70º
 BAD = 35º

Aditya Ranjan (Excise Inspector) Selected Selection 192


https://sscstudy.com/
Join Telegram- Maths by Aditya Ranjan TRIANGLE

53. (d) A D 57. (c) A

36
92 120

B 48 C E 200 F
B C
ABC ~ DEF
AC BC
 =
DF EF Side 36
R= =
= 12 3
3 3
120 48
 = 58. (d) Let ABC ~ PQR
DF 200
 5 = 5
2

ar ABC (AB)²

r
 DF = 500  Longest side   =
ar PQR (PQ)²
 7 7

si
54. (b) Let, equal augles be x°
59. (c) A
 2x° + 78 = 180°
an by
[Angle sum property]
x+3 x+1

n
 x = 51°
ja D E
55. (c) A D
R s
2x–3 2x–1
a th

25° 25°

B C
25° DE || BC
ty a

B C F E AD AE
 =
AB AC
A = E, B = F
di M

x3 x 1
AC = DE  =
3x 3x – 1
 BAC  FED  (x + 3) (3x – 1) = 3x (x + 1)
 3x² + 8x – 3 = 3x² + 3x
or  5x = 3
3
ABC  EFD x=
5
56. (c) 60. (a)
A P A

75°
6 8
A

B C D 16 F
B 6 C Q R ABC ~ EDF
ABC  PQR AB BC
 =
ED DF
 A = P = 75° 6 BC
 =
BC = QR = 6 8 16
 BC = 12 cm.

Aditya Ranjan (Excise Inspector) Selected Selection 193


https://sscstudy.com/
Join Telegram- Maths by Aditya Ranjan TRIANGLE

61. (b) ASA  2x + 60º + 90º = 180º


P  x = 15º
[Refer question 44 for detailed solution]
65. (d) P

8
Q R B C
When,
45°
P = A Q R
Q = B QPR = 180 – 90 – 45
PQ = AB = 45°
 PQR  ABC (By ASA)  PQ = QR
[Sides opp. to equal angles]
62. (a)

r
A D
 QR = 8 cm.

si
ar (ABC) 49 (BC)²
66. (b) ar (DEF) = 144 = (EF)²
an by 
BC
=
7

n
EF 12
B P C E Q F
ja 7
 BC =  16.8
R s
ar ABC x²  AP 
2 12
= =  = 9.8 cm.
ar DEF y²  
a th

 DQ 
Side
67. (d) Circumradius =
AP x 3
 =
DQ y
ty a

9
= =3 3
P ( ABC) AB 3
di M

63. (a) P (DEF) = DE


ar (ABC) 324 (AP)²
68. (d) = =
ar (DEF) 289 (DQ)²
32 AB
 =
12 6 AP 18
 =
 AB = 16 cm. DQ 17

64. (d) A 69. (d) A P

B C
A

B C Q R
ABC = x + 60º = PQR = 85º – 4x
 x + 60º = 85º – 4x
 5x = 25º
E D
x = 5º
BC = AC = CD
 ABC = x + 60º
 CAD = CDA = x (say) = 65º

Aditya Ranjan (Excise Inspector) Selected Selection 194


https://sscstudy.com/
Join Telegram- Maths by Aditya Ranjan TRIANGLE

70. (c) K 74. (b)


Number of circles passing through a given
triangle is
Since, for formation of a  AB + BC > AC
E Which is not true in given case.
150° I 75. (d)
D C
R

T
KEI  TEI 15 S Q 21
(By SSS)
 KEI = IKT = x (say)
P
 2x + 150º = 360º (complete angle)
A B
 x = 105º 18

r
71. (b) A + B + C = 180º We know, PA = AS, DS = DR1 RC = CQ, QB = PB
(Angle sum property)  DR + DS + SA + AP + PB + QB + CQ + CR =

si
A = 180º – 110º 15 + 18 + 21 + CD
an by  2 (DS + AS) + 2 (QB + CQ) = 54 + CD
A= 70º
 30 + 42 = 54 + CD
72. (a) P  CD = 18

n
76. (b)
P
ja
R s
5
a th

30°
Q R
P PQ O
ty a

tan 30º = =
B QR
Q R
di M

1 5 OP = OR
 =
3 QR  OPR is isosceles
I is true
 QR = 5 3  O is not centroid but an in centre II is false
73. (a) A  PQO  RQO (SAS) III is true
77. (d)

D
c b A
32°

o
19° C
A

B
B a C
2
Side = (P1  P2  P3 )
3

2 BAC = 90° [Angle formed by diameter]


=
3  32 35 3  A + C = 180°
 16  90 + 32 + 19 + ACD = 180°
 Perimeter = 3 × 16 = 48 cm.  ACD = 39°

Aditya Ranjan (Excise Inspector) Selected Selection 195


https://sscstudy.com/
Join Telegram- Maths by Aditya Ranjan TRIANGLE

78. (c) 82. (c)


We know, Sum of interior angles = (x – 2) × 180
180  2  2700 = (x – 2) × 180
Each external angle =
x = x = 17
360
 = 18 x (x – 3)
n  No. of diagonals =
2
n = 20
n (n – 3) 17  14
 No. of diagonals = = = 119
2 2

20  17 83. (a)
= = 170
2 Decagon = 10 sided polygon.
79. (a) A.T.Q,
Let ABC ~ PQR
(n – 2) 15 8
 180 =   180

r
ar ABC (AB)² 196 n 16 10
 = =
ar PQR (PQ)² 625

si
n–2 3
AB 14 =
 = an by n 4
PQ 25
 n = 8
80. (c)

n
A n (n – 3)
No. of diagonals =
ja 2
60°
R s
85
= = 20
a th

12 2
84. (d)
ty a

B C
BC S R
tan 60º =
di M

AB 4y x
BC
3=
12
 AB = 12 3
81. (d) 3x y
P P Q

Let R = x°
6 P = 3x°
A

L
Let, Q = y°
Q R S = 4y°
S
We know,
3
PS =  PQ = 3 3 R + P + Q + S = 4x + 5y = 360°
2
5y = 180° and 4x = 180°
2 2
PL =  PS =  3 3 = 2 3 y = 36° x = 45°
3 3
R + S = x + 4y = 45° + 144° = 189°

Aditya Ranjan (Excise Inspector) Selected Selection 196


https://sscstudy.com/
Join Telegram- Maths by Aditya Ranjan TRIANGLE

85. (a) 87. (d)


A P a1 b1 c1
For no. solution, a = b = c
2 2 2
80°
5 
18 – 72
 =
7 –m
40°
B C Q R  m = 28
C = 180° – 80° – 40° (Angle sum prop.) 88. (c)

= 60° = R For coordinate a x is  x = 0, y = 0


Then, 57x – 19y = 399 cut
86. (a)
coordinate a x is when x = 0
For infinite solutions
– 19y = 399
a1 b c y = – 21
= 1 = 1
a 2 b2 c 2
 (0, – 21)

r
a1 = 17, b1 = m, c1 = – 102 When, y = 0

si
a2 = 23, b2 = 299, c2 = – 138 57x = 399


17
=
m
23 299
an by
 m = 221
x=7
(7, 0)

n
ja ......-------......
R s
a th
ty a
di M
A

Aditya Ranjan (Excise Inspector) Selected Selection 197


https://sscstudy.com/
Join Telegram- Maths by Aditya Ranjan CIRCLE

CIRCLE
21
1. Two circles touch each other externally at P. 8 cm
AB is a direct common tangent to the two

circles, A and B are points of contact, and
PAB = 40º. The measure of ABP is:
P AB SSC CGL 01/12/2022 (Shift 04)
A B PAB (a) 83 cm (b) 3 cm
= 40º ABP (c) 23 cm (d) 43 cm
SSC CGL 01/12/2022 (Shift- 01) 6. A chord of length 42 cm is drawn in a circle

r
(a) 45º (b) 55º having diameter 58 cm. What is the minimum
(c) 50º (d) 40º distance of other parallel chord of length 40

si
2. Radius of a circle is 10 cm. Angle made by cm in the same circle from 42 cm long chord?
an by
chord AB at the centre of this circle is 60
degree. What is the length of this chord?

n

AB
ja

R s
SSC CGL 01/12/2022 (Shift- 04)
SSC CGL 01/12/2022 (Shift- 01)
a th

(a) 4 cm (b) 1 cm
(a) 40 cm (b) 20 cm
(c) 3 cm (d) 2 cm
(c) 30 cm (d) 10 cm
7. In the given figure, a circle is inscribed in
3. The side of an equilateral triangle is 12 cm.
PQR, Such that it touches the sides PQ, OR
ty a

What is the radius of the circle circumscribing


and RP at points D, E, F, respectively. If the
this equilateral triangle?
lengths of the sides PQ= 15 cm, OR = 11cm
di M

and RP = 13cm, then find the length of PD.


PQR
SSC CGL 01/12/2022 (Shift- 02)
PQ, QR RP
(a) 63 cm (b) 43 cm
D, E, F PQ
(c) 93 cm (d) 53 cm
= 15cm, QR = 11cm RP = 13 cm
4. O is the centre of this circle. Tangent drawn
from a point P, touches the circle at Q. If PQ PD
= 24 cm and OQ = 10 cm, then what is the
value of OP? R
O P
A

Q PQ = 24 F

OQ = 10 OP E
SSC CGL 01/12/2022 (Shift- 02)
(a) 26 cm (b) 52 cm P D Q
(c) 13 cm (d) 15 cm
5. Two equal circles of radius 8 cm intersect each
SSC CGL 02/12/2022 (Shift- 01)
other in such a way that each passes through
the centre of the other. The length of the (a) 9 cm (b) 8 cm
common chord is: (c) 7.5 cm (d) 8.5 cm

Aditya Ranjan (Excise Inspector) Selected Selection 198


https://sscstudy.com/
Join Telegram- Maths by Aditya Ranjan CIRCLE

8. The length of the tangent to a circle from a 12. How many circles can be drawn that pass(es)
point P is 15 cm. Point P is 17 cm away from through two fixed points?
the centre. What is the radius of the circle?
P
P 17 cm SSC CGL 06/12/2022 (Shift- 02)
(a) Infinite (b) Only two

(c) One or two (d) Only one
SSC CGL 02/12/2022 (Shift- 03)
13. A chord of length 40 cm is drawn in a circle
(a) 7 cm (b) 9 cm having diameter 50 cm. What is the minimum
(c) 8 cm (d) 4 cm distance of other parallel chord of length 30
9. Select the correct option with respect to the cm in the same circle from 40 cm long chord?
given statement. Two tangents are drawn at
the end of the diameter of a circle.


SSC CGL 06/12/2022 (Shift- 02)

r
SSC CGL 03/12/2022 (Shift- 01) (a) 10 cm (b) 15 cm

si
(a) They intersect each other. (c) 5 cm (d) 20 cm
14. A tangent is drawn from a point M to a circle,

an by which meets the circle at T such that MT =
(b) They pass through origin. 12 cm. A secant MAB intersects the circle at

n
points A and B. If MA = 8 cm, what is the
(c) They are parallel to each other. length (in cm) of the chord AB?
ja
M
R s
(d) They are perpendicular to each other. T MT
a th

= 12 cm MAB A
10. Let O be the center of a circle and P be a point B MA = 8 cm
outside the circle. If PAB is a secant of the AB
SSC CGL 06/12/2022 (Shift- 02)
ty a

circle which cuts the circle at A and B and


PT is the tangent drawn from P, then find the (a) 16 (b) 10
length of PT. PA = 3 cm and AB = 9 cm. (c) 12 (d) 9
di M

O P 15. In the figure, O is the centre of the circle. Its two


chords AB and CD intersect each other at the point
PAB, P within the circle. If AB = 20 cm, PB = 12 cm
A B PT,P and CP = 8 cm, then find the measure of PD.
PT PA = 3 O AB
cm AB = 9 cm CD P
SSC CGL 03/12/2022 (Shift- 03) AB = 20 cm, PB = 12 cm CP
(a) 33 cm (b) 43 cm = 8 cm, PD
(c) 6 cm (d) 8 cm
11. Find the number of common tangents, if r1
A

+ r2 = C1C2.(With usual notations, r1 & r2 and


C1 & C2 are the radii and centres of the two
circles.)
r1
+ r2 = c1c2 r1 r2
c1 c2
SSC CGL 05/12/2022 (Shift- 01) SSC CGL 06/12/2022 (Shift- 03)
(a) 1 (b) 0 (a) 12 cm (b) 11 cm
(c) 3 (d) 4 (c) 22 cm (d) 14 cm

Aditya Ranjan (Excise Inspector) Selected Selection 199


https://sscstudy.com/
Join Telegram- Maths by Aditya Ranjan CIRCLE

16. The number of parallel tangents of a circle 20. If two circles do not touch or intersect each
with a given tangent is: other and one does not lie inside the other,
then find the number of common tangents.



SSC CGL 06/12/2022 (Shift- 04)

(a) 1 (b) 2
SSC CGL 08/12/2022 (Shift- 02)
(c) 3 (d) 4
(a) 5 (b) 4
17. A circle is inscribed in a ABC having sides
(c) 2 (d) 3
AB = 16 cm, BC = 20 cm and AC = 24 cm,
and side AB, BC and AC touches circle at D, 21. The radius of a circle is 5 cm. Calculate the
E and F, respectively. The measure of AD is: length of a tangent drawn to this circle from
a point at a distance of 10 cm from its centre.
ABC
Ab = 16 cm, BC= 20 cm AC = 24 cm

r
AB,BC AC D, E
F AD

si
SSC CGL 08/12/2022 (Shift- 03)
SSC CGL 07/12/2022 (Shift- 02)
an by (a) 55 cm (b) 52 cm
(a) 10 cm (b) 20 cm
(c) 5 cm (d) 53 cm
(c) 6 cm (d) 14 cm

n
22. There are two circles which touch each other
18. In the given figure, O is the centre of the circle externally. The radius of the first circle with
ja
and AOC = 140º find ABC. centre O is 17 cm and radius of the second
R s
O AOC = 140º circle with centre A is 7 cm. BC is a direct
common tangent to these two circles, where
a th

ABC
B and C are points on the circles with centres
O and A, respectively. The length of BC is:

ty a

O
O A BC
di M

B C
O A BC
A C

SSC CGL 08/12/2022 (Shift- 04)
B
SSC CGL 07/12/2022 (Shift- 04) (a) 2 188 cm (b) 2 119 cm

(a) 95º (b) 110º (c) 2 113 cm (d) 2 117 cm


(c) 120º (d) 103º 23. If PT is a tangent at T to a circle whose centre
19. AB is the diametr of a circle with center O. is O and OP = 17 cm and OT = 8 cm, find
A

If P be point on the circle such that AOP = the length of the tangent segment PT.
110º, then the measure of OBP is: PT T
AB, O P O OP = 17 cm
AOP = 110º OBP OT = 8 cm PT

SSC CGL 08/12/2022 (Shift- 01) SSC CGL 09/12/2022 (Shift- 01)
(a) 50º (b) 65º (a) 13 cm (b) 14 cm
(c) 60º (d) 55º (c) 16 cm (d) 15 cm

Aditya Ranjan (Excise Inspector) Selected Selection 200


https://sscstudy.com/
Join Telegram- Maths by Aditya Ranjan CIRCLE

24. In a circle of radius 3 cm, two chords of length


2 cm and 3 cm lie on the same side of a
diameter. What is the perpendicular distance

between the two chords?
SSC CGL 12/12/2022 (Shift- 03)
(a) 24 cm (b) 14 cm
(c) 16 cm (d) 12 cm
29. The diameters of two circles are 12 cm and
SSC CGL 09/12/2022 (Shift- 02) 20 cm, respectively and the distance between
their centres is 16 cm. Find the number of
4 3–3 2 4 2–3 3 common tangents to the circles.
(a) cm (b) cm
2 2

4 2–3 3 4 2–3 3
(c) cm (d) cm
3 4 SSC CGL 12/12/2022 (Shift- 03)
25. The perpendicular distance between the centre (a) 2 (b) 3

r
of a circle and the longest chord of that circle (c) 1 (d) 4

si
is _______. 30. In the given figure, if PA and PB are tangents
to the circle with centre O such that APB

an by = 54º, then OBA = _____.
______ PA PB O

n
SSC CGL 09/12/2022 (Shift- 03) APB = 54º, OBA =
(a) 1 (b) 0 ___
(c) 2
ja (d) 
R s
A
26. Radius of a circle is 5 cm. Length of chord
a th

AB in this circle is 6 cm. What is the distance


of this chord from the centre of the circle? P
O
AB
ty a


B
di M

SSC CGL 12/12/2022 (Shift- 01)


SSC CGL 12/12/2022 (Shift- 04)
(a) 4 cm (b) 5 cm
(a) 27º (b) 40º
(c) 6 cm (d) 8 cm (c) 30º (d) 35º
27. If C1, C2 be the centres of two circles and r1,r2 31. In the figure, XYZ is a secant and ZT is a
be the respective radii such that the distance tangent to the circle at T. If TZ = 12 cm and
between the centres is equal to the sum of YZ = 8cm, then find the length of XY.
the radii of the two circles, find the number XYZ ZT, T
of common tangents.
TZ = 12cm YZ =
C1, C2 r1, r2 8 cm XY

A

X
xc
m
Y
SSC CGL 12/12/2022 (Shift- 02) 8c
m
(a) 4 (b) 2
Z
(c) 1 (d) 3 T 12 cm
28. If two circles of radii 18 cm and 8 cm touch SSC CGL 12/12/2022 (Shift- 04)
externally, then the length of a direct common (a) 8 cm (b) 9 cm
tangent is: (c) 6 cm (d) 10 cm

Aditya Ranjan (Excise Inspector) Selected Selection 201


https://sscstudy.com/
Join Telegram- Maths by Aditya Ranjan CIRCLE

32. The length of the chord of a circle is 24 cm, (c) The diameter of a circle is the longest
and the perpendicular distance between the chord of a circle.
centre and the chord is 5 cm. The radius of
the circle is: (d) The perpendicular distance from the centre
of a circle increases when the length of a
chord increases.




SSC CGL 13/12/2022 (Shift- 02) 34. In the given figure, ABC = 81º and ACB =
(a) 10 cm (b) 13 cm 9º. What is the value of BDC ?
(c) 12 cm (d) 24 cm ABC = 81º ACB = 9º
33. Select the INCORRECT statement with respect BDC
to the properties of a circle. D

r
A
SSC CGL 13/12/2022 (Shift- 02)

si
(a) Two tangents drawn at the end of the
diameter of a circle are parallel.
an by


81º 9º

n
B C
(b) The radius drawn perpendicular to a chord
ja
bisects the chord.
R s
SSC CGL 13/12/2022 (Shift- 04)
(a) 80º (b) 90º
a th

(c) 70º (d) 60º


ty a
di M

ANSWER KEY
1.(c) 2.(d) 3.(b) 4.(a) 5.(a) 6.(b) 7.(d) 8.(c) 9.(c) 10.(c)

11.(c) 12.(a) 13.(c) 14.(b) 15.(a) 16.(a) 17.(a) 18.(b) 19.(d) 20.(c)

21.(d) 22.(b) 23.(d) 24.(b) 25.(b) 26.(a) 27.(d) 28.(a) 29.(b) 30.(a)

31.(d) 32.(b) 33.(d) 34.(b)


A

Aditya Ranjan (Excise Inspector) Selected Selection 202


https://sscstudy.com/
Join Telegram- Maths by Aditya Ranjan CIRCLE

SOL U T ION S
1. (c) A B OQP = 90° (Radius  Tangent)
 OP² = 10² + 24²
= 100 + 576
= 676
P OP = 26

5. (a) C

PAB = 40° (given)


We know, A B
APB = 90° O

r
 APB + ABP + BAP = 180 (Angle sum

si
property)
D
 PBA = 50°
an by AB = 8 cm.
2. (d)  AO = 4 cm.

n
AC = 8 cm.
0
 In AOC
10ja 10
60° OC² = 8² – 4²
R s
= 64 – 16 = 48
a th

A B OC = 43
 CD = 2.OC = 83
OAB = OBA = 60°
[Angeles opp. to equal sides] 6. (b)
ty a

 AOB is an equilateral triangle


 AB = 10
di M

3. (b) O
A B
12 12
O1
C D
E O11 F
12
58
OC = = 29 cm
2
Side
r= 42
3 O1 C =  21cm.
2
A

12
= =4 3
3 OO1 = 29² – 21²
4. (a) l 001 = 20
OE = 29 cm.
24
10 O11 E = 20 cm.

P  OO11 = 29² – 20²


O
OO11 = 21
 Distance = 1 cm.

Aditya Ranjan (Excise Inspector) Selected Selection 203


https://sscstudy.com/
Join Telegram- Maths by Aditya Ranjan CIRCLE

7. (d) (2)
11. (c) (1)
R

F r1 r2

E c1 c2

(3)
P D Q
PQ = 15 Total 3 tavgents
QR = 11 12. (a)
RP = 13
PQ + QR + RP = PD + PF + FR + RE + EQ + DQ
15 + 11 + 13 = 2PD + 2RE + 2EQ
39 = 2PD + 2 (RE + EQ)

r
 2PD = 39 – 22
 PD = 8.5

si
Q
8. (c) an by There can be infinite circle drawn from 2 fixed
15
points. (with different radii)

n
P
O 17 13. (c)
ja
R s
OQP = 90°
a th

O
A B
 OQ² = 17² – 15² = (17 – 15) (17 + 15)
O 1

OQ = 8 C D
O 11
ty a

9. (c) P Q
E F
di M

O1 O11 = OO11 – 001

A B O1 O11 = OE² – (EO11 )2 – OC² – O1 C²


O

O1 O11 = 625 – 225 – 625 – 400

O1 O11 = 20 – 15
OAP = OBQ = 90°
O1 O11 = 5 cm.
PA||BQ
14. (b) T
10. (c) T 12
A

M
P 8
O 3 A
A B
9
B MT² = MA × MB
We know,
144 = 8 × (8 + AB)
PT² = PA × PB
= 3 × 12 18 = 8 + AB
 PT = 6  AB = 10

Aditya Ranjan (Excise Inspector) Selected Selection 204


https://sscstudy.com/
Join Telegram- Maths by Aditya Ranjan CIRCLE

15. (a) 19. (d) P

110°
A B
O

AP = AB – PB Q

= 20 – 12 = 8 AOP = 110°
AP × PB = CP × PD 110
AQP = = 55
8 × 12 = 8 × PD 2
PD = 12 cm. (Angle formed at circumference is half the

r
16. (a) There will always be only one tangent parallel angle at centre)

si
to a given tangent. Now,
17. (a) an by A AQP = ABP = 55°
(Angle formed by same are.)

n
20. (b)
16 D F 24
ja
R s
a th

B C No. of common tangents = 4


E
20 Q
ty a

AB + BC + AC = AD + AF + FC + EC + BE + DB 21. (d)
di M

= 2AD + 2EC + 2BE 5 P


16 + 20 + 24 = 2AD + 2 (EC + BE) 10
O
60 = 2AD + 40
AD = 10 cm.
OQP = 90°
18. (b)
 QP² = 10² – 5²
QP = 5 3
O B
22. (b) C
A

17 7
A C
O A

1 BC = 2 r1 r2
ABC = 180 – AOC
2
= 180 – 70 = 2 7  17

= 110° = 2 119

Aditya Ranjan (Excise Inspector) Selected Selection 205


https://sscstudy.com/
Join Telegram- Maths by Aditya Ranjan CIRCLE

23. (d) T (1)


27. (d)

8 r1 r2
P c1 c2
O 17

(2) (3)
OTP = 90°
PT² = 17² – 8² c1 c2 = r1 + r2
= 9 × 25 Common tangents = 3
PT = 225 28. (a) B C
= 15

24. (b) 18 8

r
O
A B

si
O1

C an by D BC = 2 r1 r2

E F = 2 18  8

n
=2×3×4
O
11

BC = 24
ja
O1 O11 = r² – O11 E² – r² – O1 C²
R s
29. (b)
9
a th

= 9 –1 – 9 – 6 10
4
3 3
= 2 2–
2
ty a

4 2–3 3 No. of common tangents = 3



di M

2
25. (b) 30. (a) A
Zero
(The biggest chords of the circle is the diameter) O
Now the perpendicular distance between the
biggest chord diameter and the centre of the
circle is 0. 54°
Since the chord passes through the centre. P
B
26. (a) We can directly say that:-

1
OBA = APB
2
A

O
A B
1
=  54 = 27
2
C D 'OR'
O1
AOB = OBA [OA = OB]
O O1 = OC² – O1 C²
180 – 126
= 5² – 3²  OBA = = 27
2
O O1 = 4

Aditya Ranjan (Excise Inspector) Selected Selection 206


https://sscstudy.com/
Join Telegram- Maths by Aditya Ranjan CIRCLE

31. (d) 24
AC = = 12
X 2
x OC = 5
Y  OA² = 12² + 5²

8 = 144 + 25 = 169 = 13
33. (d) Is incorrect
Z
T 12 because, length of chord decreases with
ZT² = (zy) × (ZX) increase in perpendicular distance
12² = 8 × (8 + x) 34. (b) In ABC

144 BAC = 180° – (81 + a)


= 8 x
8 = 90° (Angle sum prop.) and BAC = BDC = 90°
x = 10 [Angles formed by same arc]

r
32. (b)

si
an by O

n
A B
12 C
ja
R s
a th

......-------......
ty a
di M
A

Aditya Ranjan (Excise Inspector) Selected Selection 207


https://sscstudy.com/
Join Telegram- Maths by Aditya Ranjan MENSURATION-2D

MENSURATION-2D
22
2D
1. The circumference of the two circles is 198 6. In the given figure, a square ABCD is inscribed
cm and 352 cm respectively. What is the in a quadrant APCQ. If AB = 16 cm, find the
difference between their radii? area of the shaded region (take  = 3.14)
correct to two places of decimal.
APCQ ABCD

SSC CGL 01/12/2022 (Shift- 01)


AB = 16 cm
( = 3.14 )
(a) 45 cm (b) 16.5 cm
(c) 49.5 cm (d) 24.5 cm

r
2. The area of a triangle is 480 cm² and the ratio
SSC CGL 02/12/2022 (Shift- 04)

si
of its sides is 10 : 24 : 26. What is the
perimeter of the triangle? (a) 155.98 cm² (b) 179.68 cm²
an by



7.
(c) 163.85 cm² (d) 145.92 cm²
If the side of an equilateral triangle is 20 cm,

n
then what is its area?
SSC CGL 01/12/2022 (Shift- 04)
(a) 120 cm ja (b) 30 cm
R s
(c) 150 cm (d) 60 cm SSC CGL 02/12/2022 (Shift- 04)
a th

3. In the side of an equilateral triangle is 24 cm, (a) 1103 cm² (b) 1253 cm²
then what is its area? (c) 2003 cm² (d) 1003 cm²
8. The area of a sector of a circle is 66cm2 and
the angle of the sector is 60º. Find the radius
ty a


of the circle.
SSC CGL 02/12/2022 (Shift- 02)
66 cm²
di M

(a) 1693 cm² (b) 1253 cm² 60º


(c) 2563 cm² (d) 1443 cm²
4. If the ratio of corresponding sides of two SSC CGL 03/12/2022 (Shift- 01)
similar triangles is 3 : 2 then what is the
ratio of the area of the two triangles? (a) 5 15 cm (b) 6 14 cm
(c) 7 19 cm (d) 3 14 cm
3 : 2
9. Two identical circles each of radius 30 cm
intersect each other such that the
SSC CGL 02/12/2022 (Shift- 03) circumference of each one passes through the
(a) 3 : 2 (b) 9 : 4 centre of the other. What is the area of the
A

(c) 3 : 2 (d) 27 : 8 intersecting region?


5. How far will the tip of a 2cm-long minute hand
of a clock move in 45 minutes?
(tip)

SSC CGL 02/12/2022 (Shift- 03) SSC CGL 03/12/2022 (Shift- 04)
(a)  cm (b) 3 cm (a) 400 – 2503 cm² (b) 300 – 1503 cm²
(c) 4 cm (d) 2 cm (c) 500 – 3503 cm² (d) 600 – 4503 cm²

Aditya Ranjan (Excise Inspector) Selected Selection 208


https://sscstudy.com/
Join Telegram- Maths by Aditya Ranjan MENSURATION-2D

10. The area of a sector of a circle is 308cm², 15. A wire is in the form of square with side 11
with the central angle measuring 45º. The cm. It is bent to form a circle. What is the
radius of the circle is: radius of the circle?
308 cm²
45º
SSC CGL 03/12/2022 (Shift- 04)

(a) 14 cm (b) 21 cm
SSC CGL 06/12/2022 (Shift- 01)
(c) 7 cm (d) 28 cm
11. Three circles of radius 6 cm are kept touching (a) 11 cm (b) 5 cm
each other. The string is tightly tied around (c) 9 cm (d) 7 cm
these three circles. What is the length of the
16. Find the area of the sector of a circle with
string?
radius 4 cm and angle 30º.
6 cm


30º

r
SSC CGL 03/12/2022 (Shift- 02) SSC CGL 06/12/2022 (Shift- 01)

si
(a) 36 + 12 cm (b) 36 + 18 cm (a) 7.186 cm² (b) 6.186 cm²
(c) 24 + 36 cm
an by (d) 36 + 20 cm (c) 4.186 cm² (d) 5.186 cm²
12. If the ratio of area of two similar triangles 17. The height of an equilateral triangle is 73
is 3 : 2 then what is the ratio of the

n
cm. What is the area of this equilateral
corresponding sides of the two triangles? triangle?

ja 3 : 2
73
R s


a th

SSC CGL 05/12/2022 (Shift- 02)


SSC CGL 06/12/2022 (Shift- 02)
(a) 9 : 4 (b) 3 : 2
(a) 363 cm² (b) 253 cm²
(c) 3
3:32 (d) 4
3:42
(c) 493 cm² (d) 323 cm²
ty a

13. Three circles of radius 4 cm are kept touching


each other. The string is tightly tied around 18. Calculate the area of a sector of a circle with
di M

these three circles. What is the length of the radius 10 metres and angle of 90 degrees at
string? the centre.



SSC CGL 06/12/2022 (Shift- 03)
SSC CGL 05/12/2022 (Shift- 04) (a) 79.3 m² (b) 79.5 m²
(a) 24 + 8p cm (b) 24p + 16 cm (c) 78.5 m² (d) 75 m²
(c) 32 + 16p cm (d) 24 + 18p cm 19. The ratio of the outer and the inner
14. If the length of certain rectangle is decreased circumference of a circular path is 5 : 4. If
by 4 cm and breadth is increased by 2 cm, it path is 50 metres wide, then what is the radius
A

would result in a square of the same area. What of the inner circle?
is the perimeter of the original rectangle?

4 cm

2 cm


SSC CGL 06/12/2022 (Shift- 03)
SSC CGL 06/12/2022 (Shift- 01)
(a) 15 cm (b) 24 cm (a) 250 metres (b) 300 metres
(c) 20 cm (d) 10 cm (c) 200 metres (d) 210 metres

Aditya Ranjan (Excise Inspector) Selected Selection 209


https://sscstudy.com/
Join Telegram- Maths by Aditya Ranjan MENSURATION-2D

20. The diagonal of the square is 82 cm. Find 26. If the side of an equilateral triangle is 8cm,
the diagonal of another square whose area is then find the area of the triangle (correct to
triple that of the first square. two decimal places).
82 cm 8 cm


SSC CGL 06/12/2022 (Shift- 04) SSC CGL 07/12/2022 (Shift- 04)
(a) 85 cm (b) 83 cm (a) 27.17 cm² (b) 27.27 cm²
(c) 82 cm (d) 86 cm (c) 27.71 cm² (d) 27.72 cm²
21. The height of an equilateral triangle is 93 27. The radius of a circle with centre at O is 6 cm
cm. What is the area of this equilateral and the central angle of a sector is 40º. Find
triangle? the area of the sector.
93 O 6 cm
40º
SSC CGL 07/12/2022 (Shift- 01)

r
(a) 923 cm² (b) 673 cm² SSC CGL 08/12/2022 (Shift- 04)

si
(c) 493 cm² (d) 813 cm² (a) 6 cm² (b) 5 cm²
22. If the circumference of a circle is 220 cm, (c) 4 cm² (d) 8 cm²
an by
then what is its radius? 28. Three circles of radius 7 cm are kept touching
each other. The string is tightly tied around

n

these three circles. What is the length of the
string?
ja SSC CGL 07/12/2022 (Shift- 01)
R s
(a) 26 cm (b) 25 cm
a th

(c) 35 cm (d) 40 cm

23. The perimeter of a square is 124 metres. What
SSC CGL 08/12/2022 (Shift- 01)
is its area?
(a) 42 + 7p cm (b) 21p + 14 cm

ty a

(c) 42 + 14p cm (d) 7 + 14p cm


29. The circumference of the two circles is 264
di M

SSC CGL 07/12/2022 (Shift- 02) cm and 396 cm respectively. What is the
(a) 448 m² (b) 884 m² difference between their radii?
(c) 764 m² (d) 961 m²
24. The circumference of the two circles is 308
cm and 440 cm respectively. What is the SSC CGL 09/12/2022 (Shift- 01)
difference between their radii?
(a) 25 cm (b) 32 cm
(c) 16 cm (d) 21 cm
30. The area of a sector of a circle is 88 sq. cm.,
SSC CGL 07/12/2022 (Shift- 02) and the angle of the sector is 45º. Find the
(a) 21 cm (b) 49 cm  22 
radius of the circle  Use  = 
A

(c) 35 cm (d) 14 cm  7 
25. The length of the altitude of an equilateral
triangle is 63 m. The perimeter of the

equilateral triangle (in m) is: 45º
63 m  22 
  = 
(m ) 7
SSC CGL 07/12/2022 (Shift- 03) SSC CGL 09/12/2022 (Shift- 04)
(a) 122 (b) 362 (a) 311 cm (b) 414 cm
(c) 36 (d) 243 (c) 613 cm (d) 514 cm

Aditya Ranjan (Excise Inspector) Selected Selection 210


https://sscstudy.com/
Join Telegram- Maths by Aditya Ranjan MENSURATION-2D

31. The minute hand of a clock is 20 cm long. 35. The circumference of the two circles is 110
Find the area on the face of the clock swept cm and 330 cm respectively. What is the
by the minute hand between 8 a.m. and 8:45 difference between their radii?
a.m.

20 cm 8
a.m. 8:45 a.m. SSC CGL 13/12/2022 (Shift- 01)

(a) 70 cm (b) 15 cm
SSC CGL 12/12/2022 (Shift- 01) (c) 46 cm (d) 35 cm

6600 6600 36. The hour hand moves through 4 hours and
(a) cm2 (b) cm 2 has a length of 6 cm. Find the area (in cm²,
7 9
rounded off to two decimal places) of the
sector covered by the hour hand.
6600 6600
(c) cm2 (d) cm 2
18 14

r
32. The area of a triangle is 96 cm² and the ratio

si
of its sides is 6 : 8 : 10. What is the perimeter
of the triangle?
an by

SSC CGL 13/12/2022 (Shift- 01)

n
(a) 32.69 (b) 30.67

SSC CGL 12/12/2022 (Shift- 02) (c) 37.71 (d) 35.75


ja
R s
(a) 48 cm (b) 56 cm 37. A triangle and a parallelogram have the same
base 28 cm and the same area. If the height
a th

(c) 64 cm (d) 44 cm of the parallelogram is 12 cm, then find the


33. A coconut tree swings with the wind in such length of the altitude of the triangle.
a manner that the angle covered by its trunk
ty a

is 18 degrees. If the topmost portion of the



tree covers a distance of 44 metres, find the

di M

length of the tree.




SSC CGL 13/12/2022 (Shift- 02)

(a) 28 cm (b) 23 cm
44 metres
(c) 24 cm (d) 21 cm

SSC CGL 12/12/2022 (Shift- 02) 38. A horse is grazing in a field. It is tied to a
pole with a rope of length 6 m. The horse
(a) 120 metres (b) 210 metres moves from point A to point B making an arch
(c) 140 metres (d) 70 metres with an angle of 70°. Find the area of the
sector grazed by the horse.
A

34. If the side of an equilateral triangle is 16 cm,


then what is its area?
A B
70º

SSC CGL 12/12/2022 (Shift- 03) SSC CGL 13/12/2022 (Shift- 02)
(a) 813 cm² (b) 483 cm² (a) 22 m (b) 21 m
(c) 323 cm (d) 643 cm² (c) 23 m (d) 20 m

Aditya Ranjan (Excise Inspector) Selected Selection 211


https://sscstudy.com/
Join Telegram- Maths by Aditya Ranjan MENSURATION-2D

39. A rectangular park is 120 m long and 104 m 41. The difference between the lengths of two
wide. A 1-m wide path runs along the boundary parallel sides of a trapezium is 12 cm. The
of the park, remaining completely inside the perpendicular distance between these two
park area. Thus, the outside edges of the path parallel sides is 60 cm. If the area of the
run along the boundary wall of the park. The trapezium is 1380 cm², then find the length
inside edges of the path are marked with a (in cm) of each parallel side.
white line of negligible thickness. If it costs
Rs. 2.50 to mark each metre with the white 12 cm.
line, then how much would it cost (in Rs.) to 60 cm. 1380
fully mark the inside edges of the path? cm² (cm. )
m m
m SSC CGL TIER- II 06/03/2023
(a) 31, 19
(b) 24, 12
(c) 27, 15

r
(d) 29, 17

si
Rs. 2.50 42. In a trapezium ABCD, AB and DC are parallel
to each other with a perpendicular distance of

an by
Rs.
SSC CGL TIER - II 02/03/2023
8 m between them. Also, (AD) = (BC) = 10 m,
and (AB) = 15 m < (DC). What is the perimeter
(in m) of the trapezium ABCD?

n
(a) 1090 (b) 1080
ABCDAB DC
(c) 1120 ja (d) 1100
8 m
R s
40. What is the perimeter of the square inscribed
in a circle of radius 5 cm?
(AD) = (BC) = 10 m, (AB) = 15 m < (DC)
a th

ABCD (m )
5 cm
SSC CGL TIER- II 07/03/2023

SSC CGL TIER- II 06/03/2023 (a) 50


ty a

(a) 20 2 cm (b) 10 2 cm (b) 66


di M

(c) 62
(c) 30 2 cm (d) 40 2 cm (d) 58

ANSWER KEY
1.(d) 2.(a) 3.(d) 4.(a) 5.(b) 6.(d) 7.(d) 8.(d) 9.(d) 10.(d)

11.(a) 12.(d) 13.(a) 14.(c) 15.(d) 16.(c) 17.(c) 18.(c) 19.(c) 20.(d)
A

21.(d) 22.(c) 23.(d) 24.(a) 25.(c) 26.(c) 27.(c) 28.(c) 29.(d) 30.(b)

31.(a) 32.(a) 33.(c) 34.(d) 35.(d) 36.(c) 37.(c) 38.(a) 39.(d) 40.(a)

41.(d) 42.(c)

Aditya Ranjan (Excise Inspector) Selected Selection 212


https://sscstudy.com/
Join Telegram- Maths by Aditya Ranjan MENSURATION-2D

SOL U TION S
1. (d) Given,
2 (r1 – r2) = 352 – 198 6. (d) Q

2 (r1 – r2) = 154


D C
154  7
=
2  22

49
=
2

r
= 24.5 A P
16 B
2 (a) From ratio of sides it is clear that the triangle

si
is a right angle triangle having triplets in the
ratio (10 : 24 : 26)
an by Radius of quadrant = diagonal of square
1

n
 Area = × 10x × 24x  r = 16 2
2
480 = 120x² Area of shaded region = (Area of quadrant –
ja
R s
Area of square)
 x=2
a th

 Perimeter = (10 + 24 + 26) × 2 1


 (16 2)² – (16)²
= 120 cm. 4

3 1 22 
ty a

3. (d) = (side)²  (16)²    2 – 1


4 4 7
 
di M

3 4 
=  24  24
4 = 256   = 145.92 cm²
7 

= 144 3 cm²
3
7. (d) Area = (side)²
 3 4
2
Area 1 3
4. (a) = =
Area 2
 2 2
2

3
=  20  20
4
5. (b) Angle formed in 45 minutes
A

45 = 100 3 cm²
  360 = 270
60

8. (d) r² = 66
Q 360
 Area = r²
360
1 22
270 22    r²  66
=  22 6 7
360 7

= 3  r = 3 14 cm

Aditya Ranjan (Excise Inspector) Selected Selection 213


https://sscstudy.com/
Join Telegram- Maths by Aditya Ranjan MENSURATION-2D

A 14. (c) Let, L = length


9. (d)
B = breadth of rectangle

c ATQ,
D r O
1

(L – 4) = (B + 2)
L–B=6 .........(1)
B Also,
OA = O1 A = O O1
LB = (L – 4) (B + 2)
 AOD1 is an equilateral 
LB = LB – 4B + 2L – 8
AOO1 = 60º
After solving (i) and (ii)
AOB = 120º
4B – 2L = – 8 ........(2)
Now,
L + B = 10
AC = OA² – OC² = 30² – 15² = 900 – 225
 2 (L + B) = 2(10) = 20 cm.

r
AC = 675 = 15 3 15. (d) ATQ,

si
AB = 30 3 4 × 11 = 22r
an by
ar. AO BA = ar. secta OAO B – ar of AOB
1 1

4  11 = 2 
22
r

n
120 1 7
= 30² –  30 3  15
360 2
r = 7 cm.
ja
R s
= (300 – 225 3 ) cm² Q
16. (c) Area = r²
Now, 360
a th

ar. of shaded region = 2 × ar. AO1 BA 30 22


=  44
= (600 – 450 3 ) cm² 360 7
ty a

10. (d) Given, 88


= = 4.186 cm²
21
di M

45
r² = 308
360
3
17. (c) height =  side
308  8 2
r² = 7
22
r = 28 cm 7 3 2
 side =
3
11. (a) length of string = 2r + n (2r)
= (12 + 36) cm = 14 cm.

 1 2 3
 Area =  14  14
area 1 ( 3)  (3) 4  4
A

12 (d) = = 1 
are 2 ( 2) 
 (2) 4 

  = 49 3 cm²
1
Side of 1  3 4 Q
 =
   18. (c) Area = r²
Side of 2  2  360

90 22
13. (a) Lenth of string = 2hr + 2nr. =   10  10
360 7

= (8 + 24) cm = 78.5 m²

Aditya Ranjan (Excise Inspector) Selected Selection 214


https://sscstudy.com/
Join Telegram- Maths by Aditya Ranjan MENSURATION-2D

19. (c) ATQ,


height
R – r = 50 25. (c) Perimeter = 3  2
3

6 3
= 3 2
3

R = 36 m.

r 3
26. (c) Area =  8  8 = 16 3
4
= 16 × 1.732
= 27.71 cm²

2R 5 40
= 27. (c) Area = (6)²
2r 4 360
= 4 cm²

r
R 5
 = 28. (c) length of string = 2r + 2nr
r 4

si
 (14 + 42) cm.
1  50
an by 29. (d) 2(R – r) = 396 – 264
4  200 mtr.
132  7
Diagonal 8 2 (R – r) =

n
20. (d) Side = = =8 22  2
2 2
ja = 21 cm.
ATQ,
R s
3 × 8 × 8 = a² 45
30. (b) r² = 88
a th

a=8 3 360
d = 2 a
88  8  7
r² =
 Diagonal = 8 6 cm. 22
ty a

3 r = 4 14 cm.
21. (d) Side  = height
di M

2 31. (a)
Angle formed in 45 minutes:-
9 3 2
 Side = = 18 cm.
3 45
 360 = 270
60
3
area =  18  18 = 81 3 cm² 270 22
4  Area =  20  20 
360 7
22. (c) 2r = 220
r = 35 6600
= cm²
Perimeter  2 7
A

23. (d) Area =   32. (a) Given ratio of sides, form a triplet of right

 4 
angle triangle.
2
124 
=
  = 961 cm²
 1
 4   × 6x × 8x = 96
2
24. (a) 2 (R – r) = 440 – 308  x² = 4
132  7 x=2
=
22  2 Perimeter = (6 + 8 + 10) × 2
(R – r) = 21 cm. = 48 cm.

Aditya Ranjan (Excise Inspector) Selected Selection 215


https://sscstudy.com/
Join Telegram- Maths by Aditya Ranjan MENSURATION-2D

Perimeter of inner boundary


2 r
33. (c) length of arc  of path = 2 (118 + 102)
360 = 440 m.
Total cost = 440 × 2.50 = 1100
2r  18
 = 44 40. (a)
360

44  360  7
 r= = 140 mtr.
2  22  18
5
3 3
34. (d) Area  (side)² =  16  16 = 64 3 cm²
4 4
35. (d) 2 (r1 – r2) = 330 – 110 = 220
(r1 – r2) = 35 cm.
d = 5 + 5 = 10
4
36. (c) Angle in 4 hrs. = × 360 = 120. d
12 8= =5 2

r
2
Area of sector = 120  22  6  6 = 37.71 cm²  Perimeter = 20 2

si
360 7
41. (d)
37. (c)

D
an by E

C
D C

n
60
12 ja
R s
A B
E
a th

Given,
A B
28 AB – CD = 12, DE = 60 .........(1)
Area of parallelogram = ar. of triangle 1
1 Area = × DE (DC + AB)
2
ty a

 base × height1 = × base × height2


2
1
1 1380 = × 60 (AB + CD)
di M

28 × 12 = × 28 × h2 2
2
46 = AB + CD ..........(2)
 h2 = 24 cm.
38. (a) From (1) and (2)
A
AB = 29
CD = 17
6 42. (c)
A 15 B

70°
10
B
6 8 8
70 22
A

Area of sector =   6  6 = 22 m²
360 7
D P 15 Q 6 C
39. (d) BQ = AP = 8
AD = BD = 10
 By pythagorean triplet:-
104
QC = DP = 6
 DC = 6 + 6 + 15 = 27

120
 Perimeter of ABCD = 15 + 10 + 27 + 10 = 62

Aditya Ranjan (Excise Inspector) Selected Selection 216


https://sscstudy.com/
Join Telegram- Maths by Aditya Ranjan MENSURATION-3D

MENSURATION-3D
23
3D
1. How many spherical lead shots each of
diameter 8.4 cm can be obtained from a
rectangular solid of lead with dimension 88

 22  SSC CGL 01/12/2022 (Shift- 03)
cm, 63 cm, 42 cm  take  = ?
 7 
(a) 246 cm (b) 152 cm
88 cm, 63 cm, 42 cm (c) 174 cm (d) 208 cm
8.4 cm 5. What is the edge of a cube whose volume is

r
equal to the sum of the volumes of the cubes
 22 
of edge 6 cm, 8 cm and 10 cm?

si
  = 
7 
an by
SSC CGL 01/12/2022 (Shift- 01)

n
(a) 920 (b) 750
(c) 650 (d) 860 SSC CGL 01/12/2022 (Shift- 04)
ja (a) 5 cm (b) 3 cm
R s
2. A hemisphere of lead of radius 4 cm is cast
into a right circular cone of height 72 cm. (c) 6 cm (d) 12 cm
a th

What is the radius of the base of the cone? 6. Which of the following two will provide more
4 cm valume?


ty a

I. A cuboid of edges 6 cm, 7 cm and 8 cm


di M

SSC CGL 01/12/2022 (Shift- 02)


(a) 1.63 cm (b) 1.35 cm
II. A cube of edge 7 cm
(c) 1.33 cm (d) 1.45 cm
3. A copper sphere of diameter 18 cm is drawn

into a wire of diameter 6 mm. Find the length SSC CGL 02/12/2022 (Shift- 01)
of the wire. (a) Cube will have more volume
18 cm 6 mm (b) Both will have equal volume
(c) Cuboid will have more volume
(d) Cannot be determined
A

SSC CGL 01/12/2022 (Shift- 03) 7. If the diameter of a sphere is 63 cm, then
what is the total surface area of the sphere?
(a) 143 m (b) 108 m

(c) 324 m (d) 234 m

4. A solid metallic sphere of radius 13 cm is
SSC CGL 02/12/2022 (Shift- 01)
melted and recast into a cone having diameter
of the base as 13 cm. What is the height of (a) 11824 cm² (b) 12836 cm²
the cone? (c) 12474 cm² (d) 11248 cm²

Aditya Ranjan (Excise Inspector) Selected Selection 217


https://sscstudy.com/
Join Telegram- Maths by Aditya Ranjan MENSURATION-3D

8. A hemispherical bowl made of iron has inner 36 m 18 m 9 m


diameter 84 cm. Find the cost of tin plating
it on the inside at the rate of Rs.21 per 100

 22 
cm²  take  =  correct to two places of SSC CGL 02/12/2022 (Shift- 01)
 7 
(a) 183 m (b) 153 m
decimal.
(c) 123 m (d) 173 m

13. If the volume of a sphere is 24,416.64 cm³,
 22  find its surface area (take  =3.14) correct to

  = 
7  two places of decimal.

24,416.64 cm³

SSC CGL 02/12/2022 (Shift- 01)
( = 3.14 )

r
(a) Rs.2,328.48 (b) Rs.2,425.48 SSC CGL 03/12/2022 (Shift- 01)
(a) 3069.55 cm² (b) 4069.44 cm²

si
(c) Rs.2,425.60 (d) Rs.2,355.48
9. If the diameter of a hemisphere is 28 cm, then (c) 5069.66 cm² (d) 6069.67 cm²
an by
what is the volume of hemisphere? 14. If the diameter of a sphere is 3.5 cm, then
what is the total surface area of the sphere?

n


ja
R s
SSC CGL 02/12/2022 (Shift- 02)
(a) 5749.33 cm³ (b) 6349.22 cm³ SSC CGL 03/12/2022 (Shift- 01)
a th

(c) 6728.11 cm³ (d) 5124.44 cm³ (a) 45.75 cm² (b) 42.6 cm²

10. The surface area of a cube is 864 cm2. The (c) 38.5 cm² (d) 34.25 cm²
ty a

volume of the cube is _______. 15. The radii of two cylinders are in the ratio 1
: 4 and their heights are in the ratio 4 : 3.
864 cm²
di M

Their volumes will be in the ratio ______.




SSC CGL 02/12/2022 (Shift- 02)

(a) 1728 cm³ (b) 2197 cm³

(c) 1331 cm³ (d) 729 cm³
SSC CGL 03/12/2022 (Shift- 02)
11. The radius of a right circular cylinder is five
(a) 1 : 12 (b) 2 : 9
times of its height. If the height of the cylinder
is 3.5 cm, then what is the volume of cylinder? (c) 3 : 10 (d) 1 : 9

16. The radius of a right circular cylinder is thrice


of its height. If the height of the cylinder is
A

2.1 cm, then what is the volume of cylinder?



SSC CGL 02/12/2022 (Shift- 03)


(a) 3368.75 cm³ (b) 3872.75 cm³

(c) 3146.75 cm³ (d) 3524.25 cm³
SSC CGL 03/12/2022 (Shift- 03)
12. A cuboid of length 36 m, breadth 18 m and
height 9 m is melted and recast into a cube. (a) 224.65 cm³ (b) 194.72 cm³
Find the length of the diagonal of the cube. (c) 324.86 cm³ (d) 261.95 cm³

Aditya Ranjan (Excise Inspector) Selected Selection 218


https://sscstudy.com/
Join Telegram- Maths by Aditya Ranjan MENSURATION-3D

17. A solid copper sphere of radius 9 cm is


hammered and moulded into a wire of radius
2 cm. What is the length of this wire?
SSC CGL 05/12/2022 (Shift- 02)
9 cm
(a) 26.40 cm³ (b) 20.50 cm³
2 cm
(c) 38.80 cm³ (d) 48.60 cm³

23. A hollow sphere has an outer radius of 4 cm
SSC CGL 03/12/2022 (Shift- 03) and inner radius of 1 cm. What is the volume
(a) 224 cm (b) 183 cm of this hollow sphere?
(c) 198 cm (d) 243 cm
18. If the length of the diagonal of a cube is 73
cm, then the surface area of the cube is. SSC CGL 05/12/2022 (Shift- 03)
73 cm (a) 84p cm³ (b) 68p cm³
(c) 48p cm³ (d) 92p cm³

r
SSC CGL 03/12/2022 (Shift- 03) 24. The volume of the hemisphere is 19,404 cm³.
(a) 216 cm² (b) 256 cm² The radius of the hemisphere is _______.

si
(c) 284 cm² (d) 294 cm² 19,404 cm³
19.
an by
The length of the side of a cube is 5.6 cm.
What is the volume of the largest sphere that

SSC CGL 05/12/2022 (Shift- 03)

n
can be taken out of the cube? (a) 14 cm (b) 19 cm
(c) 21 cm (d) 17 cm
ja
R s
25. Three cubes of equal volume are joined end
to end. Find the surface area of the resulting
a th

SSC CGL 05/12/2022 (Shift- 01) cuboid if the diagonal of the cube is 63 cm.

(a) 91.98 cm³ (b) 99.96 cm³


(c) 96.98 cm³ (d) 90.69 cm³ 63 cm
ty a

20. The curved surface area of the sphere is 154


cm². Find the volume of the sphere (rounded
di M

SSC CGL 05/12/2022 (Shift- 04)


off to one digit after decimal). (a) 509 cm² (b) 504 cm²
154 cm² (c) 516 cm² (d) 512 cm²
26. A solid hemisphere has radius 21 cm. It is
melted to form a cylinder such that the ratio
SSC CGL 05/12/2022 (Shift- 01) of its curved surface area to total surface area
is 2 : 5. What is the radius (in cm) of its base
(a) 156.9 cm³ (b) 179.7 cm³
(c) 161.1 cm³ (d) 147.8 cm³  22 
 take  = ?
21. The total surface area of a cube is 1536 m². 7 
Find its volume.
21 cm
A

1536 m²

SSC CGL 05/12/2022 (Shift- 02)
 22
(a) 216 m³ (b) 125 m³ (cm )   = 
 7 
(c) 4096 m³ (d) 729 m³
22. The length of the side of a cube is 4.2 cm. SSC CGL 06/12/2022 (Shift- 01)
What is the volume of the largest sphere that (a) 23 (b) 21
can be taken out of the cube? (c) 17 (d) 19

Aditya Ranjan (Excise Inspector) Selected Selection 219


https://sscstudy.com/
Join Telegram- Maths by Aditya Ranjan MENSURATION-3D

27. The external diameter of an iron pipe is 20


cm and its length is 12 cm. If the thickness 0.37 cm
of the pipe is 1 cm, find the surface area of
6 cm
 22 
the pipe  take  =  correct to two places  22
 7    =  .
7 
of decimal.
SSC CGL 07/12/2022 (Shift- 01)
(a) 532 cm² (b) 255.0548 cm²
(c) 484 cm² (d) 523.4107 cm²

22  32. A copper sphere of diameter 18 metre is drawn



  =  into a cylindrical wire of length 12 metre.
7 
What is the radius of the wire?
SSC CGL 06/12/2022 (Shift- 02)

(a) 1,662.67 cm² (b) 1,552.57 cm²

r

(c) 1,442.48 cm² (d) 1,772.76 cm²

si
2 SSC CGL 07/12/2022 (Shift- 03)
28. If the volume of a sphere is 179
an by m³, what
3
(a) 5 metre (b) 9 metre
is its surface area?

n
(c) 3 metre (d) 12 metre
2
179 m³ 33. What is the total surface area of a cone, if
ja 3
the slant height and radius of a cone is 30
R s
cm and 14 cm, respectively?
a th

SSC CGL 06/12/2022 (Shift- 03) 30


(a) 221 m² (b) 154 m² cm 14 cm
(c) 144 m² (d) 151 m²
ty a

29. The radius of a hemisphere is 6.3 cm. What


SSC CGL 07/12/2022 (Shift- 03)
will be its volume?
di M

(a) 1980 cm³ (b) 1963 cm³


6.3
(c) 1908 cm³ (d) 1936 cm³
SSC CGL 06/12/2022 (Shift- 04) 34. The volume of the sphere is 38,808 cm³. What
is the surface area of the sphere?
(a) 572.80 cm³ (b) 643.50 cm³
(c) 523.90 cm³ (d) 353.38 cm³ 38,808 cm³
30. If the diameter of a hemisphere is 35 cm, then
what is the volume of hemisphere? SSC CGL 07/12/2022 (Shift- 04)
(a) 4455. cm² (b) 4433 cm²

(c) 5544 cm² (d) 3344 cm²
A

SSC CGL 07/12/2022 (Shift- 01)


35. The radius of a hemisphere is 5.6 cm. What
(a) 11229.17 cm³ (b) 15248.46 cm³ will be its volume?
(c) 17428.33 cm³ (d) 9478.26 cm³
5.6 cm
31. Steel is used to make a hemispherical bowl
that is 0.37 cm thick. The bow's inner radius
is 6 cm. Find the bowl's outside curved surface SSC CGL 07/12/2022 (Shift- 04)
 22  (a) 367.95 cm³ (b) 195.95 cm²
 Take  = .
7  (c) 380 cm³ (d) 265.65 cm³

Aditya Ranjan (Excise Inspector) Selected Selection 220


https://sscstudy.com/
Join Telegram- Maths by Aditya Ranjan MENSURATION-3D

36. What is the area of the curved surface of a 41. A solid cone of height 42 cm with diameter
right circular cone with height 8 m and slant of its base 42 cm is cut out from a wooden
height 10 m? solid sphere of radius 24 cm. Find the
8 m 10 m percentage of wood wasted correct to two
places of decimal.

SSC CGL 08/12/2022 (Shift- 01) 42 cm
(a) 60 m² (b) 60 m² 42 cm 24 cm
(c) 40 m² (d) 40 m²
37. An iron rod with diameter 4 cm and length
12 cm is drawn into a wire of length 12 m of SSC CGL 09/12/2022 (Shift- 01)
uniform thickness. Determine the thickness
of the wire. (a) 75.56% (b) 56.65%

(c) 66.50% (d) 67.50%


42. The length of the side of a cube is 2.8 cm.

r
What is the volume of the largest sphere that

si
SSC CGL 08/12/2022 (Shift- 02) can be taken out of the cube?

(a) 0.6 cm
an by (b) 0.7 cm
(c) 0.3 cm (d) 0.4 cm

n
38. Find the total surface area of cube whose
SSC CGL 09/12/2022 (Shift- 02)
volume is 343 m³.
ja (a) 11.50 cm³ (b) 1.15 cm³
R s

343 m³ (c) 11.55 cm³ (d) 115 cm³
a th

SSC CGL 08/12/2022 (Shift- 03) 43. A conical vessel (solid) is made of iron. Its
(a) 186 m² (b) 294 m² base radius is 7 cm and height is 15 cm. If
the weight of the iron per cubic centimetre
(c) 196 m² (d) 210 m²
ty a

is 15 g, what is the weight of the vessel?


39. A hollow sphere has an outer radius of 6 cm
and inner radius of 3 cm. What is the volume
di M

of this hollow sphere? 7 cm 15 cm


15 g

SSC CGL 08/12/2022 (Shift- 03) SSC CGL 09/12/2022 (Shift- 02)
(a) 252p cm³ (b) 356p cm³
(a) 13.55 kg (b) 12.55 kg
(c) 144p cm³ (d) 175p cm³
40. A cuboid of sides 12 cm, 18 cm and 27 cm (c) 14.55 kg (d) 11.55 kg
is melted to form a cube. What is the ratio 44. The curved surface area of a right circular cone
of the total surface area of the cuboid to that of base radius 21 cm is 594 sq. cm. What is
A

of the cube? the slant height of the cone?


12 cm, 18 cm 27 cm

cm


SSC CGL 09/12/2022 (Shift- 03)
SSC CGL 08/12/2022 (Shift- 04)
(a) 18 : 17 (b) 17 : 19 (a) 15 cm (b) 11 cm

(c) 19 : 18 (d) 17 : 23 (c) 9 cm (d) 6 cm

Aditya Ranjan (Excise Inspector) Selected Selection 221


https://sscstudy.com/
Join Telegram- Maths by Aditya Ranjan MENSURATION-3D

45. The number of solid spheres, each of diameter 7 cm


6 cm, that could be moulded to form a solid
5 cm 3 cm
metal cylinder of height 90cm and diameter
4 cm is:
6 cm  22 
90 m 4 cm   =

7

SSC CGL 12/12/2022 (Shift- 02)
SSC CGL 09/12/2022 (Shift- 04)
(a) 10 (b) 14 (a) 345.6 cm³ (b) 359.3 cm³
(c) 16 (d) 13 (c) 379.3 cm³ (d) 369.3 cm³
46. The length of the side of a cube is 1.4 cm.
50. Area of the floor of a cubical room is 64 m².
What is the volume of the largest sphere that
The length of the longest rod that can be kept
can be taken out of the cube?
in the room is:
1.4 cm

r
64 m²

si

an by SSC CGL 09/12/2022 (Shift- 04) SSC CGL 12/12/2022 (Shift- 03)
(a) 1.44 cm³ (b) 3.26 cm³ (a) 163 m (b) 43 m

n
(c) 2.66 cm³ (d) 5.64 cm³
(c) 123 m (d) 83 m
47. How many metres of 2-m-wide cloth will be
ja
required to make a conical tent with the 51. A conical vessel, whose internal radius is 20
R s
diameter of the base as 14 m and slant height cm and height is 27 cm, is full of water. If
a th

as 9 m ignore wastage? this water is poured into a cylindrical vessel


with internal radius 15cm, what will be the
14 m 9 m
height to which the water rises in it?
2m
ty a

20 cm
27 cm
di M

SSC CGL 12/12/2022 (Shift- 01) 15 cm


(a) 66 m (b) 88 m
(c) 99 m (d) 77 m SSC CGL 12/12/2022 (Shift- 04)
48. The radius of a right circular cylinder is four
(a) 16 cm (b) 10 cm
times of its height. If the height of the cylinder
is 14 cm, then what is the volume of cylinder? (c) 12 cm (d) 14 cm
52. A solid metallic sphere of radius 12 cm is
melted and recast into a cone having diameter
of the base as 12 cm. What is the height of
the cone?
A

SSC CGL 12/12/2022 (Shift- 02)


(a) 15468 cm³ (b) 14262 cm³ 12 cm
(c) 137984 cm³ (d) 11296 cm³
49. The radii of the ends of a frustum of a cone 12 cm
7 cm height are 5 cm and 3 cm. Find its
volume correct to one decimal place. SSC CGL 12/12/2022 (Shift- 04)

22  (a) 258 cm (b) 192 cm



 Use  = 
7  (c) 166 cm (d) 224 cm

Aditya Ranjan (Excise Inspector) Selected Selection 222


https://sscstudy.com/
Join Telegram- Maths by Aditya Ranjan MENSURATION-3D

53. If the numerical value of twice the curved cm


surface area of a right circular cylinder is equal
to the numerical value of its volume, then  22 
 = 
what is the numerical value of the radius of  7 
the base of the cylinder? SSC CGL TIER - II 02/03/2023
(a) 3851cm³ (b) 6858cm³

(c) 4851cm³ (d) 5821cm³

57. A hemispherical bowl of internal radius 18 cm
is full of liquid. This liquid is to be filled in
cylindrical bottles each of radius 3 cm and
height 6 cm. How many bottles are required
SSC CGL 13/12/2022 (Shift- 01)
to empty the bowl?
(a) 2 (b) 3
18 cm
(c) 5 (d) 4 3 cm 6 cm
54. If the diameter of a solid hemisphere is 12.6

r

 22 

si
cm, then its volume is  take  = :
 7  SSC CGL TIER- II 03/03/2023
an by
12.6 cm (a) 72 (b) 70

n
 22  (c) 68 (d) 66
  =  :
7 58. What is the total surface area of a pyramid
ja whose base is a square with side 8 cm and
R s
SSC CGL 13/12/2022 (Shift- 3)
height of the pyramid is 3 cm?
a th

(a) 1,259 cm³


(b) 987 cm³ 8 cm 3
cm
(c) 523.908 cm³
ty a

SSC CGL TIER- II 03/03/2023


(d) 1,235.621 cm³
di M

(a) 169 cm² (b) 121 cm²


55. The area of two similar triangles are 324 cm²
(c) 144 cm² (d) 184 cm²
and 289 cm², respectively. What is the ratio
59. The breadth ‘b’ of a room is twice its height
of their corresponding altitudes?
and half of its length. Find the length of the
324 cm² longest diagonal of the room.

289 cm² 'b'




SSC CGL 13/12/2022 (Shift- 3)
SSC CGL TIER- II 03/03/2023
17 17
(a) (b) 20b
A

18 19 (a)
2

19 18 b
(c) (d) (b)
2
17 17
21b
56. What is the volume of the largest sphere that (c)
can be carved out of a wooden cube of sides 2

 22  19b
21 cm?   =  (d)
 7  2

Aditya Ranjan (Excise Inspector) Selected Selection 223


https://sscstudy.com/
Join Telegram- Maths by Aditya Ranjan MENSURATION-3D

60. If the curved surface area of a cylinder is 126 62. A prism and a pyramid have the same base and
cm² and its height is 14 cm. then, what is the the same height. Find the ratio of the volumes
volume of the cylinder? of the prism and the pyramid.

126 cm²
14 cm
SSC CGL TIER- II 06/03/2023
1 1 SSC CGL TIER- II 07/03/2023
(a) 283 cm³ (b) 128 cm³
2 2
(a) 2 : 3 (b) 3 : 1
1 1 (c) 1 : 3 (d) 3 : 2
(c) 137 cm³ (d) 125 cm³
2 2
63. What is the volume of a cylinder if the radius
61. The base of a pyramid is an equilateral triangle
whose each side is 8 cm. Is. Its (slant edge) is of the cylinder is 10 cm and height is 20 cm?
24 cm. What is the total surface area (in cm²) (Take  = 3.14)
of the pyramid?

r
10 cm

20 cm ( = 3.14 )

si
8 cm. (Slant
edge) 24 cm.
an by (cm² SSC CGL TIER- II 07/03/2023
)
(a) 6280 cm³

n
SSC CGL TIER- II 06/03/2023
(b) 5306 cm³
(a) 24 3  36 35
ja  (b) 16 3  48 35  (c) 6260 cm³
R s
(c) 24 3  24 35  
(d) 12 3  24 35  (d) 5280 cm³
a th

ANSWER KEY
ty a
di M

1.(b) 2.(c) 3.(b) 4.(d) 5.(d) 6.(a) 7.(c) 8.(a) 9.(a) 10.(a)

11.(a) 12.(a) 13.(b) 14.(c) 15.(a) 16.(d) 17.(d) 18.(d) 19.(a) 20.(b)

21.(c) 22.(c) 23.(a) 24.(c) 25.(b) 26.(b) 27.(b) 28.(b) 29.(c) 30.(a)

31.(b) 32.(b) 33.(d) 34.(c) 35.(a) 36.(b) 37.(d) 38.(b) 39.(a) 40.(c)

41.(c) 42.(a) 43.(d) 44.(c) 45.(a) 46.(a) 47.(c) 48.(c) 49.(b) 50.(d)
A

51.(a) 52.(b) 53.(d) 54.(c) 55.(d) 56.(c) 57.(a) 58.(c) 59.(c) 60.(a)

61.(b) 62.(b) 63.(a)

Aditya Ranjan (Excise Inspector) Selected Selection 224


https://sscstudy.com/
Join Telegram- Maths by Aditya Ranjan MENSURATION-3D

SOL U TION S
1. (b) Volume of cuboid = volume of spheres
8. (a) T.S.A = 4r² = 2  22  42  42
3 7
4 22  8.4 
 88 × 63 × 42 = x ×  
 
 = 11088 cm²
3 7  2 
Rate = 21 per 100 cm²
88  63  42  3  7  1000 T. Price = 110.88 × 21
 x=
4  22  4.2  4.2  4.2 = Rs. 2328.48
 x = 750 2 22
2. (c) ATQ, 9. (a) Volume =   14  14  14
3 7

r
2 22 1 22 = 5749.33 cm²
  4³ =   r²  72
10. (a) 6a² = 864

si
3 7 3 7

an by
2×4×4×4 864
 = r² a= = 144 = 12
72 6

n
 Volume = a³ = 1728 cm³
4
= =r 11. (a) r = 5h
3 ja
R s
= 5 × 3.5
 r = 1.33 cm.
22
a th

v= × 5 × 5 × 3.5 × 3.5 × 3.5


4 7
3. (b) (9)³ =  (3)² × length
3 = 22 × 125 × 1225 × 10–3
= 3368.75 cm³
ty a

4 9 9 9
 l=  12. (a) ATQ,
3 33
di M

18 × 36 × 9 = a³
= 108 m
a = 9 × 2 = 18 m.
2
4 1 13  Diagonal = 18 3 m.
4. (d) ×  × (13)³ =     h
3 3 2 
4
4 × 13 × 4 = h 13. (b) × 3.14 × r³ = 24416.64
3
 h = 208 cm.
5. (d) a³ = 6³ + 8³ + 10³ 7776  3
r³ =
4
a³ = 1728
 a = 12 r³ = 1944 × 3 = 216 × 27
A

6. (a) I. Culoid volume = 6 × 7 × 8 r = 18

= 336 Surface area = 4r²

II. Cube volume = 7 × 7 × 7 = 4 × 3.14 × 18 × 18


= 343 = 4069.44 cm²

22 3.5 3.5
7. (c) T.S.A = 4r² = 4  22  63  63 14. (c) T.S.A = 4   
7 2 2 7 2 2

= 12474 cm² = 38.5 cm²

Aditya Ranjan (Excise Inspector) Selected Selection 225


https://sscstudy.com/
Join Telegram- Maths by Aditya Ranjan MENSURATION-3D

21. (c) 6a² = 1536


r1 1 h1 4
15. (a) = ; =
r2 4 h2 3 a = 256 = 16
 a³ = 16³
v1 r12 h1 1 4 = 4096
= =
v 2 r2 h 2 16  3
2
22. (c) a = 4.2
r = 2.1
1
=
12 4 22
Volume =   2.1  2.1  2.1
16. (d) r = 3h = 3 × 2.1 3 7
22 88  441
v = r² h = × 3 × 2.1 × 2.1 × 2.1 × 3 =
7 1000
66  3  21  21  3 = 38.808 cm³
=
1000 4 22
23. (a) V =  (4³ – 1³)
198  441  3 3 7
=
1000

r
4 22
=   63
= 261.95 cm³ 3 7

si
17. (d) ATQ,
= 264 or 84p cm³
4 22 an by 22
  9 9 9 =  (2)²  l 2 22
3 7 7 24. (c)   r³ = 19404
3 7

n
 l = 81 × 3 = 243 cm
r³ = 441 × 21
18. (d) Side = Diagonal = 7 r = 21
ja
3
R s
25. (b) Diagonal = 6 3
 S.A = 6a² = 6 × 49 = 294 cm² Side = 6 cm.
a th

19. (a) a = 5.6


ty a
di M

5.6
r= a
2
Surface area of visible faces:-
 14a² = 14 × 6² = 504 cm²
26. (b) r = 21
4 22 5.6 5.6 5.6 2
Volume =      r³ = r12 h
3 7 2 2 2 3
= 91.98 cm³
20. (b) C.S.A = 154 2
 (21)³ = r12 h .........(1)
3
22
 4×  r² = 154
A

7 C.S.A 2r1 h h 2
= = =
7 T.S.A 2r1 (h  r1 ) h  r1 5
 r=
2 h = 2x  r1 = 3x .......(2)
Put (2) in (1)
22 4 7 7 7
Volume =     2
7 3 2 2 2 × 21 × 21 × 21 = 9x² × 2x
3
11  49 539
= =  x³ = 7 × 7 × 7
3 3  x=7
= 179.7 cm³ r = 3 × 7 = 21

Aditya Ranjan (Excise Inspector) Selected Selection 226


https://sscstudy.com/
Join Telegram- Maths by Aditya Ranjan MENSURATION-3D

27. (b) R = 10 m.
4 22
t = 1 cm. 34. (c)   r³ = 38808
3 7
inner radius, r = 10 – 1 = 9 cm.
S.A = 2 Rh + 2 rh + 2 (R² – r²) 38808  3  7
r³ =
4  22
= 2 [Rh + rh + R² – r²]
r = 21
= 2 [(r + R) (h + R – r)]
22
22  4r² = 4 × × 21 × 21
= 2 [19 (13)] 7
7
= 5544 cm²
= 1552.57 cm²
2 22
4 22 539 35. (a) V =   (5.6)³
28. (b)   r³ = 3 7
3 7 3
= 367.95 cm³
7 36. (b)
 r= m

r
2
l = 10, h = 8

si
S.A = 4r²
 r = 100 – 64
22 7 7
an by r=6
= 4  
7 2 2  rl = × 6 × 10

n
= 154 m² = 60 m²
2 22
29. (c) Volume = ja   6.3 × 6.3 × 6.3
37. (b)  × 4 × 12 =  × r² ×
12
R s
3 7 100
= 523.90 cm³
a th

 r = 20 cm.
2 22 35 35 35  Thickness = 40 cm.
30. (a) Volume =    
3 7 2 2 2 = 0.4 cm.
ty a

= 11229.17 cm³ 38. (b) v = a³ = 343


31. (b) t = 0.37 = R – r  a=7
di M

r=6 T.S.A = 6 × 7²
 R = 6.37 = 294 m²
C.S.A = 2R²
4 22
39. (a) v =  (R³ – r³)
22 3 7
=2× × 6.37 × 6.37
7
4 22
= 255.0548 cm² =  (6³ – 3³)
3 7
4 18 3 = 252 cm³
32. (b)  
   = r²  12
3 2 40. (c) 12 × 18 × 27 = a³
A

4 9 9 9  a = 18
 r² = 
3 12
T.S.A of cuboid 2(lb bh lh)
 =
r = 9 mtr. T.S.A of cube 6a²
33. (d) l = 30, r = 14
T.S.A = r (l + r) 2(12  18  18  27  27  12)

6  18  18
22
=  14 (30  14)
7 19
=
= 1936 cm³ 18

Aditya Ranjan (Excise Inspector) Selected Selection 227


https://sscstudy.com/
Join Telegram- Maths by Aditya Ranjan MENSURATION-3D

41. (c) Wood wasted:-


4 22 7  7  7
Consides:- Total =  
3 7 1000
Volume of wood : Volume of cone
= 1.44 cm³
4 1
(24)³ : (21)²  42
3 3 22
47. (c) rl = × 7 × 9 = 198
7
1024 : 343
Let, length of cloath be l mtr.
1024 – 343
Required% =  100
1024  2 × l = 198

l = 99m
681
=  100
1024 48. (c) r = 4h = 4 × 14

= 66.50% Volume = r²h

r
2.8 22
42. (a) Radius of sphere = = 1.4

si
2 = × 4 × 14 × 4 × 14 × 14
7
an by
 volume =
4 22

3 7
 1.4 × 1.4 × 1.4 = 137984 cm³

n
= 11.50 cm³ 3
ja 49. (b)
R s
1 22
43. (d) Volume =   7  7  15
3 7
a th

= 770 cm³ 7

Weight of vessel = 770 × 15 5


ty a

= 11550 g.
= 11.55 kg.
di M

44. (c) rl = 594


1
Volume of frustum = (R² + r² + Rr)h
22 3
  21 × l = 594
7
l=9 1 22
  [25  9  15] 7
3 7
4 3
45. (a) r1 h = n  r2
2

3
22
 [49] = 359.3 cm³
3
4
 × 2 × 2 × 90 = n × ×  × (3)³
3
A

50. (d) Given,

4  90  3 L × B = 64 m²
 n= = 10
4 93  L = L = 64
Side  L=B=H=8
46. (a) Radius =
2
 Length of lougest rod = 8 3
4 4 22
 r³ =   (0.7)³
3 3 7 = Diagonal of cube

Aditya Ranjan (Excise Inspector) Selected Selection 228


https://sscstudy.com/
Join Telegram- Maths by Aditya Ranjan MENSURATION-3D

4
 Volume =  R³
3
51. (a)
4 22 21 21 21
=
   
3 7 2 2 2
27 = 4851 cm³
h 57. (a)

20 15

1
  r² h = r12 h1
3 R = 18 cm.

1
   × (20)² × 27 =  × (15)² × h1

r
3

si
 h1 = 16 cm.
52. (b) Let height of cone be h


4 22

an by 1 22
 12  12  12 =  66h 6

n
3 7 3 7
 h = 192 cm.
ja
R s
53. (d) ATQ,
2(2rh) = r²h
a th

3
4=r 2
 R³ = n ×  r² h
2 22 3
54. (c) Volume =  × 6.3 × 6.3 × 6.3
ty a

3 7 2
× 18³ = n × 3² × 6
= 523.908 cm³ 3
di M

 n = 4 × 18 = 72
Ar. 1 324 (18)² 58. (c)
55. (d) Ar.  = 289 = (17)²
2

Side of 1 18
 =
Side of 2 17
3
56. (c)

8
A

h = 3 cm, s = 8 cm.

21 1
T.S.A = × 4 × s × l + s²
2
Also,
We know,
 s 2
l² = h² +   = 9 + 16
Diagonal = 2 3 Radius 2 
21 3 = 2 3  Radius l=5

Aditya Ranjan (Excise Inspector) Selected Selection 229


https://sscstudy.com/
Join Telegram- Maths by Aditya Ranjan MENSURATION-3D

61. (b)
1
 T.S.A = × 4 × 8 × 5 + 64
2
= 80 + 64
= 144
24
59. (c)

1
b = 2h = l
2

Diagonal = l²  b²  h² 8


= 4b²  b² 
4
1 3
21 T.S.A = ×3×8×l (8)²
= b² 2 4
4

r
= 12 l + 16 3

si
b
= 21
2 l = (24)² – 4² = 560
60. (a)
Given,
an by = 4 35

n
 T.S.A = 48 35  16 3
2  rh = 126 
ja 62. (b)
R s
 2 ×  × r × 14 = 126
1
Volume of pyramid = × ar. of base × h
a th

9 3
r=
2
prism = Area of base × h
Volume =  r²h
Ar  h 3
ty a

 Requaired ratio = =
81 1 1
=  14  ar  h
4 3
di M

567 63. (a)


= 
2 r = 10 cm, h = 20 cm.
V = r² h
1
= 283 cm³  3.14 × 100 × 20
2
 314 × 20 = 6280 cm³.

......-------......
A

Aditya Ranjan (Excise Inspector) Selected Selection 230


https://sscstudy.com/
Join Telegram- Maths by Aditya Ranjan DATA INTERPRETATION

DATA INTERPRETATION
24
1. The following table shows the number of pages The number of trucks sold by B, C, F and H
printed by 3 printers during 3 days. is how much percent less than the number
of trucks sold by all these 8 companies?
B, C, F H
Printers

Days X Y Z
Monday 130 140 210 SSC CGL 01/12/2022 (Shift- 01)

r
Tuesday 110 145 160 (a) 46 percent

si
Wednesday 180 90 218 (b) 22 percent
an by
What is the average number of pages printed
by printer Z during the 3 days?
(c) 34 percent
(d) 14 percent

n
Z 3. The table given below shows the number of

ja spoon manufactured by five factories.
R s
SSC CGL 01/12/2022 (Shift- 01)

a th

(a) 196 (b) 194


(c) 192 (d) 190
2. The pie chart given below shows the number Factory Spoon
of truck sold by 8 different companies. The total
ty a

P 100
number of truck sold by all these 8 companies
Q 200
are 4000. Number of truck sold by a particular
di M

company is shown as a percent of total number R 150


of truck sold by all these 8 companies. S 50
T 250

What are the ratio of number of spoon
manufactured by P to the number of spoon
manufactured by R?

P R


A

SSC CGL 01/12/2022 (Shift- 01)


H A
G 14% 13% (a) 3 : 2 (b) 2 : 3
6%
F
(c) 14 : 1 (d) 1 : 3
B
7%
27% 4. Study the given chart and answer the following
3%
E

question.
D
12% C
18%

Aditya Ranjan (Excise Inspector) Selected Selection 231


https://sscstudy.com/
Join Telegram- Maths by Aditya Ranjan DATA INTERPRETATION

6. The following table shows the marks (in


Production of steel (in mn units) percentages) obtained by six students in four
different subjects in an examination.
Maximum marks in each subject are 100.
220 240 Rajasthan

Gujarat

Delhi

200 180 Kerala
SUBJECT
160 Tamil Nadu Student
SST PHYSICS CHEMISTRY MATHS
A 92 90 90 80
B 90 80 85 85
What is the central angle corresponding to
the production steel by Kerala? C 80 80 65 70
D 85 80 82 75

E 80 75 75 85

r
F 90 90 90 85
SSC CGL 01/12/2022 (Shift- 02)

si
(a) 82º (b) 65.8º Answer the following question based on the table:

5.
(c) 75.2ºan by (d) 72º
The following table shows the number of

What is the number of students whose overall

n
different items in different shops and their percentage obtained is 80% and above?
respective selling prices per unit.
ja
80%
R s
SSC CGL 01/12/2022 (Shift- 02)
a th

(a) 4 (b) 5
(c) 2 (d) 3
Total No. AC : Cooler Selling Price per unit 7. The table given below shows the number of
Shops of Items : Fan Cooler AC Fan persons participating in a survey from 6
ty a

A 5000 4:5:1 8000 25000 8500 different states.


B 1800 3:2:4 10000 20000 16000
di M

C 3400 6:4:7 6000 42000 15000


D 3600 4:2:3 12000 32000 8000
E 4000 5:1:4 8000 26500 12200 States Persons
F 1210 2:4:5 11000 28000 11100 S1 100
S2 200
Find the percentage of total revenue which comes
S3 400
from Cooler from shop E, considering all given
items are being sold from shop E and from all S4 500
the given shops only given three items are being S5 600
sold. (Rounded off to three decimal places) S6 800
E What is the ratio of number of person
A

participating in a survey from state S3 to the


number of person participating in a survey
E from state S4?
S3
S4
SSC CGL 01/12/2022 (Shift- 02)
SSC CGL 01/12/2022 (Shift- 02)
(a) 4.226% (b) 3.516%
(a) 3 : 5 (b) 5 : 4
(c) 10.45% (d) 8.910% (c) 4 : 3 (d) 4 : 5

Aditya Ranjan (Excise Inspector) Selected Selection 232


https://sscstudy.com/
Join Telegram- Maths by Aditya Ranjan DATA INTERPRETATION

8. The pie chart given below shows the number If the interest on loans amounted to Rs.3.15
of bike sold by 8 different companies. The total crores, then the total amount of expenditure
number of bike sold by all these 8 companies on salary, taxes and infrastructure is:
are 2000. Number of bikes sold by a particular
company is shown as a percent of total number
of bike sold by all these 8 companies.
SSC CGL 01/12/2022 (Shift- 03)

(a) 9 crores (b) 7.8 crores
(c) 5.5 crores (d) 8.5 crores
10. The table given below shows the income of
two companies C1 and C2 in 6 years.
CI
C2
V Company
2% W
10% P Year C1 C2

r
21% P 750 850
U

si
16% Q 200 250
Q R 330 350
an by T
7% 19% S 550 650
S
R T 530 270
20%

n
5% U 370 390

ja
What is the difference between the average Which of the following statement is NOT correct?
R s
number of bikes sold by P, Q, R and S and the
average number of bikes sold by T, U, V and W?
a th

I. The income of C1 in year P is 33.33


P, Q, R S percent of the income of C2 in year Q.
T, U, V W P CI Q 33.33
II. The average income of C1 and C2 in year
ty a

T is 400.
SSC CGL 01/12/2022 (Shift- 03)
T CI C2 400
di M

(a) 150 (b) 200


(c) 175 (d) 125 SSC CGL 01/12/2022 (Shift- 03)
9. The bar graph shows the percentage (a) Only I (b) Both I and II
distribution of the expenditure of a company (c) Neither I nor II (d) Only II
under various expense heads during 2003. 11. Study of the following table and answer the
question below.

25

20 20 17.5 Total number of Percentage of enrolled Ratio of male to female
20 School Name
Percent Spent

students enrolled students, opted Biology students who opted Biology

12.5 15 A 900 30 7:8


15 B 400 36 5:7
A

10 C 1000 24 5 : 19
10 D 800 18 3:9

5
5 Find the ratio of the number of female students
0
who opted biology in school B and school D.
B D
Interstion
Infrastructure

Transport

Advertisement

R&D
Taxes

Salary

Loans


SSC CGL 01/12/2022 (Shift- 03)
(a) 7 : 9 (b) 9 : 7
(c) 3 : 5 (d) 5 : 3

Aditya Ranjan (Excise Inspector) Selected Selection 233


https://sscstudy.com/
Join Telegram- Maths by Aditya Ranjan DATA INTERPRETATION

12. The table given below shows the number of


bicycle sold by six companies.


Companies Bicycle
A 60
F
B 90 10%
C 120 E
A
D 165 16%
37%
E 180
D
F 120 4% C
B
20%
What are the ratio of number of bicycle sold 13%
by B to the number of bicycle sold by D?

r
B D P1 = The value of average production of A,
C, D and F.

si

P1 = A, C, D F
an by SSC CGL 01/12/2022 (Shift- 04)
P2 = The difference between the production
(a) 11 : 5 (b) 11 : 6 of B and E.

n
(c) 5 : 13 (d) 6 : 11 P2 = B E
13. The following table gives the subscription of What is the value of (P1 + P2)?
ja
different schemes of a Mutual Fund Company
R s
(P1 + P2)
over th months.
SSC CGL 01/12/2022 (Shift- 04)
a th

(a) 2525 (b) 6456


(c) 3525 (d) 4565
(Rupees in crores) 15. The pie chart given below shows the sales of 7
ty a

Schemes different companies. The total sales of all these


Months 7 companies are 2880. The sale of a particular
di M

V W X Y Z Total
company is shown in terms of degree with respect
September 200 70 30 290 10 600
to the total sales of all these 7 companies.
October 120 130 70 150 290 760
November 45 35 25 125 160 390

December 160 110 40 115 130 555
January 80 90 70 100 140 480
February 130 150 30 40 390 740
What is the different in the subscription of
D
17º
scheme X between December and February?
X J
A

34º
I
SSC CGL 01/12/2022 (Shift- 04) 53º E
(a) 10 crores (b) 18 crores 119º
(c) 8 crores (d) 25 crores H
14. The pie chart given below shows the production 73º F
of 6 different factories. The total production of 51º
all these 6 factories are 22000. The production
of a particular factory is shown as a percent of G
total production of all these 6 factories. 13º

Aditya Ranjan (Excise Inspector) Selected Selection 234


https://sscstudy.com/
Join Telegram- Maths by Aditya Ranjan DATA INTERPRETATION

The average sales of G and H is how much percent SSC CGL 02/12/2022 (Shift- 01)
more than the average sales of D and I? (a) 56 (b) 55
G H D I (c) 57 (d) 58
18. The table given below shows the number of

pencils sold by 7 shops.
SSC CGL 01/12/2022 (Shift- 04)
(a) 18.86 percent (b) 24.25 percent
(c) 56.21 percent (d) 22.86 percent
Shops Pencils
16. The table given below shows the production S1 54
of beauty products by five different companies.
S2 68
S3 70
S4 120
Companies Production S5 84
P 80 S6 130

r
Q 120 S7 36

si
R 84 What is the ratio of number of pencils sold
an byS 50 by S3 to the number of pencils sold by S6?
T 70 S3 S6

n

The production of beauty product by T is what
SSC CGL 02/12/2022 (Shift- 01)
percent of the production of beauty product
by S?
ja (a) 8 : 15 (b) 13 : 7
R s
(c) 15 : 7 (d) 7 : 13
T S
19. The pie chart given below shows the population of
a th

8 cities in a year. Total population of all the 8 cities


SSC CGL 02/12/2022 (Shift- 01) is 30,000. Population in a particular city is shown
(a) 40 percent (b) 140 percent as a percent of total population in all these 8 cities.
ty a

(c) 240 percent (d) 120 percent


17. Observe the given graph and answer the

di M

following question.




C7
C6 3%
10 6% C8
9 7% C1
18 C5 15%
8 5%
No. of students

7
6 C2
5
15 24%
14 C4
4
A

32%
3 13 C3
12 8%
2
10 11
1
0
What is the average population (approximately)
125 130 135 140 145 150 160 of cities C3, C5 and C7?
Marks Obtained C3, C5 C7
How many students obtained more than 135 SSC CGL 02/12/2022 (Shift- 01)
marks? (a) 1200 (b) 1800
(c) 1600 (d) 2000

Aditya Ranjan (Excise Inspector) Selected Selection 235


https://sscstudy.com/
Join Telegram- Maths by Aditya Ranjan DATA INTERPRETATION

ANSWER KEY
1.(a) 2.(c) 3.(b) 5.(d) 6.(a) 7.(a) 8.(d) 9.(a) 10.(a) 10.(a)

11.(a) 12.(d) 13.(a) 14.(d) 15.(d) 16.(b) 17.(d) 18.(d) 19.(c)

SOL U TION S
1. (a)
4  8000

(4  8000)  (20  26500)  (16  12200)

r
Total pages by z
Avg. =
3

si
32000
=
210  160  218 32000  530000  195200
= an by 3
=
320

n
320  5300  1950
588
= = 196
3 320
ja =
7572
= 4.226%
R s
2. (c)
Total trucks sold by B, C, F, H 6. (a)
a th

= 27 + 18 + 7 + 14 = 66% 92 + 90 + 90 + 80
% of A = × 100 > 80%
Total trucks = 100% 400
Required% = 100 – 66 = 34% % of B > 80%
ty a

3. (b) % of C < 80%


% of D > 80%
di M

100 2 % of E < 80%


Required ratio = =
150 3 % of F > 80%
4. (d)  Required number = 4
Total production 7. (d)
= 220 + 240 + 200 + 180 + 160 400 4
Required Ratio = =
= 1000 500 5
8. (a)
 1000  360°
No. of Bikes by P, Q, R, S = 21 + 19 + 5 + 20 = 65%
360
200   200 65
1000  Avg. = % of 2000
4
= 72
A

No. of Bikes by T, U, V, W = 7 + 16 + 2 + 10 = 35
(Question incomplete)
35
5. (a) Avg. = % of 2000
4
Let total items be 40
(65 – 35)
 Difference = % of 2000
4 4
Fan =  40 = 16
10
30
AC = 20 =  2000
400
Cooler = 4 = 150

Aditya Ranjan (Excise Inspector) Selected Selection 236


https://sscstudy.com/
Join Telegram- Maths by Aditya Ranjan DATA INTERPRETATION

9. (a) 14. (d)


Salary + Taxes + Infra = 20 + 10 + 20 = 50% 37  20  4  10 71
P1 = = %
17.5  3.15 4 4
P2 = 16 – 13 = 3%
3.15
50   50 = 9 cr.
17.5  71 
P1  P2 = 
  3
  22000
10. (d)  4 

I. C1 in P = 750 83 22000
=  = 4565
C2 in Q = 250 4 100
 Required% = 300% 15. (d)
 Not True 73  13
Avg. of G and H =
2
CI  C2 530  270
II. =
2 2 = 43º

r
= 400 53  17

si
Avg. of D and I =
 True. 2

11. (a)
an by
Biology Female in school B
= 35°

43 – 35

n
Required% =  100
35
7 36
=  ja
 400
800
R s
12 100 = = 22.86%
35
a th

= 84
16. (b)
Biology Female in school D
70
Required% =  100
9 18 50
ty a

=   800
12 100 = 140%
di M

= 108 17. (d)


From graph, number of students with marks
84 7 > 135 = 18 + 15 + 14 + 11 = 58
Required ratio = =
108 9 18. (d)
12. (d) 70 7
Required ratio = =
130 13
90 18 6
Required ratio = = = 19. (c)
165 33 11
8  5  3 16
13. (a) Avg. = = %
3 3
A

Scheme X in December = 40 Cr.


16
Scheme X in February = 30 Cr.  % of 30, 000
3
 Difference = 40 – 30 = 10 Cr.
 1600

......-------......

Aditya Ranjan (Excise Inspector) Selected Selection 237


https://sscstudy.com/
Join Telegram- Maths by Aditya Ranjan STATISTICS

STATISTICS
25
1. Find the mode for the given distribution 5. Mean of a sample data = 60 and median = 48.
(rounded off to two decimal places). Find the mode of this distribution.
Class 5 – 10 10 – 15 15 – 20 20 – 25 25 – 30 30 – 35 = 60 = 48
Interval
Frequency 8 7 6 9 11 10
SSC CGL TIER- II 06/03/2023
(a) 36 (b) 18

(c) 24 (d) 48

6. What will be the mode of the following data?

r
13, 15, 31, 12, 27, 13, 27, 30, 27, 28 and 16
C
S

si
CGL TIER - II 02/03/2023
(a) 35.25
(c) 30.33
an by (b) 40.25
(d) 28.33 (a) 28
SSC CGL TIER- II 06/03/2023
(b) 31

n
2. The arithmetic mean of the following data is
(c) 30 (d) 27
_________.
7. The median of a set of 11 distinct observations
23, 17,20,19,21
ja is 73.2. If each of the largest five observations
R s
____ of the set is increased by 3, then the median
of the new set:
a th

23, 17,20,19,21
SSC CGL TIER - II 02/03/2023
(a) 20 (b) 19
(c) 23 (d) 21
ty a

3. Find the standard deviation of the following SSC CGL TIER- II 07/03/2023
data (rounded off to two decimal places).
(a) Is 3 times that of the original set
di M



(b) Is increased by 3/
SSC CGL TIER- II 03/03/2023
(c) Remains the same as that of the original set
(a) 1.48 (b) 3.21

(c) 4.12 (d) 2.45
4. The median of the following data will be (d) Is decreased by 3/
_________. 8. What is the mode of the given data?
32, 25,33,27, 35, 29 and 30 5, 7, 9, 7, 3, 7, 5, 7, 8, 6, 7

32, 25,33,27, 35, 29 30 5, 7, 9, 7, 3, 7, 5, 7, 8, 6, 7
A

SSC CGL TIER- II 03/03/2023 SSC CGL TIER- II 07/03/2023


(a) 32 (b) 27 (a) 7 (b) 6
(c) 30 (d) 29 (c) 5 (d) 3

ANSWER KEY
1.(d) 2.(a) 3.(a) 4.(c) 5.(c) 6.(d) 7.(c) 8.(a)

Aditya Ranjan (Excise Inspector) Selected Selection 238


https://sscstudy.com/
Join Telegram- Maths by Aditya Ranjan STATISTICS

SOL U TION S
1. (d) Step:- 3
Modal class = max. frequency group.
1 49 9 81
Here, modal class = 25 – 30   
16 16 16 16 = 140
 f1 – f0  4 64
Mode = l   
 2f – f – f   h
 1 0 2 Step:- 4

l = lower limit of modal class = 25


140
h = size of interval =5 Standard deviation =  1.48
64

r
f1 = frequency of modal class = 11
4. (c)

si
f0 = rrequency of class preceeding modal class =
9 Median = middle term
an by
f² = frequency of class suceeding modal class =
10
arranging data increasing order:-
25, 27, 29, 30, 32, 33, 35

n
 11 – 9  Median = 30
 Mode = 25 + 
  5

 22 – 9 – 10 
ja 5. (c)
R s
We know,
85
= = 28.33
a th

3 3 × Median = Mode + 2 Mean


 3 × 48 = Mode + (2 × 60)
2. (a)
 Mode = 24
23  17  20  19  21
ty a

Arithmetic mean = 6. (d)


5
From given data
di M

= 20
27 appears maximum no. of times.
3. (a)
Standard deviation of 5, 3, 4, 7  Mode = 27
Step:- 1 7. (c)
Since, median is the middle term, increase in
19 largest five observation does not affect the
Mean of data 
4 median.
Step:- 2  It will remain same
2 2 2 2
 19  19  19   19  8. (a)
5 –
  ,
  – 3
 ,
 – 4
 ,
7 – 

 4  4  4   4  Maximum repetition = 7
A

 Mode = 7
1 49 9 81
 , , ,
16 16 16 16

......-------......

Aditya Ranjan (Excise Inspector) Selected Selection 239


https://sscstudy.com/
Join Telegram- Maths by Aditya Ranjan PROBABILITY

PROBABILITY
26

1. A glass jar contains 6 white, 8 black, 4 red 3. The probability that three students A, B and
and 3 blue marbles. If a single marble is chosen
3 5 1
at random from the jar, what is the probability C solve a problem are , and
that it is black or blue? 7 9 5
respectively. What is the probability that the
problem is solved?
A, B C

3 5 1
,
7 9 5

r
SSC CGL TIER - II 02/03/2023

si
8 11 SSC CGL TIER- II 06/03/2023
(a) an by (b)
21 21 64 32
(a) (b)
315 315

n
5 1
(c) (d)
21 7 251 155
ja (c) (d)
315 315
R s
2. A speaks the truth 5 out of 7 times and B
speaks truth 8 out of 9 times. What is the 4. A person can hit a target 5 times out of 8
a th

probability that they contradict each other in shots. If he fires 10 shots, what is the
stating the same fact? probability that he will hit the target twice?

A, 7 B, 9
ty a



di M

SSC CGL TIER- II 07/03/2023


SSC CGL TIER- II 03/03/2023
1135  38 1165  38
1 1 (a) (b)
(a) (b) 810 810
7 9
1175  38 1125  38
(c) (d)
1 1 810 810
(c) (d)
4 3
A

ANSWER KEY
1.(b) 2.(d) 3.(c) 4.(d)

Aditya Ranjan (Excise Inspector) Selected Selection 240


https://sscstudy.com/
Join Telegram- Maths by Aditya Ranjan PROBABILITY

SOL U TION S
1. (b)
4 4
Black = 8, Blue = 3, total = 21 P (A1 ) = , P (B1 ) =
7 9
8 3 11
P (Black or Blue) =  = 4
21 21 21 P (C1 ) =
5
2. (d)
 P (question not solved)
5 P (A1) × P (B1) × P (C1)
Probability that A speaks truth = P (A) =
7
4 4 4 64
=   =
8 7 9 5 315
and B speaks truth = P (B) =

r
9
64
 P (question solved) = 1 –

si
5 2 8 1 315
P (A lies) = 1 – = , P (B lies) = 1 – =
an by 7 7 9 9
251
Required probability =
315

n
= [P (A) × P (B1)] + [P (A1) × P (B)]
4. (d)
5 1  2 8 ja
=
   
   5
R s
 P (A hits target) = P (A) =
7 9  7 9 
8
a th

5 16 21 1
=  = = 3
63 63 63 3 P (A does not hit target) = P (A1 ) = ,
8
3. (c)
ty a

 3 10
3 P (does not hit in 10 shots) =  
P (A) = 8
di M

7
P (hit the target twice in 10 shots)
5
P (B) =  5 2  3 8
9 = 10
C2  
   
  
8  8 
1
P (C) =
5 10  9 25  38 1125  38
=  =
2 810 810

......-------......
A

Aditya Ranjan (Excise Inspector) Selected Selection 241


https://sscstudy.com/

You might also like